diff --git a/TST/DS/DS_21_04_07/TST1/01_210407_DS8.tex b/TST/DS/DS_21_04_07/TST1/01_210407_DS8.tex new file mode 100644 index 0000000..70bcec9 --- /dev/null +++ b/TST/DS/DS_21_04_07/TST1/01_210407_DS8.tex @@ -0,0 +1,262 @@ +\documentclass[a4paper,10pt]{article} +\usepackage{myXsim} + +% Title Page +\title{DS8 \hfill AIOUAZ Ahmed} +\tribe{TST} +\date{\hfillÀ render pour le Mercredi 7 avril} + +\xsimsetup{ + solution/print = false +} + +\begin{document} +\maketitle + +\begin{exercise}[subtitle={Automatismes}] + \textit{Toutes les questions de cette exercice sont indépendantes et peuvent être répondus séparément} + \begin{enumerate} + \item De janvier à septembre, une quantité a augmenté de $20\,\%$. Faire un schéma pour représenter la situation puis calculer le taux d'évolution moyen mensuel. + \item Une quantité augmente de $20\,\%$ par ans. En 2020, elle est de 110\euro. Quelle était sa valeur en 2019? Faire un schéma pour représenter la situation. + \item Déterminer l'équation de la droite \\ + \begin{tikzpicture}[xscale=0.8, yscale=0.5] + \tkzInit[xmin=-5,xmax=5,xstep=1, + ymin=-5,ymax=5,ystep=1] + \tkzGrid + \tkzAxeXY + \tkzFct[domain=-5:5,color=red,very thick]% + {3.0*\x -3}; + \end{tikzpicture} + \item Résoudre l'équation $2 \times 0.07^x = 20$ + \end{enumerate} +\end{exercise} + +\begin{solution} + \begin{enumerate} + \item On veut partager cette évolution en 8 évolutions. + \[ + \left(1 + \frac{20}{100}\right)^{\frac{1}{8}} = 1.0231 + \] + Donc le taux d'évolution moyen est + \[ + t_m = 1.0231 - 1 = 0.0230999999999999 + \] + \item Coefficient multiplicateur pour revenir en arrière + \[ + CM = (1 + \frac{20}{100})^{-1} = 0.8333 + \] + On en déduit la quantité en 2019 + \[ + 110 * 0.8333 = 91.66300000000001 + \] + \item L'équation de la droite est + \[ + y = 3.0 x -3 + \] + \item Il faut penser à faire la division à par $2$ avant d'utiliser le log car sinon, on ne peut pas utiliser la formule $\log(a^n) = n\times \log(a)$. + + \[x = \frac{\log(10.0)}{\log(0.07)}\] + \end{enumerate} +\end{solution} + +\begin{exercise}[subtitle={Restaurant}] + Un \emph{food truck}, ouvert le midi et le soir, propose deux types de formules : + + \setlength\parindent{10mm} + \begin{itemize} + \item la formule \emph{Burger} ; + \item la formule \emph{Wok}. + \end{itemize} + \setlength\parindent{0mm} + + \medskip + + Le gérant a remarqué que 9\,\% de ses ventes ont lieu le midi. Le quart des ventes du midi correspondent à la formule \emph{Burger}, alors que 3\,\% des ventes du soir correspondent à la formule \emph{Wok}. + + Le gérant se constitue un fichier en notant, pour chaque vente, la formule choisie et le moment de cette vente (midi ou soir). + + On prélève une fiche de façon équiprobable. On définit les quatre évènements suivants: + + \begin{enumerate} + \item $M$ : \og la fiche correspond à une vente du midi\fg{} ; + \item $S$ : \og la fiche correspond à une vente du soir\fg {}; + \item $W$ : \og la fiche correspond à une formule \emph{Wok} \fg{} ; + \item $B$ : \og la fiche correspond à une formule \emph{Burger} \fg. + \end{enumerate} + \setlength\parindent{0mm} + + \medskip + + \begin{enumerate} + \item Recopier puis compléter l'arbre pondéré + + \begin{center} + \begin{tikzpicture}[sloped] + \node {.} + child {node {$M$} + child {node {$W$} + edge from parent + node[above] {...} + } + child {node {$B$} + edge from parent + node[above] {...} + } + edge from parent + node[above] {...} + } + child[missing] {} + child { node {$S$} + child {node {$W$} + edge from parent + node[above] {...} + } + child {node {$B$} + edge from parent + node[above] {...} + } + edge from parent + node[above] {...} + } ; + \end{tikzpicture} + \end{center} + + \item Calculer la probabilité de l'évènement $M \cap W$. Interpréter ce résultat dans le contexte de l'exercice. + \item Montrer que la probabilité que la fiche choisie corresponde à une formule \emph{Burger} est égale à $0.9052$. + \item On a prélevé une fiche correspondant à la formule \emph{Burger}. Quelle est la probabilité, arrondie au millième, que la vente ait eu lieu le soir? + \end{enumerate} +\end{exercise} + +\begin{solution} + \begin{enumerate} + \item + \begin{center} + \begin{tikzpicture}[sloped] + \node {.} + child {node {$M$} + child {node {$W$} + edge from parent + node[above] {$0.75$} + } + child {node {$B$} + edge from parent + node[above] {$0.25$} + } + edge from parent + node[above] {$0.09$} + } + child[missing] {} + child { node {$S$} + child {node {$W$} + edge from parent + node[above] {$0.03$} + } + child {node {$B$} + edge from parent + node[above] {$0.97$} + } + edge from parent + node[above] {$0.91$} + } ; + \end{tikzpicture} + \end{center} + \item On calcule la probabilité que la vente soit un wok et ait eu lieu à midi + \[ P(M\cap W) = P(M) \times P_M(W) = 0.09 \times 0.75 = 0.0675 \] + \item Probabilité que la vente soit un burger. + \[ + P(B) = P(M\cap B) + P(S\cap B) = 0.09 \times 0.75 + 0.91 \times 0.03 = 0.9052 + \] + \item On cherche à calculer la quantité $P_B(S)$. Pour cela on utilise la formule de Bayes + \[ + P_B(S) = \frac{P(B\cap S)}{P(B)} = \frac{P_S(B) \times P(S)}{P(B)} = \frac{0.97\times 0.91}{0.9052} = 0.975143614670791 \approx 0.975 + \] + \end{enumerate} +\end{solution} + +\begin{exercise}[subtitle={Continent plastique}] + \textit{Les quantités évoqués dans cette exercice sont générés au hasard et sont donc complètement farfelus.} + \medskip + Le \og continent de plastique\fg{} est la plus grande des plaques de déchets plastiques évoluant sur les océans. Elle occupe actuellement dans l'océan Pacifique une surface dont l'aire est évaluée à plus de $1,6$ million de km$^2$, entre Hawaï et la Californie. + + En 2017, des scientifiques ont estimé qu'il y avait $14$ millions de tonnes de déchets plastiques qui était déversé chaque année dans les océans et que cette quantité augmentait de $28\n\%$ par chaque année. + + On modélise l'évolution de la masse de ces déchets plastiques déversée chaque année, si rien n'est fait pour la réduire, par une suite géométrique $\left(u_n\right)$. L'arrondi au centième du terme $u_n$ représente la masse de ces déchets déversée chaque année, exprimée en million de tonnes, pour l'année $(2017 + n)$. + + \medskip + + \begin{enumerate} + \item Expliquer pourquoi la suite $u_n$ est géométrique? + \item Calculer $u_1$ et $u_2$. + \item Exprimer $u_n$ en fonction de $n$. + \item Au début de l'année 2017, il y avait $300$ millions de tonnes de déchets plastique. Calculer la quantité totale de déchets plastiques en 2030. + \item On souhaite déterminer en quelle année la masse totale de ces déchets plastiques aura pour la première fois augmenté de $50$\,\% par rapport à sa valeur de 2017. + \begin{enumerate} + \item Recopier et compléter l'algorithme ci-dessous pour que la variable $N$ contienne la réponse au problème posé. + + \begin{center} + \begin{tabularx}{0.4\linewidth}{|X|}\hline + $N = 2017$\\ + $U = 14$ \\ + $S = 300 + U$ \\ + while $S < 450$: \\ + \hspace{1cm} $N = \ldots$\\ + \hspace{1cm} $U = \ldots$\\ + \hspace{1cm} $S = \ldots$\\ + \hline + \end{tabularx} + \end{center} + \item Que contiennent les variables $S$, $U$ et $N$ après exécution de cet algorithme ? + + Interpréter les résultats dans le contexte de l'exercice. + \end{enumerate} + \end{enumerate} +\end{exercise} + +\begin{solution} + \begin{enumerate} + \item Une augmentation de $28\,\%$ revient à multiplier la quantité par $1.28$. La suite est donc bien géométrique. Son premier terme est $u_0 = 14$ et sa raison est $q = 1.28$ + \item + \[ + u_1 = u_0 * 1.28 = 17.92 + \] + \[ + u_2 = u_0 * 1.28^2 = 22.9376 + \] + \item + \[ + u_n = u_0 \times q^n = 14 \times 1.28^n + \] + \item On calcule la quantité totale déversée entre 2017 et 2030. + \[ + \sum_{n = 0}^{13} u_n = u_0 \times \frac{1-q^{13}}{1-q} = 14 \times \frac{1 - 1.28^{13}}{1 - 1.28} = 1187.94 + \] + On en déduit la quantité totale de déchets en 2030 + \[ + 300 + 1187.94 = 1487.94 + \] + \item + \begin{enumerate} + \item ~ + \begin{center} + \begin{tabularx}{0.4\linewidth}{|X|}\hline + $N \gets 2017$\\ + $U \gets 14$ \\ + $S \gets 300 + U$ \\ + Tant que $S < 450$ \\ + \hspace{1cm} $N \gets N + 1$\\ + \hspace{1cm} $U \gets U * 1.28$\\ + \hspace{1cm} $S \gets S + u$\\ + Fin Tant que\\\hline + \end{tabularx} + \end{center} + \item \textit{Pas de correction automatisé} + \end{enumerate} + \end{enumerate} +\end{solution} + +\end{document} + +%%% Local Variables: +%%% mode: latex +%%% TeX-master: "master" +%%% End: diff --git a/TST/DS/DS_21_04_07/TST1/02_210407_DS8.tex b/TST/DS/DS_21_04_07/TST1/02_210407_DS8.tex new file mode 100644 index 0000000..6b481a7 --- /dev/null +++ b/TST/DS/DS_21_04_07/TST1/02_210407_DS8.tex @@ -0,0 +1,262 @@ +\documentclass[a4paper,10pt]{article} +\usepackage{myXsim} + +% Title Page +\title{DS8 \hfill BAHBAH Zakaria} +\tribe{TST} +\date{\hfillÀ render pour le Mercredi 7 avril} + +\xsimsetup{ + solution/print = false +} + +\begin{document} +\maketitle + +\begin{exercise}[subtitle={Automatismes}] + \textit{Toutes les questions de cette exercice sont indépendantes et peuvent être répondus séparément} + \begin{enumerate} + \item De janvier à septembre, une quantité a augmenté de $28\,\%$. Faire un schéma pour représenter la situation puis calculer le taux d'évolution moyen mensuel. + \item Une quantité augmente de $28\,\%$ par ans. En 2020, elle est de 138\euro. Quelle était sa valeur en 2019? Faire un schéma pour représenter la situation. + \item Déterminer l'équation de la droite \\ + \begin{tikzpicture}[xscale=0.8, yscale=0.5] + \tkzInit[xmin=-5,xmax=5,xstep=1, + ymin=-5,ymax=5,ystep=1] + \tkzGrid + \tkzAxeXY + \tkzFct[domain=-5:5,color=red,very thick]% + {1.0*\x -2}; + \end{tikzpicture} + \item Résoudre l'équation $7 \times 0.54^x = 18$ + \end{enumerate} +\end{exercise} + +\begin{solution} + \begin{enumerate} + \item On veut partager cette évolution en 8 évolutions. + \[ + \left(1 + \frac{28}{100}\right)^{\frac{1}{8}} = 1.0313 + \] + Donc le taux d'évolution moyen est + \[ + t_m = 1.0313 - 1 = 0.031300000000000106 + \] + \item Coefficient multiplicateur pour revenir en arrière + \[ + CM = (1 + \frac{28}{100})^{-1} = 0.7812 + \] + On en déduit la quantité en 2019 + \[ + 138 * 0.7812 = 107.8056 + \] + \item L'équation de la droite est + \[ + y = 1.0 x -2 + \] + \item Il faut penser à faire la division à par $7$ avant d'utiliser le log car sinon, on ne peut pas utiliser la formule $\log(a^n) = n\times \log(a)$. + + \[x = \frac{\log(2.57)}{\log(0.54)}\] + \end{enumerate} +\end{solution} + +\begin{exercise}[subtitle={Restaurant}] + Un \emph{food truck}, ouvert le midi et le soir, propose deux types de formules : + + \setlength\parindent{10mm} + \begin{itemize} + \item la formule \emph{Burger} ; + \item la formule \emph{Wok}. + \end{itemize} + \setlength\parindent{0mm} + + \medskip + + Le gérant a remarqué que 57\,\% de ses ventes ont lieu le midi. Le quart des ventes du midi correspondent à la formule \emph{Burger}, alors que 49\,\% des ventes du soir correspondent à la formule \emph{Wok}. + + Le gérant se constitue un fichier en notant, pour chaque vente, la formule choisie et le moment de cette vente (midi ou soir). + + On prélève une fiche de façon équiprobable. On définit les quatre évènements suivants: + + \begin{enumerate} + \item $M$ : \og la fiche correspond à une vente du midi\fg{} ; + \item $S$ : \og la fiche correspond à une vente du soir\fg {}; + \item $W$ : \og la fiche correspond à une formule \emph{Wok} \fg{} ; + \item $B$ : \og la fiche correspond à une formule \emph{Burger} \fg. + \end{enumerate} + \setlength\parindent{0mm} + + \medskip + + \begin{enumerate} + \item Recopier puis compléter l'arbre pondéré + + \begin{center} + \begin{tikzpicture}[sloped] + \node {.} + child {node {$M$} + child {node {$W$} + edge from parent + node[above] {...} + } + child {node {$B$} + edge from parent + node[above] {...} + } + edge from parent + node[above] {...} + } + child[missing] {} + child { node {$S$} + child {node {$W$} + edge from parent + node[above] {...} + } + child {node {$B$} + edge from parent + node[above] {...} + } + edge from parent + node[above] {...} + } ; + \end{tikzpicture} + \end{center} + + \item Calculer la probabilité de l'évènement $M \cap W$. Interpréter ce résultat dans le contexte de l'exercice. + \item Montrer que la probabilité que la fiche choisie corresponde à une formule \emph{Burger} est égale à $0.3592$. + \item On a prélevé une fiche correspondant à la formule \emph{Burger}. Quelle est la probabilité, arrondie au millième, que la vente ait eu lieu le soir? + \end{enumerate} +\end{exercise} + +\begin{solution} + \begin{enumerate} + \item + \begin{center} + \begin{tikzpicture}[sloped] + \node {.} + child {node {$M$} + child {node {$W$} + edge from parent + node[above] {$0.75$} + } + child {node {$B$} + edge from parent + node[above] {$0.25$} + } + edge from parent + node[above] {$0.58$} + } + child[missing] {} + child { node {$S$} + child {node {$W$} + edge from parent + node[above] {$0.49$} + } + child {node {$B$} + edge from parent + node[above] {$0.51$} + } + edge from parent + node[above] {$0.42$} + } ; + \end{tikzpicture} + \end{center} + \item On calcule la probabilité que la vente soit un wok et ait eu lieu à midi + \[ P(M\cap W) = P(M) \times P_M(W) = 0.58 \times 0.75 = 0.435 \] + \item Probabilité que la vente soit un burger. + \[ + P(B) = P(M\cap B) + P(S\cap B) = 0.58 \times 0.75 + 0.42 \times 0.49 = 0.3592 + \] + \item On cherche à calculer la quantité $P_B(S)$. Pour cela on utilise la formule de Bayes + \[ + P_B(S) = \frac{P(B\cap S)}{P(B)} = \frac{P_S(B) \times P(S)}{P(B)} = \frac{0.51\times 0.42}{0.3592} = 0.5963251670378619 \approx 0.596 + \] + \end{enumerate} +\end{solution} + +\begin{exercise}[subtitle={Continent plastique}] + \textit{Les quantités évoqués dans cette exercice sont générés au hasard et sont donc complètement farfelus.} + \medskip + Le \og continent de plastique\fg{} est la plus grande des plaques de déchets plastiques évoluant sur les océans. Elle occupe actuellement dans l'océan Pacifique une surface dont l'aire est évaluée à plus de $1,6$ million de km$^2$, entre Hawaï et la Californie. + + En 2017, des scientifiques ont estimé qu'il y avait $9$ millions de tonnes de déchets plastiques qui était déversé chaque année dans les océans et que cette quantité augmentait de $26\n\%$ par chaque année. + + On modélise l'évolution de la masse de ces déchets plastiques déversée chaque année, si rien n'est fait pour la réduire, par une suite géométrique $\left(u_n\right)$. L'arrondi au centième du terme $u_n$ représente la masse de ces déchets déversée chaque année, exprimée en million de tonnes, pour l'année $(2017 + n)$. + + \medskip + + \begin{enumerate} + \item Expliquer pourquoi la suite $u_n$ est géométrique? + \item Calculer $u_1$ et $u_2$. + \item Exprimer $u_n$ en fonction de $n$. + \item Au début de l'année 2017, il y avait $300$ millions de tonnes de déchets plastique. Calculer la quantité totale de déchets plastiques en 2030. + \item On souhaite déterminer en quelle année la masse totale de ces déchets plastiques aura pour la première fois augmenté de $50$\,\% par rapport à sa valeur de 2017. + \begin{enumerate} + \item Recopier et compléter l'algorithme ci-dessous pour que la variable $N$ contienne la réponse au problème posé. + + \begin{center} + \begin{tabularx}{0.4\linewidth}{|X|}\hline + $N = 2017$\\ + $U = 9$ \\ + $S = 300 + U$ \\ + while $S < 450$: \\ + \hspace{1cm} $N = \ldots$\\ + \hspace{1cm} $U = \ldots$\\ + \hspace{1cm} $S = \ldots$\\ + \hline + \end{tabularx} + \end{center} + \item Que contiennent les variables $S$, $U$ et $N$ après exécution de cet algorithme ? + + Interpréter les résultats dans le contexte de l'exercice. + \end{enumerate} + \end{enumerate} +\end{exercise} + +\begin{solution} + \begin{enumerate} + \item Une augmentation de $26\,\%$ revient à multiplier la quantité par $1.26$. La suite est donc bien géométrique. Son premier terme est $u_0 = 9$ et sa raison est $q = 1.26$ + \item + \[ + u_1 = u_0 * 1.26 = 11.34 + \] + \[ + u_2 = u_0 * 1.26^2 = 14.2884 + \] + \item + \[ + u_n = u_0 \times q^n = 9 \times 1.26^n + \] + \item On calcule la quantité totale déversée entre 2017 et 2030. + \[ + \sum_{n = 0}^{13} u_n = u_0 \times \frac{1-q^{13}}{1-q} = 9 \times \frac{1 - 1.26^{13}}{1 - 1.26} = 663.76 + \] + On en déduit la quantité totale de déchets en 2030 + \[ + 300 + 663.76 = 963.76 + \] + \item + \begin{enumerate} + \item ~ + \begin{center} + \begin{tabularx}{0.4\linewidth}{|X|}\hline + $N \gets 2017$\\ + $U \gets 9$ \\ + $S \gets 300 + U$ \\ + Tant que $S < 450$ \\ + \hspace{1cm} $N \gets N + 1$\\ + \hspace{1cm} $U \gets U * 1.26$\\ + \hspace{1cm} $S \gets S + u$\\ + Fin Tant que\\\hline + \end{tabularx} + \end{center} + \item \textit{Pas de correction automatisé} + \end{enumerate} + \end{enumerate} +\end{solution} + +\end{document} + +%%% Local Variables: +%%% mode: latex +%%% TeX-master: "master" +%%% End: diff --git a/TST/DS/DS_21_04_07/TST1/03_210407_DS8.tex b/TST/DS/DS_21_04_07/TST1/03_210407_DS8.tex new file mode 100644 index 0000000..ca829e6 --- /dev/null +++ b/TST/DS/DS_21_04_07/TST1/03_210407_DS8.tex @@ -0,0 +1,262 @@ +\documentclass[a4paper,10pt]{article} +\usepackage{myXsim} + +% Title Page +\title{DS8 \hfill BALLOFFET Kenza} +\tribe{TST} +\date{\hfillÀ render pour le Mercredi 7 avril} + +\xsimsetup{ + solution/print = false +} + +\begin{document} +\maketitle + +\begin{exercise}[subtitle={Automatismes}] + \textit{Toutes les questions de cette exercice sont indépendantes et peuvent être répondus séparément} + \begin{enumerate} + \item De janvier à septembre, une quantité a augmenté de $12\,\%$. Faire un schéma pour représenter la situation puis calculer le taux d'évolution moyen mensuel. + \item Une quantité augmente de $12\,\%$ par ans. En 2020, elle est de 126\euro. Quelle était sa valeur en 2019? Faire un schéma pour représenter la situation. + \item Déterminer l'équation de la droite \\ + \begin{tikzpicture}[xscale=0.8, yscale=0.5] + \tkzInit[xmin=-5,xmax=5,xstep=1, + ymin=-5,ymax=5,ystep=1] + \tkzGrid + \tkzAxeXY + \tkzFct[domain=-5:5,color=red,very thick]% + {2.6666666666666665*\x -4}; + \end{tikzpicture} + \item Résoudre l'équation $8 \times 0.54^x = 18$ + \end{enumerate} +\end{exercise} + +\begin{solution} + \begin{enumerate} + \item On veut partager cette évolution en 8 évolutions. + \[ + \left(1 + \frac{12}{100}\right)^{\frac{1}{8}} = 1.0143 + \] + Donc le taux d'évolution moyen est + \[ + t_m = 1.0143 - 1 = 0.01429999999999998 + \] + \item Coefficient multiplicateur pour revenir en arrière + \[ + CM = (1 + \frac{12}{100})^{-1} = 0.8929 + \] + On en déduit la quantité en 2019 + \[ + 126 * 0.8929 = 112.50540000000001 + \] + \item L'équation de la droite est + \[ + y = 2.6666666666666665 x -4 + \] + \item Il faut penser à faire la division à par $8$ avant d'utiliser le log car sinon, on ne peut pas utiliser la formule $\log(a^n) = n\times \log(a)$. + + \[x = \frac{\log(2.25)}{\log(0.54)}\] + \end{enumerate} +\end{solution} + +\begin{exercise}[subtitle={Restaurant}] + Un \emph{food truck}, ouvert le midi et le soir, propose deux types de formules : + + \setlength\parindent{10mm} + \begin{itemize} + \item la formule \emph{Burger} ; + \item la formule \emph{Wok}. + \end{itemize} + \setlength\parindent{0mm} + + \medskip + + Le gérant a remarqué que 94\,\% de ses ventes ont lieu le midi. Le quart des ventes du midi correspondent à la formule \emph{Burger}, alors que 37\,\% des ventes du soir correspondent à la formule \emph{Wok}. + + Le gérant se constitue un fichier en notant, pour chaque vente, la formule choisie et le moment de cette vente (midi ou soir). + + On prélève une fiche de façon équiprobable. On définit les quatre évènements suivants: + + \begin{enumerate} + \item $M$ : \og la fiche correspond à une vente du midi\fg{} ; + \item $S$ : \og la fiche correspond à une vente du soir\fg {}; + \item $W$ : \og la fiche correspond à une formule \emph{Wok} \fg{} ; + \item $B$ : \og la fiche correspond à une formule \emph{Burger} \fg. + \end{enumerate} + \setlength\parindent{0mm} + + \medskip + + \begin{enumerate} + \item Recopier puis compléter l'arbre pondéré + + \begin{center} + \begin{tikzpicture}[sloped] + \node {.} + child {node {$M$} + child {node {$W$} + edge from parent + node[above] {...} + } + child {node {$B$} + edge from parent + node[above] {...} + } + edge from parent + node[above] {...} + } + child[missing] {} + child { node {$S$} + child {node {$W$} + edge from parent + node[above] {...} + } + child {node {$B$} + edge from parent + node[above] {...} + } + edge from parent + node[above] {...} + } ; + \end{tikzpicture} + \end{center} + + \item Calculer la probabilité de l'évènement $M \cap W$. Interpréter ce résultat dans le contexte de l'exercice. + \item Montrer que la probabilité que la fiche choisie corresponde à une formule \emph{Burger} est égale à $0.2728$. + \item On a prélevé une fiche correspondant à la formule \emph{Burger}. Quelle est la probabilité, arrondie au millième, que la vente ait eu lieu le soir? + \end{enumerate} +\end{exercise} + +\begin{solution} + \begin{enumerate} + \item + \begin{center} + \begin{tikzpicture}[sloped] + \node {.} + child {node {$M$} + child {node {$W$} + edge from parent + node[above] {$0.75$} + } + child {node {$B$} + edge from parent + node[above] {$0.25$} + } + edge from parent + node[above] {$0.94$} + } + child[missing] {} + child { node {$S$} + child {node {$W$} + edge from parent + node[above] {$0.37$} + } + child {node {$B$} + edge from parent + node[above] {$0.63$} + } + edge from parent + node[above] {$0.06$} + } ; + \end{tikzpicture} + \end{center} + \item On calcule la probabilité que la vente soit un wok et ait eu lieu à midi + \[ P(M\cap W) = P(M) \times P_M(W) = 0.94 \times 0.75 = 0.705 \] + \item Probabilité que la vente soit un burger. + \[ + P(B) = P(M\cap B) + P(S\cap B) = 0.94 \times 0.75 + 0.06 \times 0.37 = 0.2728 + \] + \item On cherche à calculer la quantité $P_B(S)$. Pour cela on utilise la formule de Bayes + \[ + P_B(S) = \frac{P(B\cap S)}{P(B)} = \frac{P_S(B) \times P(S)}{P(B)} = \frac{0.63\times 0.06}{0.2728} = 0.13856304985337245 \approx 0.139 + \] + \end{enumerate} +\end{solution} + +\begin{exercise}[subtitle={Continent plastique}] + \textit{Les quantités évoqués dans cette exercice sont générés au hasard et sont donc complètement farfelus.} + \medskip + Le \og continent de plastique\fg{} est la plus grande des plaques de déchets plastiques évoluant sur les océans. Elle occupe actuellement dans l'océan Pacifique une surface dont l'aire est évaluée à plus de $1,6$ million de km$^2$, entre Hawaï et la Californie. + + En 2017, des scientifiques ont estimé qu'il y avait $15$ millions de tonnes de déchets plastiques qui était déversé chaque année dans les océans et que cette quantité augmentait de $19\n\%$ par chaque année. + + On modélise l'évolution de la masse de ces déchets plastiques déversée chaque année, si rien n'est fait pour la réduire, par une suite géométrique $\left(u_n\right)$. L'arrondi au centième du terme $u_n$ représente la masse de ces déchets déversée chaque année, exprimée en million de tonnes, pour l'année $(2017 + n)$. + + \medskip + + \begin{enumerate} + \item Expliquer pourquoi la suite $u_n$ est géométrique? + \item Calculer $u_1$ et $u_2$. + \item Exprimer $u_n$ en fonction de $n$. + \item Au début de l'année 2017, il y avait $300$ millions de tonnes de déchets plastique. Calculer la quantité totale de déchets plastiques en 2030. + \item On souhaite déterminer en quelle année la masse totale de ces déchets plastiques aura pour la première fois augmenté de $50$\,\% par rapport à sa valeur de 2017. + \begin{enumerate} + \item Recopier et compléter l'algorithme ci-dessous pour que la variable $N$ contienne la réponse au problème posé. + + \begin{center} + \begin{tabularx}{0.4\linewidth}{|X|}\hline + $N = 2017$\\ + $U = 15$ \\ + $S = 300 + U$ \\ + while $S < 450$: \\ + \hspace{1cm} $N = \ldots$\\ + \hspace{1cm} $U = \ldots$\\ + \hspace{1cm} $S = \ldots$\\ + \hline + \end{tabularx} + \end{center} + \item Que contiennent les variables $S$, $U$ et $N$ après exécution de cet algorithme ? + + Interpréter les résultats dans le contexte de l'exercice. + \end{enumerate} + \end{enumerate} +\end{exercise} + +\begin{solution} + \begin{enumerate} + \item Une augmentation de $19\,\%$ revient à multiplier la quantité par $1.19$. La suite est donc bien géométrique. Son premier terme est $u_0 = 15$ et sa raison est $q = 1.19$ + \item + \[ + u_1 = u_0 * 1.19 = 17.849999999999998 + \] + \[ + u_2 = u_0 * 1.19^2 = 21.2415 + \] + \item + \[ + u_n = u_0 \times q^n = 15 \times 1.19^n + \] + \item On calcule la quantité totale déversée entre 2017 et 2030. + \[ + \sum_{n = 0}^{13} u_n = u_0 \times \frac{1-q^{13}}{1-q} = 15 \times \frac{1 - 1.19^{13}}{1 - 1.19} = 678.67 + \] + On en déduit la quantité totale de déchets en 2030 + \[ + 300 + 678.67 = 978.67 + \] + \item + \begin{enumerate} + \item ~ + \begin{center} + \begin{tabularx}{0.4\linewidth}{|X|}\hline + $N \gets 2017$\\ + $U \gets 15$ \\ + $S \gets 300 + U$ \\ + Tant que $S < 450$ \\ + \hspace{1cm} $N \gets N + 1$\\ + \hspace{1cm} $U \gets U * 1.19$\\ + \hspace{1cm} $S \gets S + u$\\ + Fin Tant que\\\hline + \end{tabularx} + \end{center} + \item \textit{Pas de correction automatisé} + \end{enumerate} + \end{enumerate} +\end{solution} + +\end{document} + +%%% Local Variables: +%%% mode: latex +%%% TeX-master: "master" +%%% End: diff --git a/TST/DS/DS_21_04_07/TST1/04_210407_DS8.tex b/TST/DS/DS_21_04_07/TST1/04_210407_DS8.tex new file mode 100644 index 0000000..93a6ec0 --- /dev/null +++ b/TST/DS/DS_21_04_07/TST1/04_210407_DS8.tex @@ -0,0 +1,262 @@ +\documentclass[a4paper,10pt]{article} +\usepackage{myXsim} + +% Title Page +\title{DS8 \hfill BENHATTAL Chakir} +\tribe{TST} +\date{\hfillÀ render pour le Mercredi 7 avril} + +\xsimsetup{ + solution/print = false +} + +\begin{document} +\maketitle + +\begin{exercise}[subtitle={Automatismes}] + \textit{Toutes les questions de cette exercice sont indépendantes et peuvent être répondus séparément} + \begin{enumerate} + \item De janvier à septembre, une quantité a augmenté de $28\,\%$. Faire un schéma pour représenter la situation puis calculer le taux d'évolution moyen mensuel. + \item Une quantité augmente de $28\,\%$ par ans. En 2020, elle est de 111\euro. Quelle était sa valeur en 2019? Faire un schéma pour représenter la situation. + \item Déterminer l'équation de la droite \\ + \begin{tikzpicture}[xscale=0.8, yscale=0.5] + \tkzInit[xmin=-5,xmax=5,xstep=1, + ymin=-5,ymax=5,ystep=1] + \tkzGrid + \tkzAxeXY + \tkzFct[domain=-5:5,color=red,very thick]% + {1.0*\x -1}; + \end{tikzpicture} + \item Résoudre l'équation $4 \times 0.77^x = 25$ + \end{enumerate} +\end{exercise} + +\begin{solution} + \begin{enumerate} + \item On veut partager cette évolution en 8 évolutions. + \[ + \left(1 + \frac{28}{100}\right)^{\frac{1}{8}} = 1.0313 + \] + Donc le taux d'évolution moyen est + \[ + t_m = 1.0313 - 1 = 0.031300000000000106 + \] + \item Coefficient multiplicateur pour revenir en arrière + \[ + CM = (1 + \frac{28}{100})^{-1} = 0.7812 + \] + On en déduit la quantité en 2019 + \[ + 111 * 0.7812 = 86.7132 + \] + \item L'équation de la droite est + \[ + y = 1.0 x -1 + \] + \item Il faut penser à faire la division à par $4$ avant d'utiliser le log car sinon, on ne peut pas utiliser la formule $\log(a^n) = n\times \log(a)$. + + \[x = \frac{\log(6.25)}{\log(0.77)}\] + \end{enumerate} +\end{solution} + +\begin{exercise}[subtitle={Restaurant}] + Un \emph{food truck}, ouvert le midi et le soir, propose deux types de formules : + + \setlength\parindent{10mm} + \begin{itemize} + \item la formule \emph{Burger} ; + \item la formule \emph{Wok}. + \end{itemize} + \setlength\parindent{0mm} + + \medskip + + Le gérant a remarqué que 89\,\% de ses ventes ont lieu le midi. Le quart des ventes du midi correspondent à la formule \emph{Burger}, alors que 43\,\% des ventes du soir correspondent à la formule \emph{Wok}. + + Le gérant se constitue un fichier en notant, pour chaque vente, la formule choisie et le moment de cette vente (midi ou soir). + + On prélève une fiche de façon équiprobable. On définit les quatre évènements suivants: + + \begin{enumerate} + \item $M$ : \og la fiche correspond à une vente du midi\fg{} ; + \item $S$ : \og la fiche correspond à une vente du soir\fg {}; + \item $W$ : \og la fiche correspond à une formule \emph{Wok} \fg{} ; + \item $B$ : \og la fiche correspond à une formule \emph{Burger} \fg. + \end{enumerate} + \setlength\parindent{0mm} + + \medskip + + \begin{enumerate} + \item Recopier puis compléter l'arbre pondéré + + \begin{center} + \begin{tikzpicture}[sloped] + \node {.} + child {node {$M$} + child {node {$W$} + edge from parent + node[above] {...} + } + child {node {$B$} + edge from parent + node[above] {...} + } + edge from parent + node[above] {...} + } + child[missing] {} + child { node {$S$} + child {node {$W$} + edge from parent + node[above] {...} + } + child {node {$B$} + edge from parent + node[above] {...} + } + edge from parent + node[above] {...} + } ; + \end{tikzpicture} + \end{center} + + \item Calculer la probabilité de l'évènement $M \cap W$. Interpréter ce résultat dans le contexte de l'exercice. + \item Montrer que la probabilité que la fiche choisie corresponde à une formule \emph{Burger} est égale à $0.2852$. + \item On a prélevé une fiche correspondant à la formule \emph{Burger}. Quelle est la probabilité, arrondie au millième, que la vente ait eu lieu le soir? + \end{enumerate} +\end{exercise} + +\begin{solution} + \begin{enumerate} + \item + \begin{center} + \begin{tikzpicture}[sloped] + \node {.} + child {node {$M$} + child {node {$W$} + edge from parent + node[above] {$0.75$} + } + child {node {$B$} + edge from parent + node[above] {$0.25$} + } + edge from parent + node[above] {$0.89$} + } + child[missing] {} + child { node {$S$} + child {node {$W$} + edge from parent + node[above] {$0.43$} + } + child {node {$B$} + edge from parent + node[above] {$0.57$} + } + edge from parent + node[above] {$0.11$} + } ; + \end{tikzpicture} + \end{center} + \item On calcule la probabilité que la vente soit un wok et ait eu lieu à midi + \[ P(M\cap W) = P(M) \times P_M(W) = 0.89 \times 0.75 = 0.6675 \] + \item Probabilité que la vente soit un burger. + \[ + P(B) = P(M\cap B) + P(S\cap B) = 0.89 \times 0.75 + 0.11 \times 0.43 = 0.2852 + \] + \item On cherche à calculer la quantité $P_B(S)$. Pour cela on utilise la formule de Bayes + \[ + P_B(S) = \frac{P(B\cap S)}{P(B)} = \frac{P_S(B) \times P(S)}{P(B)} = \frac{0.57\times 0.11}{0.2852} = 0.2198457223001402 \approx 0.22 + \] + \end{enumerate} +\end{solution} + +\begin{exercise}[subtitle={Continent plastique}] + \textit{Les quantités évoqués dans cette exercice sont générés au hasard et sont donc complètement farfelus.} + \medskip + Le \og continent de plastique\fg{} est la plus grande des plaques de déchets plastiques évoluant sur les océans. Elle occupe actuellement dans l'océan Pacifique une surface dont l'aire est évaluée à plus de $1,6$ million de km$^2$, entre Hawaï et la Californie. + + En 2017, des scientifiques ont estimé qu'il y avait $6$ millions de tonnes de déchets plastiques qui était déversé chaque année dans les océans et que cette quantité augmentait de $13\n\%$ par chaque année. + + On modélise l'évolution de la masse de ces déchets plastiques déversée chaque année, si rien n'est fait pour la réduire, par une suite géométrique $\left(u_n\right)$. L'arrondi au centième du terme $u_n$ représente la masse de ces déchets déversée chaque année, exprimée en million de tonnes, pour l'année $(2017 + n)$. + + \medskip + + \begin{enumerate} + \item Expliquer pourquoi la suite $u_n$ est géométrique? + \item Calculer $u_1$ et $u_2$. + \item Exprimer $u_n$ en fonction de $n$. + \item Au début de l'année 2017, il y avait $300$ millions de tonnes de déchets plastique. Calculer la quantité totale de déchets plastiques en 2030. + \item On souhaite déterminer en quelle année la masse totale de ces déchets plastiques aura pour la première fois augmenté de $50$\,\% par rapport à sa valeur de 2017. + \begin{enumerate} + \item Recopier et compléter l'algorithme ci-dessous pour que la variable $N$ contienne la réponse au problème posé. + + \begin{center} + \begin{tabularx}{0.4\linewidth}{|X|}\hline + $N = 2017$\\ + $U = 6$ \\ + $S = 300 + U$ \\ + while $S < 450$: \\ + \hspace{1cm} $N = \ldots$\\ + \hspace{1cm} $U = \ldots$\\ + \hspace{1cm} $S = \ldots$\\ + \hline + \end{tabularx} + \end{center} + \item Que contiennent les variables $S$, $U$ et $N$ après exécution de cet algorithme ? + + Interpréter les résultats dans le contexte de l'exercice. + \end{enumerate} + \end{enumerate} +\end{exercise} + +\begin{solution} + \begin{enumerate} + \item Une augmentation de $13\,\%$ revient à multiplier la quantité par $1.13$. La suite est donc bien géométrique. Son premier terme est $u_0 = 6$ et sa raison est $q = 1.13$ + \item + \[ + u_1 = u_0 * 1.13 = 6.779999999999999 + \] + \[ + u_2 = u_0 * 1.13^2 = 7.6614 + \] + \item + \[ + u_n = u_0 \times q^n = 6 \times 1.13^n + \] + \item On calcule la quantité totale déversée entre 2017 et 2030. + \[ + \sum_{n = 0}^{13} u_n = u_0 \times \frac{1-q^{13}}{1-q} = 6 \times \frac{1 - 1.13^{13}}{1 - 1.13} = 179.91 + \] + On en déduit la quantité totale de déchets en 2030 + \[ + 300 + 179.91 = 479.90999999999997 + \] + \item + \begin{enumerate} + \item ~ + \begin{center} + \begin{tabularx}{0.4\linewidth}{|X|}\hline + $N \gets 2017$\\ + $U \gets 6$ \\ + $S \gets 300 + U$ \\ + Tant que $S < 450$ \\ + \hspace{1cm} $N \gets N + 1$\\ + \hspace{1cm} $U \gets U * 1.13$\\ + \hspace{1cm} $S \gets S + u$\\ + Fin Tant que\\\hline + \end{tabularx} + \end{center} + \item \textit{Pas de correction automatisé} + \end{enumerate} + \end{enumerate} +\end{solution} + +\end{document} + +%%% Local Variables: +%%% mode: latex +%%% TeX-master: "master" +%%% End: diff --git a/TST/DS/DS_21_04_07/TST1/05_210407_DS8.tex b/TST/DS/DS_21_04_07/TST1/05_210407_DS8.tex new file mode 100644 index 0000000..56beec7 --- /dev/null +++ b/TST/DS/DS_21_04_07/TST1/05_210407_DS8.tex @@ -0,0 +1,262 @@ +\documentclass[a4paper,10pt]{article} +\usepackage{myXsim} + +% Title Page +\title{DS8 \hfill CLAIN Avinash} +\tribe{TST} +\date{\hfillÀ render pour le Mercredi 7 avril} + +\xsimsetup{ + solution/print = false +} + +\begin{document} +\maketitle + +\begin{exercise}[subtitle={Automatismes}] + \textit{Toutes les questions de cette exercice sont indépendantes et peuvent être répondus séparément} + \begin{enumerate} + \item De janvier à septembre, une quantité a augmenté de $15\,\%$. Faire un schéma pour représenter la situation puis calculer le taux d'évolution moyen mensuel. + \item Une quantité augmente de $15\,\%$ par ans. En 2020, elle est de 112\euro. Quelle était sa valeur en 2019? Faire un schéma pour représenter la situation. + \item Déterminer l'équation de la droite \\ + \begin{tikzpicture}[xscale=0.8, yscale=0.5] + \tkzInit[xmin=-5,xmax=5,xstep=1, + ymin=-5,ymax=5,ystep=1] + \tkzGrid + \tkzAxeXY + \tkzFct[domain=-5:5,color=red,very thick]% + {4.0*\x -4}; + \end{tikzpicture} + \item Résoudre l'équation $6 \times 0.77^x = 23$ + \end{enumerate} +\end{exercise} + +\begin{solution} + \begin{enumerate} + \item On veut partager cette évolution en 8 évolutions. + \[ + \left(1 + \frac{15}{100}\right)^{\frac{1}{8}} = 1.0176 + \] + Donc le taux d'évolution moyen est + \[ + t_m = 1.0176 - 1 = 0.01760000000000006 + \] + \item Coefficient multiplicateur pour revenir en arrière + \[ + CM = (1 + \frac{15}{100})^{-1} = 0.8696 + \] + On en déduit la quantité en 2019 + \[ + 112 * 0.8696 = 97.3952 + \] + \item L'équation de la droite est + \[ + y = 4.0 x -4 + \] + \item Il faut penser à faire la division à par $6$ avant d'utiliser le log car sinon, on ne peut pas utiliser la formule $\log(a^n) = n\times \log(a)$. + + \[x = \frac{\log(3.83)}{\log(0.77)}\] + \end{enumerate} +\end{solution} + +\begin{exercise}[subtitle={Restaurant}] + Un \emph{food truck}, ouvert le midi et le soir, propose deux types de formules : + + \setlength\parindent{10mm} + \begin{itemize} + \item la formule \emph{Burger} ; + \item la formule \emph{Wok}. + \end{itemize} + \setlength\parindent{0mm} + + \medskip + + Le gérant a remarqué que 92\,\% de ses ventes ont lieu le midi. Le quart des ventes du midi correspondent à la formule \emph{Burger}, alors que 54\,\% des ventes du soir correspondent à la formule \emph{Wok}. + + Le gérant se constitue un fichier en notant, pour chaque vente, la formule choisie et le moment de cette vente (midi ou soir). + + On prélève une fiche de façon équiprobable. On définit les quatre évènements suivants: + + \begin{enumerate} + \item $M$ : \og la fiche correspond à une vente du midi\fg{} ; + \item $S$ : \og la fiche correspond à une vente du soir\fg {}; + \item $W$ : \og la fiche correspond à une formule \emph{Wok} \fg{} ; + \item $B$ : \og la fiche correspond à une formule \emph{Burger} \fg. + \end{enumerate} + \setlength\parindent{0mm} + + \medskip + + \begin{enumerate} + \item Recopier puis compléter l'arbre pondéré + + \begin{center} + \begin{tikzpicture}[sloped] + \node {.} + child {node {$M$} + child {node {$W$} + edge from parent + node[above] {...} + } + child {node {$B$} + edge from parent + node[above] {...} + } + edge from parent + node[above] {...} + } + child[missing] {} + child { node {$S$} + child {node {$W$} + edge from parent + node[above] {...} + } + child {node {$B$} + edge from parent + node[above] {...} + } + edge from parent + node[above] {...} + } ; + \end{tikzpicture} + \end{center} + + \item Calculer la probabilité de l'évènement $M \cap W$. Interpréter ce résultat dans le contexte de l'exercice. + \item Montrer que la probabilité que la fiche choisie corresponde à une formule \emph{Burger} est égale à $0.2668$. + \item On a prélevé une fiche correspondant à la formule \emph{Burger}. Quelle est la probabilité, arrondie au millième, que la vente ait eu lieu le soir? + \end{enumerate} +\end{exercise} + +\begin{solution} + \begin{enumerate} + \item + \begin{center} + \begin{tikzpicture}[sloped] + \node {.} + child {node {$M$} + child {node {$W$} + edge from parent + node[above] {$0.75$} + } + child {node {$B$} + edge from parent + node[above] {$0.25$} + } + edge from parent + node[above] {$0.92$} + } + child[missing] {} + child { node {$S$} + child {node {$W$} + edge from parent + node[above] {$0.54$} + } + child {node {$B$} + edge from parent + node[above] {$0.46$} + } + edge from parent + node[above] {$0.08$} + } ; + \end{tikzpicture} + \end{center} + \item On calcule la probabilité que la vente soit un wok et ait eu lieu à midi + \[ P(M\cap W) = P(M) \times P_M(W) = 0.92 \times 0.75 = 0.69 \] + \item Probabilité que la vente soit un burger. + \[ + P(B) = P(M\cap B) + P(S\cap B) = 0.92 \times 0.75 + 0.08 \times 0.54 = 0.2668 + \] + \item On cherche à calculer la quantité $P_B(S)$. Pour cela on utilise la formule de Bayes + \[ + P_B(S) = \frac{P(B\cap S)}{P(B)} = \frac{P_S(B) \times P(S)}{P(B)} = \frac{0.46\times 0.08}{0.2668} = 0.13793103448275862 \approx 0.138 + \] + \end{enumerate} +\end{solution} + +\begin{exercise}[subtitle={Continent plastique}] + \textit{Les quantités évoqués dans cette exercice sont générés au hasard et sont donc complètement farfelus.} + \medskip + Le \og continent de plastique\fg{} est la plus grande des plaques de déchets plastiques évoluant sur les océans. Elle occupe actuellement dans l'océan Pacifique une surface dont l'aire est évaluée à plus de $1,6$ million de km$^2$, entre Hawaï et la Californie. + + En 2017, des scientifiques ont estimé qu'il y avait $18$ millions de tonnes de déchets plastiques qui était déversé chaque année dans les océans et que cette quantité augmentait de $21\n\%$ par chaque année. + + On modélise l'évolution de la masse de ces déchets plastiques déversée chaque année, si rien n'est fait pour la réduire, par une suite géométrique $\left(u_n\right)$. L'arrondi au centième du terme $u_n$ représente la masse de ces déchets déversée chaque année, exprimée en million de tonnes, pour l'année $(2017 + n)$. + + \medskip + + \begin{enumerate} + \item Expliquer pourquoi la suite $u_n$ est géométrique? + \item Calculer $u_1$ et $u_2$. + \item Exprimer $u_n$ en fonction de $n$. + \item Au début de l'année 2017, il y avait $300$ millions de tonnes de déchets plastique. Calculer la quantité totale de déchets plastiques en 2030. + \item On souhaite déterminer en quelle année la masse totale de ces déchets plastiques aura pour la première fois augmenté de $50$\,\% par rapport à sa valeur de 2017. + \begin{enumerate} + \item Recopier et compléter l'algorithme ci-dessous pour que la variable $N$ contienne la réponse au problème posé. + + \begin{center} + \begin{tabularx}{0.4\linewidth}{|X|}\hline + $N = 2017$\\ + $U = 18$ \\ + $S = 300 + U$ \\ + while $S < 450$: \\ + \hspace{1cm} $N = \ldots$\\ + \hspace{1cm} $U = \ldots$\\ + \hspace{1cm} $S = \ldots$\\ + \hline + \end{tabularx} + \end{center} + \item Que contiennent les variables $S$, $U$ et $N$ après exécution de cet algorithme ? + + Interpréter les résultats dans le contexte de l'exercice. + \end{enumerate} + \end{enumerate} +\end{exercise} + +\begin{solution} + \begin{enumerate} + \item Une augmentation de $21\,\%$ revient à multiplier la quantité par $1.21$. La suite est donc bien géométrique. Son premier terme est $u_0 = 18$ et sa raison est $q = 1.21$ + \item + \[ + u_1 = u_0 * 1.21 = 21.78 + \] + \[ + u_2 = u_0 * 1.21^2 = 26.3538 + \] + \item + \[ + u_n = u_0 \times q^n = 18 \times 1.21^n + \] + \item On calcule la quantité totale déversée entre 2017 et 2030. + \[ + \sum_{n = 0}^{13} u_n = u_0 \times \frac{1-q^{13}}{1-q} = 18 \times \frac{1 - 1.21^{13}}{1 - 1.21} = 935.84 + \] + On en déduit la quantité totale de déchets en 2030 + \[ + 300 + 935.84 = 1235.8400000000001 + \] + \item + \begin{enumerate} + \item ~ + \begin{center} + \begin{tabularx}{0.4\linewidth}{|X|}\hline + $N \gets 2017$\\ + $U \gets 18$ \\ + $S \gets 300 + U$ \\ + Tant que $S < 450$ \\ + \hspace{1cm} $N \gets N + 1$\\ + \hspace{1cm} $U \gets U * 1.21$\\ + \hspace{1cm} $S \gets S + u$\\ + Fin Tant que\\\hline + \end{tabularx} + \end{center} + \item \textit{Pas de correction automatisé} + \end{enumerate} + \end{enumerate} +\end{solution} + +\end{document} + +%%% Local Variables: +%%% mode: latex +%%% TeX-master: "master" +%%% End: diff --git a/TST/DS/DS_21_04_07/TST1/06_210407_DS8.tex b/TST/DS/DS_21_04_07/TST1/06_210407_DS8.tex new file mode 100644 index 0000000..d2574d8 --- /dev/null +++ b/TST/DS/DS_21_04_07/TST1/06_210407_DS8.tex @@ -0,0 +1,262 @@ +\documentclass[a4paper,10pt]{article} +\usepackage{myXsim} + +% Title Page +\title{DS8 \hfill COLASSI Alexis} +\tribe{TST} +\date{\hfillÀ render pour le Mercredi 7 avril} + +\xsimsetup{ + solution/print = false +} + +\begin{document} +\maketitle + +\begin{exercise}[subtitle={Automatismes}] + \textit{Toutes les questions de cette exercice sont indépendantes et peuvent être répondus séparément} + \begin{enumerate} + \item De janvier à septembre, une quantité a augmenté de $27\,\%$. Faire un schéma pour représenter la situation puis calculer le taux d'évolution moyen mensuel. + \item Une quantité augmente de $27\,\%$ par ans. En 2020, elle est de 116\euro. Quelle était sa valeur en 2019? Faire un schéma pour représenter la situation. + \item Déterminer l'équation de la droite \\ + \begin{tikzpicture}[xscale=0.8, yscale=0.5] + \tkzInit[xmin=-5,xmax=5,xstep=1, + ymin=-5,ymax=5,ystep=1] + \tkzGrid + \tkzAxeXY + \tkzFct[domain=-5:5,color=red,very thick]% + {2.0*\x -2}; + \end{tikzpicture} + \item Résoudre l'équation $7 \times 0.23^x = 16$ + \end{enumerate} +\end{exercise} + +\begin{solution} + \begin{enumerate} + \item On veut partager cette évolution en 8 évolutions. + \[ + \left(1 + \frac{27}{100}\right)^{\frac{1}{8}} = 1.0303 + \] + Donc le taux d'évolution moyen est + \[ + t_m = 1.0303 - 1 = 0.030299999999999994 + \] + \item Coefficient multiplicateur pour revenir en arrière + \[ + CM = (1 + \frac{27}{100})^{-1} = 0.7874 + \] + On en déduit la quantité en 2019 + \[ + 116 * 0.7874 = 91.3384 + \] + \item L'équation de la droite est + \[ + y = 2.0 x -2 + \] + \item Il faut penser à faire la division à par $7$ avant d'utiliser le log car sinon, on ne peut pas utiliser la formule $\log(a^n) = n\times \log(a)$. + + \[x = \frac{\log(2.29)}{\log(0.23)}\] + \end{enumerate} +\end{solution} + +\begin{exercise}[subtitle={Restaurant}] + Un \emph{food truck}, ouvert le midi et le soir, propose deux types de formules : + + \setlength\parindent{10mm} + \begin{itemize} + \item la formule \emph{Burger} ; + \item la formule \emph{Wok}. + \end{itemize} + \setlength\parindent{0mm} + + \medskip + + Le gérant a remarqué que 53\,\% de ses ventes ont lieu le midi. Le quart des ventes du midi correspondent à la formule \emph{Burger}, alors que 26\,\% des ventes du soir correspondent à la formule \emph{Wok}. + + Le gérant se constitue un fichier en notant, pour chaque vente, la formule choisie et le moment de cette vente (midi ou soir). + + On prélève une fiche de façon équiprobable. On définit les quatre évènements suivants: + + \begin{enumerate} + \item $M$ : \og la fiche correspond à une vente du midi\fg{} ; + \item $S$ : \og la fiche correspond à une vente du soir\fg {}; + \item $W$ : \og la fiche correspond à une formule \emph{Wok} \fg{} ; + \item $B$ : \og la fiche correspond à une formule \emph{Burger} \fg. + \end{enumerate} + \setlength\parindent{0mm} + + \medskip + + \begin{enumerate} + \item Recopier puis compléter l'arbre pondéré + + \begin{center} + \begin{tikzpicture}[sloped] + \node {.} + child {node {$M$} + child {node {$W$} + edge from parent + node[above] {...} + } + child {node {$B$} + edge from parent + node[above] {...} + } + edge from parent + node[above] {...} + } + child[missing] {} + child { node {$S$} + child {node {$W$} + edge from parent + node[above] {...} + } + child {node {$B$} + edge from parent + node[above] {...} + } + edge from parent + node[above] {...} + } ; + \end{tikzpicture} + \end{center} + + \item Calculer la probabilité de l'évènement $M \cap W$. Interpréter ce résultat dans le contexte de l'exercice. + \item Montrer que la probabilité que la fiche choisie corresponde à une formule \emph{Burger} est égale à $0.4803$. + \item On a prélevé une fiche correspondant à la formule \emph{Burger}. Quelle est la probabilité, arrondie au millième, que la vente ait eu lieu le soir? + \end{enumerate} +\end{exercise} + +\begin{solution} + \begin{enumerate} + \item + \begin{center} + \begin{tikzpicture}[sloped] + \node {.} + child {node {$M$} + child {node {$W$} + edge from parent + node[above] {$0.75$} + } + child {node {$B$} + edge from parent + node[above] {$0.25$} + } + edge from parent + node[above] {$0.53$} + } + child[missing] {} + child { node {$S$} + child {node {$W$} + edge from parent + node[above] {$0.26$} + } + child {node {$B$} + edge from parent + node[above] {$0.74$} + } + edge from parent + node[above] {$0.47$} + } ; + \end{tikzpicture} + \end{center} + \item On calcule la probabilité que la vente soit un wok et ait eu lieu à midi + \[ P(M\cap W) = P(M) \times P_M(W) = 0.53 \times 0.75 = 0.3975 \] + \item Probabilité que la vente soit un burger. + \[ + P(B) = P(M\cap B) + P(S\cap B) = 0.53 \times 0.75 + 0.47 \times 0.26 = 0.4803 + \] + \item On cherche à calculer la quantité $P_B(S)$. Pour cela on utilise la formule de Bayes + \[ + P_B(S) = \frac{P(B\cap S)}{P(B)} = \frac{P_S(B) \times P(S)}{P(B)} = \frac{0.74\times 0.47}{0.4803} = 0.7241307516135749 \approx 0.724 + \] + \end{enumerate} +\end{solution} + +\begin{exercise}[subtitle={Continent plastique}] + \textit{Les quantités évoqués dans cette exercice sont générés au hasard et sont donc complètement farfelus.} + \medskip + Le \og continent de plastique\fg{} est la plus grande des plaques de déchets plastiques évoluant sur les océans. Elle occupe actuellement dans l'océan Pacifique une surface dont l'aire est évaluée à plus de $1,6$ million de km$^2$, entre Hawaï et la Californie. + + En 2017, des scientifiques ont estimé qu'il y avait $3$ millions de tonnes de déchets plastiques qui était déversé chaque année dans les océans et que cette quantité augmentait de $17\n\%$ par chaque année. + + On modélise l'évolution de la masse de ces déchets plastiques déversée chaque année, si rien n'est fait pour la réduire, par une suite géométrique $\left(u_n\right)$. L'arrondi au centième du terme $u_n$ représente la masse de ces déchets déversée chaque année, exprimée en million de tonnes, pour l'année $(2017 + n)$. + + \medskip + + \begin{enumerate} + \item Expliquer pourquoi la suite $u_n$ est géométrique? + \item Calculer $u_1$ et $u_2$. + \item Exprimer $u_n$ en fonction de $n$. + \item Au début de l'année 2017, il y avait $300$ millions de tonnes de déchets plastique. Calculer la quantité totale de déchets plastiques en 2030. + \item On souhaite déterminer en quelle année la masse totale de ces déchets plastiques aura pour la première fois augmenté de $50$\,\% par rapport à sa valeur de 2017. + \begin{enumerate} + \item Recopier et compléter l'algorithme ci-dessous pour que la variable $N$ contienne la réponse au problème posé. + + \begin{center} + \begin{tabularx}{0.4\linewidth}{|X|}\hline + $N = 2017$\\ + $U = 3$ \\ + $S = 300 + U$ \\ + while $S < 450$: \\ + \hspace{1cm} $N = \ldots$\\ + \hspace{1cm} $U = \ldots$\\ + \hspace{1cm} $S = \ldots$\\ + \hline + \end{tabularx} + \end{center} + \item Que contiennent les variables $S$, $U$ et $N$ après exécution de cet algorithme ? + + Interpréter les résultats dans le contexte de l'exercice. + \end{enumerate} + \end{enumerate} +\end{exercise} + +\begin{solution} + \begin{enumerate} + \item Une augmentation de $17\,\%$ revient à multiplier la quantité par $1.17$. La suite est donc bien géométrique. Son premier terme est $u_0 = 3$ et sa raison est $q = 1.17$ + \item + \[ + u_1 = u_0 * 1.17 = 3.51 + \] + \[ + u_2 = u_0 * 1.17^2 = 4.1067 + \] + \item + \[ + u_n = u_0 \times q^n = 3 \times 1.17^n + \] + \item On calcule la quantité totale déversée entre 2017 et 2030. + \[ + \sum_{n = 0}^{13} u_n = u_0 \times \frac{1-q^{13}}{1-q} = 3 \times \frac{1 - 1.17^{13}}{1 - 1.17} = 118.21 + \] + On en déduit la quantité totale de déchets en 2030 + \[ + 300 + 118.21 = 418.21 + \] + \item + \begin{enumerate} + \item ~ + \begin{center} + \begin{tabularx}{0.4\linewidth}{|X|}\hline + $N \gets 2017$\\ + $U \gets 3$ \\ + $S \gets 300 + U$ \\ + Tant que $S < 450$ \\ + \hspace{1cm} $N \gets N + 1$\\ + \hspace{1cm} $U \gets U * 1.17$\\ + \hspace{1cm} $S \gets S + u$\\ + Fin Tant que\\\hline + \end{tabularx} + \end{center} + \item \textit{Pas de correction automatisé} + \end{enumerate} + \end{enumerate} +\end{solution} + +\end{document} + +%%% Local Variables: +%%% mode: latex +%%% TeX-master: "master" +%%% End: diff --git a/TST/DS/DS_21_04_07/TST1/07_210407_DS8.tex b/TST/DS/DS_21_04_07/TST1/07_210407_DS8.tex new file mode 100644 index 0000000..3eadc07 --- /dev/null +++ b/TST/DS/DS_21_04_07/TST1/07_210407_DS8.tex @@ -0,0 +1,262 @@ +\documentclass[a4paper,10pt]{article} +\usepackage{myXsim} + +% Title Page +\title{DS8 \hfill COUBAT Alexis} +\tribe{TST} +\date{\hfillÀ render pour le Mercredi 7 avril} + +\xsimsetup{ + solution/print = false +} + +\begin{document} +\maketitle + +\begin{exercise}[subtitle={Automatismes}] + \textit{Toutes les questions de cette exercice sont indépendantes et peuvent être répondus séparément} + \begin{enumerate} + \item De janvier à septembre, une quantité a augmenté de $16\,\%$. Faire un schéma pour représenter la situation puis calculer le taux d'évolution moyen mensuel. + \item Une quantité augmente de $16\,\%$ par ans. En 2020, elle est de 120\euro. Quelle était sa valeur en 2019? Faire un schéma pour représenter la situation. + \item Déterminer l'équation de la droite \\ + \begin{tikzpicture}[xscale=0.8, yscale=0.5] + \tkzInit[xmin=-5,xmax=5,xstep=1, + ymin=-5,ymax=5,ystep=1] + \tkzGrid + \tkzAxeXY + \tkzFct[domain=-5:5,color=red,very thick]% + {4.0*\x -4}; + \end{tikzpicture} + \item Résoudre l'équation $10 \times 0.06^x = 34$ + \end{enumerate} +\end{exercise} + +\begin{solution} + \begin{enumerate} + \item On veut partager cette évolution en 8 évolutions. + \[ + \left(1 + \frac{16}{100}\right)^{\frac{1}{8}} = 1.0187 + \] + Donc le taux d'évolution moyen est + \[ + t_m = 1.0187 - 1 = 0.01869999999999994 + \] + \item Coefficient multiplicateur pour revenir en arrière + \[ + CM = (1 + \frac{16}{100})^{-1} = 0.8621 + \] + On en déduit la quantité en 2019 + \[ + 120 * 0.8621 = 103.452 + \] + \item L'équation de la droite est + \[ + y = 4.0 x -4 + \] + \item Il faut penser à faire la division à par $10$ avant d'utiliser le log car sinon, on ne peut pas utiliser la formule $\log(a^n) = n\times \log(a)$. + + \[x = \frac{\log(3.4)}{\log(0.06)}\] + \end{enumerate} +\end{solution} + +\begin{exercise}[subtitle={Restaurant}] + Un \emph{food truck}, ouvert le midi et le soir, propose deux types de formules : + + \setlength\parindent{10mm} + \begin{itemize} + \item la formule \emph{Burger} ; + \item la formule \emph{Wok}. + \end{itemize} + \setlength\parindent{0mm} + + \medskip + + Le gérant a remarqué que 1\,\% de ses ventes ont lieu le midi. Le quart des ventes du midi correspondent à la formule \emph{Burger}, alors que 92\,\% des ventes du soir correspondent à la formule \emph{Wok}. + + Le gérant se constitue un fichier en notant, pour chaque vente, la formule choisie et le moment de cette vente (midi ou soir). + + On prélève une fiche de façon équiprobable. On définit les quatre évènements suivants: + + \begin{enumerate} + \item $M$ : \og la fiche correspond à une vente du midi\fg{} ; + \item $S$ : \og la fiche correspond à une vente du soir\fg {}; + \item $W$ : \og la fiche correspond à une formule \emph{Wok} \fg{} ; + \item $B$ : \og la fiche correspond à une formule \emph{Burger} \fg. + \end{enumerate} + \setlength\parindent{0mm} + + \medskip + + \begin{enumerate} + \item Recopier puis compléter l'arbre pondéré + + \begin{center} + \begin{tikzpicture}[sloped] + \node {.} + child {node {$M$} + child {node {$W$} + edge from parent + node[above] {...} + } + child {node {$B$} + edge from parent + node[above] {...} + } + edge from parent + node[above] {...} + } + child[missing] {} + child { node {$S$} + child {node {$W$} + edge from parent + node[above] {...} + } + child {node {$B$} + edge from parent + node[above] {...} + } + edge from parent + node[above] {...} + } ; + \end{tikzpicture} + \end{center} + + \item Calculer la probabilité de l'évènement $M \cap W$. Interpréter ce résultat dans le contexte de l'exercice. + \item Montrer que la probabilité que la fiche choisie corresponde à une formule \emph{Burger} est égale à $0.0817$. + \item On a prélevé une fiche correspondant à la formule \emph{Burger}. Quelle est la probabilité, arrondie au millième, que la vente ait eu lieu le soir? + \end{enumerate} +\end{exercise} + +\begin{solution} + \begin{enumerate} + \item + \begin{center} + \begin{tikzpicture}[sloped] + \node {.} + child {node {$M$} + child {node {$W$} + edge from parent + node[above] {$0.75$} + } + child {node {$B$} + edge from parent + node[above] {$0.25$} + } + edge from parent + node[above] {$0.01$} + } + child[missing] {} + child { node {$S$} + child {node {$W$} + edge from parent + node[above] {$0.92$} + } + child {node {$B$} + edge from parent + node[above] {$0.08$} + } + edge from parent + node[above] {$0.99$} + } ; + \end{tikzpicture} + \end{center} + \item On calcule la probabilité que la vente soit un wok et ait eu lieu à midi + \[ P(M\cap W) = P(M) \times P_M(W) = 0.01 \times 0.75 = 0.0075 \] + \item Probabilité que la vente soit un burger. + \[ + P(B) = P(M\cap B) + P(S\cap B) = 0.01 \times 0.75 + 0.99 \times 0.92 = 0.0817 + \] + \item On cherche à calculer la quantité $P_B(S)$. Pour cela on utilise la formule de Bayes + \[ + P_B(S) = \frac{P(B\cap S)}{P(B)} = \frac{P_S(B) \times P(S)}{P(B)} = \frac{0.08\times 0.99}{0.0817} = 0.9694002447980418 \approx 0.969 + \] + \end{enumerate} +\end{solution} + +\begin{exercise}[subtitle={Continent plastique}] + \textit{Les quantités évoqués dans cette exercice sont générés au hasard et sont donc complètement farfelus.} + \medskip + Le \og continent de plastique\fg{} est la plus grande des plaques de déchets plastiques évoluant sur les océans. Elle occupe actuellement dans l'océan Pacifique une surface dont l'aire est évaluée à plus de $1,6$ million de km$^2$, entre Hawaï et la Californie. + + En 2017, des scientifiques ont estimé qu'il y avait $12$ millions de tonnes de déchets plastiques qui était déversé chaque année dans les océans et que cette quantité augmentait de $14\n\%$ par chaque année. + + On modélise l'évolution de la masse de ces déchets plastiques déversée chaque année, si rien n'est fait pour la réduire, par une suite géométrique $\left(u_n\right)$. L'arrondi au centième du terme $u_n$ représente la masse de ces déchets déversée chaque année, exprimée en million de tonnes, pour l'année $(2017 + n)$. + + \medskip + + \begin{enumerate} + \item Expliquer pourquoi la suite $u_n$ est géométrique? + \item Calculer $u_1$ et $u_2$. + \item Exprimer $u_n$ en fonction de $n$. + \item Au début de l'année 2017, il y avait $300$ millions de tonnes de déchets plastique. Calculer la quantité totale de déchets plastiques en 2030. + \item On souhaite déterminer en quelle année la masse totale de ces déchets plastiques aura pour la première fois augmenté de $50$\,\% par rapport à sa valeur de 2017. + \begin{enumerate} + \item Recopier et compléter l'algorithme ci-dessous pour que la variable $N$ contienne la réponse au problème posé. + + \begin{center} + \begin{tabularx}{0.4\linewidth}{|X|}\hline + $N = 2017$\\ + $U = 12$ \\ + $S = 300 + U$ \\ + while $S < 450$: \\ + \hspace{1cm} $N = \ldots$\\ + \hspace{1cm} $U = \ldots$\\ + \hspace{1cm} $S = \ldots$\\ + \hline + \end{tabularx} + \end{center} + \item Que contiennent les variables $S$, $U$ et $N$ après exécution de cet algorithme ? + + Interpréter les résultats dans le contexte de l'exercice. + \end{enumerate} + \end{enumerate} +\end{exercise} + +\begin{solution} + \begin{enumerate} + \item Une augmentation de $14\,\%$ revient à multiplier la quantité par $1.1400000000000001$. La suite est donc bien géométrique. Son premier terme est $u_0 = 12$ et sa raison est $q = 1.1400000000000001$ + \item + \[ + u_1 = u_0 * 1.1400000000000001 = 13.680000000000001 + \] + \[ + u_2 = u_0 * 1.1400000000000001^2 = 15.5952 + \] + \item + \[ + u_n = u_0 \times q^n = 12 \times 1.1400000000000001^n + \] + \item On calcule la quantité totale déversée entre 2017 et 2030. + \[ + \sum_{n = 0}^{13} u_n = u_0 \times \frac{1-q^{13}}{1-q} = 12 \times \frac{1 - 1.1400000000000001^{13}}{1 - 1.1400000000000001} = 385.06 + \] + On en déduit la quantité totale de déchets en 2030 + \[ + 300 + 385.06 = 685.06 + \] + \item + \begin{enumerate} + \item ~ + \begin{center} + \begin{tabularx}{0.4\linewidth}{|X|}\hline + $N \gets 2017$\\ + $U \gets 12$ \\ + $S \gets 300 + U$ \\ + Tant que $S < 450$ \\ + \hspace{1cm} $N \gets N + 1$\\ + \hspace{1cm} $U \gets U * 1.1400000000000001$\\ + \hspace{1cm} $S \gets S + u$\\ + Fin Tant que\\\hline + \end{tabularx} + \end{center} + \item \textit{Pas de correction automatisé} + \end{enumerate} + \end{enumerate} +\end{solution} + +\end{document} + +%%% Local Variables: +%%% mode: latex +%%% TeX-master: "master" +%%% End: diff --git a/TST/DS/DS_21_04_07/TST1/08_210407_DS8.tex b/TST/DS/DS_21_04_07/TST1/08_210407_DS8.tex new file mode 100644 index 0000000..19202a4 --- /dev/null +++ b/TST/DS/DS_21_04_07/TST1/08_210407_DS8.tex @@ -0,0 +1,262 @@ +\documentclass[a4paper,10pt]{article} +\usepackage{myXsim} + +% Title Page +\title{DS8 \hfill COULLON Anis} +\tribe{TST} +\date{\hfillÀ render pour le Mercredi 7 avril} + +\xsimsetup{ + solution/print = false +} + +\begin{document} +\maketitle + +\begin{exercise}[subtitle={Automatismes}] + \textit{Toutes les questions de cette exercice sont indépendantes et peuvent être répondus séparément} + \begin{enumerate} + \item De janvier à septembre, une quantité a augmenté de $14\,\%$. Faire un schéma pour représenter la situation puis calculer le taux d'évolution moyen mensuel. + \item Une quantité augmente de $14\,\%$ par ans. En 2020, elle est de 125\euro. Quelle était sa valeur en 2019? Faire un schéma pour représenter la situation. + \item Déterminer l'équation de la droite \\ + \begin{tikzpicture}[xscale=0.8, yscale=0.5] + \tkzInit[xmin=-5,xmax=5,xstep=1, + ymin=-5,ymax=5,ystep=1] + \tkzGrid + \tkzAxeXY + \tkzFct[domain=-5:5,color=red,very thick]% + {0.5*\x -1}; + \end{tikzpicture} + \item Résoudre l'équation $6 \times 0.82^x = 19$ + \end{enumerate} +\end{exercise} + +\begin{solution} + \begin{enumerate} + \item On veut partager cette évolution en 8 évolutions. + \[ + \left(1 + \frac{14}{100}\right)^{\frac{1}{8}} = 1.0165 + \] + Donc le taux d'évolution moyen est + \[ + t_m = 1.0165 - 1 = 0.01649999999999996 + \] + \item Coefficient multiplicateur pour revenir en arrière + \[ + CM = (1 + \frac{14}{100})^{-1} = 0.8772 + \] + On en déduit la quantité en 2019 + \[ + 125 * 0.8772 = 109.64999999999999 + \] + \item L'équation de la droite est + \[ + y = 0.5 x -1 + \] + \item Il faut penser à faire la division à par $6$ avant d'utiliser le log car sinon, on ne peut pas utiliser la formule $\log(a^n) = n\times \log(a)$. + + \[x = \frac{\log(3.17)}{\log(0.82)}\] + \end{enumerate} +\end{solution} + +\begin{exercise}[subtitle={Restaurant}] + Un \emph{food truck}, ouvert le midi et le soir, propose deux types de formules : + + \setlength\parindent{10mm} + \begin{itemize} + \item la formule \emph{Burger} ; + \item la formule \emph{Wok}. + \end{itemize} + \setlength\parindent{0mm} + + \medskip + + Le gérant a remarqué que 30\,\% de ses ventes ont lieu le midi. Le quart des ventes du midi correspondent à la formule \emph{Burger}, alors que 27\,\% des ventes du soir correspondent à la formule \emph{Wok}. + + Le gérant se constitue un fichier en notant, pour chaque vente, la formule choisie et le moment de cette vente (midi ou soir). + + On prélève une fiche de façon équiprobable. On définit les quatre évènements suivants: + + \begin{enumerate} + \item $M$ : \og la fiche correspond à une vente du midi\fg{} ; + \item $S$ : \og la fiche correspond à une vente du soir\fg {}; + \item $W$ : \og la fiche correspond à une formule \emph{Wok} \fg{} ; + \item $B$ : \og la fiche correspond à une formule \emph{Burger} \fg. + \end{enumerate} + \setlength\parindent{0mm} + + \medskip + + \begin{enumerate} + \item Recopier puis compléter l'arbre pondéré + + \begin{center} + \begin{tikzpicture}[sloped] + \node {.} + child {node {$M$} + child {node {$W$} + edge from parent + node[above] {...} + } + child {node {$B$} + edge from parent + node[above] {...} + } + edge from parent + node[above] {...} + } + child[missing] {} + child { node {$S$} + child {node {$W$} + edge from parent + node[above] {...} + } + child {node {$B$} + edge from parent + node[above] {...} + } + edge from parent + node[above] {...} + } ; + \end{tikzpicture} + \end{center} + + \item Calculer la probabilité de l'évènement $M \cap W$. Interpréter ce résultat dans le contexte de l'exercice. + \item Montrer que la probabilité que la fiche choisie corresponde à une formule \emph{Burger} est égale à $0.586$. + \item On a prélevé une fiche correspondant à la formule \emph{Burger}. Quelle est la probabilité, arrondie au millième, que la vente ait eu lieu le soir? + \end{enumerate} +\end{exercise} + +\begin{solution} + \begin{enumerate} + \item + \begin{center} + \begin{tikzpicture}[sloped] + \node {.} + child {node {$M$} + child {node {$W$} + edge from parent + node[above] {$0.75$} + } + child {node {$B$} + edge from parent + node[above] {$0.25$} + } + edge from parent + node[above] {$0.3$} + } + child[missing] {} + child { node {$S$} + child {node {$W$} + edge from parent + node[above] {$0.27$} + } + child {node {$B$} + edge from parent + node[above] {$0.73$} + } + edge from parent + node[above] {$0.7$} + } ; + \end{tikzpicture} + \end{center} + \item On calcule la probabilité que la vente soit un wok et ait eu lieu à midi + \[ P(M\cap W) = P(M) \times P_M(W) = 0.3 \times 0.75 = 0.225 \] + \item Probabilité que la vente soit un burger. + \[ + P(B) = P(M\cap B) + P(S\cap B) = 0.3 \times 0.75 + 0.7 \times 0.27 = 0.586 + \] + \item On cherche à calculer la quantité $P_B(S)$. Pour cela on utilise la formule de Bayes + \[ + P_B(S) = \frac{P(B\cap S)}{P(B)} = \frac{P_S(B) \times P(S)}{P(B)} = \frac{0.73\times 0.7}{0.586} = 0.8720136518771332 \approx 0.872 + \] + \end{enumerate} +\end{solution} + +\begin{exercise}[subtitle={Continent plastique}] + \textit{Les quantités évoqués dans cette exercice sont générés au hasard et sont donc complètement farfelus.} + \medskip + Le \og continent de plastique\fg{} est la plus grande des plaques de déchets plastiques évoluant sur les océans. Elle occupe actuellement dans l'océan Pacifique une surface dont l'aire est évaluée à plus de $1,6$ million de km$^2$, entre Hawaï et la Californie. + + En 2017, des scientifiques ont estimé qu'il y avait $14$ millions de tonnes de déchets plastiques qui était déversé chaque année dans les océans et que cette quantité augmentait de $25\n\%$ par chaque année. + + On modélise l'évolution de la masse de ces déchets plastiques déversée chaque année, si rien n'est fait pour la réduire, par une suite géométrique $\left(u_n\right)$. L'arrondi au centième du terme $u_n$ représente la masse de ces déchets déversée chaque année, exprimée en million de tonnes, pour l'année $(2017 + n)$. + + \medskip + + \begin{enumerate} + \item Expliquer pourquoi la suite $u_n$ est géométrique? + \item Calculer $u_1$ et $u_2$. + \item Exprimer $u_n$ en fonction de $n$. + \item Au début de l'année 2017, il y avait $300$ millions de tonnes de déchets plastique. Calculer la quantité totale de déchets plastiques en 2030. + \item On souhaite déterminer en quelle année la masse totale de ces déchets plastiques aura pour la première fois augmenté de $50$\,\% par rapport à sa valeur de 2017. + \begin{enumerate} + \item Recopier et compléter l'algorithme ci-dessous pour que la variable $N$ contienne la réponse au problème posé. + + \begin{center} + \begin{tabularx}{0.4\linewidth}{|X|}\hline + $N = 2017$\\ + $U = 14$ \\ + $S = 300 + U$ \\ + while $S < 450$: \\ + \hspace{1cm} $N = \ldots$\\ + \hspace{1cm} $U = \ldots$\\ + \hspace{1cm} $S = \ldots$\\ + \hline + \end{tabularx} + \end{center} + \item Que contiennent les variables $S$, $U$ et $N$ après exécution de cet algorithme ? + + Interpréter les résultats dans le contexte de l'exercice. + \end{enumerate} + \end{enumerate} +\end{exercise} + +\begin{solution} + \begin{enumerate} + \item Une augmentation de $25\,\%$ revient à multiplier la quantité par $1.25$. La suite est donc bien géométrique. Son premier terme est $u_0 = 14$ et sa raison est $q = 1.25$ + \item + \[ + u_1 = u_0 * 1.25 = 17.5 + \] + \[ + u_2 = u_0 * 1.25^2 = 21.875 + \] + \item + \[ + u_n = u_0 \times q^n = 14 \times 1.25^n + \] + \item On calcule la quantité totale déversée entre 2017 et 2030. + \[ + \sum_{n = 0}^{13} u_n = u_0 \times \frac{1-q^{13}}{1-q} = 14 \times \frac{1 - 1.25^{13}}{1 - 1.25} = 962.63 + \] + On en déduit la quantité totale de déchets en 2030 + \[ + 300 + 962.63 = 1262.63 + \] + \item + \begin{enumerate} + \item ~ + \begin{center} + \begin{tabularx}{0.4\linewidth}{|X|}\hline + $N \gets 2017$\\ + $U \gets 14$ \\ + $S \gets 300 + U$ \\ + Tant que $S < 450$ \\ + \hspace{1cm} $N \gets N + 1$\\ + \hspace{1cm} $U \gets U * 1.25$\\ + \hspace{1cm} $S \gets S + u$\\ + Fin Tant que\\\hline + \end{tabularx} + \end{center} + \item \textit{Pas de correction automatisé} + \end{enumerate} + \end{enumerate} +\end{solution} + +\end{document} + +%%% Local Variables: +%%% mode: latex +%%% TeX-master: "master" +%%% End: diff --git a/TST/DS/DS_21_04_07/TST1/09_210407_DS8.tex b/TST/DS/DS_21_04_07/TST1/09_210407_DS8.tex new file mode 100644 index 0000000..6d635b2 --- /dev/null +++ b/TST/DS/DS_21_04_07/TST1/09_210407_DS8.tex @@ -0,0 +1,262 @@ +\documentclass[a4paper,10pt]{article} +\usepackage{myXsim} + +% Title Page +\title{DS8 \hfill DINGER Sölen} +\tribe{TST} +\date{\hfillÀ render pour le Mercredi 7 avril} + +\xsimsetup{ + solution/print = false +} + +\begin{document} +\maketitle + +\begin{exercise}[subtitle={Automatismes}] + \textit{Toutes les questions de cette exercice sont indépendantes et peuvent être répondus séparément} + \begin{enumerate} + \item De janvier à septembre, une quantité a augmenté de $30\,\%$. Faire un schéma pour représenter la situation puis calculer le taux d'évolution moyen mensuel. + \item Une quantité augmente de $30\,\%$ par ans. En 2020, elle est de 141\euro. Quelle était sa valeur en 2019? Faire un schéma pour représenter la situation. + \item Déterminer l'équation de la droite \\ + \begin{tikzpicture}[xscale=0.8, yscale=0.5] + \tkzInit[xmin=-5,xmax=5,xstep=1, + ymin=-5,ymax=5,ystep=1] + \tkzGrid + \tkzAxeXY + \tkzFct[domain=-5:5,color=red,very thick]% + {2.0*\x -3}; + \end{tikzpicture} + \item Résoudre l'équation $5 \times 0.8^x = 11$ + \end{enumerate} +\end{exercise} + +\begin{solution} + \begin{enumerate} + \item On veut partager cette évolution en 8 évolutions. + \[ + \left(1 + \frac{30}{100}\right)^{\frac{1}{8}} = 1.0333 + \] + Donc le taux d'évolution moyen est + \[ + t_m = 1.0333 - 1 = 0.03330000000000011 + \] + \item Coefficient multiplicateur pour revenir en arrière + \[ + CM = (1 + \frac{30}{100})^{-1} = 0.7692 + \] + On en déduit la quantité en 2019 + \[ + 141 * 0.7692 = 108.4572 + \] + \item L'équation de la droite est + \[ + y = 2.0 x -3 + \] + \item Il faut penser à faire la division à par $5$ avant d'utiliser le log car sinon, on ne peut pas utiliser la formule $\log(a^n) = n\times \log(a)$. + + \[x = \frac{\log(2.2)}{\log(0.8)}\] + \end{enumerate} +\end{solution} + +\begin{exercise}[subtitle={Restaurant}] + Un \emph{food truck}, ouvert le midi et le soir, propose deux types de formules : + + \setlength\parindent{10mm} + \begin{itemize} + \item la formule \emph{Burger} ; + \item la formule \emph{Wok}. + \end{itemize} + \setlength\parindent{0mm} + + \medskip + + Le gérant a remarqué que 57\,\% de ses ventes ont lieu le midi. Le quart des ventes du midi correspondent à la formule \emph{Burger}, alors que 90\,\% des ventes du soir correspondent à la formule \emph{Wok}. + + Le gérant se constitue un fichier en notant, pour chaque vente, la formule choisie et le moment de cette vente (midi ou soir). + + On prélève une fiche de façon équiprobable. On définit les quatre évènements suivants: + + \begin{enumerate} + \item $M$ : \og la fiche correspond à une vente du midi\fg{} ; + \item $S$ : \og la fiche correspond à une vente du soir\fg {}; + \item $W$ : \og la fiche correspond à une formule \emph{Wok} \fg{} ; + \item $B$ : \og la fiche correspond à une formule \emph{Burger} \fg. + \end{enumerate} + \setlength\parindent{0mm} + + \medskip + + \begin{enumerate} + \item Recopier puis compléter l'arbre pondéré + + \begin{center} + \begin{tikzpicture}[sloped] + \node {.} + child {node {$M$} + child {node {$W$} + edge from parent + node[above] {...} + } + child {node {$B$} + edge from parent + node[above] {...} + } + edge from parent + node[above] {...} + } + child[missing] {} + child { node {$S$} + child {node {$W$} + edge from parent + node[above] {...} + } + child {node {$B$} + edge from parent + node[above] {...} + } + edge from parent + node[above] {...} + } ; + \end{tikzpicture} + \end{center} + + \item Calculer la probabilité de l'évènement $M \cap W$. Interpréter ce résultat dans le contexte de l'exercice. + \item Montrer que la probabilité que la fiche choisie corresponde à une formule \emph{Burger} est égale à $0.187$. + \item On a prélevé une fiche correspondant à la formule \emph{Burger}. Quelle est la probabilité, arrondie au millième, que la vente ait eu lieu le soir? + \end{enumerate} +\end{exercise} + +\begin{solution} + \begin{enumerate} + \item + \begin{center} + \begin{tikzpicture}[sloped] + \node {.} + child {node {$M$} + child {node {$W$} + edge from parent + node[above] {$0.75$} + } + child {node {$B$} + edge from parent + node[above] {$0.25$} + } + edge from parent + node[above] {$0.58$} + } + child[missing] {} + child { node {$S$} + child {node {$W$} + edge from parent + node[above] {$0.9$} + } + child {node {$B$} + edge from parent + node[above] {$0.1$} + } + edge from parent + node[above] {$0.42$} + } ; + \end{tikzpicture} + \end{center} + \item On calcule la probabilité que la vente soit un wok et ait eu lieu à midi + \[ P(M\cap W) = P(M) \times P_M(W) = 0.58 \times 0.75 = 0.435 \] + \item Probabilité que la vente soit un burger. + \[ + P(B) = P(M\cap B) + P(S\cap B) = 0.58 \times 0.75 + 0.42 \times 0.9 = 0.187 + \] + \item On cherche à calculer la quantité $P_B(S)$. Pour cela on utilise la formule de Bayes + \[ + P_B(S) = \frac{P(B\cap S)}{P(B)} = \frac{P_S(B) \times P(S)}{P(B)} = \frac{0.1\times 0.42}{0.187} = 0.22459893048128343 \approx 0.225 + \] + \end{enumerate} +\end{solution} + +\begin{exercise}[subtitle={Continent plastique}] + \textit{Les quantités évoqués dans cette exercice sont générés au hasard et sont donc complètement farfelus.} + \medskip + Le \og continent de plastique\fg{} est la plus grande des plaques de déchets plastiques évoluant sur les océans. Elle occupe actuellement dans l'océan Pacifique une surface dont l'aire est évaluée à plus de $1,6$ million de km$^2$, entre Hawaï et la Californie. + + En 2017, des scientifiques ont estimé qu'il y avait $14$ millions de tonnes de déchets plastiques qui était déversé chaque année dans les océans et que cette quantité augmentait de $19\n\%$ par chaque année. + + On modélise l'évolution de la masse de ces déchets plastiques déversée chaque année, si rien n'est fait pour la réduire, par une suite géométrique $\left(u_n\right)$. L'arrondi au centième du terme $u_n$ représente la masse de ces déchets déversée chaque année, exprimée en million de tonnes, pour l'année $(2017 + n)$. + + \medskip + + \begin{enumerate} + \item Expliquer pourquoi la suite $u_n$ est géométrique? + \item Calculer $u_1$ et $u_2$. + \item Exprimer $u_n$ en fonction de $n$. + \item Au début de l'année 2017, il y avait $300$ millions de tonnes de déchets plastique. Calculer la quantité totale de déchets plastiques en 2030. + \item On souhaite déterminer en quelle année la masse totale de ces déchets plastiques aura pour la première fois augmenté de $50$\,\% par rapport à sa valeur de 2017. + \begin{enumerate} + \item Recopier et compléter l'algorithme ci-dessous pour que la variable $N$ contienne la réponse au problème posé. + + \begin{center} + \begin{tabularx}{0.4\linewidth}{|X|}\hline + $N = 2017$\\ + $U = 14$ \\ + $S = 300 + U$ \\ + while $S < 450$: \\ + \hspace{1cm} $N = \ldots$\\ + \hspace{1cm} $U = \ldots$\\ + \hspace{1cm} $S = \ldots$\\ + \hline + \end{tabularx} + \end{center} + \item Que contiennent les variables $S$, $U$ et $N$ après exécution de cet algorithme ? + + Interpréter les résultats dans le contexte de l'exercice. + \end{enumerate} + \end{enumerate} +\end{exercise} + +\begin{solution} + \begin{enumerate} + \item Une augmentation de $19\,\%$ revient à multiplier la quantité par $1.19$. La suite est donc bien géométrique. Son premier terme est $u_0 = 14$ et sa raison est $q = 1.19$ + \item + \[ + u_1 = u_0 * 1.19 = 16.66 + \] + \[ + u_2 = u_0 * 1.19^2 = 19.8254 + \] + \item + \[ + u_n = u_0 \times q^n = 14 \times 1.19^n + \] + \item On calcule la quantité totale déversée entre 2017 et 2030. + \[ + \sum_{n = 0}^{13} u_n = u_0 \times \frac{1-q^{13}}{1-q} = 14 \times \frac{1 - 1.19^{13}}{1 - 1.19} = 633.42 + \] + On en déduit la quantité totale de déchets en 2030 + \[ + 300 + 633.42 = 933.42 + \] + \item + \begin{enumerate} + \item ~ + \begin{center} + \begin{tabularx}{0.4\linewidth}{|X|}\hline + $N \gets 2017$\\ + $U \gets 14$ \\ + $S \gets 300 + U$ \\ + Tant que $S < 450$ \\ + \hspace{1cm} $N \gets N + 1$\\ + \hspace{1cm} $U \gets U * 1.19$\\ + \hspace{1cm} $S \gets S + u$\\ + Fin Tant que\\\hline + \end{tabularx} + \end{center} + \item \textit{Pas de correction automatisé} + \end{enumerate} + \end{enumerate} +\end{solution} + +\end{document} + +%%% Local Variables: +%%% mode: latex +%%% TeX-master: "master" +%%% End: diff --git a/TST/DS/DS_21_04_07/TST1/10_210407_DS8.tex b/TST/DS/DS_21_04_07/TST1/10_210407_DS8.tex new file mode 100644 index 0000000..16856e4 --- /dev/null +++ b/TST/DS/DS_21_04_07/TST1/10_210407_DS8.tex @@ -0,0 +1,262 @@ +\documentclass[a4paper,10pt]{article} +\usepackage{myXsim} + +% Title Page +\title{DS8 \hfill EYRAUD Cynthia} +\tribe{TST} +\date{\hfillÀ render pour le Mercredi 7 avril} + +\xsimsetup{ + solution/print = false +} + +\begin{document} +\maketitle + +\begin{exercise}[subtitle={Automatismes}] + \textit{Toutes les questions de cette exercice sont indépendantes et peuvent être répondus séparément} + \begin{enumerate} + \item De janvier à septembre, une quantité a augmenté de $29\,\%$. Faire un schéma pour représenter la situation puis calculer le taux d'évolution moyen mensuel. + \item Une quantité augmente de $29\,\%$ par ans. En 2020, elle est de 130\euro. Quelle était sa valeur en 2019? Faire un schéma pour représenter la situation. + \item Déterminer l'équation de la droite \\ + \begin{tikzpicture}[xscale=0.8, yscale=0.5] + \tkzInit[xmin=-5,xmax=5,xstep=1, + ymin=-5,ymax=5,ystep=1] + \tkzGrid + \tkzAxeXY + \tkzFct[domain=-5:5,color=red,very thick]% + {4.0*\x -4}; + \end{tikzpicture} + \item Résoudre l'équation $6 \times 0.48^x = 28$ + \end{enumerate} +\end{exercise} + +\begin{solution} + \begin{enumerate} + \item On veut partager cette évolution en 8 évolutions. + \[ + \left(1 + \frac{29}{100}\right)^{\frac{1}{8}} = 1.0323 + \] + Donc le taux d'évolution moyen est + \[ + t_m = 1.0323 - 1 = 0.032299999999999995 + \] + \item Coefficient multiplicateur pour revenir en arrière + \[ + CM = (1 + \frac{29}{100})^{-1} = 0.7752 + \] + On en déduit la quantité en 2019 + \[ + 130 * 0.7752 = 100.776 + \] + \item L'équation de la droite est + \[ + y = 4.0 x -4 + \] + \item Il faut penser à faire la division à par $6$ avant d'utiliser le log car sinon, on ne peut pas utiliser la formule $\log(a^n) = n\times \log(a)$. + + \[x = \frac{\log(4.67)}{\log(0.48)}\] + \end{enumerate} +\end{solution} + +\begin{exercise}[subtitle={Restaurant}] + Un \emph{food truck}, ouvert le midi et le soir, propose deux types de formules : + + \setlength\parindent{10mm} + \begin{itemize} + \item la formule \emph{Burger} ; + \item la formule \emph{Wok}. + \end{itemize} + \setlength\parindent{0mm} + + \medskip + + Le gérant a remarqué que 3\,\% de ses ventes ont lieu le midi. Le quart des ventes du midi correspondent à la formule \emph{Burger}, alors que 33\,\% des ventes du soir correspondent à la formule \emph{Wok}. + + Le gérant se constitue un fichier en notant, pour chaque vente, la formule choisie et le moment de cette vente (midi ou soir). + + On prélève une fiche de façon équiprobable. On définit les quatre évènements suivants: + + \begin{enumerate} + \item $M$ : \og la fiche correspond à une vente du midi\fg{} ; + \item $S$ : \og la fiche correspond à une vente du soir\fg {}; + \item $W$ : \og la fiche correspond à une formule \emph{Wok} \fg{} ; + \item $B$ : \og la fiche correspond à une formule \emph{Burger} \fg. + \end{enumerate} + \setlength\parindent{0mm} + + \medskip + + \begin{enumerate} + \item Recopier puis compléter l'arbre pondéré + + \begin{center} + \begin{tikzpicture}[sloped] + \node {.} + child {node {$M$} + child {node {$W$} + edge from parent + node[above] {...} + } + child {node {$B$} + edge from parent + node[above] {...} + } + edge from parent + node[above] {...} + } + child[missing] {} + child { node {$S$} + child {node {$W$} + edge from parent + node[above] {...} + } + child {node {$B$} + edge from parent + node[above] {...} + } + edge from parent + node[above] {...} + } ; + \end{tikzpicture} + \end{center} + + \item Calculer la probabilité de l'évènement $M \cap W$. Interpréter ce résultat dans le contexte de l'exercice. + \item Montrer que la probabilité que la fiche choisie corresponde à une formule \emph{Burger} est égale à $0.6574$. + \item On a prélevé une fiche correspondant à la formule \emph{Burger}. Quelle est la probabilité, arrondie au millième, que la vente ait eu lieu le soir? + \end{enumerate} +\end{exercise} + +\begin{solution} + \begin{enumerate} + \item + \begin{center} + \begin{tikzpicture}[sloped] + \node {.} + child {node {$M$} + child {node {$W$} + edge from parent + node[above] {$0.75$} + } + child {node {$B$} + edge from parent + node[above] {$0.25$} + } + edge from parent + node[above] {$0.03$} + } + child[missing] {} + child { node {$S$} + child {node {$W$} + edge from parent + node[above] {$0.33$} + } + child {node {$B$} + edge from parent + node[above] {$0.67$} + } + edge from parent + node[above] {$0.97$} + } ; + \end{tikzpicture} + \end{center} + \item On calcule la probabilité que la vente soit un wok et ait eu lieu à midi + \[ P(M\cap W) = P(M) \times P_M(W) = 0.03 \times 0.75 = 0.0225 \] + \item Probabilité que la vente soit un burger. + \[ + P(B) = P(M\cap B) + P(S\cap B) = 0.03 \times 0.75 + 0.97 \times 0.33 = 0.6574 + \] + \item On cherche à calculer la quantité $P_B(S)$. Pour cela on utilise la formule de Bayes + \[ + P_B(S) = \frac{P(B\cap S)}{P(B)} = \frac{P_S(B) \times P(S)}{P(B)} = \frac{0.67\times 0.97}{0.6574} = 0.9885914207484029 \approx 0.989 + \] + \end{enumerate} +\end{solution} + +\begin{exercise}[subtitle={Continent plastique}] + \textit{Les quantités évoqués dans cette exercice sont générés au hasard et sont donc complètement farfelus.} + \medskip + Le \og continent de plastique\fg{} est la plus grande des plaques de déchets plastiques évoluant sur les océans. Elle occupe actuellement dans l'océan Pacifique une surface dont l'aire est évaluée à plus de $1,6$ million de km$^2$, entre Hawaï et la Californie. + + En 2017, des scientifiques ont estimé qu'il y avait $18$ millions de tonnes de déchets plastiques qui était déversé chaque année dans les océans et que cette quantité augmentait de $25\n\%$ par chaque année. + + On modélise l'évolution de la masse de ces déchets plastiques déversée chaque année, si rien n'est fait pour la réduire, par une suite géométrique $\left(u_n\right)$. L'arrondi au centième du terme $u_n$ représente la masse de ces déchets déversée chaque année, exprimée en million de tonnes, pour l'année $(2017 + n)$. + + \medskip + + \begin{enumerate} + \item Expliquer pourquoi la suite $u_n$ est géométrique? + \item Calculer $u_1$ et $u_2$. + \item Exprimer $u_n$ en fonction de $n$. + \item Au début de l'année 2017, il y avait $300$ millions de tonnes de déchets plastique. Calculer la quantité totale de déchets plastiques en 2030. + \item On souhaite déterminer en quelle année la masse totale de ces déchets plastiques aura pour la première fois augmenté de $50$\,\% par rapport à sa valeur de 2017. + \begin{enumerate} + \item Recopier et compléter l'algorithme ci-dessous pour que la variable $N$ contienne la réponse au problème posé. + + \begin{center} + \begin{tabularx}{0.4\linewidth}{|X|}\hline + $N = 2017$\\ + $U = 18$ \\ + $S = 300 + U$ \\ + while $S < 450$: \\ + \hspace{1cm} $N = \ldots$\\ + \hspace{1cm} $U = \ldots$\\ + \hspace{1cm} $S = \ldots$\\ + \hline + \end{tabularx} + \end{center} + \item Que contiennent les variables $S$, $U$ et $N$ après exécution de cet algorithme ? + + Interpréter les résultats dans le contexte de l'exercice. + \end{enumerate} + \end{enumerate} +\end{exercise} + +\begin{solution} + \begin{enumerate} + \item Une augmentation de $25\,\%$ revient à multiplier la quantité par $1.25$. La suite est donc bien géométrique. Son premier terme est $u_0 = 18$ et sa raison est $q = 1.25$ + \item + \[ + u_1 = u_0 * 1.25 = 22.5 + \] + \[ + u_2 = u_0 * 1.25^2 = 28.125 + \] + \item + \[ + u_n = u_0 \times q^n = 18 \times 1.25^n + \] + \item On calcule la quantité totale déversée entre 2017 et 2030. + \[ + \sum_{n = 0}^{13} u_n = u_0 \times \frac{1-q^{13}}{1-q} = 18 \times \frac{1 - 1.25^{13}}{1 - 1.25} = 1237.67 + \] + On en déduit la quantité totale de déchets en 2030 + \[ + 300 + 1237.67 = 1537.67 + \] + \item + \begin{enumerate} + \item ~ + \begin{center} + \begin{tabularx}{0.4\linewidth}{|X|}\hline + $N \gets 2017$\\ + $U \gets 18$ \\ + $S \gets 300 + U$ \\ + Tant que $S < 450$ \\ + \hspace{1cm} $N \gets N + 1$\\ + \hspace{1cm} $U \gets U * 1.25$\\ + \hspace{1cm} $S \gets S + u$\\ + Fin Tant que\\\hline + \end{tabularx} + \end{center} + \item \textit{Pas de correction automatisé} + \end{enumerate} + \end{enumerate} +\end{solution} + +\end{document} + +%%% Local Variables: +%%% mode: latex +%%% TeX-master: "master" +%%% End: diff --git a/TST/DS/DS_21_04_07/TST1/11_210407_DS8.tex b/TST/DS/DS_21_04_07/TST1/11_210407_DS8.tex new file mode 100644 index 0000000..5484a98 --- /dev/null +++ b/TST/DS/DS_21_04_07/TST1/11_210407_DS8.tex @@ -0,0 +1,262 @@ +\documentclass[a4paper,10pt]{article} +\usepackage{myXsim} + +% Title Page +\title{DS8 \hfill FERREIRA Léo} +\tribe{TST} +\date{\hfillÀ render pour le Mercredi 7 avril} + +\xsimsetup{ + solution/print = false +} + +\begin{document} +\maketitle + +\begin{exercise}[subtitle={Automatismes}] + \textit{Toutes les questions de cette exercice sont indépendantes et peuvent être répondus séparément} + \begin{enumerate} + \item De janvier à septembre, une quantité a augmenté de $10\,\%$. Faire un schéma pour représenter la situation puis calculer le taux d'évolution moyen mensuel. + \item Une quantité augmente de $10\,\%$ par ans. En 2020, elle est de 123\euro. Quelle était sa valeur en 2019? Faire un schéma pour représenter la situation. + \item Déterminer l'équation de la droite \\ + \begin{tikzpicture}[xscale=0.8, yscale=0.5] + \tkzInit[xmin=-5,xmax=5,xstep=1, + ymin=-5,ymax=5,ystep=1] + \tkzGrid + \tkzAxeXY + \tkzFct[domain=-5:5,color=red,very thick]% + {1.5*\x -3}; + \end{tikzpicture} + \item Résoudre l'équation $5 \times 0.79^x = 30$ + \end{enumerate} +\end{exercise} + +\begin{solution} + \begin{enumerate} + \item On veut partager cette évolution en 8 évolutions. + \[ + \left(1 + \frac{10}{100}\right)^{\frac{1}{8}} = 1.012 + \] + Donc le taux d'évolution moyen est + \[ + t_m = 1.012 - 1 = 0.01200000000000001 + \] + \item Coefficient multiplicateur pour revenir en arrière + \[ + CM = (1 + \frac{10}{100})^{-1} = 0.9091 + \] + On en déduit la quantité en 2019 + \[ + 123 * 0.9091 = 111.8193 + \] + \item L'équation de la droite est + \[ + y = 1.5 x -3 + \] + \item Il faut penser à faire la division à par $5$ avant d'utiliser le log car sinon, on ne peut pas utiliser la formule $\log(a^n) = n\times \log(a)$. + + \[x = \frac{\log(6.0)}{\log(0.79)}\] + \end{enumerate} +\end{solution} + +\begin{exercise}[subtitle={Restaurant}] + Un \emph{food truck}, ouvert le midi et le soir, propose deux types de formules : + + \setlength\parindent{10mm} + \begin{itemize} + \item la formule \emph{Burger} ; + \item la formule \emph{Wok}. + \end{itemize} + \setlength\parindent{0mm} + + \medskip + + Le gérant a remarqué que 18\,\% de ses ventes ont lieu le midi. Le quart des ventes du midi correspondent à la formule \emph{Burger}, alors que 37\,\% des ventes du soir correspondent à la formule \emph{Wok}. + + Le gérant se constitue un fichier en notant, pour chaque vente, la formule choisie et le moment de cette vente (midi ou soir). + + On prélève une fiche de façon équiprobable. On définit les quatre évènements suivants: + + \begin{enumerate} + \item $M$ : \og la fiche correspond à une vente du midi\fg{} ; + \item $S$ : \og la fiche correspond à une vente du soir\fg {}; + \item $W$ : \og la fiche correspond à une formule \emph{Wok} \fg{} ; + \item $B$ : \og la fiche correspond à une formule \emph{Burger} \fg. + \end{enumerate} + \setlength\parindent{0mm} + + \medskip + + \begin{enumerate} + \item Recopier puis compléter l'arbre pondéré + + \begin{center} + \begin{tikzpicture}[sloped] + \node {.} + child {node {$M$} + child {node {$W$} + edge from parent + node[above] {...} + } + child {node {$B$} + edge from parent + node[above] {...} + } + edge from parent + node[above] {...} + } + child[missing] {} + child { node {$S$} + child {node {$W$} + edge from parent + node[above] {...} + } + child {node {$B$} + edge from parent + node[above] {...} + } + edge from parent + node[above] {...} + } ; + \end{tikzpicture} + \end{center} + + \item Calculer la probabilité de l'évènement $M \cap W$. Interpréter ce résultat dans le contexte de l'exercice. + \item Montrer que la probabilité que la fiche choisie corresponde à une formule \emph{Burger} est égale à $0.5616$. + \item On a prélevé une fiche correspondant à la formule \emph{Burger}. Quelle est la probabilité, arrondie au millième, que la vente ait eu lieu le soir? + \end{enumerate} +\end{exercise} + +\begin{solution} + \begin{enumerate} + \item + \begin{center} + \begin{tikzpicture}[sloped] + \node {.} + child {node {$M$} + child {node {$W$} + edge from parent + node[above] {$0.75$} + } + child {node {$B$} + edge from parent + node[above] {$0.25$} + } + edge from parent + node[above] {$0.18$} + } + child[missing] {} + child { node {$S$} + child {node {$W$} + edge from parent + node[above] {$0.37$} + } + child {node {$B$} + edge from parent + node[above] {$0.63$} + } + edge from parent + node[above] {$0.82$} + } ; + \end{tikzpicture} + \end{center} + \item On calcule la probabilité que la vente soit un wok et ait eu lieu à midi + \[ P(M\cap W) = P(M) \times P_M(W) = 0.18 \times 0.75 = 0.135 \] + \item Probabilité que la vente soit un burger. + \[ + P(B) = P(M\cap B) + P(S\cap B) = 0.18 \times 0.75 + 0.82 \times 0.37 = 0.5616 + \] + \item On cherche à calculer la quantité $P_B(S)$. Pour cela on utilise la formule de Bayes + \[ + P_B(S) = \frac{P(B\cap S)}{P(B)} = \frac{P_S(B) \times P(S)}{P(B)} = \frac{0.63\times 0.82}{0.5616} = 0.9198717948717948 \approx 0.92 + \] + \end{enumerate} +\end{solution} + +\begin{exercise}[subtitle={Continent plastique}] + \textit{Les quantités évoqués dans cette exercice sont générés au hasard et sont donc complètement farfelus.} + \medskip + Le \og continent de plastique\fg{} est la plus grande des plaques de déchets plastiques évoluant sur les océans. Elle occupe actuellement dans l'océan Pacifique une surface dont l'aire est évaluée à plus de $1,6$ million de km$^2$, entre Hawaï et la Californie. + + En 2017, des scientifiques ont estimé qu'il y avait $3$ millions de tonnes de déchets plastiques qui était déversé chaque année dans les océans et que cette quantité augmentait de $19\n\%$ par chaque année. + + On modélise l'évolution de la masse de ces déchets plastiques déversée chaque année, si rien n'est fait pour la réduire, par une suite géométrique $\left(u_n\right)$. L'arrondi au centième du terme $u_n$ représente la masse de ces déchets déversée chaque année, exprimée en million de tonnes, pour l'année $(2017 + n)$. + + \medskip + + \begin{enumerate} + \item Expliquer pourquoi la suite $u_n$ est géométrique? + \item Calculer $u_1$ et $u_2$. + \item Exprimer $u_n$ en fonction de $n$. + \item Au début de l'année 2017, il y avait $300$ millions de tonnes de déchets plastique. Calculer la quantité totale de déchets plastiques en 2030. + \item On souhaite déterminer en quelle année la masse totale de ces déchets plastiques aura pour la première fois augmenté de $50$\,\% par rapport à sa valeur de 2017. + \begin{enumerate} + \item Recopier et compléter l'algorithme ci-dessous pour que la variable $N$ contienne la réponse au problème posé. + + \begin{center} + \begin{tabularx}{0.4\linewidth}{|X|}\hline + $N = 2017$\\ + $U = 3$ \\ + $S = 300 + U$ \\ + while $S < 450$: \\ + \hspace{1cm} $N = \ldots$\\ + \hspace{1cm} $U = \ldots$\\ + \hspace{1cm} $S = \ldots$\\ + \hline + \end{tabularx} + \end{center} + \item Que contiennent les variables $S$, $U$ et $N$ après exécution de cet algorithme ? + + Interpréter les résultats dans le contexte de l'exercice. + \end{enumerate} + \end{enumerate} +\end{exercise} + +\begin{solution} + \begin{enumerate} + \item Une augmentation de $19\,\%$ revient à multiplier la quantité par $1.19$. La suite est donc bien géométrique. Son premier terme est $u_0 = 3$ et sa raison est $q = 1.19$ + \item + \[ + u_1 = u_0 * 1.19 = 3.57 + \] + \[ + u_2 = u_0 * 1.19^2 = 4.2483 + \] + \item + \[ + u_n = u_0 \times q^n = 3 \times 1.19^n + \] + \item On calcule la quantité totale déversée entre 2017 et 2030. + \[ + \sum_{n = 0}^{13} u_n = u_0 \times \frac{1-q^{13}}{1-q} = 3 \times \frac{1 - 1.19^{13}}{1 - 1.19} = 135.73 + \] + On en déduit la quantité totale de déchets en 2030 + \[ + 300 + 135.73 = 435.73 + \] + \item + \begin{enumerate} + \item ~ + \begin{center} + \begin{tabularx}{0.4\linewidth}{|X|}\hline + $N \gets 2017$\\ + $U \gets 3$ \\ + $S \gets 300 + U$ \\ + Tant que $S < 450$ \\ + \hspace{1cm} $N \gets N + 1$\\ + \hspace{1cm} $U \gets U * 1.19$\\ + \hspace{1cm} $S \gets S + u$\\ + Fin Tant que\\\hline + \end{tabularx} + \end{center} + \item \textit{Pas de correction automatisé} + \end{enumerate} + \end{enumerate} +\end{solution} + +\end{document} + +%%% Local Variables: +%%% mode: latex +%%% TeX-master: "master" +%%% End: diff --git a/TST/DS/DS_21_04_07/TST1/12_210407_DS8.tex b/TST/DS/DS_21_04_07/TST1/12_210407_DS8.tex new file mode 100644 index 0000000..ea53b8a --- /dev/null +++ b/TST/DS/DS_21_04_07/TST1/12_210407_DS8.tex @@ -0,0 +1,262 @@ +\documentclass[a4paper,10pt]{article} +\usepackage{myXsim} + +% Title Page +\title{DS8 \hfill FILALI Zakaria} +\tribe{TST} +\date{\hfillÀ render pour le Mercredi 7 avril} + +\xsimsetup{ + solution/print = false +} + +\begin{document} +\maketitle + +\begin{exercise}[subtitle={Automatismes}] + \textit{Toutes les questions de cette exercice sont indépendantes et peuvent être répondus séparément} + \begin{enumerate} + \item De janvier à septembre, une quantité a augmenté de $19\,\%$. Faire un schéma pour représenter la situation puis calculer le taux d'évolution moyen mensuel. + \item Une quantité augmente de $19\,\%$ par ans. En 2020, elle est de 122\euro. Quelle était sa valeur en 2019? Faire un schéma pour représenter la situation. + \item Déterminer l'équation de la droite \\ + \begin{tikzpicture}[xscale=0.8, yscale=0.5] + \tkzInit[xmin=-5,xmax=5,xstep=1, + ymin=-5,ymax=5,ystep=1] + \tkzGrid + \tkzAxeXY + \tkzFct[domain=-5:5,color=red,very thick]% + {1.0*\x -2}; + \end{tikzpicture} + \item Résoudre l'équation $8 \times 0.25^x = 25$ + \end{enumerate} +\end{exercise} + +\begin{solution} + \begin{enumerate} + \item On veut partager cette évolution en 8 évolutions. + \[ + \left(1 + \frac{19}{100}\right)^{\frac{1}{8}} = 1.022 + \] + Donc le taux d'évolution moyen est + \[ + t_m = 1.022 - 1 = 0.02200000000000002 + \] + \item Coefficient multiplicateur pour revenir en arrière + \[ + CM = (1 + \frac{19}{100})^{-1} = 0.8403 + \] + On en déduit la quantité en 2019 + \[ + 122 * 0.8403 = 102.51660000000001 + \] + \item L'équation de la droite est + \[ + y = 1.0 x -2 + \] + \item Il faut penser à faire la division à par $8$ avant d'utiliser le log car sinon, on ne peut pas utiliser la formule $\log(a^n) = n\times \log(a)$. + + \[x = \frac{\log(3.12)}{\log(0.25)}\] + \end{enumerate} +\end{solution} + +\begin{exercise}[subtitle={Restaurant}] + Un \emph{food truck}, ouvert le midi et le soir, propose deux types de formules : + + \setlength\parindent{10mm} + \begin{itemize} + \item la formule \emph{Burger} ; + \item la formule \emph{Wok}. + \end{itemize} + \setlength\parindent{0mm} + + \medskip + + Le gérant a remarqué que 47\,\% de ses ventes ont lieu le midi. Le quart des ventes du midi correspondent à la formule \emph{Burger}, alors que 69\,\% des ventes du soir correspondent à la formule \emph{Wok}. + + Le gérant se constitue un fichier en notant, pour chaque vente, la formule choisie et le moment de cette vente (midi ou soir). + + On prélève une fiche de façon équiprobable. On définit les quatre évènements suivants: + + \begin{enumerate} + \item $M$ : \og la fiche correspond à une vente du midi\fg{} ; + \item $S$ : \og la fiche correspond à une vente du soir\fg {}; + \item $W$ : \og la fiche correspond à une formule \emph{Wok} \fg{} ; + \item $B$ : \og la fiche correspond à une formule \emph{Burger} \fg. + \end{enumerate} + \setlength\parindent{0mm} + + \medskip + + \begin{enumerate} + \item Recopier puis compléter l'arbre pondéré + + \begin{center} + \begin{tikzpicture}[sloped] + \node {.} + child {node {$M$} + child {node {$W$} + edge from parent + node[above] {...} + } + child {node {$B$} + edge from parent + node[above] {...} + } + edge from parent + node[above] {...} + } + child[missing] {} + child { node {$S$} + child {node {$W$} + edge from parent + node[above] {...} + } + child {node {$B$} + edge from parent + node[above] {...} + } + edge from parent + node[above] {...} + } ; + \end{tikzpicture} + \end{center} + + \item Calculer la probabilité de l'évènement $M \cap W$. Interpréter ce résultat dans le contexte de l'exercice. + \item Montrer que la probabilité que la fiche choisie corresponde à une formule \emph{Burger} est égale à $0.2818$. + \item On a prélevé une fiche correspondant à la formule \emph{Burger}. Quelle est la probabilité, arrondie au millième, que la vente ait eu lieu le soir? + \end{enumerate} +\end{exercise} + +\begin{solution} + \begin{enumerate} + \item + \begin{center} + \begin{tikzpicture}[sloped] + \node {.} + child {node {$M$} + child {node {$W$} + edge from parent + node[above] {$0.75$} + } + child {node {$B$} + edge from parent + node[above] {$0.25$} + } + edge from parent + node[above] {$0.47$} + } + child[missing] {} + child { node {$S$} + child {node {$W$} + edge from parent + node[above] {$0.69$} + } + child {node {$B$} + edge from parent + node[above] {$0.31$} + } + edge from parent + node[above] {$0.53$} + } ; + \end{tikzpicture} + \end{center} + \item On calcule la probabilité que la vente soit un wok et ait eu lieu à midi + \[ P(M\cap W) = P(M) \times P_M(W) = 0.47 \times 0.75 = 0.3525 \] + \item Probabilité que la vente soit un burger. + \[ + P(B) = P(M\cap B) + P(S\cap B) = 0.47 \times 0.75 + 0.53 \times 0.69 = 0.2818 + \] + \item On cherche à calculer la quantité $P_B(S)$. Pour cela on utilise la formule de Bayes + \[ + P_B(S) = \frac{P(B\cap S)}{P(B)} = \frac{P_S(B) \times P(S)}{P(B)} = \frac{0.31\times 0.53}{0.2818} = 0.5830376153300213 \approx 0.583 + \] + \end{enumerate} +\end{solution} + +\begin{exercise}[subtitle={Continent plastique}] + \textit{Les quantités évoqués dans cette exercice sont générés au hasard et sont donc complètement farfelus.} + \medskip + Le \og continent de plastique\fg{} est la plus grande des plaques de déchets plastiques évoluant sur les océans. Elle occupe actuellement dans l'océan Pacifique une surface dont l'aire est évaluée à plus de $1,6$ million de km$^2$, entre Hawaï et la Californie. + + En 2017, des scientifiques ont estimé qu'il y avait $11$ millions de tonnes de déchets plastiques qui était déversé chaque année dans les océans et que cette quantité augmentait de $16\n\%$ par chaque année. + + On modélise l'évolution de la masse de ces déchets plastiques déversée chaque année, si rien n'est fait pour la réduire, par une suite géométrique $\left(u_n\right)$. L'arrondi au centième du terme $u_n$ représente la masse de ces déchets déversée chaque année, exprimée en million de tonnes, pour l'année $(2017 + n)$. + + \medskip + + \begin{enumerate} + \item Expliquer pourquoi la suite $u_n$ est géométrique? + \item Calculer $u_1$ et $u_2$. + \item Exprimer $u_n$ en fonction de $n$. + \item Au début de l'année 2017, il y avait $300$ millions de tonnes de déchets plastique. Calculer la quantité totale de déchets plastiques en 2030. + \item On souhaite déterminer en quelle année la masse totale de ces déchets plastiques aura pour la première fois augmenté de $50$\,\% par rapport à sa valeur de 2017. + \begin{enumerate} + \item Recopier et compléter l'algorithme ci-dessous pour que la variable $N$ contienne la réponse au problème posé. + + \begin{center} + \begin{tabularx}{0.4\linewidth}{|X|}\hline + $N = 2017$\\ + $U = 11$ \\ + $S = 300 + U$ \\ + while $S < 450$: \\ + \hspace{1cm} $N = \ldots$\\ + \hspace{1cm} $U = \ldots$\\ + \hspace{1cm} $S = \ldots$\\ + \hline + \end{tabularx} + \end{center} + \item Que contiennent les variables $S$, $U$ et $N$ après exécution de cet algorithme ? + + Interpréter les résultats dans le contexte de l'exercice. + \end{enumerate} + \end{enumerate} +\end{exercise} + +\begin{solution} + \begin{enumerate} + \item Une augmentation de $16\,\%$ revient à multiplier la quantité par $1.16$. La suite est donc bien géométrique. Son premier terme est $u_0 = 11$ et sa raison est $q = 1.16$ + \item + \[ + u_1 = u_0 * 1.16 = 12.76 + \] + \[ + u_2 = u_0 * 1.16^2 = 14.8016 + \] + \item + \[ + u_n = u_0 \times q^n = 11 \times 1.16^n + \] + \item On calcule la quantité totale déversée entre 2017 et 2030. + \[ + \sum_{n = 0}^{13} u_n = u_0 \times \frac{1-q^{13}}{1-q} = 11 \times \frac{1 - 1.16^{13}}{1 - 1.16} = 404.65 + \] + On en déduit la quantité totale de déchets en 2030 + \[ + 300 + 404.65 = 704.65 + \] + \item + \begin{enumerate} + \item ~ + \begin{center} + \begin{tabularx}{0.4\linewidth}{|X|}\hline + $N \gets 2017$\\ + $U \gets 11$ \\ + $S \gets 300 + U$ \\ + Tant que $S < 450$ \\ + \hspace{1cm} $N \gets N + 1$\\ + \hspace{1cm} $U \gets U * 1.16$\\ + \hspace{1cm} $S \gets S + u$\\ + Fin Tant que\\\hline + \end{tabularx} + \end{center} + \item \textit{Pas de correction automatisé} + \end{enumerate} + \end{enumerate} +\end{solution} + +\end{document} + +%%% Local Variables: +%%% mode: latex +%%% TeX-master: "master" +%%% End: diff --git a/TST/DS/DS_21_04_07/TST1/13_210407_DS8.tex b/TST/DS/DS_21_04_07/TST1/13_210407_DS8.tex new file mode 100644 index 0000000..a254390 --- /dev/null +++ b/TST/DS/DS_21_04_07/TST1/13_210407_DS8.tex @@ -0,0 +1,262 @@ +\documentclass[a4paper,10pt]{article} +\usepackage{myXsim} + +% Title Page +\title{DS8 \hfill FOIGNY Romain} +\tribe{TST} +\date{\hfillÀ render pour le Mercredi 7 avril} + +\xsimsetup{ + solution/print = false +} + +\begin{document} +\maketitle + +\begin{exercise}[subtitle={Automatismes}] + \textit{Toutes les questions de cette exercice sont indépendantes et peuvent être répondus séparément} + \begin{enumerate} + \item De janvier à septembre, une quantité a augmenté de $22\,\%$. Faire un schéma pour représenter la situation puis calculer le taux d'évolution moyen mensuel. + \item Une quantité augmente de $22\,\%$ par ans. En 2020, elle est de 125\euro. Quelle était sa valeur en 2019? Faire un schéma pour représenter la situation. + \item Déterminer l'équation de la droite \\ + \begin{tikzpicture}[xscale=0.8, yscale=0.5] + \tkzInit[xmin=-5,xmax=5,xstep=1, + ymin=-5,ymax=5,ystep=1] + \tkzGrid + \tkzAxeXY + \tkzFct[domain=-5:5,color=red,very thick]% + {2.6666666666666665*\x -4}; + \end{tikzpicture} + \item Résoudre l'équation $9 \times 0.35^x = 44$ + \end{enumerate} +\end{exercise} + +\begin{solution} + \begin{enumerate} + \item On veut partager cette évolution en 8 évolutions. + \[ + \left(1 + \frac{22}{100}\right)^{\frac{1}{8}} = 1.0252 + \] + Donc le taux d'évolution moyen est + \[ + t_m = 1.0252 - 1 = 0.02519999999999989 + \] + \item Coefficient multiplicateur pour revenir en arrière + \[ + CM = (1 + \frac{22}{100})^{-1} = 0.8197 + \] + On en déduit la quantité en 2019 + \[ + 125 * 0.8197 = 102.46249999999999 + \] + \item L'équation de la droite est + \[ + y = 2.6666666666666665 x -4 + \] + \item Il faut penser à faire la division à par $9$ avant d'utiliser le log car sinon, on ne peut pas utiliser la formule $\log(a^n) = n\times \log(a)$. + + \[x = \frac{\log(4.89)}{\log(0.35)}\] + \end{enumerate} +\end{solution} + +\begin{exercise}[subtitle={Restaurant}] + Un \emph{food truck}, ouvert le midi et le soir, propose deux types de formules : + + \setlength\parindent{10mm} + \begin{itemize} + \item la formule \emph{Burger} ; + \item la formule \emph{Wok}. + \end{itemize} + \setlength\parindent{0mm} + + \medskip + + Le gérant a remarqué que 14\,\% de ses ventes ont lieu le midi. Le quart des ventes du midi correspondent à la formule \emph{Burger}, alors que 17\,\% des ventes du soir correspondent à la formule \emph{Wok}. + + Le gérant se constitue un fichier en notant, pour chaque vente, la formule choisie et le moment de cette vente (midi ou soir). + + On prélève une fiche de façon équiprobable. On définit les quatre évènements suivants: + + \begin{enumerate} + \item $M$ : \og la fiche correspond à une vente du midi\fg{} ; + \item $S$ : \og la fiche correspond à une vente du soir\fg {}; + \item $W$ : \og la fiche correspond à une formule \emph{Wok} \fg{} ; + \item $B$ : \og la fiche correspond à une formule \emph{Burger} \fg. + \end{enumerate} + \setlength\parindent{0mm} + + \medskip + + \begin{enumerate} + \item Recopier puis compléter l'arbre pondéré + + \begin{center} + \begin{tikzpicture}[sloped] + \node {.} + child {node {$M$} + child {node {$W$} + edge from parent + node[above] {...} + } + child {node {$B$} + edge from parent + node[above] {...} + } + edge from parent + node[above] {...} + } + child[missing] {} + child { node {$S$} + child {node {$W$} + edge from parent + node[above] {...} + } + child {node {$B$} + edge from parent + node[above] {...} + } + edge from parent + node[above] {...} + } ; + \end{tikzpicture} + \end{center} + + \item Calculer la probabilité de l'évènement $M \cap W$. Interpréter ce résultat dans le contexte de l'exercice. + \item Montrer que la probabilité que la fiche choisie corresponde à une formule \emph{Burger} est égale à $0.7488$. + \item On a prélevé une fiche correspondant à la formule \emph{Burger}. Quelle est la probabilité, arrondie au millième, que la vente ait eu lieu le soir? + \end{enumerate} +\end{exercise} + +\begin{solution} + \begin{enumerate} + \item + \begin{center} + \begin{tikzpicture}[sloped] + \node {.} + child {node {$M$} + child {node {$W$} + edge from parent + node[above] {$0.75$} + } + child {node {$B$} + edge from parent + node[above] {$0.25$} + } + edge from parent + node[above] {$0.14$} + } + child[missing] {} + child { node {$S$} + child {node {$W$} + edge from parent + node[above] {$0.17$} + } + child {node {$B$} + edge from parent + node[above] {$0.83$} + } + edge from parent + node[above] {$0.86$} + } ; + \end{tikzpicture} + \end{center} + \item On calcule la probabilité que la vente soit un wok et ait eu lieu à midi + \[ P(M\cap W) = P(M) \times P_M(W) = 0.14 \times 0.75 = 0.105 \] + \item Probabilité que la vente soit un burger. + \[ + P(B) = P(M\cap B) + P(S\cap B) = 0.14 \times 0.75 + 0.86 \times 0.17 = 0.7488 + \] + \item On cherche à calculer la quantité $P_B(S)$. Pour cela on utilise la formule de Bayes + \[ + P_B(S) = \frac{P(B\cap S)}{P(B)} = \frac{P_S(B) \times P(S)}{P(B)} = \frac{0.83\times 0.86}{0.7488} = 0.953258547008547 \approx 0.953 + \] + \end{enumerate} +\end{solution} + +\begin{exercise}[subtitle={Continent plastique}] + \textit{Les quantités évoqués dans cette exercice sont générés au hasard et sont donc complètement farfelus.} + \medskip + Le \og continent de plastique\fg{} est la plus grande des plaques de déchets plastiques évoluant sur les océans. Elle occupe actuellement dans l'océan Pacifique une surface dont l'aire est évaluée à plus de $1,6$ million de km$^2$, entre Hawaï et la Californie. + + En 2017, des scientifiques ont estimé qu'il y avait $11$ millions de tonnes de déchets plastiques qui était déversé chaque année dans les océans et que cette quantité augmentait de $12\n\%$ par chaque année. + + On modélise l'évolution de la masse de ces déchets plastiques déversée chaque année, si rien n'est fait pour la réduire, par une suite géométrique $\left(u_n\right)$. L'arrondi au centième du terme $u_n$ représente la masse de ces déchets déversée chaque année, exprimée en million de tonnes, pour l'année $(2017 + n)$. + + \medskip + + \begin{enumerate} + \item Expliquer pourquoi la suite $u_n$ est géométrique? + \item Calculer $u_1$ et $u_2$. + \item Exprimer $u_n$ en fonction de $n$. + \item Au début de l'année 2017, il y avait $300$ millions de tonnes de déchets plastique. Calculer la quantité totale de déchets plastiques en 2030. + \item On souhaite déterminer en quelle année la masse totale de ces déchets plastiques aura pour la première fois augmenté de $50$\,\% par rapport à sa valeur de 2017. + \begin{enumerate} + \item Recopier et compléter l'algorithme ci-dessous pour que la variable $N$ contienne la réponse au problème posé. + + \begin{center} + \begin{tabularx}{0.4\linewidth}{|X|}\hline + $N = 2017$\\ + $U = 11$ \\ + $S = 300 + U$ \\ + while $S < 450$: \\ + \hspace{1cm} $N = \ldots$\\ + \hspace{1cm} $U = \ldots$\\ + \hspace{1cm} $S = \ldots$\\ + \hline + \end{tabularx} + \end{center} + \item Que contiennent les variables $S$, $U$ et $N$ après exécution de cet algorithme ? + + Interpréter les résultats dans le contexte de l'exercice. + \end{enumerate} + \end{enumerate} +\end{exercise} + +\begin{solution} + \begin{enumerate} + \item Une augmentation de $12\,\%$ revient à multiplier la quantité par $1.12$. La suite est donc bien géométrique. Son premier terme est $u_0 = 11$ et sa raison est $q = 1.12$ + \item + \[ + u_1 = u_0 * 1.12 = 12.32 + \] + \[ + u_2 = u_0 * 1.12^2 = 13.7984 + \] + \item + \[ + u_n = u_0 \times q^n = 11 \times 1.12^n + \] + \item On calcule la quantité totale déversée entre 2017 et 2030. + \[ + \sum_{n = 0}^{13} u_n = u_0 \times \frac{1-q^{13}}{1-q} = 11 \times \frac{1 - 1.12^{13}}{1 - 1.12} = 308.32 + \] + On en déduit la quantité totale de déchets en 2030 + \[ + 300 + 308.32 = 608.3199999999999 + \] + \item + \begin{enumerate} + \item ~ + \begin{center} + \begin{tabularx}{0.4\linewidth}{|X|}\hline + $N \gets 2017$\\ + $U \gets 11$ \\ + $S \gets 300 + U$ \\ + Tant que $S < 450$ \\ + \hspace{1cm} $N \gets N + 1$\\ + \hspace{1cm} $U \gets U * 1.12$\\ + \hspace{1cm} $S \gets S + u$\\ + Fin Tant que\\\hline + \end{tabularx} + \end{center} + \item \textit{Pas de correction automatisé} + \end{enumerate} + \end{enumerate} +\end{solution} + +\end{document} + +%%% Local Variables: +%%% mode: latex +%%% TeX-master: "master" +%%% End: diff --git a/TST/DS/DS_21_04_07/TST1/14_210407_DS8.tex b/TST/DS/DS_21_04_07/TST1/14_210407_DS8.tex new file mode 100644 index 0000000..614f7af --- /dev/null +++ b/TST/DS/DS_21_04_07/TST1/14_210407_DS8.tex @@ -0,0 +1,262 @@ +\documentclass[a4paper,10pt]{article} +\usepackage{myXsim} + +% Title Page +\title{DS8 \hfill HIPOLITO DA SILVA Andréa} +\tribe{TST} +\date{\hfillÀ render pour le Mercredi 7 avril} + +\xsimsetup{ + solution/print = false +} + +\begin{document} +\maketitle + +\begin{exercise}[subtitle={Automatismes}] + \textit{Toutes les questions de cette exercice sont indépendantes et peuvent être répondus séparément} + \begin{enumerate} + \item De janvier à septembre, une quantité a augmenté de $23\,\%$. Faire un schéma pour représenter la situation puis calculer le taux d'évolution moyen mensuel. + \item Une quantité augmente de $23\,\%$ par ans. En 2020, elle est de 149\euro. Quelle était sa valeur en 2019? Faire un schéma pour représenter la situation. + \item Déterminer l'équation de la droite \\ + \begin{tikzpicture}[xscale=0.8, yscale=0.5] + \tkzInit[xmin=-5,xmax=5,xstep=1, + ymin=-5,ymax=5,ystep=1] + \tkzGrid + \tkzAxeXY + \tkzFct[domain=-5:5,color=red,very thick]% + {0.6666666666666666*\x -1}; + \end{tikzpicture} + \item Résoudre l'équation $7 \times 0.38^x = 21$ + \end{enumerate} +\end{exercise} + +\begin{solution} + \begin{enumerate} + \item On veut partager cette évolution en 8 évolutions. + \[ + \left(1 + \frac{23}{100}\right)^{\frac{1}{8}} = 1.0262 + \] + Donc le taux d'évolution moyen est + \[ + t_m = 1.0262 - 1 = 0.0262 + \] + \item Coefficient multiplicateur pour revenir en arrière + \[ + CM = (1 + \frac{23}{100})^{-1} = 0.813 + \] + On en déduit la quantité en 2019 + \[ + 149 * 0.813 = 121.13699999999999 + \] + \item L'équation de la droite est + \[ + y = 0.6666666666666666 x -1 + \] + \item Il faut penser à faire la division à par $7$ avant d'utiliser le log car sinon, on ne peut pas utiliser la formule $\log(a^n) = n\times \log(a)$. + + \[x = \frac{\log(3.0)}{\log(0.38)}\] + \end{enumerate} +\end{solution} + +\begin{exercise}[subtitle={Restaurant}] + Un \emph{food truck}, ouvert le midi et le soir, propose deux types de formules : + + \setlength\parindent{10mm} + \begin{itemize} + \item la formule \emph{Burger} ; + \item la formule \emph{Wok}. + \end{itemize} + \setlength\parindent{0mm} + + \medskip + + Le gérant a remarqué que 35\,\% de ses ventes ont lieu le midi. Le quart des ventes du midi correspondent à la formule \emph{Burger}, alors que 13\,\% des ventes du soir correspondent à la formule \emph{Wok}. + + Le gérant se constitue un fichier en notant, pour chaque vente, la formule choisie et le moment de cette vente (midi ou soir). + + On prélève une fiche de façon équiprobable. On définit les quatre évènements suivants: + + \begin{enumerate} + \item $M$ : \og la fiche correspond à une vente du midi\fg{} ; + \item $S$ : \og la fiche correspond à une vente du soir\fg {}; + \item $W$ : \og la fiche correspond à une formule \emph{Wok} \fg{} ; + \item $B$ : \og la fiche correspond à une formule \emph{Burger} \fg. + \end{enumerate} + \setlength\parindent{0mm} + + \medskip + + \begin{enumerate} + \item Recopier puis compléter l'arbre pondéré + + \begin{center} + \begin{tikzpicture}[sloped] + \node {.} + child {node {$M$} + child {node {$W$} + edge from parent + node[above] {...} + } + child {node {$B$} + edge from parent + node[above] {...} + } + edge from parent + node[above] {...} + } + child[missing] {} + child { node {$S$} + child {node {$W$} + edge from parent + node[above] {...} + } + child {node {$B$} + edge from parent + node[above] {...} + } + edge from parent + node[above] {...} + } ; + \end{tikzpicture} + \end{center} + + \item Calculer la probabilité de l'évènement $M \cap W$. Interpréter ce résultat dans le contexte de l'exercice. + \item Montrer que la probabilité que la fiche choisie corresponde à une formule \emph{Burger} est égale à $0.653$. + \item On a prélevé une fiche correspondant à la formule \emph{Burger}. Quelle est la probabilité, arrondie au millième, que la vente ait eu lieu le soir? + \end{enumerate} +\end{exercise} + +\begin{solution} + \begin{enumerate} + \item + \begin{center} + \begin{tikzpicture}[sloped] + \node {.} + child {node {$M$} + child {node {$W$} + edge from parent + node[above] {$0.75$} + } + child {node {$B$} + edge from parent + node[above] {$0.25$} + } + edge from parent + node[above] {$0.35$} + } + child[missing] {} + child { node {$S$} + child {node {$W$} + edge from parent + node[above] {$0.13$} + } + child {node {$B$} + edge from parent + node[above] {$0.87$} + } + edge from parent + node[above] {$0.65$} + } ; + \end{tikzpicture} + \end{center} + \item On calcule la probabilité que la vente soit un wok et ait eu lieu à midi + \[ P(M\cap W) = P(M) \times P_M(W) = 0.35 \times 0.75 = 0.2625 \] + \item Probabilité que la vente soit un burger. + \[ + P(B) = P(M\cap B) + P(S\cap B) = 0.35 \times 0.75 + 0.65 \times 0.13 = 0.653 + \] + \item On cherche à calculer la quantité $P_B(S)$. Pour cela on utilise la formule de Bayes + \[ + P_B(S) = \frac{P(B\cap S)}{P(B)} = \frac{P_S(B) \times P(S)}{P(B)} = \frac{0.87\times 0.65}{0.653} = 0.8660030627871362 \approx 0.866 + \] + \end{enumerate} +\end{solution} + +\begin{exercise}[subtitle={Continent plastique}] + \textit{Les quantités évoqués dans cette exercice sont générés au hasard et sont donc complètement farfelus.} + \medskip + Le \og continent de plastique\fg{} est la plus grande des plaques de déchets plastiques évoluant sur les océans. Elle occupe actuellement dans l'océan Pacifique une surface dont l'aire est évaluée à plus de $1,6$ million de km$^2$, entre Hawaï et la Californie. + + En 2017, des scientifiques ont estimé qu'il y avait $4$ millions de tonnes de déchets plastiques qui était déversé chaque année dans les océans et que cette quantité augmentait de $23\n\%$ par chaque année. + + On modélise l'évolution de la masse de ces déchets plastiques déversée chaque année, si rien n'est fait pour la réduire, par une suite géométrique $\left(u_n\right)$. L'arrondi au centième du terme $u_n$ représente la masse de ces déchets déversée chaque année, exprimée en million de tonnes, pour l'année $(2017 + n)$. + + \medskip + + \begin{enumerate} + \item Expliquer pourquoi la suite $u_n$ est géométrique? + \item Calculer $u_1$ et $u_2$. + \item Exprimer $u_n$ en fonction de $n$. + \item Au début de l'année 2017, il y avait $300$ millions de tonnes de déchets plastique. Calculer la quantité totale de déchets plastiques en 2030. + \item On souhaite déterminer en quelle année la masse totale de ces déchets plastiques aura pour la première fois augmenté de $50$\,\% par rapport à sa valeur de 2017. + \begin{enumerate} + \item Recopier et compléter l'algorithme ci-dessous pour que la variable $N$ contienne la réponse au problème posé. + + \begin{center} + \begin{tabularx}{0.4\linewidth}{|X|}\hline + $N = 2017$\\ + $U = 4$ \\ + $S = 300 + U$ \\ + while $S < 450$: \\ + \hspace{1cm} $N = \ldots$\\ + \hspace{1cm} $U = \ldots$\\ + \hspace{1cm} $S = \ldots$\\ + \hline + \end{tabularx} + \end{center} + \item Que contiennent les variables $S$, $U$ et $N$ après exécution de cet algorithme ? + + Interpréter les résultats dans le contexte de l'exercice. + \end{enumerate} + \end{enumerate} +\end{exercise} + +\begin{solution} + \begin{enumerate} + \item Une augmentation de $23\,\%$ revient à multiplier la quantité par $1.23$. La suite est donc bien géométrique. Son premier terme est $u_0 = 4$ et sa raison est $q = 1.23$ + \item + \[ + u_1 = u_0 * 1.23 = 4.92 + \] + \[ + u_2 = u_0 * 1.23^2 = 6.0516 + \] + \item + \[ + u_n = u_0 \times q^n = 4 \times 1.23^n + \] + \item On calcule la quantité totale déversée entre 2017 et 2030. + \[ + \sum_{n = 0}^{13} u_n = u_0 \times \frac{1-q^{13}}{1-q} = 4 \times \frac{1 - 1.23^{13}}{1 - 1.23} = 239.12 + \] + On en déduit la quantité totale de déchets en 2030 + \[ + 300 + 239.12 = 539.12 + \] + \item + \begin{enumerate} + \item ~ + \begin{center} + \begin{tabularx}{0.4\linewidth}{|X|}\hline + $N \gets 2017$\\ + $U \gets 4$ \\ + $S \gets 300 + U$ \\ + Tant que $S < 450$ \\ + \hspace{1cm} $N \gets N + 1$\\ + \hspace{1cm} $U \gets U * 1.23$\\ + \hspace{1cm} $S \gets S + u$\\ + Fin Tant que\\\hline + \end{tabularx} + \end{center} + \item \textit{Pas de correction automatisé} + \end{enumerate} + \end{enumerate} +\end{solution} + +\end{document} + +%%% Local Variables: +%%% mode: latex +%%% TeX-master: "master" +%%% End: diff --git a/TST/DS/DS_21_04_07/TST1/15_210407_DS8.tex b/TST/DS/DS_21_04_07/TST1/15_210407_DS8.tex new file mode 100644 index 0000000..61ad8fe --- /dev/null +++ b/TST/DS/DS_21_04_07/TST1/15_210407_DS8.tex @@ -0,0 +1,262 @@ +\documentclass[a4paper,10pt]{article} +\usepackage{myXsim} + +% Title Page +\title{DS8 \hfill HUMBERT Rayan} +\tribe{TST} +\date{\hfillÀ render pour le Mercredi 7 avril} + +\xsimsetup{ + solution/print = false +} + +\begin{document} +\maketitle + +\begin{exercise}[subtitle={Automatismes}] + \textit{Toutes les questions de cette exercice sont indépendantes et peuvent être répondus séparément} + \begin{enumerate} + \item De janvier à septembre, une quantité a augmenté de $17\,\%$. Faire un schéma pour représenter la situation puis calculer le taux d'évolution moyen mensuel. + \item Une quantité augmente de $17\,\%$ par ans. En 2020, elle est de 149\euro. Quelle était sa valeur en 2019? Faire un schéma pour représenter la situation. + \item Déterminer l'équation de la droite \\ + \begin{tikzpicture}[xscale=0.8, yscale=0.5] + \tkzInit[xmin=-5,xmax=5,xstep=1, + ymin=-5,ymax=5,ystep=1] + \tkzGrid + \tkzAxeXY + \tkzFct[domain=-5:5,color=red,very thick]% + {0.5*\x -1}; + \end{tikzpicture} + \item Résoudre l'équation $5 \times 0.02^x = 46$ + \end{enumerate} +\end{exercise} + +\begin{solution} + \begin{enumerate} + \item On veut partager cette évolution en 8 évolutions. + \[ + \left(1 + \frac{17}{100}\right)^{\frac{1}{8}} = 1.0198 + \] + Donc le taux d'évolution moyen est + \[ + t_m = 1.0198 - 1 = 0.01980000000000004 + \] + \item Coefficient multiplicateur pour revenir en arrière + \[ + CM = (1 + \frac{17}{100})^{-1} = 0.8547 + \] + On en déduit la quantité en 2019 + \[ + 149 * 0.8547 = 127.3503 + \] + \item L'équation de la droite est + \[ + y = 0.5 x -1 + \] + \item Il faut penser à faire la division à par $5$ avant d'utiliser le log car sinon, on ne peut pas utiliser la formule $\log(a^n) = n\times \log(a)$. + + \[x = \frac{\log(9.2)}{\log(0.02)}\] + \end{enumerate} +\end{solution} + +\begin{exercise}[subtitle={Restaurant}] + Un \emph{food truck}, ouvert le midi et le soir, propose deux types de formules : + + \setlength\parindent{10mm} + \begin{itemize} + \item la formule \emph{Burger} ; + \item la formule \emph{Wok}. + \end{itemize} + \setlength\parindent{0mm} + + \medskip + + Le gérant a remarqué que 65\,\% de ses ventes ont lieu le midi. Le quart des ventes du midi correspondent à la formule \emph{Burger}, alors que 93\,\% des ventes du soir correspondent à la formule \emph{Wok}. + + Le gérant se constitue un fichier en notant, pour chaque vente, la formule choisie et le moment de cette vente (midi ou soir). + + On prélève une fiche de façon équiprobable. On définit les quatre évènements suivants: + + \begin{enumerate} + \item $M$ : \og la fiche correspond à une vente du midi\fg{} ; + \item $S$ : \og la fiche correspond à une vente du soir\fg {}; + \item $W$ : \og la fiche correspond à une formule \emph{Wok} \fg{} ; + \item $B$ : \og la fiche correspond à une formule \emph{Burger} \fg. + \end{enumerate} + \setlength\parindent{0mm} + + \medskip + + \begin{enumerate} + \item Recopier puis compléter l'arbre pondéré + + \begin{center} + \begin{tikzpicture}[sloped] + \node {.} + child {node {$M$} + child {node {$W$} + edge from parent + node[above] {...} + } + child {node {$B$} + edge from parent + node[above] {...} + } + edge from parent + node[above] {...} + } + child[missing] {} + child { node {$S$} + child {node {$W$} + edge from parent + node[above] {...} + } + child {node {$B$} + edge from parent + node[above] {...} + } + edge from parent + node[above] {...} + } ; + \end{tikzpicture} + \end{center} + + \item Calculer la probabilité de l'évènement $M \cap W$. Interpréter ce résultat dans le contexte de l'exercice. + \item Montrer que la probabilité que la fiche choisie corresponde à une formule \emph{Burger} est égale à $0.187$. + \item On a prélevé une fiche correspondant à la formule \emph{Burger}. Quelle est la probabilité, arrondie au millième, que la vente ait eu lieu le soir? + \end{enumerate} +\end{exercise} + +\begin{solution} + \begin{enumerate} + \item + \begin{center} + \begin{tikzpicture}[sloped] + \node {.} + child {node {$M$} + child {node {$W$} + edge from parent + node[above] {$0.75$} + } + child {node {$B$} + edge from parent + node[above] {$0.25$} + } + edge from parent + node[above] {$0.65$} + } + child[missing] {} + child { node {$S$} + child {node {$W$} + edge from parent + node[above] {$0.93$} + } + child {node {$B$} + edge from parent + node[above] {$0.07$} + } + edge from parent + node[above] {$0.35$} + } ; + \end{tikzpicture} + \end{center} + \item On calcule la probabilité que la vente soit un wok et ait eu lieu à midi + \[ P(M\cap W) = P(M) \times P_M(W) = 0.65 \times 0.75 = 0.4875 \] + \item Probabilité que la vente soit un burger. + \[ + P(B) = P(M\cap B) + P(S\cap B) = 0.65 \times 0.75 + 0.35 \times 0.93 = 0.187 + \] + \item On cherche à calculer la quantité $P_B(S)$. Pour cela on utilise la formule de Bayes + \[ + P_B(S) = \frac{P(B\cap S)}{P(B)} = \frac{P_S(B) \times P(S)}{P(B)} = \frac{0.07\times 0.35}{0.187} = 0.13101604278074866 \approx 0.131 + \] + \end{enumerate} +\end{solution} + +\begin{exercise}[subtitle={Continent plastique}] + \textit{Les quantités évoqués dans cette exercice sont générés au hasard et sont donc complètement farfelus.} + \medskip + Le \og continent de plastique\fg{} est la plus grande des plaques de déchets plastiques évoluant sur les océans. Elle occupe actuellement dans l'océan Pacifique une surface dont l'aire est évaluée à plus de $1,6$ million de km$^2$, entre Hawaï et la Californie. + + En 2017, des scientifiques ont estimé qu'il y avait $15$ millions de tonnes de déchets plastiques qui était déversé chaque année dans les océans et que cette quantité augmentait de $12\n\%$ par chaque année. + + On modélise l'évolution de la masse de ces déchets plastiques déversée chaque année, si rien n'est fait pour la réduire, par une suite géométrique $\left(u_n\right)$. L'arrondi au centième du terme $u_n$ représente la masse de ces déchets déversée chaque année, exprimée en million de tonnes, pour l'année $(2017 + n)$. + + \medskip + + \begin{enumerate} + \item Expliquer pourquoi la suite $u_n$ est géométrique? + \item Calculer $u_1$ et $u_2$. + \item Exprimer $u_n$ en fonction de $n$. + \item Au début de l'année 2017, il y avait $300$ millions de tonnes de déchets plastique. Calculer la quantité totale de déchets plastiques en 2030. + \item On souhaite déterminer en quelle année la masse totale de ces déchets plastiques aura pour la première fois augmenté de $50$\,\% par rapport à sa valeur de 2017. + \begin{enumerate} + \item Recopier et compléter l'algorithme ci-dessous pour que la variable $N$ contienne la réponse au problème posé. + + \begin{center} + \begin{tabularx}{0.4\linewidth}{|X|}\hline + $N = 2017$\\ + $U = 15$ \\ + $S = 300 + U$ \\ + while $S < 450$: \\ + \hspace{1cm} $N = \ldots$\\ + \hspace{1cm} $U = \ldots$\\ + \hspace{1cm} $S = \ldots$\\ + \hline + \end{tabularx} + \end{center} + \item Que contiennent les variables $S$, $U$ et $N$ après exécution de cet algorithme ? + + Interpréter les résultats dans le contexte de l'exercice. + \end{enumerate} + \end{enumerate} +\end{exercise} + +\begin{solution} + \begin{enumerate} + \item Une augmentation de $12\,\%$ revient à multiplier la quantité par $1.12$. La suite est donc bien géométrique. Son premier terme est $u_0 = 15$ et sa raison est $q = 1.12$ + \item + \[ + u_1 = u_0 * 1.12 = 16.8 + \] + \[ + u_2 = u_0 * 1.12^2 = 18.816 + \] + \item + \[ + u_n = u_0 \times q^n = 15 \times 1.12^n + \] + \item On calcule la quantité totale déversée entre 2017 et 2030. + \[ + \sum_{n = 0}^{13} u_n = u_0 \times \frac{1-q^{13}}{1-q} = 15 \times \frac{1 - 1.12^{13}}{1 - 1.12} = 420.44 + \] + On en déduit la quantité totale de déchets en 2030 + \[ + 300 + 420.44 = 720.44 + \] + \item + \begin{enumerate} + \item ~ + \begin{center} + \begin{tabularx}{0.4\linewidth}{|X|}\hline + $N \gets 2017$\\ + $U \gets 15$ \\ + $S \gets 300 + U$ \\ + Tant que $S < 450$ \\ + \hspace{1cm} $N \gets N + 1$\\ + \hspace{1cm} $U \gets U * 1.12$\\ + \hspace{1cm} $S \gets S + u$\\ + Fin Tant que\\\hline + \end{tabularx} + \end{center} + \item \textit{Pas de correction automatisé} + \end{enumerate} + \end{enumerate} +\end{solution} + +\end{document} + +%%% Local Variables: +%%% mode: latex +%%% TeX-master: "master" +%%% End: diff --git a/TST/DS/DS_21_04_07/TST1/16_210407_DS8.tex b/TST/DS/DS_21_04_07/TST1/16_210407_DS8.tex new file mode 100644 index 0000000..c653e35 --- /dev/null +++ b/TST/DS/DS_21_04_07/TST1/16_210407_DS8.tex @@ -0,0 +1,262 @@ +\documentclass[a4paper,10pt]{article} +\usepackage{myXsim} + +% Title Page +\title{DS8 \hfill MASSON Grace} +\tribe{TST} +\date{\hfillÀ render pour le Mercredi 7 avril} + +\xsimsetup{ + solution/print = false +} + +\begin{document} +\maketitle + +\begin{exercise}[subtitle={Automatismes}] + \textit{Toutes les questions de cette exercice sont indépendantes et peuvent être répondus séparément} + \begin{enumerate} + \item De janvier à septembre, une quantité a augmenté de $24\,\%$. Faire un schéma pour représenter la situation puis calculer le taux d'évolution moyen mensuel. + \item Une quantité augmente de $24\,\%$ par ans. En 2020, elle est de 145\euro. Quelle était sa valeur en 2019? Faire un schéma pour représenter la situation. + \item Déterminer l'équation de la droite \\ + \begin{tikzpicture}[xscale=0.8, yscale=0.5] + \tkzInit[xmin=-5,xmax=5,xstep=1, + ymin=-5,ymax=5,ystep=1] + \tkzGrid + \tkzAxeXY + \tkzFct[domain=-5:5,color=red,very thick]% + {3.0*\x -3}; + \end{tikzpicture} + \item Résoudre l'équation $5 \times 0.53^x = 18$ + \end{enumerate} +\end{exercise} + +\begin{solution} + \begin{enumerate} + \item On veut partager cette évolution en 8 évolutions. + \[ + \left(1 + \frac{24}{100}\right)^{\frac{1}{8}} = 1.0273 + \] + Donc le taux d'évolution moyen est + \[ + t_m = 1.0273 - 1 = 0.027300000000000102 + \] + \item Coefficient multiplicateur pour revenir en arrière + \[ + CM = (1 + \frac{24}{100})^{-1} = 0.8065 + \] + On en déduit la quantité en 2019 + \[ + 145 * 0.8065 = 116.9425 + \] + \item L'équation de la droite est + \[ + y = 3.0 x -3 + \] + \item Il faut penser à faire la division à par $5$ avant d'utiliser le log car sinon, on ne peut pas utiliser la formule $\log(a^n) = n\times \log(a)$. + + \[x = \frac{\log(3.6)}{\log(0.53)}\] + \end{enumerate} +\end{solution} + +\begin{exercise}[subtitle={Restaurant}] + Un \emph{food truck}, ouvert le midi et le soir, propose deux types de formules : + + \setlength\parindent{10mm} + \begin{itemize} + \item la formule \emph{Burger} ; + \item la formule \emph{Wok}. + \end{itemize} + \setlength\parindent{0mm} + + \medskip + + Le gérant a remarqué que 73\,\% de ses ventes ont lieu le midi. Le quart des ventes du midi correspondent à la formule \emph{Burger}, alors que 56\,\% des ventes du soir correspondent à la formule \emph{Wok}. + + Le gérant se constitue un fichier en notant, pour chaque vente, la formule choisie et le moment de cette vente (midi ou soir). + + On prélève une fiche de façon équiprobable. On définit les quatre évènements suivants: + + \begin{enumerate} + \item $M$ : \og la fiche correspond à une vente du midi\fg{} ; + \item $S$ : \og la fiche correspond à une vente du soir\fg {}; + \item $W$ : \og la fiche correspond à une formule \emph{Wok} \fg{} ; + \item $B$ : \og la fiche correspond à une formule \emph{Burger} \fg. + \end{enumerate} + \setlength\parindent{0mm} + + \medskip + + \begin{enumerate} + \item Recopier puis compléter l'arbre pondéré + + \begin{center} + \begin{tikzpicture}[sloped] + \node {.} + child {node {$M$} + child {node {$W$} + edge from parent + node[above] {...} + } + child {node {$B$} + edge from parent + node[above] {...} + } + edge from parent + node[above] {...} + } + child[missing] {} + child { node {$S$} + child {node {$W$} + edge from parent + node[above] {...} + } + child {node {$B$} + edge from parent + node[above] {...} + } + edge from parent + node[above] {...} + } ; + \end{tikzpicture} + \end{center} + + \item Calculer la probabilité de l'évènement $M \cap W$. Interpréter ce résultat dans le contexte de l'exercice. + \item Montrer que la probabilité que la fiche choisie corresponde à une formule \emph{Burger} est égale à $0.2986$. + \item On a prélevé une fiche correspondant à la formule \emph{Burger}. Quelle est la probabilité, arrondie au millième, que la vente ait eu lieu le soir? + \end{enumerate} +\end{exercise} + +\begin{solution} + \begin{enumerate} + \item + \begin{center} + \begin{tikzpicture}[sloped] + \node {.} + child {node {$M$} + child {node {$W$} + edge from parent + node[above] {$0.75$} + } + child {node {$B$} + edge from parent + node[above] {$0.25$} + } + edge from parent + node[above] {$0.73$} + } + child[missing] {} + child { node {$S$} + child {node {$W$} + edge from parent + node[above] {$0.57$} + } + child {node {$B$} + edge from parent + node[above] {$0.43$} + } + edge from parent + node[above] {$0.27$} + } ; + \end{tikzpicture} + \end{center} + \item On calcule la probabilité que la vente soit un wok et ait eu lieu à midi + \[ P(M\cap W) = P(M) \times P_M(W) = 0.73 \times 0.75 = 0.5475 \] + \item Probabilité que la vente soit un burger. + \[ + P(B) = P(M\cap B) + P(S\cap B) = 0.73 \times 0.75 + 0.27 \times 0.57 = 0.2986 + \] + \item On cherche à calculer la quantité $P_B(S)$. Pour cela on utilise la formule de Bayes + \[ + P_B(S) = \frac{P(B\cap S)}{P(B)} = \frac{P_S(B) \times P(S)}{P(B)} = \frac{0.43\times 0.27}{0.2986} = 0.38881446751507037 \approx 0.389 + \] + \end{enumerate} +\end{solution} + +\begin{exercise}[subtitle={Continent plastique}] + \textit{Les quantités évoqués dans cette exercice sont générés au hasard et sont donc complètement farfelus.} + \medskip + Le \og continent de plastique\fg{} est la plus grande des plaques de déchets plastiques évoluant sur les océans. Elle occupe actuellement dans l'océan Pacifique une surface dont l'aire est évaluée à plus de $1,6$ million de km$^2$, entre Hawaï et la Californie. + + En 2017, des scientifiques ont estimé qu'il y avait $4$ millions de tonnes de déchets plastiques qui était déversé chaque année dans les océans et que cette quantité augmentait de $19\n\%$ par chaque année. + + On modélise l'évolution de la masse de ces déchets plastiques déversée chaque année, si rien n'est fait pour la réduire, par une suite géométrique $\left(u_n\right)$. L'arrondi au centième du terme $u_n$ représente la masse de ces déchets déversée chaque année, exprimée en million de tonnes, pour l'année $(2017 + n)$. + + \medskip + + \begin{enumerate} + \item Expliquer pourquoi la suite $u_n$ est géométrique? + \item Calculer $u_1$ et $u_2$. + \item Exprimer $u_n$ en fonction de $n$. + \item Au début de l'année 2017, il y avait $300$ millions de tonnes de déchets plastique. Calculer la quantité totale de déchets plastiques en 2030. + \item On souhaite déterminer en quelle année la masse totale de ces déchets plastiques aura pour la première fois augmenté de $50$\,\% par rapport à sa valeur de 2017. + \begin{enumerate} + \item Recopier et compléter l'algorithme ci-dessous pour que la variable $N$ contienne la réponse au problème posé. + + \begin{center} + \begin{tabularx}{0.4\linewidth}{|X|}\hline + $N = 2017$\\ + $U = 4$ \\ + $S = 300 + U$ \\ + while $S < 450$: \\ + \hspace{1cm} $N = \ldots$\\ + \hspace{1cm} $U = \ldots$\\ + \hspace{1cm} $S = \ldots$\\ + \hline + \end{tabularx} + \end{center} + \item Que contiennent les variables $S$, $U$ et $N$ après exécution de cet algorithme ? + + Interpréter les résultats dans le contexte de l'exercice. + \end{enumerate} + \end{enumerate} +\end{exercise} + +\begin{solution} + \begin{enumerate} + \item Une augmentation de $19\,\%$ revient à multiplier la quantité par $1.19$. La suite est donc bien géométrique. Son premier terme est $u_0 = 4$ et sa raison est $q = 1.19$ + \item + \[ + u_1 = u_0 * 1.19 = 4.76 + \] + \[ + u_2 = u_0 * 1.19^2 = 5.6644 + \] + \item + \[ + u_n = u_0 \times q^n = 4 \times 1.19^n + \] + \item On calcule la quantité totale déversée entre 2017 et 2030. + \[ + \sum_{n = 0}^{13} u_n = u_0 \times \frac{1-q^{13}}{1-q} = 4 \times \frac{1 - 1.19^{13}}{1 - 1.19} = 180.98 + \] + On en déduit la quantité totale de déchets en 2030 + \[ + 300 + 180.98 = 480.98 + \] + \item + \begin{enumerate} + \item ~ + \begin{center} + \begin{tabularx}{0.4\linewidth}{|X|}\hline + $N \gets 2017$\\ + $U \gets 4$ \\ + $S \gets 300 + U$ \\ + Tant que $S < 450$ \\ + \hspace{1cm} $N \gets N + 1$\\ + \hspace{1cm} $U \gets U * 1.19$\\ + \hspace{1cm} $S \gets S + u$\\ + Fin Tant que\\\hline + \end{tabularx} + \end{center} + \item \textit{Pas de correction automatisé} + \end{enumerate} + \end{enumerate} +\end{solution} + +\end{document} + +%%% Local Variables: +%%% mode: latex +%%% TeX-master: "master" +%%% End: diff --git a/TST/DS/DS_21_04_07/TST1/17_210407_DS8.tex b/TST/DS/DS_21_04_07/TST1/17_210407_DS8.tex new file mode 100644 index 0000000..4e55741 --- /dev/null +++ b/TST/DS/DS_21_04_07/TST1/17_210407_DS8.tex @@ -0,0 +1,262 @@ +\documentclass[a4paper,10pt]{article} +\usepackage{myXsim} + +% Title Page +\title{DS8 \hfill MOKHTARI Nissrine} +\tribe{TST} +\date{\hfillÀ render pour le Mercredi 7 avril} + +\xsimsetup{ + solution/print = false +} + +\begin{document} +\maketitle + +\begin{exercise}[subtitle={Automatismes}] + \textit{Toutes les questions de cette exercice sont indépendantes et peuvent être répondus séparément} + \begin{enumerate} + \item De janvier à septembre, une quantité a augmenté de $11\,\%$. Faire un schéma pour représenter la situation puis calculer le taux d'évolution moyen mensuel. + \item Une quantité augmente de $11\,\%$ par ans. En 2020, elle est de 138\euro. Quelle était sa valeur en 2019? Faire un schéma pour représenter la situation. + \item Déterminer l'équation de la droite \\ + \begin{tikzpicture}[xscale=0.8, yscale=0.5] + \tkzInit[xmin=-5,xmax=5,xstep=1, + ymin=-5,ymax=5,ystep=1] + \tkzGrid + \tkzAxeXY + \tkzFct[domain=-5:5,color=red,very thick]% + {2.0*\x -3}; + \end{tikzpicture} + \item Résoudre l'équation $8 \times 0.17^x = 35$ + \end{enumerate} +\end{exercise} + +\begin{solution} + \begin{enumerate} + \item On veut partager cette évolution en 8 évolutions. + \[ + \left(1 + \frac{11}{100}\right)^{\frac{1}{8}} = 1.0131 + \] + Donc le taux d'évolution moyen est + \[ + t_m = 1.0131 - 1 = 0.01309999999999989 + \] + \item Coefficient multiplicateur pour revenir en arrière + \[ + CM = (1 + \frac{11}{100})^{-1} = 0.9009 + \] + On en déduit la quantité en 2019 + \[ + 138 * 0.9009 = 124.3242 + \] + \item L'équation de la droite est + \[ + y = 2.0 x -3 + \] + \item Il faut penser à faire la division à par $8$ avant d'utiliser le log car sinon, on ne peut pas utiliser la formule $\log(a^n) = n\times \log(a)$. + + \[x = \frac{\log(4.38)}{\log(0.17)}\] + \end{enumerate} +\end{solution} + +\begin{exercise}[subtitle={Restaurant}] + Un \emph{food truck}, ouvert le midi et le soir, propose deux types de formules : + + \setlength\parindent{10mm} + \begin{itemize} + \item la formule \emph{Burger} ; + \item la formule \emph{Wok}. + \end{itemize} + \setlength\parindent{0mm} + + \medskip + + Le gérant a remarqué que 62\,\% de ses ventes ont lieu le midi. Le quart des ventes du midi correspondent à la formule \emph{Burger}, alors que 28\,\% des ventes du soir correspondent à la formule \emph{Wok}. + + Le gérant se constitue un fichier en notant, pour chaque vente, la formule choisie et le moment de cette vente (midi ou soir). + + On prélève une fiche de façon équiprobable. On définit les quatre évènements suivants: + + \begin{enumerate} + \item $M$ : \og la fiche correspond à une vente du midi\fg{} ; + \item $S$ : \og la fiche correspond à une vente du soir\fg {}; + \item $W$ : \og la fiche correspond à une formule \emph{Wok} \fg{} ; + \item $B$ : \og la fiche correspond à une formule \emph{Burger} \fg. + \end{enumerate} + \setlength\parindent{0mm} + + \medskip + + \begin{enumerate} + \item Recopier puis compléter l'arbre pondéré + + \begin{center} + \begin{tikzpicture}[sloped] + \node {.} + child {node {$M$} + child {node {$W$} + edge from parent + node[above] {...} + } + child {node {$B$} + edge from parent + node[above] {...} + } + edge from parent + node[above] {...} + } + child[missing] {} + child { node {$S$} + child {node {$W$} + edge from parent + node[above] {...} + } + child {node {$B$} + edge from parent + node[above] {...} + } + edge from parent + node[above] {...} + } ; + \end{tikzpicture} + \end{center} + + \item Calculer la probabilité de l'évènement $M \cap W$. Interpréter ce résultat dans le contexte de l'exercice. + \item Montrer que la probabilité que la fiche choisie corresponde à une formule \emph{Burger} est égale à $0.4248$. + \item On a prélevé une fiche correspondant à la formule \emph{Burger}. Quelle est la probabilité, arrondie au millième, que la vente ait eu lieu le soir? + \end{enumerate} +\end{exercise} + +\begin{solution} + \begin{enumerate} + \item + \begin{center} + \begin{tikzpicture}[sloped] + \node {.} + child {node {$M$} + child {node {$W$} + edge from parent + node[above] {$0.75$} + } + child {node {$B$} + edge from parent + node[above] {$0.25$} + } + edge from parent + node[above] {$0.62$} + } + child[missing] {} + child { node {$S$} + child {node {$W$} + edge from parent + node[above] {$0.29$} + } + child {node {$B$} + edge from parent + node[above] {$0.71$} + } + edge from parent + node[above] {$0.38$} + } ; + \end{tikzpicture} + \end{center} + \item On calcule la probabilité que la vente soit un wok et ait eu lieu à midi + \[ P(M\cap W) = P(M) \times P_M(W) = 0.62 \times 0.75 = 0.465 \] + \item Probabilité que la vente soit un burger. + \[ + P(B) = P(M\cap B) + P(S\cap B) = 0.62 \times 0.75 + 0.38 \times 0.29 = 0.4248 + \] + \item On cherche à calculer la quantité $P_B(S)$. Pour cela on utilise la formule de Bayes + \[ + P_B(S) = \frac{P(B\cap S)}{P(B)} = \frac{P_S(B) \times P(S)}{P(B)} = \frac{0.71\times 0.38}{0.4248} = 0.6351224105461393 \approx 0.635 + \] + \end{enumerate} +\end{solution} + +\begin{exercise}[subtitle={Continent plastique}] + \textit{Les quantités évoqués dans cette exercice sont générés au hasard et sont donc complètement farfelus.} + \medskip + Le \og continent de plastique\fg{} est la plus grande des plaques de déchets plastiques évoluant sur les océans. Elle occupe actuellement dans l'océan Pacifique une surface dont l'aire est évaluée à plus de $1,6$ million de km$^2$, entre Hawaï et la Californie. + + En 2017, des scientifiques ont estimé qu'il y avait $4$ millions de tonnes de déchets plastiques qui était déversé chaque année dans les océans et que cette quantité augmentait de $30\n\%$ par chaque année. + + On modélise l'évolution de la masse de ces déchets plastiques déversée chaque année, si rien n'est fait pour la réduire, par une suite géométrique $\left(u_n\right)$. L'arrondi au centième du terme $u_n$ représente la masse de ces déchets déversée chaque année, exprimée en million de tonnes, pour l'année $(2017 + n)$. + + \medskip + + \begin{enumerate} + \item Expliquer pourquoi la suite $u_n$ est géométrique? + \item Calculer $u_1$ et $u_2$. + \item Exprimer $u_n$ en fonction de $n$. + \item Au début de l'année 2017, il y avait $300$ millions de tonnes de déchets plastique. Calculer la quantité totale de déchets plastiques en 2030. + \item On souhaite déterminer en quelle année la masse totale de ces déchets plastiques aura pour la première fois augmenté de $50$\,\% par rapport à sa valeur de 2017. + \begin{enumerate} + \item Recopier et compléter l'algorithme ci-dessous pour que la variable $N$ contienne la réponse au problème posé. + + \begin{center} + \begin{tabularx}{0.4\linewidth}{|X|}\hline + $N = 2017$\\ + $U = 4$ \\ + $S = 300 + U$ \\ + while $S < 450$: \\ + \hspace{1cm} $N = \ldots$\\ + \hspace{1cm} $U = \ldots$\\ + \hspace{1cm} $S = \ldots$\\ + \hline + \end{tabularx} + \end{center} + \item Que contiennent les variables $S$, $U$ et $N$ après exécution de cet algorithme ? + + Interpréter les résultats dans le contexte de l'exercice. + \end{enumerate} + \end{enumerate} +\end{exercise} + +\begin{solution} + \begin{enumerate} + \item Une augmentation de $30\,\%$ revient à multiplier la quantité par $1.3$. La suite est donc bien géométrique. Son premier terme est $u_0 = 4$ et sa raison est $q = 1.3$ + \item + \[ + u_1 = u_0 * 1.3 = 5.2 + \] + \[ + u_2 = u_0 * 1.3^2 = 6.76 + \] + \item + \[ + u_n = u_0 \times q^n = 4 \times 1.3^n + \] + \item On calcule la quantité totale déversée entre 2017 et 2030. + \[ + \sum_{n = 0}^{13} u_n = u_0 \times \frac{1-q^{13}}{1-q} = 4 \times \frac{1 - 1.3^{13}}{1 - 1.3} = 390.5 + \] + On en déduit la quantité totale de déchets en 2030 + \[ + 300 + 390.5 = 690.5 + \] + \item + \begin{enumerate} + \item ~ + \begin{center} + \begin{tabularx}{0.4\linewidth}{|X|}\hline + $N \gets 2017$\\ + $U \gets 4$ \\ + $S \gets 300 + U$ \\ + Tant que $S < 450$ \\ + \hspace{1cm} $N \gets N + 1$\\ + \hspace{1cm} $U \gets U * 1.3$\\ + \hspace{1cm} $S \gets S + u$\\ + Fin Tant que\\\hline + \end{tabularx} + \end{center} + \item \textit{Pas de correction automatisé} + \end{enumerate} + \end{enumerate} +\end{solution} + +\end{document} + +%%% Local Variables: +%%% mode: latex +%%% TeX-master: "master" +%%% End: diff --git a/TST/DS/DS_21_04_07/TST1/18_210407_DS8.tex b/TST/DS/DS_21_04_07/TST1/18_210407_DS8.tex new file mode 100644 index 0000000..c9d3244 --- /dev/null +++ b/TST/DS/DS_21_04_07/TST1/18_210407_DS8.tex @@ -0,0 +1,262 @@ +\documentclass[a4paper,10pt]{article} +\usepackage{myXsim} + +% Title Page +\title{DS8 \hfill MOUFAQ Amine} +\tribe{TST} +\date{\hfillÀ render pour le Mercredi 7 avril} + +\xsimsetup{ + solution/print = false +} + +\begin{document} +\maketitle + +\begin{exercise}[subtitle={Automatismes}] + \textit{Toutes les questions de cette exercice sont indépendantes et peuvent être répondus séparément} + \begin{enumerate} + \item De janvier à septembre, une quantité a augmenté de $16\,\%$. Faire un schéma pour représenter la situation puis calculer le taux d'évolution moyen mensuel. + \item Une quantité augmente de $16\,\%$ par ans. En 2020, elle est de 115\euro. Quelle était sa valeur en 2019? Faire un schéma pour représenter la situation. + \item Déterminer l'équation de la droite \\ + \begin{tikzpicture}[xscale=0.8, yscale=0.5] + \tkzInit[xmin=-5,xmax=5,xstep=1, + ymin=-5,ymax=5,ystep=1] + \tkzGrid + \tkzAxeXY + \tkzFct[domain=-5:5,color=red,very thick]% + {0.5*\x -1}; + \end{tikzpicture} + \item Résoudre l'équation $6 \times 0.24^x = 41$ + \end{enumerate} +\end{exercise} + +\begin{solution} + \begin{enumerate} + \item On veut partager cette évolution en 8 évolutions. + \[ + \left(1 + \frac{16}{100}\right)^{\frac{1}{8}} = 1.0187 + \] + Donc le taux d'évolution moyen est + \[ + t_m = 1.0187 - 1 = 0.01869999999999994 + \] + \item Coefficient multiplicateur pour revenir en arrière + \[ + CM = (1 + \frac{16}{100})^{-1} = 0.8621 + \] + On en déduit la quantité en 2019 + \[ + 115 * 0.8621 = 99.1415 + \] + \item L'équation de la droite est + \[ + y = 0.5 x -1 + \] + \item Il faut penser à faire la division à par $6$ avant d'utiliser le log car sinon, on ne peut pas utiliser la formule $\log(a^n) = n\times \log(a)$. + + \[x = \frac{\log(6.83)}{\log(0.24)}\] + \end{enumerate} +\end{solution} + +\begin{exercise}[subtitle={Restaurant}] + Un \emph{food truck}, ouvert le midi et le soir, propose deux types de formules : + + \setlength\parindent{10mm} + \begin{itemize} + \item la formule \emph{Burger} ; + \item la formule \emph{Wok}. + \end{itemize} + \setlength\parindent{0mm} + + \medskip + + Le gérant a remarqué que 78\,\% de ses ventes ont lieu le midi. Le quart des ventes du midi correspondent à la formule \emph{Burger}, alors que 37\,\% des ventes du soir correspondent à la formule \emph{Wok}. + + Le gérant se constitue un fichier en notant, pour chaque vente, la formule choisie et le moment de cette vente (midi ou soir). + + On prélève une fiche de façon équiprobable. On définit les quatre évènements suivants: + + \begin{enumerate} + \item $M$ : \og la fiche correspond à une vente du midi\fg{} ; + \item $S$ : \og la fiche correspond à une vente du soir\fg {}; + \item $W$ : \og la fiche correspond à une formule \emph{Wok} \fg{} ; + \item $B$ : \og la fiche correspond à une formule \emph{Burger} \fg. + \end{enumerate} + \setlength\parindent{0mm} + + \medskip + + \begin{enumerate} + \item Recopier puis compléter l'arbre pondéré + + \begin{center} + \begin{tikzpicture}[sloped] + \node {.} + child {node {$M$} + child {node {$W$} + edge from parent + node[above] {...} + } + child {node {$B$} + edge from parent + node[above] {...} + } + edge from parent + node[above] {...} + } + child[missing] {} + child { node {$S$} + child {node {$W$} + edge from parent + node[above] {...} + } + child {node {$B$} + edge from parent + node[above] {...} + } + edge from parent + node[above] {...} + } ; + \end{tikzpicture} + \end{center} + + \item Calculer la probabilité de l'évènement $M \cap W$. Interpréter ce résultat dans le contexte de l'exercice. + \item Montrer que la probabilité que la fiche choisie corresponde à une formule \emph{Burger} est égale à $0.3336$. + \item On a prélevé une fiche correspondant à la formule \emph{Burger}. Quelle est la probabilité, arrondie au millième, que la vente ait eu lieu le soir? + \end{enumerate} +\end{exercise} + +\begin{solution} + \begin{enumerate} + \item + \begin{center} + \begin{tikzpicture}[sloped] + \node {.} + child {node {$M$} + child {node {$W$} + edge from parent + node[above] {$0.75$} + } + child {node {$B$} + edge from parent + node[above] {$0.25$} + } + edge from parent + node[above] {$0.78$} + } + child[missing] {} + child { node {$S$} + child {node {$W$} + edge from parent + node[above] {$0.37$} + } + child {node {$B$} + edge from parent + node[above] {$0.63$} + } + edge from parent + node[above] {$0.22$} + } ; + \end{tikzpicture} + \end{center} + \item On calcule la probabilité que la vente soit un wok et ait eu lieu à midi + \[ P(M\cap W) = P(M) \times P_M(W) = 0.78 \times 0.75 = 0.585 \] + \item Probabilité que la vente soit un burger. + \[ + P(B) = P(M\cap B) + P(S\cap B) = 0.78 \times 0.75 + 0.22 \times 0.37 = 0.3336 + \] + \item On cherche à calculer la quantité $P_B(S)$. Pour cela on utilise la formule de Bayes + \[ + P_B(S) = \frac{P(B\cap S)}{P(B)} = \frac{P_S(B) \times P(S)}{P(B)} = \frac{0.63\times 0.22}{0.3336} = 0.41546762589928055 \approx 0.415 + \] + \end{enumerate} +\end{solution} + +\begin{exercise}[subtitle={Continent plastique}] + \textit{Les quantités évoqués dans cette exercice sont générés au hasard et sont donc complètement farfelus.} + \medskip + Le \og continent de plastique\fg{} est la plus grande des plaques de déchets plastiques évoluant sur les océans. Elle occupe actuellement dans l'océan Pacifique une surface dont l'aire est évaluée à plus de $1,6$ million de km$^2$, entre Hawaï et la Californie. + + En 2017, des scientifiques ont estimé qu'il y avait $10$ millions de tonnes de déchets plastiques qui était déversé chaque année dans les océans et que cette quantité augmentait de $24\n\%$ par chaque année. + + On modélise l'évolution de la masse de ces déchets plastiques déversée chaque année, si rien n'est fait pour la réduire, par une suite géométrique $\left(u_n\right)$. L'arrondi au centième du terme $u_n$ représente la masse de ces déchets déversée chaque année, exprimée en million de tonnes, pour l'année $(2017 + n)$. + + \medskip + + \begin{enumerate} + \item Expliquer pourquoi la suite $u_n$ est géométrique? + \item Calculer $u_1$ et $u_2$. + \item Exprimer $u_n$ en fonction de $n$. + \item Au début de l'année 2017, il y avait $300$ millions de tonnes de déchets plastique. Calculer la quantité totale de déchets plastiques en 2030. + \item On souhaite déterminer en quelle année la masse totale de ces déchets plastiques aura pour la première fois augmenté de $50$\,\% par rapport à sa valeur de 2017. + \begin{enumerate} + \item Recopier et compléter l'algorithme ci-dessous pour que la variable $N$ contienne la réponse au problème posé. + + \begin{center} + \begin{tabularx}{0.4\linewidth}{|X|}\hline + $N = 2017$\\ + $U = 10$ \\ + $S = 300 + U$ \\ + while $S < 450$: \\ + \hspace{1cm} $N = \ldots$\\ + \hspace{1cm} $U = \ldots$\\ + \hspace{1cm} $S = \ldots$\\ + \hline + \end{tabularx} + \end{center} + \item Que contiennent les variables $S$, $U$ et $N$ après exécution de cet algorithme ? + + Interpréter les résultats dans le contexte de l'exercice. + \end{enumerate} + \end{enumerate} +\end{exercise} + +\begin{solution} + \begin{enumerate} + \item Une augmentation de $24\,\%$ revient à multiplier la quantité par $1.24$. La suite est donc bien géométrique. Son premier terme est $u_0 = 10$ et sa raison est $q = 1.24$ + \item + \[ + u_1 = u_0 * 1.24 = 12.4 + \] + \[ + u_2 = u_0 * 1.24^2 = 15.376 + \] + \item + \[ + u_n = u_0 \times q^n = 10 \times 1.24^n + \] + \item On calcule la quantité totale déversée entre 2017 et 2030. + \[ + \sum_{n = 0}^{13} u_n = u_0 \times \frac{1-q^{13}}{1-q} = 10 \times \frac{1 - 1.24^{13}}{1 - 1.24} = 641.1 + \] + On en déduit la quantité totale de déchets en 2030 + \[ + 300 + 641.1 = 941.1 + \] + \item + \begin{enumerate} + \item ~ + \begin{center} + \begin{tabularx}{0.4\linewidth}{|X|}\hline + $N \gets 2017$\\ + $U \gets 10$ \\ + $S \gets 300 + U$ \\ + Tant que $S < 450$ \\ + \hspace{1cm} $N \gets N + 1$\\ + \hspace{1cm} $U \gets U * 1.24$\\ + \hspace{1cm} $S \gets S + u$\\ + Fin Tant que\\\hline + \end{tabularx} + \end{center} + \item \textit{Pas de correction automatisé} + \end{enumerate} + \end{enumerate} +\end{solution} + +\end{document} + +%%% Local Variables: +%%% mode: latex +%%% TeX-master: "master" +%%% End: diff --git a/TST/DS/DS_21_04_07/TST1/19_210407_DS8.tex b/TST/DS/DS_21_04_07/TST1/19_210407_DS8.tex new file mode 100644 index 0000000..38bc7c9 --- /dev/null +++ b/TST/DS/DS_21_04_07/TST1/19_210407_DS8.tex @@ -0,0 +1,262 @@ +\documentclass[a4paper,10pt]{article} +\usepackage{myXsim} + +% Title Page +\title{DS8 \hfill ONAL Yakub} +\tribe{TST} +\date{\hfillÀ render pour le Mercredi 7 avril} + +\xsimsetup{ + solution/print = false +} + +\begin{document} +\maketitle + +\begin{exercise}[subtitle={Automatismes}] + \textit{Toutes les questions de cette exercice sont indépendantes et peuvent être répondus séparément} + \begin{enumerate} + \item De janvier à septembre, une quantité a augmenté de $27\,\%$. Faire un schéma pour représenter la situation puis calculer le taux d'évolution moyen mensuel. + \item Une quantité augmente de $27\,\%$ par ans. En 2020, elle est de 132\euro. Quelle était sa valeur en 2019? Faire un schéma pour représenter la situation. + \item Déterminer l'équation de la droite \\ + \begin{tikzpicture}[xscale=0.8, yscale=0.5] + \tkzInit[xmin=-5,xmax=5,xstep=1, + ymin=-5,ymax=5,ystep=1] + \tkzGrid + \tkzAxeXY + \tkzFct[domain=-5:5,color=red,very thick]% + {1.0*\x -1}; + \end{tikzpicture} + \item Résoudre l'équation $10 \times 0.18^x = 7$ + \end{enumerate} +\end{exercise} + +\begin{solution} + \begin{enumerate} + \item On veut partager cette évolution en 8 évolutions. + \[ + \left(1 + \frac{27}{100}\right)^{\frac{1}{8}} = 1.0303 + \] + Donc le taux d'évolution moyen est + \[ + t_m = 1.0303 - 1 = 0.030299999999999994 + \] + \item Coefficient multiplicateur pour revenir en arrière + \[ + CM = (1 + \frac{27}{100})^{-1} = 0.7874 + \] + On en déduit la quantité en 2019 + \[ + 132 * 0.7874 = 103.9368 + \] + \item L'équation de la droite est + \[ + y = 1.0 x -1 + \] + \item Il faut penser à faire la division à par $10$ avant d'utiliser le log car sinon, on ne peut pas utiliser la formule $\log(a^n) = n\times \log(a)$. + + \[x = \frac{\log(0.7)}{\log(0.18)}\] + \end{enumerate} +\end{solution} + +\begin{exercise}[subtitle={Restaurant}] + Un \emph{food truck}, ouvert le midi et le soir, propose deux types de formules : + + \setlength\parindent{10mm} + \begin{itemize} + \item la formule \emph{Burger} ; + \item la formule \emph{Wok}. + \end{itemize} + \setlength\parindent{0mm} + + \medskip + + Le gérant a remarqué que 78\,\% de ses ventes ont lieu le midi. Le quart des ventes du midi correspondent à la formule \emph{Burger}, alors que 28\,\% des ventes du soir correspondent à la formule \emph{Wok}. + + Le gérant se constitue un fichier en notant, pour chaque vente, la formule choisie et le moment de cette vente (midi ou soir). + + On prélève une fiche de façon équiprobable. On définit les quatre évènements suivants: + + \begin{enumerate} + \item $M$ : \og la fiche correspond à une vente du midi\fg{} ; + \item $S$ : \og la fiche correspond à une vente du soir\fg {}; + \item $W$ : \og la fiche correspond à une formule \emph{Wok} \fg{} ; + \item $B$ : \og la fiche correspond à une formule \emph{Burger} \fg. + \end{enumerate} + \setlength\parindent{0mm} + + \medskip + + \begin{enumerate} + \item Recopier puis compléter l'arbre pondéré + + \begin{center} + \begin{tikzpicture}[sloped] + \node {.} + child {node {$M$} + child {node {$W$} + edge from parent + node[above] {...} + } + child {node {$B$} + edge from parent + node[above] {...} + } + edge from parent + node[above] {...} + } + child[missing] {} + child { node {$S$} + child {node {$W$} + edge from parent + node[above] {...} + } + child {node {$B$} + edge from parent + node[above] {...} + } + edge from parent + node[above] {...} + } ; + \end{tikzpicture} + \end{center} + + \item Calculer la probabilité de l'évènement $M \cap W$. Interpréter ce résultat dans le contexte de l'exercice. + \item Montrer que la probabilité que la fiche choisie corresponde à une formule \emph{Burger} est égale à $0.3534$. + \item On a prélevé une fiche correspondant à la formule \emph{Burger}. Quelle est la probabilité, arrondie au millième, que la vente ait eu lieu le soir? + \end{enumerate} +\end{exercise} + +\begin{solution} + \begin{enumerate} + \item + \begin{center} + \begin{tikzpicture}[sloped] + \node {.} + child {node {$M$} + child {node {$W$} + edge from parent + node[above] {$0.75$} + } + child {node {$B$} + edge from parent + node[above] {$0.25$} + } + edge from parent + node[above] {$0.78$} + } + child[missing] {} + child { node {$S$} + child {node {$W$} + edge from parent + node[above] {$0.28$} + } + child {node {$B$} + edge from parent + node[above] {$0.72$} + } + edge from parent + node[above] {$0.22$} + } ; + \end{tikzpicture} + \end{center} + \item On calcule la probabilité que la vente soit un wok et ait eu lieu à midi + \[ P(M\cap W) = P(M) \times P_M(W) = 0.78 \times 0.75 = 0.585 \] + \item Probabilité que la vente soit un burger. + \[ + P(B) = P(M\cap B) + P(S\cap B) = 0.78 \times 0.75 + 0.22 \times 0.28 = 0.3534 + \] + \item On cherche à calculer la quantité $P_B(S)$. Pour cela on utilise la formule de Bayes + \[ + P_B(S) = \frac{P(B\cap S)}{P(B)} = \frac{P_S(B) \times P(S)}{P(B)} = \frac{0.72\times 0.22}{0.3534} = 0.4482173174872665 \approx 0.448 + \] + \end{enumerate} +\end{solution} + +\begin{exercise}[subtitle={Continent plastique}] + \textit{Les quantités évoqués dans cette exercice sont générés au hasard et sont donc complètement farfelus.} + \medskip + Le \og continent de plastique\fg{} est la plus grande des plaques de déchets plastiques évoluant sur les océans. Elle occupe actuellement dans l'océan Pacifique une surface dont l'aire est évaluée à plus de $1,6$ million de km$^2$, entre Hawaï et la Californie. + + En 2017, des scientifiques ont estimé qu'il y avait $3$ millions de tonnes de déchets plastiques qui était déversé chaque année dans les océans et que cette quantité augmentait de $21\n\%$ par chaque année. + + On modélise l'évolution de la masse de ces déchets plastiques déversée chaque année, si rien n'est fait pour la réduire, par une suite géométrique $\left(u_n\right)$. L'arrondi au centième du terme $u_n$ représente la masse de ces déchets déversée chaque année, exprimée en million de tonnes, pour l'année $(2017 + n)$. + + \medskip + + \begin{enumerate} + \item Expliquer pourquoi la suite $u_n$ est géométrique? + \item Calculer $u_1$ et $u_2$. + \item Exprimer $u_n$ en fonction de $n$. + \item Au début de l'année 2017, il y avait $300$ millions de tonnes de déchets plastique. Calculer la quantité totale de déchets plastiques en 2030. + \item On souhaite déterminer en quelle année la masse totale de ces déchets plastiques aura pour la première fois augmenté de $50$\,\% par rapport à sa valeur de 2017. + \begin{enumerate} + \item Recopier et compléter l'algorithme ci-dessous pour que la variable $N$ contienne la réponse au problème posé. + + \begin{center} + \begin{tabularx}{0.4\linewidth}{|X|}\hline + $N = 2017$\\ + $U = 3$ \\ + $S = 300 + U$ \\ + while $S < 450$: \\ + \hspace{1cm} $N = \ldots$\\ + \hspace{1cm} $U = \ldots$\\ + \hspace{1cm} $S = \ldots$\\ + \hline + \end{tabularx} + \end{center} + \item Que contiennent les variables $S$, $U$ et $N$ après exécution de cet algorithme ? + + Interpréter les résultats dans le contexte de l'exercice. + \end{enumerate} + \end{enumerate} +\end{exercise} + +\begin{solution} + \begin{enumerate} + \item Une augmentation de $21\,\%$ revient à multiplier la quantité par $1.21$. La suite est donc bien géométrique. Son premier terme est $u_0 = 3$ et sa raison est $q = 1.21$ + \item + \[ + u_1 = u_0 * 1.21 = 3.63 + \] + \[ + u_2 = u_0 * 1.21^2 = 4.3923 + \] + \item + \[ + u_n = u_0 \times q^n = 3 \times 1.21^n + \] + \item On calcule la quantité totale déversée entre 2017 et 2030. + \[ + \sum_{n = 0}^{13} u_n = u_0 \times \frac{1-q^{13}}{1-q} = 3 \times \frac{1 - 1.21^{13}}{1 - 1.21} = 155.97 + \] + On en déduit la quantité totale de déchets en 2030 + \[ + 300 + 155.97 = 455.97 + \] + \item + \begin{enumerate} + \item ~ + \begin{center} + \begin{tabularx}{0.4\linewidth}{|X|}\hline + $N \gets 2017$\\ + $U \gets 3$ \\ + $S \gets 300 + U$ \\ + Tant que $S < 450$ \\ + \hspace{1cm} $N \gets N + 1$\\ + \hspace{1cm} $U \gets U * 1.21$\\ + \hspace{1cm} $S \gets S + u$\\ + Fin Tant que\\\hline + \end{tabularx} + \end{center} + \item \textit{Pas de correction automatisé} + \end{enumerate} + \end{enumerate} +\end{solution} + +\end{document} + +%%% Local Variables: +%%% mode: latex +%%% TeX-master: "master" +%%% End: diff --git a/TST/DS/DS_21_04_07/TST1/20_210407_DS8.tex b/TST/DS/DS_21_04_07/TST1/20_210407_DS8.tex new file mode 100644 index 0000000..f1623e4 --- /dev/null +++ b/TST/DS/DS_21_04_07/TST1/20_210407_DS8.tex @@ -0,0 +1,262 @@ +\documentclass[a4paper,10pt]{article} +\usepackage{myXsim} + +% Title Page +\title{DS8 \hfill SORIANO Laura} +\tribe{TST} +\date{\hfillÀ render pour le Mercredi 7 avril} + +\xsimsetup{ + solution/print = false +} + +\begin{document} +\maketitle + +\begin{exercise}[subtitle={Automatismes}] + \textit{Toutes les questions de cette exercice sont indépendantes et peuvent être répondus séparément} + \begin{enumerate} + \item De janvier à septembre, une quantité a augmenté de $26\,\%$. Faire un schéma pour représenter la situation puis calculer le taux d'évolution moyen mensuel. + \item Une quantité augmente de $26\,\%$ par ans. En 2020, elle est de 122\euro. Quelle était sa valeur en 2019? Faire un schéma pour représenter la situation. + \item Déterminer l'équation de la droite \\ + \begin{tikzpicture}[xscale=0.8, yscale=0.5] + \tkzInit[xmin=-5,xmax=5,xstep=1, + ymin=-5,ymax=5,ystep=1] + \tkzGrid + \tkzAxeXY + \tkzFct[domain=-5:5,color=red,very thick]% + {3.0*\x -3}; + \end{tikzpicture} + \item Résoudre l'équation $2 \times 0.02^x = 10$ + \end{enumerate} +\end{exercise} + +\begin{solution} + \begin{enumerate} + \item On veut partager cette évolution en 8 évolutions. + \[ + \left(1 + \frac{26}{100}\right)^{\frac{1}{8}} = 1.0293 + \] + Donc le taux d'évolution moyen est + \[ + t_m = 1.0293 - 1 = 0.029300000000000104 + \] + \item Coefficient multiplicateur pour revenir en arrière + \[ + CM = (1 + \frac{26}{100})^{-1} = 0.7937 + \] + On en déduit la quantité en 2019 + \[ + 122 * 0.7937 = 96.8314 + \] + \item L'équation de la droite est + \[ + y = 3.0 x -3 + \] + \item Il faut penser à faire la division à par $2$ avant d'utiliser le log car sinon, on ne peut pas utiliser la formule $\log(a^n) = n\times \log(a)$. + + \[x = \frac{\log(5.0)}{\log(0.02)}\] + \end{enumerate} +\end{solution} + +\begin{exercise}[subtitle={Restaurant}] + Un \emph{food truck}, ouvert le midi et le soir, propose deux types de formules : + + \setlength\parindent{10mm} + \begin{itemize} + \item la formule \emph{Burger} ; + \item la formule \emph{Wok}. + \end{itemize} + \setlength\parindent{0mm} + + \medskip + + Le gérant a remarqué que 0\,\% de ses ventes ont lieu le midi. Le quart des ventes du midi correspondent à la formule \emph{Burger}, alors que 21\,\% des ventes du soir correspondent à la formule \emph{Wok}. + + Le gérant se constitue un fichier en notant, pour chaque vente, la formule choisie et le moment de cette vente (midi ou soir). + + On prélève une fiche de façon équiprobable. On définit les quatre évènements suivants: + + \begin{enumerate} + \item $M$ : \og la fiche correspond à une vente du midi\fg{} ; + \item $S$ : \og la fiche correspond à une vente du soir\fg {}; + \item $W$ : \og la fiche correspond à une formule \emph{Wok} \fg{} ; + \item $B$ : \og la fiche correspond à une formule \emph{Burger} \fg. + \end{enumerate} + \setlength\parindent{0mm} + + \medskip + + \begin{enumerate} + \item Recopier puis compléter l'arbre pondéré + + \begin{center} + \begin{tikzpicture}[sloped] + \node {.} + child {node {$M$} + child {node {$W$} + edge from parent + node[above] {...} + } + child {node {$B$} + edge from parent + node[above] {...} + } + edge from parent + node[above] {...} + } + child[missing] {} + child { node {$S$} + child {node {$W$} + edge from parent + node[above] {...} + } + child {node {$B$} + edge from parent + node[above] {...} + } + edge from parent + node[above] {...} + } ; + \end{tikzpicture} + \end{center} + + \item Calculer la probabilité de l'évènement $M \cap W$. Interpréter ce résultat dans le contexte de l'exercice. + \item Montrer que la probabilité que la fiche choisie corresponde à une formule \emph{Burger} est égale à $0.79$. + \item On a prélevé une fiche correspondant à la formule \emph{Burger}. Quelle est la probabilité, arrondie au millième, que la vente ait eu lieu le soir? + \end{enumerate} +\end{exercise} + +\begin{solution} + \begin{enumerate} + \item + \begin{center} + \begin{tikzpicture}[sloped] + \node {.} + child {node {$M$} + child {node {$W$} + edge from parent + node[above] {$0.75$} + } + child {node {$B$} + edge from parent + node[above] {$0.25$} + } + edge from parent + node[above] {$0.0$} + } + child[missing] {} + child { node {$S$} + child {node {$W$} + edge from parent + node[above] {$0.21$} + } + child {node {$B$} + edge from parent + node[above] {$0.79$} + } + edge from parent + node[above] {$1.0$} + } ; + \end{tikzpicture} + \end{center} + \item On calcule la probabilité que la vente soit un wok et ait eu lieu à midi + \[ P(M\cap W) = P(M) \times P_M(W) = 0.0 \times 0.75 = 0.0 \] + \item Probabilité que la vente soit un burger. + \[ + P(B) = P(M\cap B) + P(S\cap B) = 0.0 \times 0.75 + 1.0 \times 0.21 = 0.79 + \] + \item On cherche à calculer la quantité $P_B(S)$. Pour cela on utilise la formule de Bayes + \[ + P_B(S) = \frac{P(B\cap S)}{P(B)} = \frac{P_S(B) \times P(S)}{P(B)} = \frac{0.79\times 1.0}{0.79} = 1.0 \approx 1.0 + \] + \end{enumerate} +\end{solution} + +\begin{exercise}[subtitle={Continent plastique}] + \textit{Les quantités évoqués dans cette exercice sont générés au hasard et sont donc complètement farfelus.} + \medskip + Le \og continent de plastique\fg{} est la plus grande des plaques de déchets plastiques évoluant sur les océans. Elle occupe actuellement dans l'océan Pacifique une surface dont l'aire est évaluée à plus de $1,6$ million de km$^2$, entre Hawaï et la Californie. + + En 2017, des scientifiques ont estimé qu'il y avait $11$ millions de tonnes de déchets plastiques qui était déversé chaque année dans les océans et que cette quantité augmentait de $16\n\%$ par chaque année. + + On modélise l'évolution de la masse de ces déchets plastiques déversée chaque année, si rien n'est fait pour la réduire, par une suite géométrique $\left(u_n\right)$. L'arrondi au centième du terme $u_n$ représente la masse de ces déchets déversée chaque année, exprimée en million de tonnes, pour l'année $(2017 + n)$. + + \medskip + + \begin{enumerate} + \item Expliquer pourquoi la suite $u_n$ est géométrique? + \item Calculer $u_1$ et $u_2$. + \item Exprimer $u_n$ en fonction de $n$. + \item Au début de l'année 2017, il y avait $300$ millions de tonnes de déchets plastique. Calculer la quantité totale de déchets plastiques en 2030. + \item On souhaite déterminer en quelle année la masse totale de ces déchets plastiques aura pour la première fois augmenté de $50$\,\% par rapport à sa valeur de 2017. + \begin{enumerate} + \item Recopier et compléter l'algorithme ci-dessous pour que la variable $N$ contienne la réponse au problème posé. + + \begin{center} + \begin{tabularx}{0.4\linewidth}{|X|}\hline + $N = 2017$\\ + $U = 11$ \\ + $S = 300 + U$ \\ + while $S < 450$: \\ + \hspace{1cm} $N = \ldots$\\ + \hspace{1cm} $U = \ldots$\\ + \hspace{1cm} $S = \ldots$\\ + \hline + \end{tabularx} + \end{center} + \item Que contiennent les variables $S$, $U$ et $N$ après exécution de cet algorithme ? + + Interpréter les résultats dans le contexte de l'exercice. + \end{enumerate} + \end{enumerate} +\end{exercise} + +\begin{solution} + \begin{enumerate} + \item Une augmentation de $16\,\%$ revient à multiplier la quantité par $1.16$. La suite est donc bien géométrique. Son premier terme est $u_0 = 11$ et sa raison est $q = 1.16$ + \item + \[ + u_1 = u_0 * 1.16 = 12.76 + \] + \[ + u_2 = u_0 * 1.16^2 = 14.8016 + \] + \item + \[ + u_n = u_0 \times q^n = 11 \times 1.16^n + \] + \item On calcule la quantité totale déversée entre 2017 et 2030. + \[ + \sum_{n = 0}^{13} u_n = u_0 \times \frac{1-q^{13}}{1-q} = 11 \times \frac{1 - 1.16^{13}}{1 - 1.16} = 404.65 + \] + On en déduit la quantité totale de déchets en 2030 + \[ + 300 + 404.65 = 704.65 + \] + \item + \begin{enumerate} + \item ~ + \begin{center} + \begin{tabularx}{0.4\linewidth}{|X|}\hline + $N \gets 2017$\\ + $U \gets 11$ \\ + $S \gets 300 + U$ \\ + Tant que $S < 450$ \\ + \hspace{1cm} $N \gets N + 1$\\ + \hspace{1cm} $U \gets U * 1.16$\\ + \hspace{1cm} $S \gets S + u$\\ + Fin Tant que\\\hline + \end{tabularx} + \end{center} + \item \textit{Pas de correction automatisé} + \end{enumerate} + \end{enumerate} +\end{solution} + +\end{document} + +%%% Local Variables: +%%% mode: latex +%%% TeX-master: "master" +%%% End: diff --git a/TST/DS/DS_21_04_07/TST1/21_210407_DS8.tex b/TST/DS/DS_21_04_07/TST1/21_210407_DS8.tex new file mode 100644 index 0000000..5611692 --- /dev/null +++ b/TST/DS/DS_21_04_07/TST1/21_210407_DS8.tex @@ -0,0 +1,262 @@ +\documentclass[a4paper,10pt]{article} +\usepackage{myXsim} + +% Title Page +\title{DS8 \hfill VECCHIO Léa} +\tribe{TST} +\date{\hfillÀ render pour le Mercredi 7 avril} + +\xsimsetup{ + solution/print = false +} + +\begin{document} +\maketitle + +\begin{exercise}[subtitle={Automatismes}] + \textit{Toutes les questions de cette exercice sont indépendantes et peuvent être répondus séparément} + \begin{enumerate} + \item De janvier à septembre, une quantité a augmenté de $28\,\%$. Faire un schéma pour représenter la situation puis calculer le taux d'évolution moyen mensuel. + \item Une quantité augmente de $28\,\%$ par ans. En 2020, elle est de 113\euro. Quelle était sa valeur en 2019? Faire un schéma pour représenter la situation. + \item Déterminer l'équation de la droite \\ + \begin{tikzpicture}[xscale=0.8, yscale=0.5] + \tkzInit[xmin=-5,xmax=5,xstep=1, + ymin=-5,ymax=5,ystep=1] + \tkzGrid + \tkzAxeXY + \tkzFct[domain=-5:5,color=red,very thick]% + {2.0*\x -2}; + \end{tikzpicture} + \item Résoudre l'équation $8 \times 0.84^x = 26$ + \end{enumerate} +\end{exercise} + +\begin{solution} + \begin{enumerate} + \item On veut partager cette évolution en 8 évolutions. + \[ + \left(1 + \frac{28}{100}\right)^{\frac{1}{8}} = 1.0313 + \] + Donc le taux d'évolution moyen est + \[ + t_m = 1.0313 - 1 = 0.031300000000000106 + \] + \item Coefficient multiplicateur pour revenir en arrière + \[ + CM = (1 + \frac{28}{100})^{-1} = 0.7812 + \] + On en déduit la quantité en 2019 + \[ + 113 * 0.7812 = 88.2756 + \] + \item L'équation de la droite est + \[ + y = 2.0 x -2 + \] + \item Il faut penser à faire la division à par $8$ avant d'utiliser le log car sinon, on ne peut pas utiliser la formule $\log(a^n) = n\times \log(a)$. + + \[x = \frac{\log(3.25)}{\log(0.84)}\] + \end{enumerate} +\end{solution} + +\begin{exercise}[subtitle={Restaurant}] + Un \emph{food truck}, ouvert le midi et le soir, propose deux types de formules : + + \setlength\parindent{10mm} + \begin{itemize} + \item la formule \emph{Burger} ; + \item la formule \emph{Wok}. + \end{itemize} + \setlength\parindent{0mm} + + \medskip + + Le gérant a remarqué que 18\,\% de ses ventes ont lieu le midi. Le quart des ventes du midi correspondent à la formule \emph{Burger}, alors que 16\,\% des ventes du soir correspondent à la formule \emph{Wok}. + + Le gérant se constitue un fichier en notant, pour chaque vente, la formule choisie et le moment de cette vente (midi ou soir). + + On prélève une fiche de façon équiprobable. On définit les quatre évènements suivants: + + \begin{enumerate} + \item $M$ : \og la fiche correspond à une vente du midi\fg{} ; + \item $S$ : \og la fiche correspond à une vente du soir\fg {}; + \item $W$ : \og la fiche correspond à une formule \emph{Wok} \fg{} ; + \item $B$ : \og la fiche correspond à une formule \emph{Burger} \fg. + \end{enumerate} + \setlength\parindent{0mm} + + \medskip + + \begin{enumerate} + \item Recopier puis compléter l'arbre pondéré + + \begin{center} + \begin{tikzpicture}[sloped] + \node {.} + child {node {$M$} + child {node {$W$} + edge from parent + node[above] {...} + } + child {node {$B$} + edge from parent + node[above] {...} + } + edge from parent + node[above] {...} + } + child[missing] {} + child { node {$S$} + child {node {$W$} + edge from parent + node[above] {...} + } + child {node {$B$} + edge from parent + node[above] {...} + } + edge from parent + node[above] {...} + } ; + \end{tikzpicture} + \end{center} + + \item Calculer la probabilité de l'évènement $M \cap W$. Interpréter ce résultat dans le contexte de l'exercice. + \item Montrer que la probabilité que la fiche choisie corresponde à une formule \emph{Burger} est égale à $0.7338$. + \item On a prélevé une fiche correspondant à la formule \emph{Burger}. Quelle est la probabilité, arrondie au millième, que la vente ait eu lieu le soir? + \end{enumerate} +\end{exercise} + +\begin{solution} + \begin{enumerate} + \item + \begin{center} + \begin{tikzpicture}[sloped] + \node {.} + child {node {$M$} + child {node {$W$} + edge from parent + node[above] {$0.75$} + } + child {node {$B$} + edge from parent + node[above] {$0.25$} + } + edge from parent + node[above] {$0.18$} + } + child[missing] {} + child { node {$S$} + child {node {$W$} + edge from parent + node[above] {$0.16$} + } + child {node {$B$} + edge from parent + node[above] {$0.84$} + } + edge from parent + node[above] {$0.82$} + } ; + \end{tikzpicture} + \end{center} + \item On calcule la probabilité que la vente soit un wok et ait eu lieu à midi + \[ P(M\cap W) = P(M) \times P_M(W) = 0.18 \times 0.75 = 0.135 \] + \item Probabilité que la vente soit un burger. + \[ + P(B) = P(M\cap B) + P(S\cap B) = 0.18 \times 0.75 + 0.82 \times 0.16 = 0.7338 + \] + \item On cherche à calculer la quantité $P_B(S)$. Pour cela on utilise la formule de Bayes + \[ + P_B(S) = \frac{P(B\cap S)}{P(B)} = \frac{P_S(B) \times P(S)}{P(B)} = \frac{0.84\times 0.82}{0.7338} = 0.9386753883892068 \approx 0.939 + \] + \end{enumerate} +\end{solution} + +\begin{exercise}[subtitle={Continent plastique}] + \textit{Les quantités évoqués dans cette exercice sont générés au hasard et sont donc complètement farfelus.} + \medskip + Le \og continent de plastique\fg{} est la plus grande des plaques de déchets plastiques évoluant sur les océans. Elle occupe actuellement dans l'océan Pacifique une surface dont l'aire est évaluée à plus de $1,6$ million de km$^2$, entre Hawaï et la Californie. + + En 2017, des scientifiques ont estimé qu'il y avait $2$ millions de tonnes de déchets plastiques qui était déversé chaque année dans les océans et que cette quantité augmentait de $24\n\%$ par chaque année. + + On modélise l'évolution de la masse de ces déchets plastiques déversée chaque année, si rien n'est fait pour la réduire, par une suite géométrique $\left(u_n\right)$. L'arrondi au centième du terme $u_n$ représente la masse de ces déchets déversée chaque année, exprimée en million de tonnes, pour l'année $(2017 + n)$. + + \medskip + + \begin{enumerate} + \item Expliquer pourquoi la suite $u_n$ est géométrique? + \item Calculer $u_1$ et $u_2$. + \item Exprimer $u_n$ en fonction de $n$. + \item Au début de l'année 2017, il y avait $300$ millions de tonnes de déchets plastique. Calculer la quantité totale de déchets plastiques en 2030. + \item On souhaite déterminer en quelle année la masse totale de ces déchets plastiques aura pour la première fois augmenté de $50$\,\% par rapport à sa valeur de 2017. + \begin{enumerate} + \item Recopier et compléter l'algorithme ci-dessous pour que la variable $N$ contienne la réponse au problème posé. + + \begin{center} + \begin{tabularx}{0.4\linewidth}{|X|}\hline + $N = 2017$\\ + $U = 2$ \\ + $S = 300 + U$ \\ + while $S < 450$: \\ + \hspace{1cm} $N = \ldots$\\ + \hspace{1cm} $U = \ldots$\\ + \hspace{1cm} $S = \ldots$\\ + \hline + \end{tabularx} + \end{center} + \item Que contiennent les variables $S$, $U$ et $N$ après exécution de cet algorithme ? + + Interpréter les résultats dans le contexte de l'exercice. + \end{enumerate} + \end{enumerate} +\end{exercise} + +\begin{solution} + \begin{enumerate} + \item Une augmentation de $24\,\%$ revient à multiplier la quantité par $1.24$. La suite est donc bien géométrique. Son premier terme est $u_0 = 2$ et sa raison est $q = 1.24$ + \item + \[ + u_1 = u_0 * 1.24 = 2.48 + \] + \[ + u_2 = u_0 * 1.24^2 = 3.0752 + \] + \item + \[ + u_n = u_0 \times q^n = 2 \times 1.24^n + \] + \item On calcule la quantité totale déversée entre 2017 et 2030. + \[ + \sum_{n = 0}^{13} u_n = u_0 \times \frac{1-q^{13}}{1-q} = 2 \times \frac{1 - 1.24^{13}}{1 - 1.24} = 128.22 + \] + On en déduit la quantité totale de déchets en 2030 + \[ + 300 + 128.22 = 428.22 + \] + \item + \begin{enumerate} + \item ~ + \begin{center} + \begin{tabularx}{0.4\linewidth}{|X|}\hline + $N \gets 2017$\\ + $U \gets 2$ \\ + $S \gets 300 + U$ \\ + Tant que $S < 450$ \\ + \hspace{1cm} $N \gets N + 1$\\ + \hspace{1cm} $U \gets U * 1.24$\\ + \hspace{1cm} $S \gets S + u$\\ + Fin Tant que\\\hline + \end{tabularx} + \end{center} + \item \textit{Pas de correction automatisé} + \end{enumerate} + \end{enumerate} +\end{solution} + +\end{document} + +%%% Local Variables: +%%% mode: latex +%%% TeX-master: "master" +%%% End: diff --git a/TST/DS/DS_21_04_07/TST1/all_210407_DS8.pdf b/TST/DS/DS_21_04_07/TST1/all_210407_DS8.pdf new file mode 100644 index 0000000..9681f58 Binary files /dev/null and b/TST/DS/DS_21_04_07/TST1/all_210407_DS8.pdf differ diff --git a/TST/DS/DS_21_04_07/TST1/corr_01_210407_DS8.tex b/TST/DS/DS_21_04_07/TST1/corr_01_210407_DS8.tex new file mode 100644 index 0000000..30025b2 --- /dev/null +++ b/TST/DS/DS_21_04_07/TST1/corr_01_210407_DS8.tex @@ -0,0 +1,262 @@ +\documentclass[a4paper,10pt]{article} +\usepackage{myXsim} + +% Title Page +\title{DS8 \hfill AIOUAZ Ahmed} +\tribe{TST} +\date{\hfillÀ render pour le Mercredi 7 avril} + +\xsimsetup{ + solution/print = true +} + +\begin{document} +\maketitle + +\begin{exercise}[subtitle={Automatismes}] + \textit{Toutes les questions de cette exercice sont indépendantes et peuvent être répondus séparément} + \begin{enumerate} + \item De janvier à septembre, une quantité a augmenté de $20\,\%$. Faire un schéma pour représenter la situation puis calculer le taux d'évolution moyen mensuel. + \item Une quantité augmente de $20\,\%$ par ans. En 2020, elle est de 110\euro. Quelle était sa valeur en 2019? Faire un schéma pour représenter la situation. + \item Déterminer l'équation de la droite \\ + \begin{tikzpicture}[xscale=0.8, yscale=0.5] + \tkzInit[xmin=-5,xmax=5,xstep=1, + ymin=-5,ymax=5,ystep=1] + \tkzGrid + \tkzAxeXY + \tkzFct[domain=-5:5,color=red,very thick]% + {3.0*\x -3}; + \end{tikzpicture} + \item Résoudre l'équation $2 \times 0.07^x = 20$ + \end{enumerate} +\end{exercise} + +\begin{solution} + \begin{enumerate} + \item On veut partager cette évolution en 8 évolutions. + \[ + \left(1 + \frac{20}{100}\right)^{\frac{1}{8}} = 1.0231 + \] + Donc le taux d'évolution moyen est + \[ + t_m = 1.0231 - 1 = 0.0230999999999999 + \] + \item Coefficient multiplicateur pour revenir en arrière + \[ + CM = (1 + \frac{20}{100})^{-1} = 0.8333 + \] + On en déduit la quantité en 2019 + \[ + 110 * 0.8333 = 91.66300000000001 + \] + \item L'équation de la droite est + \[ + y = 3.0 x -3 + \] + \item Il faut penser à faire la division à par $2$ avant d'utiliser le log car sinon, on ne peut pas utiliser la formule $\log(a^n) = n\times \log(a)$. + + \[x = \frac{\log(10.0)}{\log(0.07)}\] + \end{enumerate} +\end{solution} + +\begin{exercise}[subtitle={Restaurant}] + Un \emph{food truck}, ouvert le midi et le soir, propose deux types de formules : + + \setlength\parindent{10mm} + \begin{itemize} + \item la formule \emph{Burger} ; + \item la formule \emph{Wok}. + \end{itemize} + \setlength\parindent{0mm} + + \medskip + + Le gérant a remarqué que 9\,\% de ses ventes ont lieu le midi. Le quart des ventes du midi correspondent à la formule \emph{Burger}, alors que 3\,\% des ventes du soir correspondent à la formule \emph{Wok}. + + Le gérant se constitue un fichier en notant, pour chaque vente, la formule choisie et le moment de cette vente (midi ou soir). + + On prélève une fiche de façon équiprobable. On définit les quatre évènements suivants: + + \begin{enumerate} + \item $M$ : \og la fiche correspond à une vente du midi\fg{} ; + \item $S$ : \og la fiche correspond à une vente du soir\fg {}; + \item $W$ : \og la fiche correspond à une formule \emph{Wok} \fg{} ; + \item $B$ : \og la fiche correspond à une formule \emph{Burger} \fg. + \end{enumerate} + \setlength\parindent{0mm} + + \medskip + + \begin{enumerate} + \item Recopier puis compléter l'arbre pondéré + + \begin{center} + \begin{tikzpicture}[sloped] + \node {.} + child {node {$M$} + child {node {$W$} + edge from parent + node[above] {...} + } + child {node {$B$} + edge from parent + node[above] {...} + } + edge from parent + node[above] {...} + } + child[missing] {} + child { node {$S$} + child {node {$W$} + edge from parent + node[above] {...} + } + child {node {$B$} + edge from parent + node[above] {...} + } + edge from parent + node[above] {...} + } ; + \end{tikzpicture} + \end{center} + + \item Calculer la probabilité de l'évènement $M \cap W$. Interpréter ce résultat dans le contexte de l'exercice. + \item Montrer que la probabilité que la fiche choisie corresponde à une formule \emph{Burger} est égale à $0.9052$. + \item On a prélevé une fiche correspondant à la formule \emph{Burger}. Quelle est la probabilité, arrondie au millième, que la vente ait eu lieu le soir? + \end{enumerate} +\end{exercise} + +\begin{solution} + \begin{enumerate} + \item + \begin{center} + \begin{tikzpicture}[sloped] + \node {.} + child {node {$M$} + child {node {$W$} + edge from parent + node[above] {$0.75$} + } + child {node {$B$} + edge from parent + node[above] {$0.25$} + } + edge from parent + node[above] {$0.09$} + } + child[missing] {} + child { node {$S$} + child {node {$W$} + edge from parent + node[above] {$0.03$} + } + child {node {$B$} + edge from parent + node[above] {$0.97$} + } + edge from parent + node[above] {$0.91$} + } ; + \end{tikzpicture} + \end{center} + \item On calcule la probabilité que la vente soit un wok et ait eu lieu à midi + \[ P(M\cap W) = P(M) \times P_M(W) = 0.09 \times 0.75 = 0.0675 \] + \item Probabilité que la vente soit un burger. + \[ + P(B) = P(M\cap B) + P(S\cap B) = 0.09 \times 0.75 + 0.91 \times 0.03 = 0.9052 + \] + \item On cherche à calculer la quantité $P_B(S)$. Pour cela on utilise la formule de Bayes + \[ + P_B(S) = \frac{P(B\cap S)}{P(B)} = \frac{P_S(B) \times P(S)}{P(B)} = \frac{0.97\times 0.91}{0.9052} = 0.975143614670791 \approx 0.975 + \] + \end{enumerate} +\end{solution} + +\begin{exercise}[subtitle={Continent plastique}] + \textit{Les quantités évoqués dans cette exercice sont générés au hasard et sont donc complètement farfelus.} + \medskip + Le \og continent de plastique\fg{} est la plus grande des plaques de déchets plastiques évoluant sur les océans. Elle occupe actuellement dans l'océan Pacifique une surface dont l'aire est évaluée à plus de $1,6$ million de km$^2$, entre Hawaï et la Californie. + + En 2017, des scientifiques ont estimé qu'il y avait $14$ millions de tonnes de déchets plastiques qui était déversé chaque année dans les océans et que cette quantité augmentait de $28\n\%$ par chaque année. + + On modélise l'évolution de la masse de ces déchets plastiques déversée chaque année, si rien n'est fait pour la réduire, par une suite géométrique $\left(u_n\right)$. L'arrondi au centième du terme $u_n$ représente la masse de ces déchets déversée chaque année, exprimée en million de tonnes, pour l'année $(2017 + n)$. + + \medskip + + \begin{enumerate} + \item Expliquer pourquoi la suite $u_n$ est géométrique? + \item Calculer $u_1$ et $u_2$. + \item Exprimer $u_n$ en fonction de $n$. + \item Au début de l'année 2017, il y avait $300$ millions de tonnes de déchets plastique. Calculer la quantité totale de déchets plastiques en 2030. + \item On souhaite déterminer en quelle année la masse totale de ces déchets plastiques aura pour la première fois augmenté de $50$\,\% par rapport à sa valeur de 2017. + \begin{enumerate} + \item Recopier et compléter l'algorithme ci-dessous pour que la variable $N$ contienne la réponse au problème posé. + + \begin{center} + \begin{tabularx}{0.4\linewidth}{|X|}\hline + $N = 2017$\\ + $U = 14$ \\ + $S = 300 + U$ \\ + while $S < 450$: \\ + \hspace{1cm} $N = \ldots$\\ + \hspace{1cm} $U = \ldots$\\ + \hspace{1cm} $S = \ldots$\\ + \hline + \end{tabularx} + \end{center} + \item Que contiennent les variables $S$, $U$ et $N$ après exécution de cet algorithme ? + + Interpréter les résultats dans le contexte de l'exercice. + \end{enumerate} + \end{enumerate} +\end{exercise} + +\begin{solution} + \begin{enumerate} + \item Une augmentation de $28\,\%$ revient à multiplier la quantité par $1.28$. La suite est donc bien géométrique. Son premier terme est $u_0 = 14$ et sa raison est $q = 1.28$ + \item + \[ + u_1 = u_0 * 1.28 = 17.92 + \] + \[ + u_2 = u_0 * 1.28^2 = 22.9376 + \] + \item + \[ + u_n = u_0 \times q^n = 14 \times 1.28^n + \] + \item On calcule la quantité totale déversée entre 2017 et 2030. + \[ + \sum_{n = 0}^{13} u_n = u_0 \times \frac{1-q^{13}}{1-q} = 14 \times \frac{1 - 1.28^{13}}{1 - 1.28} = 1187.94 + \] + On en déduit la quantité totale de déchets en 2030 + \[ + 300 + 1187.94 = 1487.94 + \] + \item + \begin{enumerate} + \item ~ + \begin{center} + \begin{tabularx}{0.4\linewidth}{|X|}\hline + $N \gets 2017$\\ + $U \gets 14$ \\ + $S \gets 300 + U$ \\ + Tant que $S < 450$ \\ + \hspace{1cm} $N \gets N + 1$\\ + \hspace{1cm} $U \gets U * 1.28$\\ + \hspace{1cm} $S \gets S + u$\\ + Fin Tant que\\\hline + \end{tabularx} + \end{center} + \item \textit{Pas de correction automatisé} + \end{enumerate} + \end{enumerate} +\end{solution} + +\end{document} + +%%% Local Variables: +%%% mode: latex +%%% TeX-master: "master" +%%% End: diff --git a/TST/DS/DS_21_04_07/TST1/corr_02_210407_DS8.tex b/TST/DS/DS_21_04_07/TST1/corr_02_210407_DS8.tex new file mode 100644 index 0000000..322cba3 --- /dev/null +++ b/TST/DS/DS_21_04_07/TST1/corr_02_210407_DS8.tex @@ -0,0 +1,262 @@ +\documentclass[a4paper,10pt]{article} +\usepackage{myXsim} + +% Title Page +\title{DS8 \hfill BAHBAH Zakaria} +\tribe{TST} +\date{\hfillÀ render pour le Mercredi 7 avril} + +\xsimsetup{ + solution/print = true +} + +\begin{document} +\maketitle + +\begin{exercise}[subtitle={Automatismes}] + \textit{Toutes les questions de cette exercice sont indépendantes et peuvent être répondus séparément} + \begin{enumerate} + \item De janvier à septembre, une quantité a augmenté de $28\,\%$. Faire un schéma pour représenter la situation puis calculer le taux d'évolution moyen mensuel. + \item Une quantité augmente de $28\,\%$ par ans. En 2020, elle est de 138\euro. Quelle était sa valeur en 2019? Faire un schéma pour représenter la situation. + \item Déterminer l'équation de la droite \\ + \begin{tikzpicture}[xscale=0.8, yscale=0.5] + \tkzInit[xmin=-5,xmax=5,xstep=1, + ymin=-5,ymax=5,ystep=1] + \tkzGrid + \tkzAxeXY + \tkzFct[domain=-5:5,color=red,very thick]% + {1.0*\x -2}; + \end{tikzpicture} + \item Résoudre l'équation $7 \times 0.54^x = 18$ + \end{enumerate} +\end{exercise} + +\begin{solution} + \begin{enumerate} + \item On veut partager cette évolution en 8 évolutions. + \[ + \left(1 + \frac{28}{100}\right)^{\frac{1}{8}} = 1.0313 + \] + Donc le taux d'évolution moyen est + \[ + t_m = 1.0313 - 1 = 0.031300000000000106 + \] + \item Coefficient multiplicateur pour revenir en arrière + \[ + CM = (1 + \frac{28}{100})^{-1} = 0.7812 + \] + On en déduit la quantité en 2019 + \[ + 138 * 0.7812 = 107.8056 + \] + \item L'équation de la droite est + \[ + y = 1.0 x -2 + \] + \item Il faut penser à faire la division à par $7$ avant d'utiliser le log car sinon, on ne peut pas utiliser la formule $\log(a^n) = n\times \log(a)$. + + \[x = \frac{\log(2.57)}{\log(0.54)}\] + \end{enumerate} +\end{solution} + +\begin{exercise}[subtitle={Restaurant}] + Un \emph{food truck}, ouvert le midi et le soir, propose deux types de formules : + + \setlength\parindent{10mm} + \begin{itemize} + \item la formule \emph{Burger} ; + \item la formule \emph{Wok}. + \end{itemize} + \setlength\parindent{0mm} + + \medskip + + Le gérant a remarqué que 57\,\% de ses ventes ont lieu le midi. Le quart des ventes du midi correspondent à la formule \emph{Burger}, alors que 49\,\% des ventes du soir correspondent à la formule \emph{Wok}. + + Le gérant se constitue un fichier en notant, pour chaque vente, la formule choisie et le moment de cette vente (midi ou soir). + + On prélève une fiche de façon équiprobable. On définit les quatre évènements suivants: + + \begin{enumerate} + \item $M$ : \og la fiche correspond à une vente du midi\fg{} ; + \item $S$ : \og la fiche correspond à une vente du soir\fg {}; + \item $W$ : \og la fiche correspond à une formule \emph{Wok} \fg{} ; + \item $B$ : \og la fiche correspond à une formule \emph{Burger} \fg. + \end{enumerate} + \setlength\parindent{0mm} + + \medskip + + \begin{enumerate} + \item Recopier puis compléter l'arbre pondéré + + \begin{center} + \begin{tikzpicture}[sloped] + \node {.} + child {node {$M$} + child {node {$W$} + edge from parent + node[above] {...} + } + child {node {$B$} + edge from parent + node[above] {...} + } + edge from parent + node[above] {...} + } + child[missing] {} + child { node {$S$} + child {node {$W$} + edge from parent + node[above] {...} + } + child {node {$B$} + edge from parent + node[above] {...} + } + edge from parent + node[above] {...} + } ; + \end{tikzpicture} + \end{center} + + \item Calculer la probabilité de l'évènement $M \cap W$. Interpréter ce résultat dans le contexte de l'exercice. + \item Montrer que la probabilité que la fiche choisie corresponde à une formule \emph{Burger} est égale à $0.3592$. + \item On a prélevé une fiche correspondant à la formule \emph{Burger}. Quelle est la probabilité, arrondie au millième, que la vente ait eu lieu le soir? + \end{enumerate} +\end{exercise} + +\begin{solution} + \begin{enumerate} + \item + \begin{center} + \begin{tikzpicture}[sloped] + \node {.} + child {node {$M$} + child {node {$W$} + edge from parent + node[above] {$0.75$} + } + child {node {$B$} + edge from parent + node[above] {$0.25$} + } + edge from parent + node[above] {$0.58$} + } + child[missing] {} + child { node {$S$} + child {node {$W$} + edge from parent + node[above] {$0.49$} + } + child {node {$B$} + edge from parent + node[above] {$0.51$} + } + edge from parent + node[above] {$0.42$} + } ; + \end{tikzpicture} + \end{center} + \item On calcule la probabilité que la vente soit un wok et ait eu lieu à midi + \[ P(M\cap W) = P(M) \times P_M(W) = 0.58 \times 0.75 = 0.435 \] + \item Probabilité que la vente soit un burger. + \[ + P(B) = P(M\cap B) + P(S\cap B) = 0.58 \times 0.75 + 0.42 \times 0.49 = 0.3592 + \] + \item On cherche à calculer la quantité $P_B(S)$. Pour cela on utilise la formule de Bayes + \[ + P_B(S) = \frac{P(B\cap S)}{P(B)} = \frac{P_S(B) \times P(S)}{P(B)} = \frac{0.51\times 0.42}{0.3592} = 0.5963251670378619 \approx 0.596 + \] + \end{enumerate} +\end{solution} + +\begin{exercise}[subtitle={Continent plastique}] + \textit{Les quantités évoqués dans cette exercice sont générés au hasard et sont donc complètement farfelus.} + \medskip + Le \og continent de plastique\fg{} est la plus grande des plaques de déchets plastiques évoluant sur les océans. Elle occupe actuellement dans l'océan Pacifique une surface dont l'aire est évaluée à plus de $1,6$ million de km$^2$, entre Hawaï et la Californie. + + En 2017, des scientifiques ont estimé qu'il y avait $9$ millions de tonnes de déchets plastiques qui était déversé chaque année dans les océans et que cette quantité augmentait de $26\n\%$ par chaque année. + + On modélise l'évolution de la masse de ces déchets plastiques déversée chaque année, si rien n'est fait pour la réduire, par une suite géométrique $\left(u_n\right)$. L'arrondi au centième du terme $u_n$ représente la masse de ces déchets déversée chaque année, exprimée en million de tonnes, pour l'année $(2017 + n)$. + + \medskip + + \begin{enumerate} + \item Expliquer pourquoi la suite $u_n$ est géométrique? + \item Calculer $u_1$ et $u_2$. + \item Exprimer $u_n$ en fonction de $n$. + \item Au début de l'année 2017, il y avait $300$ millions de tonnes de déchets plastique. Calculer la quantité totale de déchets plastiques en 2030. + \item On souhaite déterminer en quelle année la masse totale de ces déchets plastiques aura pour la première fois augmenté de $50$\,\% par rapport à sa valeur de 2017. + \begin{enumerate} + \item Recopier et compléter l'algorithme ci-dessous pour que la variable $N$ contienne la réponse au problème posé. + + \begin{center} + \begin{tabularx}{0.4\linewidth}{|X|}\hline + $N = 2017$\\ + $U = 9$ \\ + $S = 300 + U$ \\ + while $S < 450$: \\ + \hspace{1cm} $N = \ldots$\\ + \hspace{1cm} $U = \ldots$\\ + \hspace{1cm} $S = \ldots$\\ + \hline + \end{tabularx} + \end{center} + \item Que contiennent les variables $S$, $U$ et $N$ après exécution de cet algorithme ? + + Interpréter les résultats dans le contexte de l'exercice. + \end{enumerate} + \end{enumerate} +\end{exercise} + +\begin{solution} + \begin{enumerate} + \item Une augmentation de $26\,\%$ revient à multiplier la quantité par $1.26$. La suite est donc bien géométrique. Son premier terme est $u_0 = 9$ et sa raison est $q = 1.26$ + \item + \[ + u_1 = u_0 * 1.26 = 11.34 + \] + \[ + u_2 = u_0 * 1.26^2 = 14.2884 + \] + \item + \[ + u_n = u_0 \times q^n = 9 \times 1.26^n + \] + \item On calcule la quantité totale déversée entre 2017 et 2030. + \[ + \sum_{n = 0}^{13} u_n = u_0 \times \frac{1-q^{13}}{1-q} = 9 \times \frac{1 - 1.26^{13}}{1 - 1.26} = 663.76 + \] + On en déduit la quantité totale de déchets en 2030 + \[ + 300 + 663.76 = 963.76 + \] + \item + \begin{enumerate} + \item ~ + \begin{center} + \begin{tabularx}{0.4\linewidth}{|X|}\hline + $N \gets 2017$\\ + $U \gets 9$ \\ + $S \gets 300 + U$ \\ + Tant que $S < 450$ \\ + \hspace{1cm} $N \gets N + 1$\\ + \hspace{1cm} $U \gets U * 1.26$\\ + \hspace{1cm} $S \gets S + u$\\ + Fin Tant que\\\hline + \end{tabularx} + \end{center} + \item \textit{Pas de correction automatisé} + \end{enumerate} + \end{enumerate} +\end{solution} + +\end{document} + +%%% Local Variables: +%%% mode: latex +%%% TeX-master: "master" +%%% End: diff --git a/TST/DS/DS_21_04_07/TST1/corr_03_210407_DS8.tex b/TST/DS/DS_21_04_07/TST1/corr_03_210407_DS8.tex new file mode 100644 index 0000000..7759b46 --- /dev/null +++ b/TST/DS/DS_21_04_07/TST1/corr_03_210407_DS8.tex @@ -0,0 +1,262 @@ +\documentclass[a4paper,10pt]{article} +\usepackage{myXsim} + +% Title Page +\title{DS8 \hfill BALLOFFET Kenza} +\tribe{TST} +\date{\hfillÀ render pour le Mercredi 7 avril} + +\xsimsetup{ + solution/print = true +} + +\begin{document} +\maketitle + +\begin{exercise}[subtitle={Automatismes}] + \textit{Toutes les questions de cette exercice sont indépendantes et peuvent être répondus séparément} + \begin{enumerate} + \item De janvier à septembre, une quantité a augmenté de $12\,\%$. Faire un schéma pour représenter la situation puis calculer le taux d'évolution moyen mensuel. + \item Une quantité augmente de $12\,\%$ par ans. En 2020, elle est de 126\euro. Quelle était sa valeur en 2019? Faire un schéma pour représenter la situation. + \item Déterminer l'équation de la droite \\ + \begin{tikzpicture}[xscale=0.8, yscale=0.5] + \tkzInit[xmin=-5,xmax=5,xstep=1, + ymin=-5,ymax=5,ystep=1] + \tkzGrid + \tkzAxeXY + \tkzFct[domain=-5:5,color=red,very thick]% + {2.6666666666666665*\x -4}; + \end{tikzpicture} + \item Résoudre l'équation $8 \times 0.54^x = 18$ + \end{enumerate} +\end{exercise} + +\begin{solution} + \begin{enumerate} + \item On veut partager cette évolution en 8 évolutions. + \[ + \left(1 + \frac{12}{100}\right)^{\frac{1}{8}} = 1.0143 + \] + Donc le taux d'évolution moyen est + \[ + t_m = 1.0143 - 1 = 0.01429999999999998 + \] + \item Coefficient multiplicateur pour revenir en arrière + \[ + CM = (1 + \frac{12}{100})^{-1} = 0.8929 + \] + On en déduit la quantité en 2019 + \[ + 126 * 0.8929 = 112.50540000000001 + \] + \item L'équation de la droite est + \[ + y = 2.6666666666666665 x -4 + \] + \item Il faut penser à faire la division à par $8$ avant d'utiliser le log car sinon, on ne peut pas utiliser la formule $\log(a^n) = n\times \log(a)$. + + \[x = \frac{\log(2.25)}{\log(0.54)}\] + \end{enumerate} +\end{solution} + +\begin{exercise}[subtitle={Restaurant}] + Un \emph{food truck}, ouvert le midi et le soir, propose deux types de formules : + + \setlength\parindent{10mm} + \begin{itemize} + \item la formule \emph{Burger} ; + \item la formule \emph{Wok}. + \end{itemize} + \setlength\parindent{0mm} + + \medskip + + Le gérant a remarqué que 94\,\% de ses ventes ont lieu le midi. Le quart des ventes du midi correspondent à la formule \emph{Burger}, alors que 37\,\% des ventes du soir correspondent à la formule \emph{Wok}. + + Le gérant se constitue un fichier en notant, pour chaque vente, la formule choisie et le moment de cette vente (midi ou soir). + + On prélève une fiche de façon équiprobable. On définit les quatre évènements suivants: + + \begin{enumerate} + \item $M$ : \og la fiche correspond à une vente du midi\fg{} ; + \item $S$ : \og la fiche correspond à une vente du soir\fg {}; + \item $W$ : \og la fiche correspond à une formule \emph{Wok} \fg{} ; + \item $B$ : \og la fiche correspond à une formule \emph{Burger} \fg. + \end{enumerate} + \setlength\parindent{0mm} + + \medskip + + \begin{enumerate} + \item Recopier puis compléter l'arbre pondéré + + \begin{center} + \begin{tikzpicture}[sloped] + \node {.} + child {node {$M$} + child {node {$W$} + edge from parent + node[above] {...} + } + child {node {$B$} + edge from parent + node[above] {...} + } + edge from parent + node[above] {...} + } + child[missing] {} + child { node {$S$} + child {node {$W$} + edge from parent + node[above] {...} + } + child {node {$B$} + edge from parent + node[above] {...} + } + edge from parent + node[above] {...} + } ; + \end{tikzpicture} + \end{center} + + \item Calculer la probabilité de l'évènement $M \cap W$. Interpréter ce résultat dans le contexte de l'exercice. + \item Montrer que la probabilité que la fiche choisie corresponde à une formule \emph{Burger} est égale à $0.2728$. + \item On a prélevé une fiche correspondant à la formule \emph{Burger}. Quelle est la probabilité, arrondie au millième, que la vente ait eu lieu le soir? + \end{enumerate} +\end{exercise} + +\begin{solution} + \begin{enumerate} + \item + \begin{center} + \begin{tikzpicture}[sloped] + \node {.} + child {node {$M$} + child {node {$W$} + edge from parent + node[above] {$0.75$} + } + child {node {$B$} + edge from parent + node[above] {$0.25$} + } + edge from parent + node[above] {$0.94$} + } + child[missing] {} + child { node {$S$} + child {node {$W$} + edge from parent + node[above] {$0.37$} + } + child {node {$B$} + edge from parent + node[above] {$0.63$} + } + edge from parent + node[above] {$0.06$} + } ; + \end{tikzpicture} + \end{center} + \item On calcule la probabilité que la vente soit un wok et ait eu lieu à midi + \[ P(M\cap W) = P(M) \times P_M(W) = 0.94 \times 0.75 = 0.705 \] + \item Probabilité que la vente soit un burger. + \[ + P(B) = P(M\cap B) + P(S\cap B) = 0.94 \times 0.75 + 0.06 \times 0.37 = 0.2728 + \] + \item On cherche à calculer la quantité $P_B(S)$. Pour cela on utilise la formule de Bayes + \[ + P_B(S) = \frac{P(B\cap S)}{P(B)} = \frac{P_S(B) \times P(S)}{P(B)} = \frac{0.63\times 0.06}{0.2728} = 0.13856304985337245 \approx 0.139 + \] + \end{enumerate} +\end{solution} + +\begin{exercise}[subtitle={Continent plastique}] + \textit{Les quantités évoqués dans cette exercice sont générés au hasard et sont donc complètement farfelus.} + \medskip + Le \og continent de plastique\fg{} est la plus grande des plaques de déchets plastiques évoluant sur les océans. Elle occupe actuellement dans l'océan Pacifique une surface dont l'aire est évaluée à plus de $1,6$ million de km$^2$, entre Hawaï et la Californie. + + En 2017, des scientifiques ont estimé qu'il y avait $15$ millions de tonnes de déchets plastiques qui était déversé chaque année dans les océans et que cette quantité augmentait de $19\n\%$ par chaque année. + + On modélise l'évolution de la masse de ces déchets plastiques déversée chaque année, si rien n'est fait pour la réduire, par une suite géométrique $\left(u_n\right)$. L'arrondi au centième du terme $u_n$ représente la masse de ces déchets déversée chaque année, exprimée en million de tonnes, pour l'année $(2017 + n)$. + + \medskip + + \begin{enumerate} + \item Expliquer pourquoi la suite $u_n$ est géométrique? + \item Calculer $u_1$ et $u_2$. + \item Exprimer $u_n$ en fonction de $n$. + \item Au début de l'année 2017, il y avait $300$ millions de tonnes de déchets plastique. Calculer la quantité totale de déchets plastiques en 2030. + \item On souhaite déterminer en quelle année la masse totale de ces déchets plastiques aura pour la première fois augmenté de $50$\,\% par rapport à sa valeur de 2017. + \begin{enumerate} + \item Recopier et compléter l'algorithme ci-dessous pour que la variable $N$ contienne la réponse au problème posé. + + \begin{center} + \begin{tabularx}{0.4\linewidth}{|X|}\hline + $N = 2017$\\ + $U = 15$ \\ + $S = 300 + U$ \\ + while $S < 450$: \\ + \hspace{1cm} $N = \ldots$\\ + \hspace{1cm} $U = \ldots$\\ + \hspace{1cm} $S = \ldots$\\ + \hline + \end{tabularx} + \end{center} + \item Que contiennent les variables $S$, $U$ et $N$ après exécution de cet algorithme ? + + Interpréter les résultats dans le contexte de l'exercice. + \end{enumerate} + \end{enumerate} +\end{exercise} + +\begin{solution} + \begin{enumerate} + \item Une augmentation de $19\,\%$ revient à multiplier la quantité par $1.19$. La suite est donc bien géométrique. Son premier terme est $u_0 = 15$ et sa raison est $q = 1.19$ + \item + \[ + u_1 = u_0 * 1.19 = 17.849999999999998 + \] + \[ + u_2 = u_0 * 1.19^2 = 21.2415 + \] + \item + \[ + u_n = u_0 \times q^n = 15 \times 1.19^n + \] + \item On calcule la quantité totale déversée entre 2017 et 2030. + \[ + \sum_{n = 0}^{13} u_n = u_0 \times \frac{1-q^{13}}{1-q} = 15 \times \frac{1 - 1.19^{13}}{1 - 1.19} = 678.67 + \] + On en déduit la quantité totale de déchets en 2030 + \[ + 300 + 678.67 = 978.67 + \] + \item + \begin{enumerate} + \item ~ + \begin{center} + \begin{tabularx}{0.4\linewidth}{|X|}\hline + $N \gets 2017$\\ + $U \gets 15$ \\ + $S \gets 300 + U$ \\ + Tant que $S < 450$ \\ + \hspace{1cm} $N \gets N + 1$\\ + \hspace{1cm} $U \gets U * 1.19$\\ + \hspace{1cm} $S \gets S + u$\\ + Fin Tant que\\\hline + \end{tabularx} + \end{center} + \item \textit{Pas de correction automatisé} + \end{enumerate} + \end{enumerate} +\end{solution} + +\end{document} + +%%% Local Variables: +%%% mode: latex +%%% TeX-master: "master" +%%% End: diff --git a/TST/DS/DS_21_04_07/TST1/corr_04_210407_DS8.tex b/TST/DS/DS_21_04_07/TST1/corr_04_210407_DS8.tex new file mode 100644 index 0000000..6a00b05 --- /dev/null +++ b/TST/DS/DS_21_04_07/TST1/corr_04_210407_DS8.tex @@ -0,0 +1,262 @@ +\documentclass[a4paper,10pt]{article} +\usepackage{myXsim} + +% Title Page +\title{DS8 \hfill BENHATTAL Chakir} +\tribe{TST} +\date{\hfillÀ render pour le Mercredi 7 avril} + +\xsimsetup{ + solution/print = true +} + +\begin{document} +\maketitle + +\begin{exercise}[subtitle={Automatismes}] + \textit{Toutes les questions de cette exercice sont indépendantes et peuvent être répondus séparément} + \begin{enumerate} + \item De janvier à septembre, une quantité a augmenté de $28\,\%$. Faire un schéma pour représenter la situation puis calculer le taux d'évolution moyen mensuel. + \item Une quantité augmente de $28\,\%$ par ans. En 2020, elle est de 111\euro. Quelle était sa valeur en 2019? Faire un schéma pour représenter la situation. + \item Déterminer l'équation de la droite \\ + \begin{tikzpicture}[xscale=0.8, yscale=0.5] + \tkzInit[xmin=-5,xmax=5,xstep=1, + ymin=-5,ymax=5,ystep=1] + \tkzGrid + \tkzAxeXY + \tkzFct[domain=-5:5,color=red,very thick]% + {1.0*\x -1}; + \end{tikzpicture} + \item Résoudre l'équation $4 \times 0.77^x = 25$ + \end{enumerate} +\end{exercise} + +\begin{solution} + \begin{enumerate} + \item On veut partager cette évolution en 8 évolutions. + \[ + \left(1 + \frac{28}{100}\right)^{\frac{1}{8}} = 1.0313 + \] + Donc le taux d'évolution moyen est + \[ + t_m = 1.0313 - 1 = 0.031300000000000106 + \] + \item Coefficient multiplicateur pour revenir en arrière + \[ + CM = (1 + \frac{28}{100})^{-1} = 0.7812 + \] + On en déduit la quantité en 2019 + \[ + 111 * 0.7812 = 86.7132 + \] + \item L'équation de la droite est + \[ + y = 1.0 x -1 + \] + \item Il faut penser à faire la division à par $4$ avant d'utiliser le log car sinon, on ne peut pas utiliser la formule $\log(a^n) = n\times \log(a)$. + + \[x = \frac{\log(6.25)}{\log(0.77)}\] + \end{enumerate} +\end{solution} + +\begin{exercise}[subtitle={Restaurant}] + Un \emph{food truck}, ouvert le midi et le soir, propose deux types de formules : + + \setlength\parindent{10mm} + \begin{itemize} + \item la formule \emph{Burger} ; + \item la formule \emph{Wok}. + \end{itemize} + \setlength\parindent{0mm} + + \medskip + + Le gérant a remarqué que 89\,\% de ses ventes ont lieu le midi. Le quart des ventes du midi correspondent à la formule \emph{Burger}, alors que 43\,\% des ventes du soir correspondent à la formule \emph{Wok}. + + Le gérant se constitue un fichier en notant, pour chaque vente, la formule choisie et le moment de cette vente (midi ou soir). + + On prélève une fiche de façon équiprobable. On définit les quatre évènements suivants: + + \begin{enumerate} + \item $M$ : \og la fiche correspond à une vente du midi\fg{} ; + \item $S$ : \og la fiche correspond à une vente du soir\fg {}; + \item $W$ : \og la fiche correspond à une formule \emph{Wok} \fg{} ; + \item $B$ : \og la fiche correspond à une formule \emph{Burger} \fg. + \end{enumerate} + \setlength\parindent{0mm} + + \medskip + + \begin{enumerate} + \item Recopier puis compléter l'arbre pondéré + + \begin{center} + \begin{tikzpicture}[sloped] + \node {.} + child {node {$M$} + child {node {$W$} + edge from parent + node[above] {...} + } + child {node {$B$} + edge from parent + node[above] {...} + } + edge from parent + node[above] {...} + } + child[missing] {} + child { node {$S$} + child {node {$W$} + edge from parent + node[above] {...} + } + child {node {$B$} + edge from parent + node[above] {...} + } + edge from parent + node[above] {...} + } ; + \end{tikzpicture} + \end{center} + + \item Calculer la probabilité de l'évènement $M \cap W$. Interpréter ce résultat dans le contexte de l'exercice. + \item Montrer que la probabilité que la fiche choisie corresponde à une formule \emph{Burger} est égale à $0.2852$. + \item On a prélevé une fiche correspondant à la formule \emph{Burger}. Quelle est la probabilité, arrondie au millième, que la vente ait eu lieu le soir? + \end{enumerate} +\end{exercise} + +\begin{solution} + \begin{enumerate} + \item + \begin{center} + \begin{tikzpicture}[sloped] + \node {.} + child {node {$M$} + child {node {$W$} + edge from parent + node[above] {$0.75$} + } + child {node {$B$} + edge from parent + node[above] {$0.25$} + } + edge from parent + node[above] {$0.89$} + } + child[missing] {} + child { node {$S$} + child {node {$W$} + edge from parent + node[above] {$0.43$} + } + child {node {$B$} + edge from parent + node[above] {$0.57$} + } + edge from parent + node[above] {$0.11$} + } ; + \end{tikzpicture} + \end{center} + \item On calcule la probabilité que la vente soit un wok et ait eu lieu à midi + \[ P(M\cap W) = P(M) \times P_M(W) = 0.89 \times 0.75 = 0.6675 \] + \item Probabilité que la vente soit un burger. + \[ + P(B) = P(M\cap B) + P(S\cap B) = 0.89 \times 0.75 + 0.11 \times 0.43 = 0.2852 + \] + \item On cherche à calculer la quantité $P_B(S)$. Pour cela on utilise la formule de Bayes + \[ + P_B(S) = \frac{P(B\cap S)}{P(B)} = \frac{P_S(B) \times P(S)}{P(B)} = \frac{0.57\times 0.11}{0.2852} = 0.2198457223001402 \approx 0.22 + \] + \end{enumerate} +\end{solution} + +\begin{exercise}[subtitle={Continent plastique}] + \textit{Les quantités évoqués dans cette exercice sont générés au hasard et sont donc complètement farfelus.} + \medskip + Le \og continent de plastique\fg{} est la plus grande des plaques de déchets plastiques évoluant sur les océans. Elle occupe actuellement dans l'océan Pacifique une surface dont l'aire est évaluée à plus de $1,6$ million de km$^2$, entre Hawaï et la Californie. + + En 2017, des scientifiques ont estimé qu'il y avait $6$ millions de tonnes de déchets plastiques qui était déversé chaque année dans les océans et que cette quantité augmentait de $13\n\%$ par chaque année. + + On modélise l'évolution de la masse de ces déchets plastiques déversée chaque année, si rien n'est fait pour la réduire, par une suite géométrique $\left(u_n\right)$. L'arrondi au centième du terme $u_n$ représente la masse de ces déchets déversée chaque année, exprimée en million de tonnes, pour l'année $(2017 + n)$. + + \medskip + + \begin{enumerate} + \item Expliquer pourquoi la suite $u_n$ est géométrique? + \item Calculer $u_1$ et $u_2$. + \item Exprimer $u_n$ en fonction de $n$. + \item Au début de l'année 2017, il y avait $300$ millions de tonnes de déchets plastique. Calculer la quantité totale de déchets plastiques en 2030. + \item On souhaite déterminer en quelle année la masse totale de ces déchets plastiques aura pour la première fois augmenté de $50$\,\% par rapport à sa valeur de 2017. + \begin{enumerate} + \item Recopier et compléter l'algorithme ci-dessous pour que la variable $N$ contienne la réponse au problème posé. + + \begin{center} + \begin{tabularx}{0.4\linewidth}{|X|}\hline + $N = 2017$\\ + $U = 6$ \\ + $S = 300 + U$ \\ + while $S < 450$: \\ + \hspace{1cm} $N = \ldots$\\ + \hspace{1cm} $U = \ldots$\\ + \hspace{1cm} $S = \ldots$\\ + \hline + \end{tabularx} + \end{center} + \item Que contiennent les variables $S$, $U$ et $N$ après exécution de cet algorithme ? + + Interpréter les résultats dans le contexte de l'exercice. + \end{enumerate} + \end{enumerate} +\end{exercise} + +\begin{solution} + \begin{enumerate} + \item Une augmentation de $13\,\%$ revient à multiplier la quantité par $1.13$. La suite est donc bien géométrique. Son premier terme est $u_0 = 6$ et sa raison est $q = 1.13$ + \item + \[ + u_1 = u_0 * 1.13 = 6.779999999999999 + \] + \[ + u_2 = u_0 * 1.13^2 = 7.6614 + \] + \item + \[ + u_n = u_0 \times q^n = 6 \times 1.13^n + \] + \item On calcule la quantité totale déversée entre 2017 et 2030. + \[ + \sum_{n = 0}^{13} u_n = u_0 \times \frac{1-q^{13}}{1-q} = 6 \times \frac{1 - 1.13^{13}}{1 - 1.13} = 179.91 + \] + On en déduit la quantité totale de déchets en 2030 + \[ + 300 + 179.91 = 479.90999999999997 + \] + \item + \begin{enumerate} + \item ~ + \begin{center} + \begin{tabularx}{0.4\linewidth}{|X|}\hline + $N \gets 2017$\\ + $U \gets 6$ \\ + $S \gets 300 + U$ \\ + Tant que $S < 450$ \\ + \hspace{1cm} $N \gets N + 1$\\ + \hspace{1cm} $U \gets U * 1.13$\\ + \hspace{1cm} $S \gets S + u$\\ + Fin Tant que\\\hline + \end{tabularx} + \end{center} + \item \textit{Pas de correction automatisé} + \end{enumerate} + \end{enumerate} +\end{solution} + +\end{document} + +%%% Local Variables: +%%% mode: latex +%%% TeX-master: "master" +%%% End: diff --git a/TST/DS/DS_21_04_07/TST1/corr_05_210407_DS8.tex b/TST/DS/DS_21_04_07/TST1/corr_05_210407_DS8.tex new file mode 100644 index 0000000..6375b9b --- /dev/null +++ b/TST/DS/DS_21_04_07/TST1/corr_05_210407_DS8.tex @@ -0,0 +1,262 @@ +\documentclass[a4paper,10pt]{article} +\usepackage{myXsim} + +% Title Page +\title{DS8 \hfill CLAIN Avinash} +\tribe{TST} +\date{\hfillÀ render pour le Mercredi 7 avril} + +\xsimsetup{ + solution/print = true +} + +\begin{document} +\maketitle + +\begin{exercise}[subtitle={Automatismes}] + \textit{Toutes les questions de cette exercice sont indépendantes et peuvent être répondus séparément} + \begin{enumerate} + \item De janvier à septembre, une quantité a augmenté de $15\,\%$. Faire un schéma pour représenter la situation puis calculer le taux d'évolution moyen mensuel. + \item Une quantité augmente de $15\,\%$ par ans. En 2020, elle est de 112\euro. Quelle était sa valeur en 2019? Faire un schéma pour représenter la situation. + \item Déterminer l'équation de la droite \\ + \begin{tikzpicture}[xscale=0.8, yscale=0.5] + \tkzInit[xmin=-5,xmax=5,xstep=1, + ymin=-5,ymax=5,ystep=1] + \tkzGrid + \tkzAxeXY + \tkzFct[domain=-5:5,color=red,very thick]% + {4.0*\x -4}; + \end{tikzpicture} + \item Résoudre l'équation $6 \times 0.77^x = 23$ + \end{enumerate} +\end{exercise} + +\begin{solution} + \begin{enumerate} + \item On veut partager cette évolution en 8 évolutions. + \[ + \left(1 + \frac{15}{100}\right)^{\frac{1}{8}} = 1.0176 + \] + Donc le taux d'évolution moyen est + \[ + t_m = 1.0176 - 1 = 0.01760000000000006 + \] + \item Coefficient multiplicateur pour revenir en arrière + \[ + CM = (1 + \frac{15}{100})^{-1} = 0.8696 + \] + On en déduit la quantité en 2019 + \[ + 112 * 0.8696 = 97.3952 + \] + \item L'équation de la droite est + \[ + y = 4.0 x -4 + \] + \item Il faut penser à faire la division à par $6$ avant d'utiliser le log car sinon, on ne peut pas utiliser la formule $\log(a^n) = n\times \log(a)$. + + \[x = \frac{\log(3.83)}{\log(0.77)}\] + \end{enumerate} +\end{solution} + +\begin{exercise}[subtitle={Restaurant}] + Un \emph{food truck}, ouvert le midi et le soir, propose deux types de formules : + + \setlength\parindent{10mm} + \begin{itemize} + \item la formule \emph{Burger} ; + \item la formule \emph{Wok}. + \end{itemize} + \setlength\parindent{0mm} + + \medskip + + Le gérant a remarqué que 92\,\% de ses ventes ont lieu le midi. Le quart des ventes du midi correspondent à la formule \emph{Burger}, alors que 54\,\% des ventes du soir correspondent à la formule \emph{Wok}. + + Le gérant se constitue un fichier en notant, pour chaque vente, la formule choisie et le moment de cette vente (midi ou soir). + + On prélève une fiche de façon équiprobable. On définit les quatre évènements suivants: + + \begin{enumerate} + \item $M$ : \og la fiche correspond à une vente du midi\fg{} ; + \item $S$ : \og la fiche correspond à une vente du soir\fg {}; + \item $W$ : \og la fiche correspond à une formule \emph{Wok} \fg{} ; + \item $B$ : \og la fiche correspond à une formule \emph{Burger} \fg. + \end{enumerate} + \setlength\parindent{0mm} + + \medskip + + \begin{enumerate} + \item Recopier puis compléter l'arbre pondéré + + \begin{center} + \begin{tikzpicture}[sloped] + \node {.} + child {node {$M$} + child {node {$W$} + edge from parent + node[above] {...} + } + child {node {$B$} + edge from parent + node[above] {...} + } + edge from parent + node[above] {...} + } + child[missing] {} + child { node {$S$} + child {node {$W$} + edge from parent + node[above] {...} + } + child {node {$B$} + edge from parent + node[above] {...} + } + edge from parent + node[above] {...} + } ; + \end{tikzpicture} + \end{center} + + \item Calculer la probabilité de l'évènement $M \cap W$. Interpréter ce résultat dans le contexte de l'exercice. + \item Montrer que la probabilité que la fiche choisie corresponde à une formule \emph{Burger} est égale à $0.2668$. + \item On a prélevé une fiche correspondant à la formule \emph{Burger}. Quelle est la probabilité, arrondie au millième, que la vente ait eu lieu le soir? + \end{enumerate} +\end{exercise} + +\begin{solution} + \begin{enumerate} + \item + \begin{center} + \begin{tikzpicture}[sloped] + \node {.} + child {node {$M$} + child {node {$W$} + edge from parent + node[above] {$0.75$} + } + child {node {$B$} + edge from parent + node[above] {$0.25$} + } + edge from parent + node[above] {$0.92$} + } + child[missing] {} + child { node {$S$} + child {node {$W$} + edge from parent + node[above] {$0.54$} + } + child {node {$B$} + edge from parent + node[above] {$0.46$} + } + edge from parent + node[above] {$0.08$} + } ; + \end{tikzpicture} + \end{center} + \item On calcule la probabilité que la vente soit un wok et ait eu lieu à midi + \[ P(M\cap W) = P(M) \times P_M(W) = 0.92 \times 0.75 = 0.69 \] + \item Probabilité que la vente soit un burger. + \[ + P(B) = P(M\cap B) + P(S\cap B) = 0.92 \times 0.75 + 0.08 \times 0.54 = 0.2668 + \] + \item On cherche à calculer la quantité $P_B(S)$. Pour cela on utilise la formule de Bayes + \[ + P_B(S) = \frac{P(B\cap S)}{P(B)} = \frac{P_S(B) \times P(S)}{P(B)} = \frac{0.46\times 0.08}{0.2668} = 0.13793103448275862 \approx 0.138 + \] + \end{enumerate} +\end{solution} + +\begin{exercise}[subtitle={Continent plastique}] + \textit{Les quantités évoqués dans cette exercice sont générés au hasard et sont donc complètement farfelus.} + \medskip + Le \og continent de plastique\fg{} est la plus grande des plaques de déchets plastiques évoluant sur les océans. Elle occupe actuellement dans l'océan Pacifique une surface dont l'aire est évaluée à plus de $1,6$ million de km$^2$, entre Hawaï et la Californie. + + En 2017, des scientifiques ont estimé qu'il y avait $18$ millions de tonnes de déchets plastiques qui était déversé chaque année dans les océans et que cette quantité augmentait de $21\n\%$ par chaque année. + + On modélise l'évolution de la masse de ces déchets plastiques déversée chaque année, si rien n'est fait pour la réduire, par une suite géométrique $\left(u_n\right)$. L'arrondi au centième du terme $u_n$ représente la masse de ces déchets déversée chaque année, exprimée en million de tonnes, pour l'année $(2017 + n)$. + + \medskip + + \begin{enumerate} + \item Expliquer pourquoi la suite $u_n$ est géométrique? + \item Calculer $u_1$ et $u_2$. + \item Exprimer $u_n$ en fonction de $n$. + \item Au début de l'année 2017, il y avait $300$ millions de tonnes de déchets plastique. Calculer la quantité totale de déchets plastiques en 2030. + \item On souhaite déterminer en quelle année la masse totale de ces déchets plastiques aura pour la première fois augmenté de $50$\,\% par rapport à sa valeur de 2017. + \begin{enumerate} + \item Recopier et compléter l'algorithme ci-dessous pour que la variable $N$ contienne la réponse au problème posé. + + \begin{center} + \begin{tabularx}{0.4\linewidth}{|X|}\hline + $N = 2017$\\ + $U = 18$ \\ + $S = 300 + U$ \\ + while $S < 450$: \\ + \hspace{1cm} $N = \ldots$\\ + \hspace{1cm} $U = \ldots$\\ + \hspace{1cm} $S = \ldots$\\ + \hline + \end{tabularx} + \end{center} + \item Que contiennent les variables $S$, $U$ et $N$ après exécution de cet algorithme ? + + Interpréter les résultats dans le contexte de l'exercice. + \end{enumerate} + \end{enumerate} +\end{exercise} + +\begin{solution} + \begin{enumerate} + \item Une augmentation de $21\,\%$ revient à multiplier la quantité par $1.21$. La suite est donc bien géométrique. Son premier terme est $u_0 = 18$ et sa raison est $q = 1.21$ + \item + \[ + u_1 = u_0 * 1.21 = 21.78 + \] + \[ + u_2 = u_0 * 1.21^2 = 26.3538 + \] + \item + \[ + u_n = u_0 \times q^n = 18 \times 1.21^n + \] + \item On calcule la quantité totale déversée entre 2017 et 2030. + \[ + \sum_{n = 0}^{13} u_n = u_0 \times \frac{1-q^{13}}{1-q} = 18 \times \frac{1 - 1.21^{13}}{1 - 1.21} = 935.84 + \] + On en déduit la quantité totale de déchets en 2030 + \[ + 300 + 935.84 = 1235.8400000000001 + \] + \item + \begin{enumerate} + \item ~ + \begin{center} + \begin{tabularx}{0.4\linewidth}{|X|}\hline + $N \gets 2017$\\ + $U \gets 18$ \\ + $S \gets 300 + U$ \\ + Tant que $S < 450$ \\ + \hspace{1cm} $N \gets N + 1$\\ + \hspace{1cm} $U \gets U * 1.21$\\ + \hspace{1cm} $S \gets S + u$\\ + Fin Tant que\\\hline + \end{tabularx} + \end{center} + \item \textit{Pas de correction automatisé} + \end{enumerate} + \end{enumerate} +\end{solution} + +\end{document} + +%%% Local Variables: +%%% mode: latex +%%% TeX-master: "master" +%%% End: diff --git a/TST/DS/DS_21_04_07/TST1/corr_06_210407_DS8.tex b/TST/DS/DS_21_04_07/TST1/corr_06_210407_DS8.tex new file mode 100644 index 0000000..d54012c --- /dev/null +++ b/TST/DS/DS_21_04_07/TST1/corr_06_210407_DS8.tex @@ -0,0 +1,262 @@ +\documentclass[a4paper,10pt]{article} +\usepackage{myXsim} + +% Title Page +\title{DS8 \hfill COLASSI Alexis} +\tribe{TST} +\date{\hfillÀ render pour le Mercredi 7 avril} + +\xsimsetup{ + solution/print = true +} + +\begin{document} +\maketitle + +\begin{exercise}[subtitle={Automatismes}] + \textit{Toutes les questions de cette exercice sont indépendantes et peuvent être répondus séparément} + \begin{enumerate} + \item De janvier à septembre, une quantité a augmenté de $27\,\%$. Faire un schéma pour représenter la situation puis calculer le taux d'évolution moyen mensuel. + \item Une quantité augmente de $27\,\%$ par ans. En 2020, elle est de 116\euro. Quelle était sa valeur en 2019? Faire un schéma pour représenter la situation. + \item Déterminer l'équation de la droite \\ + \begin{tikzpicture}[xscale=0.8, yscale=0.5] + \tkzInit[xmin=-5,xmax=5,xstep=1, + ymin=-5,ymax=5,ystep=1] + \tkzGrid + \tkzAxeXY + \tkzFct[domain=-5:5,color=red,very thick]% + {2.0*\x -2}; + \end{tikzpicture} + \item Résoudre l'équation $7 \times 0.23^x = 16$ + \end{enumerate} +\end{exercise} + +\begin{solution} + \begin{enumerate} + \item On veut partager cette évolution en 8 évolutions. + \[ + \left(1 + \frac{27}{100}\right)^{\frac{1}{8}} = 1.0303 + \] + Donc le taux d'évolution moyen est + \[ + t_m = 1.0303 - 1 = 0.030299999999999994 + \] + \item Coefficient multiplicateur pour revenir en arrière + \[ + CM = (1 + \frac{27}{100})^{-1} = 0.7874 + \] + On en déduit la quantité en 2019 + \[ + 116 * 0.7874 = 91.3384 + \] + \item L'équation de la droite est + \[ + y = 2.0 x -2 + \] + \item Il faut penser à faire la division à par $7$ avant d'utiliser le log car sinon, on ne peut pas utiliser la formule $\log(a^n) = n\times \log(a)$. + + \[x = \frac{\log(2.29)}{\log(0.23)}\] + \end{enumerate} +\end{solution} + +\begin{exercise}[subtitle={Restaurant}] + Un \emph{food truck}, ouvert le midi et le soir, propose deux types de formules : + + \setlength\parindent{10mm} + \begin{itemize} + \item la formule \emph{Burger} ; + \item la formule \emph{Wok}. + \end{itemize} + \setlength\parindent{0mm} + + \medskip + + Le gérant a remarqué que 53\,\% de ses ventes ont lieu le midi. Le quart des ventes du midi correspondent à la formule \emph{Burger}, alors que 26\,\% des ventes du soir correspondent à la formule \emph{Wok}. + + Le gérant se constitue un fichier en notant, pour chaque vente, la formule choisie et le moment de cette vente (midi ou soir). + + On prélève une fiche de façon équiprobable. On définit les quatre évènements suivants: + + \begin{enumerate} + \item $M$ : \og la fiche correspond à une vente du midi\fg{} ; + \item $S$ : \og la fiche correspond à une vente du soir\fg {}; + \item $W$ : \og la fiche correspond à une formule \emph{Wok} \fg{} ; + \item $B$ : \og la fiche correspond à une formule \emph{Burger} \fg. + \end{enumerate} + \setlength\parindent{0mm} + + \medskip + + \begin{enumerate} + \item Recopier puis compléter l'arbre pondéré + + \begin{center} + \begin{tikzpicture}[sloped] + \node {.} + child {node {$M$} + child {node {$W$} + edge from parent + node[above] {...} + } + child {node {$B$} + edge from parent + node[above] {...} + } + edge from parent + node[above] {...} + } + child[missing] {} + child { node {$S$} + child {node {$W$} + edge from parent + node[above] {...} + } + child {node {$B$} + edge from parent + node[above] {...} + } + edge from parent + node[above] {...} + } ; + \end{tikzpicture} + \end{center} + + \item Calculer la probabilité de l'évènement $M \cap W$. Interpréter ce résultat dans le contexte de l'exercice. + \item Montrer que la probabilité que la fiche choisie corresponde à une formule \emph{Burger} est égale à $0.4803$. + \item On a prélevé une fiche correspondant à la formule \emph{Burger}. Quelle est la probabilité, arrondie au millième, que la vente ait eu lieu le soir? + \end{enumerate} +\end{exercise} + +\begin{solution} + \begin{enumerate} + \item + \begin{center} + \begin{tikzpicture}[sloped] + \node {.} + child {node {$M$} + child {node {$W$} + edge from parent + node[above] {$0.75$} + } + child {node {$B$} + edge from parent + node[above] {$0.25$} + } + edge from parent + node[above] {$0.53$} + } + child[missing] {} + child { node {$S$} + child {node {$W$} + edge from parent + node[above] {$0.26$} + } + child {node {$B$} + edge from parent + node[above] {$0.74$} + } + edge from parent + node[above] {$0.47$} + } ; + \end{tikzpicture} + \end{center} + \item On calcule la probabilité que la vente soit un wok et ait eu lieu à midi + \[ P(M\cap W) = P(M) \times P_M(W) = 0.53 \times 0.75 = 0.3975 \] + \item Probabilité que la vente soit un burger. + \[ + P(B) = P(M\cap B) + P(S\cap B) = 0.53 \times 0.75 + 0.47 \times 0.26 = 0.4803 + \] + \item On cherche à calculer la quantité $P_B(S)$. Pour cela on utilise la formule de Bayes + \[ + P_B(S) = \frac{P(B\cap S)}{P(B)} = \frac{P_S(B) \times P(S)}{P(B)} = \frac{0.74\times 0.47}{0.4803} = 0.7241307516135749 \approx 0.724 + \] + \end{enumerate} +\end{solution} + +\begin{exercise}[subtitle={Continent plastique}] + \textit{Les quantités évoqués dans cette exercice sont générés au hasard et sont donc complètement farfelus.} + \medskip + Le \og continent de plastique\fg{} est la plus grande des plaques de déchets plastiques évoluant sur les océans. Elle occupe actuellement dans l'océan Pacifique une surface dont l'aire est évaluée à plus de $1,6$ million de km$^2$, entre Hawaï et la Californie. + + En 2017, des scientifiques ont estimé qu'il y avait $3$ millions de tonnes de déchets plastiques qui était déversé chaque année dans les océans et que cette quantité augmentait de $17\n\%$ par chaque année. + + On modélise l'évolution de la masse de ces déchets plastiques déversée chaque année, si rien n'est fait pour la réduire, par une suite géométrique $\left(u_n\right)$. L'arrondi au centième du terme $u_n$ représente la masse de ces déchets déversée chaque année, exprimée en million de tonnes, pour l'année $(2017 + n)$. + + \medskip + + \begin{enumerate} + \item Expliquer pourquoi la suite $u_n$ est géométrique? + \item Calculer $u_1$ et $u_2$. + \item Exprimer $u_n$ en fonction de $n$. + \item Au début de l'année 2017, il y avait $300$ millions de tonnes de déchets plastique. Calculer la quantité totale de déchets plastiques en 2030. + \item On souhaite déterminer en quelle année la masse totale de ces déchets plastiques aura pour la première fois augmenté de $50$\,\% par rapport à sa valeur de 2017. + \begin{enumerate} + \item Recopier et compléter l'algorithme ci-dessous pour que la variable $N$ contienne la réponse au problème posé. + + \begin{center} + \begin{tabularx}{0.4\linewidth}{|X|}\hline + $N = 2017$\\ + $U = 3$ \\ + $S = 300 + U$ \\ + while $S < 450$: \\ + \hspace{1cm} $N = \ldots$\\ + \hspace{1cm} $U = \ldots$\\ + \hspace{1cm} $S = \ldots$\\ + \hline + \end{tabularx} + \end{center} + \item Que contiennent les variables $S$, $U$ et $N$ après exécution de cet algorithme ? + + Interpréter les résultats dans le contexte de l'exercice. + \end{enumerate} + \end{enumerate} +\end{exercise} + +\begin{solution} + \begin{enumerate} + \item Une augmentation de $17\,\%$ revient à multiplier la quantité par $1.17$. La suite est donc bien géométrique. Son premier terme est $u_0 = 3$ et sa raison est $q = 1.17$ + \item + \[ + u_1 = u_0 * 1.17 = 3.51 + \] + \[ + u_2 = u_0 * 1.17^2 = 4.1067 + \] + \item + \[ + u_n = u_0 \times q^n = 3 \times 1.17^n + \] + \item On calcule la quantité totale déversée entre 2017 et 2030. + \[ + \sum_{n = 0}^{13} u_n = u_0 \times \frac{1-q^{13}}{1-q} = 3 \times \frac{1 - 1.17^{13}}{1 - 1.17} = 118.21 + \] + On en déduit la quantité totale de déchets en 2030 + \[ + 300 + 118.21 = 418.21 + \] + \item + \begin{enumerate} + \item ~ + \begin{center} + \begin{tabularx}{0.4\linewidth}{|X|}\hline + $N \gets 2017$\\ + $U \gets 3$ \\ + $S \gets 300 + U$ \\ + Tant que $S < 450$ \\ + \hspace{1cm} $N \gets N + 1$\\ + \hspace{1cm} $U \gets U * 1.17$\\ + \hspace{1cm} $S \gets S + u$\\ + Fin Tant que\\\hline + \end{tabularx} + \end{center} + \item \textit{Pas de correction automatisé} + \end{enumerate} + \end{enumerate} +\end{solution} + +\end{document} + +%%% Local Variables: +%%% mode: latex +%%% TeX-master: "master" +%%% End: diff --git a/TST/DS/DS_21_04_07/TST1/corr_07_210407_DS8.tex b/TST/DS/DS_21_04_07/TST1/corr_07_210407_DS8.tex new file mode 100644 index 0000000..d3b2a73 --- /dev/null +++ b/TST/DS/DS_21_04_07/TST1/corr_07_210407_DS8.tex @@ -0,0 +1,262 @@ +\documentclass[a4paper,10pt]{article} +\usepackage{myXsim} + +% Title Page +\title{DS8 \hfill COUBAT Alexis} +\tribe{TST} +\date{\hfillÀ render pour le Mercredi 7 avril} + +\xsimsetup{ + solution/print = true +} + +\begin{document} +\maketitle + +\begin{exercise}[subtitle={Automatismes}] + \textit{Toutes les questions de cette exercice sont indépendantes et peuvent être répondus séparément} + \begin{enumerate} + \item De janvier à septembre, une quantité a augmenté de $16\,\%$. Faire un schéma pour représenter la situation puis calculer le taux d'évolution moyen mensuel. + \item Une quantité augmente de $16\,\%$ par ans. En 2020, elle est de 120\euro. Quelle était sa valeur en 2019? Faire un schéma pour représenter la situation. + \item Déterminer l'équation de la droite \\ + \begin{tikzpicture}[xscale=0.8, yscale=0.5] + \tkzInit[xmin=-5,xmax=5,xstep=1, + ymin=-5,ymax=5,ystep=1] + \tkzGrid + \tkzAxeXY + \tkzFct[domain=-5:5,color=red,very thick]% + {4.0*\x -4}; + \end{tikzpicture} + \item Résoudre l'équation $10 \times 0.06^x = 34$ + \end{enumerate} +\end{exercise} + +\begin{solution} + \begin{enumerate} + \item On veut partager cette évolution en 8 évolutions. + \[ + \left(1 + \frac{16}{100}\right)^{\frac{1}{8}} = 1.0187 + \] + Donc le taux d'évolution moyen est + \[ + t_m = 1.0187 - 1 = 0.01869999999999994 + \] + \item Coefficient multiplicateur pour revenir en arrière + \[ + CM = (1 + \frac{16}{100})^{-1} = 0.8621 + \] + On en déduit la quantité en 2019 + \[ + 120 * 0.8621 = 103.452 + \] + \item L'équation de la droite est + \[ + y = 4.0 x -4 + \] + \item Il faut penser à faire la division à par $10$ avant d'utiliser le log car sinon, on ne peut pas utiliser la formule $\log(a^n) = n\times \log(a)$. + + \[x = \frac{\log(3.4)}{\log(0.06)}\] + \end{enumerate} +\end{solution} + +\begin{exercise}[subtitle={Restaurant}] + Un \emph{food truck}, ouvert le midi et le soir, propose deux types de formules : + + \setlength\parindent{10mm} + \begin{itemize} + \item la formule \emph{Burger} ; + \item la formule \emph{Wok}. + \end{itemize} + \setlength\parindent{0mm} + + \medskip + + Le gérant a remarqué que 1\,\% de ses ventes ont lieu le midi. Le quart des ventes du midi correspondent à la formule \emph{Burger}, alors que 92\,\% des ventes du soir correspondent à la formule \emph{Wok}. + + Le gérant se constitue un fichier en notant, pour chaque vente, la formule choisie et le moment de cette vente (midi ou soir). + + On prélève une fiche de façon équiprobable. On définit les quatre évènements suivants: + + \begin{enumerate} + \item $M$ : \og la fiche correspond à une vente du midi\fg{} ; + \item $S$ : \og la fiche correspond à une vente du soir\fg {}; + \item $W$ : \og la fiche correspond à une formule \emph{Wok} \fg{} ; + \item $B$ : \og la fiche correspond à une formule \emph{Burger} \fg. + \end{enumerate} + \setlength\parindent{0mm} + + \medskip + + \begin{enumerate} + \item Recopier puis compléter l'arbre pondéré + + \begin{center} + \begin{tikzpicture}[sloped] + \node {.} + child {node {$M$} + child {node {$W$} + edge from parent + node[above] {...} + } + child {node {$B$} + edge from parent + node[above] {...} + } + edge from parent + node[above] {...} + } + child[missing] {} + child { node {$S$} + child {node {$W$} + edge from parent + node[above] {...} + } + child {node {$B$} + edge from parent + node[above] {...} + } + edge from parent + node[above] {...} + } ; + \end{tikzpicture} + \end{center} + + \item Calculer la probabilité de l'évènement $M \cap W$. Interpréter ce résultat dans le contexte de l'exercice. + \item Montrer que la probabilité que la fiche choisie corresponde à une formule \emph{Burger} est égale à $0.0817$. + \item On a prélevé une fiche correspondant à la formule \emph{Burger}. Quelle est la probabilité, arrondie au millième, que la vente ait eu lieu le soir? + \end{enumerate} +\end{exercise} + +\begin{solution} + \begin{enumerate} + \item + \begin{center} + \begin{tikzpicture}[sloped] + \node {.} + child {node {$M$} + child {node {$W$} + edge from parent + node[above] {$0.75$} + } + child {node {$B$} + edge from parent + node[above] {$0.25$} + } + edge from parent + node[above] {$0.01$} + } + child[missing] {} + child { node {$S$} + child {node {$W$} + edge from parent + node[above] {$0.92$} + } + child {node {$B$} + edge from parent + node[above] {$0.08$} + } + edge from parent + node[above] {$0.99$} + } ; + \end{tikzpicture} + \end{center} + \item On calcule la probabilité que la vente soit un wok et ait eu lieu à midi + \[ P(M\cap W) = P(M) \times P_M(W) = 0.01 \times 0.75 = 0.0075 \] + \item Probabilité que la vente soit un burger. + \[ + P(B) = P(M\cap B) + P(S\cap B) = 0.01 \times 0.75 + 0.99 \times 0.92 = 0.0817 + \] + \item On cherche à calculer la quantité $P_B(S)$. Pour cela on utilise la formule de Bayes + \[ + P_B(S) = \frac{P(B\cap S)}{P(B)} = \frac{P_S(B) \times P(S)}{P(B)} = \frac{0.08\times 0.99}{0.0817} = 0.9694002447980418 \approx 0.969 + \] + \end{enumerate} +\end{solution} + +\begin{exercise}[subtitle={Continent plastique}] + \textit{Les quantités évoqués dans cette exercice sont générés au hasard et sont donc complètement farfelus.} + \medskip + Le \og continent de plastique\fg{} est la plus grande des plaques de déchets plastiques évoluant sur les océans. Elle occupe actuellement dans l'océan Pacifique une surface dont l'aire est évaluée à plus de $1,6$ million de km$^2$, entre Hawaï et la Californie. + + En 2017, des scientifiques ont estimé qu'il y avait $12$ millions de tonnes de déchets plastiques qui était déversé chaque année dans les océans et que cette quantité augmentait de $14\n\%$ par chaque année. + + On modélise l'évolution de la masse de ces déchets plastiques déversée chaque année, si rien n'est fait pour la réduire, par une suite géométrique $\left(u_n\right)$. L'arrondi au centième du terme $u_n$ représente la masse de ces déchets déversée chaque année, exprimée en million de tonnes, pour l'année $(2017 + n)$. + + \medskip + + \begin{enumerate} + \item Expliquer pourquoi la suite $u_n$ est géométrique? + \item Calculer $u_1$ et $u_2$. + \item Exprimer $u_n$ en fonction de $n$. + \item Au début de l'année 2017, il y avait $300$ millions de tonnes de déchets plastique. Calculer la quantité totale de déchets plastiques en 2030. + \item On souhaite déterminer en quelle année la masse totale de ces déchets plastiques aura pour la première fois augmenté de $50$\,\% par rapport à sa valeur de 2017. + \begin{enumerate} + \item Recopier et compléter l'algorithme ci-dessous pour que la variable $N$ contienne la réponse au problème posé. + + \begin{center} + \begin{tabularx}{0.4\linewidth}{|X|}\hline + $N = 2017$\\ + $U = 12$ \\ + $S = 300 + U$ \\ + while $S < 450$: \\ + \hspace{1cm} $N = \ldots$\\ + \hspace{1cm} $U = \ldots$\\ + \hspace{1cm} $S = \ldots$\\ + \hline + \end{tabularx} + \end{center} + \item Que contiennent les variables $S$, $U$ et $N$ après exécution de cet algorithme ? + + Interpréter les résultats dans le contexte de l'exercice. + \end{enumerate} + \end{enumerate} +\end{exercise} + +\begin{solution} + \begin{enumerate} + \item Une augmentation de $14\,\%$ revient à multiplier la quantité par $1.1400000000000001$. La suite est donc bien géométrique. Son premier terme est $u_0 = 12$ et sa raison est $q = 1.1400000000000001$ + \item + \[ + u_1 = u_0 * 1.1400000000000001 = 13.680000000000001 + \] + \[ + u_2 = u_0 * 1.1400000000000001^2 = 15.5952 + \] + \item + \[ + u_n = u_0 \times q^n = 12 \times 1.1400000000000001^n + \] + \item On calcule la quantité totale déversée entre 2017 et 2030. + \[ + \sum_{n = 0}^{13} u_n = u_0 \times \frac{1-q^{13}}{1-q} = 12 \times \frac{1 - 1.1400000000000001^{13}}{1 - 1.1400000000000001} = 385.06 + \] + On en déduit la quantité totale de déchets en 2030 + \[ + 300 + 385.06 = 685.06 + \] + \item + \begin{enumerate} + \item ~ + \begin{center} + \begin{tabularx}{0.4\linewidth}{|X|}\hline + $N \gets 2017$\\ + $U \gets 12$ \\ + $S \gets 300 + U$ \\ + Tant que $S < 450$ \\ + \hspace{1cm} $N \gets N + 1$\\ + \hspace{1cm} $U \gets U * 1.1400000000000001$\\ + \hspace{1cm} $S \gets S + u$\\ + Fin Tant que\\\hline + \end{tabularx} + \end{center} + \item \textit{Pas de correction automatisé} + \end{enumerate} + \end{enumerate} +\end{solution} + +\end{document} + +%%% Local Variables: +%%% mode: latex +%%% TeX-master: "master" +%%% End: diff --git a/TST/DS/DS_21_04_07/TST1/corr_08_210407_DS8.tex b/TST/DS/DS_21_04_07/TST1/corr_08_210407_DS8.tex new file mode 100644 index 0000000..685989a --- /dev/null +++ b/TST/DS/DS_21_04_07/TST1/corr_08_210407_DS8.tex @@ -0,0 +1,262 @@ +\documentclass[a4paper,10pt]{article} +\usepackage{myXsim} + +% Title Page +\title{DS8 \hfill COULLON Anis} +\tribe{TST} +\date{\hfillÀ render pour le Mercredi 7 avril} + +\xsimsetup{ + solution/print = true +} + +\begin{document} +\maketitle + +\begin{exercise}[subtitle={Automatismes}] + \textit{Toutes les questions de cette exercice sont indépendantes et peuvent être répondus séparément} + \begin{enumerate} + \item De janvier à septembre, une quantité a augmenté de $14\,\%$. Faire un schéma pour représenter la situation puis calculer le taux d'évolution moyen mensuel. + \item Une quantité augmente de $14\,\%$ par ans. En 2020, elle est de 125\euro. Quelle était sa valeur en 2019? Faire un schéma pour représenter la situation. + \item Déterminer l'équation de la droite \\ + \begin{tikzpicture}[xscale=0.8, yscale=0.5] + \tkzInit[xmin=-5,xmax=5,xstep=1, + ymin=-5,ymax=5,ystep=1] + \tkzGrid + \tkzAxeXY + \tkzFct[domain=-5:5,color=red,very thick]% + {0.5*\x -1}; + \end{tikzpicture} + \item Résoudre l'équation $6 \times 0.82^x = 19$ + \end{enumerate} +\end{exercise} + +\begin{solution} + \begin{enumerate} + \item On veut partager cette évolution en 8 évolutions. + \[ + \left(1 + \frac{14}{100}\right)^{\frac{1}{8}} = 1.0165 + \] + Donc le taux d'évolution moyen est + \[ + t_m = 1.0165 - 1 = 0.01649999999999996 + \] + \item Coefficient multiplicateur pour revenir en arrière + \[ + CM = (1 + \frac{14}{100})^{-1} = 0.8772 + \] + On en déduit la quantité en 2019 + \[ + 125 * 0.8772 = 109.64999999999999 + \] + \item L'équation de la droite est + \[ + y = 0.5 x -1 + \] + \item Il faut penser à faire la division à par $6$ avant d'utiliser le log car sinon, on ne peut pas utiliser la formule $\log(a^n) = n\times \log(a)$. + + \[x = \frac{\log(3.17)}{\log(0.82)}\] + \end{enumerate} +\end{solution} + +\begin{exercise}[subtitle={Restaurant}] + Un \emph{food truck}, ouvert le midi et le soir, propose deux types de formules : + + \setlength\parindent{10mm} + \begin{itemize} + \item la formule \emph{Burger} ; + \item la formule \emph{Wok}. + \end{itemize} + \setlength\parindent{0mm} + + \medskip + + Le gérant a remarqué que 30\,\% de ses ventes ont lieu le midi. Le quart des ventes du midi correspondent à la formule \emph{Burger}, alors que 27\,\% des ventes du soir correspondent à la formule \emph{Wok}. + + Le gérant se constitue un fichier en notant, pour chaque vente, la formule choisie et le moment de cette vente (midi ou soir). + + On prélève une fiche de façon équiprobable. On définit les quatre évènements suivants: + + \begin{enumerate} + \item $M$ : \og la fiche correspond à une vente du midi\fg{} ; + \item $S$ : \og la fiche correspond à une vente du soir\fg {}; + \item $W$ : \og la fiche correspond à une formule \emph{Wok} \fg{} ; + \item $B$ : \og la fiche correspond à une formule \emph{Burger} \fg. + \end{enumerate} + \setlength\parindent{0mm} + + \medskip + + \begin{enumerate} + \item Recopier puis compléter l'arbre pondéré + + \begin{center} + \begin{tikzpicture}[sloped] + \node {.} + child {node {$M$} + child {node {$W$} + edge from parent + node[above] {...} + } + child {node {$B$} + edge from parent + node[above] {...} + } + edge from parent + node[above] {...} + } + child[missing] {} + child { node {$S$} + child {node {$W$} + edge from parent + node[above] {...} + } + child {node {$B$} + edge from parent + node[above] {...} + } + edge from parent + node[above] {...} + } ; + \end{tikzpicture} + \end{center} + + \item Calculer la probabilité de l'évènement $M \cap W$. Interpréter ce résultat dans le contexte de l'exercice. + \item Montrer que la probabilité que la fiche choisie corresponde à une formule \emph{Burger} est égale à $0.586$. + \item On a prélevé une fiche correspondant à la formule \emph{Burger}. Quelle est la probabilité, arrondie au millième, que la vente ait eu lieu le soir? + \end{enumerate} +\end{exercise} + +\begin{solution} + \begin{enumerate} + \item + \begin{center} + \begin{tikzpicture}[sloped] + \node {.} + child {node {$M$} + child {node {$W$} + edge from parent + node[above] {$0.75$} + } + child {node {$B$} + edge from parent + node[above] {$0.25$} + } + edge from parent + node[above] {$0.3$} + } + child[missing] {} + child { node {$S$} + child {node {$W$} + edge from parent + node[above] {$0.27$} + } + child {node {$B$} + edge from parent + node[above] {$0.73$} + } + edge from parent + node[above] {$0.7$} + } ; + \end{tikzpicture} + \end{center} + \item On calcule la probabilité que la vente soit un wok et ait eu lieu à midi + \[ P(M\cap W) = P(M) \times P_M(W) = 0.3 \times 0.75 = 0.225 \] + \item Probabilité que la vente soit un burger. + \[ + P(B) = P(M\cap B) + P(S\cap B) = 0.3 \times 0.75 + 0.7 \times 0.27 = 0.586 + \] + \item On cherche à calculer la quantité $P_B(S)$. Pour cela on utilise la formule de Bayes + \[ + P_B(S) = \frac{P(B\cap S)}{P(B)} = \frac{P_S(B) \times P(S)}{P(B)} = \frac{0.73\times 0.7}{0.586} = 0.8720136518771332 \approx 0.872 + \] + \end{enumerate} +\end{solution} + +\begin{exercise}[subtitle={Continent plastique}] + \textit{Les quantités évoqués dans cette exercice sont générés au hasard et sont donc complètement farfelus.} + \medskip + Le \og continent de plastique\fg{} est la plus grande des plaques de déchets plastiques évoluant sur les océans. Elle occupe actuellement dans l'océan Pacifique une surface dont l'aire est évaluée à plus de $1,6$ million de km$^2$, entre Hawaï et la Californie. + + En 2017, des scientifiques ont estimé qu'il y avait $14$ millions de tonnes de déchets plastiques qui était déversé chaque année dans les océans et que cette quantité augmentait de $25\n\%$ par chaque année. + + On modélise l'évolution de la masse de ces déchets plastiques déversée chaque année, si rien n'est fait pour la réduire, par une suite géométrique $\left(u_n\right)$. L'arrondi au centième du terme $u_n$ représente la masse de ces déchets déversée chaque année, exprimée en million de tonnes, pour l'année $(2017 + n)$. + + \medskip + + \begin{enumerate} + \item Expliquer pourquoi la suite $u_n$ est géométrique? + \item Calculer $u_1$ et $u_2$. + \item Exprimer $u_n$ en fonction de $n$. + \item Au début de l'année 2017, il y avait $300$ millions de tonnes de déchets plastique. Calculer la quantité totale de déchets plastiques en 2030. + \item On souhaite déterminer en quelle année la masse totale de ces déchets plastiques aura pour la première fois augmenté de $50$\,\% par rapport à sa valeur de 2017. + \begin{enumerate} + \item Recopier et compléter l'algorithme ci-dessous pour que la variable $N$ contienne la réponse au problème posé. + + \begin{center} + \begin{tabularx}{0.4\linewidth}{|X|}\hline + $N = 2017$\\ + $U = 14$ \\ + $S = 300 + U$ \\ + while $S < 450$: \\ + \hspace{1cm} $N = \ldots$\\ + \hspace{1cm} $U = \ldots$\\ + \hspace{1cm} $S = \ldots$\\ + \hline + \end{tabularx} + \end{center} + \item Que contiennent les variables $S$, $U$ et $N$ après exécution de cet algorithme ? + + Interpréter les résultats dans le contexte de l'exercice. + \end{enumerate} + \end{enumerate} +\end{exercise} + +\begin{solution} + \begin{enumerate} + \item Une augmentation de $25\,\%$ revient à multiplier la quantité par $1.25$. La suite est donc bien géométrique. Son premier terme est $u_0 = 14$ et sa raison est $q = 1.25$ + \item + \[ + u_1 = u_0 * 1.25 = 17.5 + \] + \[ + u_2 = u_0 * 1.25^2 = 21.875 + \] + \item + \[ + u_n = u_0 \times q^n = 14 \times 1.25^n + \] + \item On calcule la quantité totale déversée entre 2017 et 2030. + \[ + \sum_{n = 0}^{13} u_n = u_0 \times \frac{1-q^{13}}{1-q} = 14 \times \frac{1 - 1.25^{13}}{1 - 1.25} = 962.63 + \] + On en déduit la quantité totale de déchets en 2030 + \[ + 300 + 962.63 = 1262.63 + \] + \item + \begin{enumerate} + \item ~ + \begin{center} + \begin{tabularx}{0.4\linewidth}{|X|}\hline + $N \gets 2017$\\ + $U \gets 14$ \\ + $S \gets 300 + U$ \\ + Tant que $S < 450$ \\ + \hspace{1cm} $N \gets N + 1$\\ + \hspace{1cm} $U \gets U * 1.25$\\ + \hspace{1cm} $S \gets S + u$\\ + Fin Tant que\\\hline + \end{tabularx} + \end{center} + \item \textit{Pas de correction automatisé} + \end{enumerate} + \end{enumerate} +\end{solution} + +\end{document} + +%%% Local Variables: +%%% mode: latex +%%% TeX-master: "master" +%%% End: diff --git a/TST/DS/DS_21_04_07/TST1/corr_09_210407_DS8.tex b/TST/DS/DS_21_04_07/TST1/corr_09_210407_DS8.tex new file mode 100644 index 0000000..1b56293 --- /dev/null +++ b/TST/DS/DS_21_04_07/TST1/corr_09_210407_DS8.tex @@ -0,0 +1,262 @@ +\documentclass[a4paper,10pt]{article} +\usepackage{myXsim} + +% Title Page +\title{DS8 \hfill DINGER Sölen} +\tribe{TST} +\date{\hfillÀ render pour le Mercredi 7 avril} + +\xsimsetup{ + solution/print = true +} + +\begin{document} +\maketitle + +\begin{exercise}[subtitle={Automatismes}] + \textit{Toutes les questions de cette exercice sont indépendantes et peuvent être répondus séparément} + \begin{enumerate} + \item De janvier à septembre, une quantité a augmenté de $30\,\%$. Faire un schéma pour représenter la situation puis calculer le taux d'évolution moyen mensuel. + \item Une quantité augmente de $30\,\%$ par ans. En 2020, elle est de 141\euro. Quelle était sa valeur en 2019? Faire un schéma pour représenter la situation. + \item Déterminer l'équation de la droite \\ + \begin{tikzpicture}[xscale=0.8, yscale=0.5] + \tkzInit[xmin=-5,xmax=5,xstep=1, + ymin=-5,ymax=5,ystep=1] + \tkzGrid + \tkzAxeXY + \tkzFct[domain=-5:5,color=red,very thick]% + {2.0*\x -3}; + \end{tikzpicture} + \item Résoudre l'équation $5 \times 0.8^x = 11$ + \end{enumerate} +\end{exercise} + +\begin{solution} + \begin{enumerate} + \item On veut partager cette évolution en 8 évolutions. + \[ + \left(1 + \frac{30}{100}\right)^{\frac{1}{8}} = 1.0333 + \] + Donc le taux d'évolution moyen est + \[ + t_m = 1.0333 - 1 = 0.03330000000000011 + \] + \item Coefficient multiplicateur pour revenir en arrière + \[ + CM = (1 + \frac{30}{100})^{-1} = 0.7692 + \] + On en déduit la quantité en 2019 + \[ + 141 * 0.7692 = 108.4572 + \] + \item L'équation de la droite est + \[ + y = 2.0 x -3 + \] + \item Il faut penser à faire la division à par $5$ avant d'utiliser le log car sinon, on ne peut pas utiliser la formule $\log(a^n) = n\times \log(a)$. + + \[x = \frac{\log(2.2)}{\log(0.8)}\] + \end{enumerate} +\end{solution} + +\begin{exercise}[subtitle={Restaurant}] + Un \emph{food truck}, ouvert le midi et le soir, propose deux types de formules : + + \setlength\parindent{10mm} + \begin{itemize} + \item la formule \emph{Burger} ; + \item la formule \emph{Wok}. + \end{itemize} + \setlength\parindent{0mm} + + \medskip + + Le gérant a remarqué que 57\,\% de ses ventes ont lieu le midi. Le quart des ventes du midi correspondent à la formule \emph{Burger}, alors que 90\,\% des ventes du soir correspondent à la formule \emph{Wok}. + + Le gérant se constitue un fichier en notant, pour chaque vente, la formule choisie et le moment de cette vente (midi ou soir). + + On prélève une fiche de façon équiprobable. On définit les quatre évènements suivants: + + \begin{enumerate} + \item $M$ : \og la fiche correspond à une vente du midi\fg{} ; + \item $S$ : \og la fiche correspond à une vente du soir\fg {}; + \item $W$ : \og la fiche correspond à une formule \emph{Wok} \fg{} ; + \item $B$ : \og la fiche correspond à une formule \emph{Burger} \fg. + \end{enumerate} + \setlength\parindent{0mm} + + \medskip + + \begin{enumerate} + \item Recopier puis compléter l'arbre pondéré + + \begin{center} + \begin{tikzpicture}[sloped] + \node {.} + child {node {$M$} + child {node {$W$} + edge from parent + node[above] {...} + } + child {node {$B$} + edge from parent + node[above] {...} + } + edge from parent + node[above] {...} + } + child[missing] {} + child { node {$S$} + child {node {$W$} + edge from parent + node[above] {...} + } + child {node {$B$} + edge from parent + node[above] {...} + } + edge from parent + node[above] {...} + } ; + \end{tikzpicture} + \end{center} + + \item Calculer la probabilité de l'évènement $M \cap W$. Interpréter ce résultat dans le contexte de l'exercice. + \item Montrer que la probabilité que la fiche choisie corresponde à une formule \emph{Burger} est égale à $0.187$. + \item On a prélevé une fiche correspondant à la formule \emph{Burger}. Quelle est la probabilité, arrondie au millième, que la vente ait eu lieu le soir? + \end{enumerate} +\end{exercise} + +\begin{solution} + \begin{enumerate} + \item + \begin{center} + \begin{tikzpicture}[sloped] + \node {.} + child {node {$M$} + child {node {$W$} + edge from parent + node[above] {$0.75$} + } + child {node {$B$} + edge from parent + node[above] {$0.25$} + } + edge from parent + node[above] {$0.58$} + } + child[missing] {} + child { node {$S$} + child {node {$W$} + edge from parent + node[above] {$0.9$} + } + child {node {$B$} + edge from parent + node[above] {$0.1$} + } + edge from parent + node[above] {$0.42$} + } ; + \end{tikzpicture} + \end{center} + \item On calcule la probabilité que la vente soit un wok et ait eu lieu à midi + \[ P(M\cap W) = P(M) \times P_M(W) = 0.58 \times 0.75 = 0.435 \] + \item Probabilité que la vente soit un burger. + \[ + P(B) = P(M\cap B) + P(S\cap B) = 0.58 \times 0.75 + 0.42 \times 0.9 = 0.187 + \] + \item On cherche à calculer la quantité $P_B(S)$. Pour cela on utilise la formule de Bayes + \[ + P_B(S) = \frac{P(B\cap S)}{P(B)} = \frac{P_S(B) \times P(S)}{P(B)} = \frac{0.1\times 0.42}{0.187} = 0.22459893048128343 \approx 0.225 + \] + \end{enumerate} +\end{solution} + +\begin{exercise}[subtitle={Continent plastique}] + \textit{Les quantités évoqués dans cette exercice sont générés au hasard et sont donc complètement farfelus.} + \medskip + Le \og continent de plastique\fg{} est la plus grande des plaques de déchets plastiques évoluant sur les océans. Elle occupe actuellement dans l'océan Pacifique une surface dont l'aire est évaluée à plus de $1,6$ million de km$^2$, entre Hawaï et la Californie. + + En 2017, des scientifiques ont estimé qu'il y avait $14$ millions de tonnes de déchets plastiques qui était déversé chaque année dans les océans et que cette quantité augmentait de $19\n\%$ par chaque année. + + On modélise l'évolution de la masse de ces déchets plastiques déversée chaque année, si rien n'est fait pour la réduire, par une suite géométrique $\left(u_n\right)$. L'arrondi au centième du terme $u_n$ représente la masse de ces déchets déversée chaque année, exprimée en million de tonnes, pour l'année $(2017 + n)$. + + \medskip + + \begin{enumerate} + \item Expliquer pourquoi la suite $u_n$ est géométrique? + \item Calculer $u_1$ et $u_2$. + \item Exprimer $u_n$ en fonction de $n$. + \item Au début de l'année 2017, il y avait $300$ millions de tonnes de déchets plastique. Calculer la quantité totale de déchets plastiques en 2030. + \item On souhaite déterminer en quelle année la masse totale de ces déchets plastiques aura pour la première fois augmenté de $50$\,\% par rapport à sa valeur de 2017. + \begin{enumerate} + \item Recopier et compléter l'algorithme ci-dessous pour que la variable $N$ contienne la réponse au problème posé. + + \begin{center} + \begin{tabularx}{0.4\linewidth}{|X|}\hline + $N = 2017$\\ + $U = 14$ \\ + $S = 300 + U$ \\ + while $S < 450$: \\ + \hspace{1cm} $N = \ldots$\\ + \hspace{1cm} $U = \ldots$\\ + \hspace{1cm} $S = \ldots$\\ + \hline + \end{tabularx} + \end{center} + \item Que contiennent les variables $S$, $U$ et $N$ après exécution de cet algorithme ? + + Interpréter les résultats dans le contexte de l'exercice. + \end{enumerate} + \end{enumerate} +\end{exercise} + +\begin{solution} + \begin{enumerate} + \item Une augmentation de $19\,\%$ revient à multiplier la quantité par $1.19$. La suite est donc bien géométrique. Son premier terme est $u_0 = 14$ et sa raison est $q = 1.19$ + \item + \[ + u_1 = u_0 * 1.19 = 16.66 + \] + \[ + u_2 = u_0 * 1.19^2 = 19.8254 + \] + \item + \[ + u_n = u_0 \times q^n = 14 \times 1.19^n + \] + \item On calcule la quantité totale déversée entre 2017 et 2030. + \[ + \sum_{n = 0}^{13} u_n = u_0 \times \frac{1-q^{13}}{1-q} = 14 \times \frac{1 - 1.19^{13}}{1 - 1.19} = 633.42 + \] + On en déduit la quantité totale de déchets en 2030 + \[ + 300 + 633.42 = 933.42 + \] + \item + \begin{enumerate} + \item ~ + \begin{center} + \begin{tabularx}{0.4\linewidth}{|X|}\hline + $N \gets 2017$\\ + $U \gets 14$ \\ + $S \gets 300 + U$ \\ + Tant que $S < 450$ \\ + \hspace{1cm} $N \gets N + 1$\\ + \hspace{1cm} $U \gets U * 1.19$\\ + \hspace{1cm} $S \gets S + u$\\ + Fin Tant que\\\hline + \end{tabularx} + \end{center} + \item \textit{Pas de correction automatisé} + \end{enumerate} + \end{enumerate} +\end{solution} + +\end{document} + +%%% Local Variables: +%%% mode: latex +%%% TeX-master: "master" +%%% End: diff --git a/TST/DS/DS_21_04_07/TST1/corr_10_210407_DS8.tex b/TST/DS/DS_21_04_07/TST1/corr_10_210407_DS8.tex new file mode 100644 index 0000000..a239d0e --- /dev/null +++ b/TST/DS/DS_21_04_07/TST1/corr_10_210407_DS8.tex @@ -0,0 +1,262 @@ +\documentclass[a4paper,10pt]{article} +\usepackage{myXsim} + +% Title Page +\title{DS8 \hfill EYRAUD Cynthia} +\tribe{TST} +\date{\hfillÀ render pour le Mercredi 7 avril} + +\xsimsetup{ + solution/print = true +} + +\begin{document} +\maketitle + +\begin{exercise}[subtitle={Automatismes}] + \textit{Toutes les questions de cette exercice sont indépendantes et peuvent être répondus séparément} + \begin{enumerate} + \item De janvier à septembre, une quantité a augmenté de $29\,\%$. Faire un schéma pour représenter la situation puis calculer le taux d'évolution moyen mensuel. + \item Une quantité augmente de $29\,\%$ par ans. En 2020, elle est de 130\euro. Quelle était sa valeur en 2019? Faire un schéma pour représenter la situation. + \item Déterminer l'équation de la droite \\ + \begin{tikzpicture}[xscale=0.8, yscale=0.5] + \tkzInit[xmin=-5,xmax=5,xstep=1, + ymin=-5,ymax=5,ystep=1] + \tkzGrid + \tkzAxeXY + \tkzFct[domain=-5:5,color=red,very thick]% + {4.0*\x -4}; + \end{tikzpicture} + \item Résoudre l'équation $6 \times 0.48^x = 28$ + \end{enumerate} +\end{exercise} + +\begin{solution} + \begin{enumerate} + \item On veut partager cette évolution en 8 évolutions. + \[ + \left(1 + \frac{29}{100}\right)^{\frac{1}{8}} = 1.0323 + \] + Donc le taux d'évolution moyen est + \[ + t_m = 1.0323 - 1 = 0.032299999999999995 + \] + \item Coefficient multiplicateur pour revenir en arrière + \[ + CM = (1 + \frac{29}{100})^{-1} = 0.7752 + \] + On en déduit la quantité en 2019 + \[ + 130 * 0.7752 = 100.776 + \] + \item L'équation de la droite est + \[ + y = 4.0 x -4 + \] + \item Il faut penser à faire la division à par $6$ avant d'utiliser le log car sinon, on ne peut pas utiliser la formule $\log(a^n) = n\times \log(a)$. + + \[x = \frac{\log(4.67)}{\log(0.48)}\] + \end{enumerate} +\end{solution} + +\begin{exercise}[subtitle={Restaurant}] + Un \emph{food truck}, ouvert le midi et le soir, propose deux types de formules : + + \setlength\parindent{10mm} + \begin{itemize} + \item la formule \emph{Burger} ; + \item la formule \emph{Wok}. + \end{itemize} + \setlength\parindent{0mm} + + \medskip + + Le gérant a remarqué que 3\,\% de ses ventes ont lieu le midi. Le quart des ventes du midi correspondent à la formule \emph{Burger}, alors que 33\,\% des ventes du soir correspondent à la formule \emph{Wok}. + + Le gérant se constitue un fichier en notant, pour chaque vente, la formule choisie et le moment de cette vente (midi ou soir). + + On prélève une fiche de façon équiprobable. On définit les quatre évènements suivants: + + \begin{enumerate} + \item $M$ : \og la fiche correspond à une vente du midi\fg{} ; + \item $S$ : \og la fiche correspond à une vente du soir\fg {}; + \item $W$ : \og la fiche correspond à une formule \emph{Wok} \fg{} ; + \item $B$ : \og la fiche correspond à une formule \emph{Burger} \fg. + \end{enumerate} + \setlength\parindent{0mm} + + \medskip + + \begin{enumerate} + \item Recopier puis compléter l'arbre pondéré + + \begin{center} + \begin{tikzpicture}[sloped] + \node {.} + child {node {$M$} + child {node {$W$} + edge from parent + node[above] {...} + } + child {node {$B$} + edge from parent + node[above] {...} + } + edge from parent + node[above] {...} + } + child[missing] {} + child { node {$S$} + child {node {$W$} + edge from parent + node[above] {...} + } + child {node {$B$} + edge from parent + node[above] {...} + } + edge from parent + node[above] {...} + } ; + \end{tikzpicture} + \end{center} + + \item Calculer la probabilité de l'évènement $M \cap W$. Interpréter ce résultat dans le contexte de l'exercice. + \item Montrer que la probabilité que la fiche choisie corresponde à une formule \emph{Burger} est égale à $0.6574$. + \item On a prélevé une fiche correspondant à la formule \emph{Burger}. Quelle est la probabilité, arrondie au millième, que la vente ait eu lieu le soir? + \end{enumerate} +\end{exercise} + +\begin{solution} + \begin{enumerate} + \item + \begin{center} + \begin{tikzpicture}[sloped] + \node {.} + child {node {$M$} + child {node {$W$} + edge from parent + node[above] {$0.75$} + } + child {node {$B$} + edge from parent + node[above] {$0.25$} + } + edge from parent + node[above] {$0.03$} + } + child[missing] {} + child { node {$S$} + child {node {$W$} + edge from parent + node[above] {$0.33$} + } + child {node {$B$} + edge from parent + node[above] {$0.67$} + } + edge from parent + node[above] {$0.97$} + } ; + \end{tikzpicture} + \end{center} + \item On calcule la probabilité que la vente soit un wok et ait eu lieu à midi + \[ P(M\cap W) = P(M) \times P_M(W) = 0.03 \times 0.75 = 0.0225 \] + \item Probabilité que la vente soit un burger. + \[ + P(B) = P(M\cap B) + P(S\cap B) = 0.03 \times 0.75 + 0.97 \times 0.33 = 0.6574 + \] + \item On cherche à calculer la quantité $P_B(S)$. Pour cela on utilise la formule de Bayes + \[ + P_B(S) = \frac{P(B\cap S)}{P(B)} = \frac{P_S(B) \times P(S)}{P(B)} = \frac{0.67\times 0.97}{0.6574} = 0.9885914207484029 \approx 0.989 + \] + \end{enumerate} +\end{solution} + +\begin{exercise}[subtitle={Continent plastique}] + \textit{Les quantités évoqués dans cette exercice sont générés au hasard et sont donc complètement farfelus.} + \medskip + Le \og continent de plastique\fg{} est la plus grande des plaques de déchets plastiques évoluant sur les océans. Elle occupe actuellement dans l'océan Pacifique une surface dont l'aire est évaluée à plus de $1,6$ million de km$^2$, entre Hawaï et la Californie. + + En 2017, des scientifiques ont estimé qu'il y avait $18$ millions de tonnes de déchets plastiques qui était déversé chaque année dans les océans et que cette quantité augmentait de $25\n\%$ par chaque année. + + On modélise l'évolution de la masse de ces déchets plastiques déversée chaque année, si rien n'est fait pour la réduire, par une suite géométrique $\left(u_n\right)$. L'arrondi au centième du terme $u_n$ représente la masse de ces déchets déversée chaque année, exprimée en million de tonnes, pour l'année $(2017 + n)$. + + \medskip + + \begin{enumerate} + \item Expliquer pourquoi la suite $u_n$ est géométrique? + \item Calculer $u_1$ et $u_2$. + \item Exprimer $u_n$ en fonction de $n$. + \item Au début de l'année 2017, il y avait $300$ millions de tonnes de déchets plastique. Calculer la quantité totale de déchets plastiques en 2030. + \item On souhaite déterminer en quelle année la masse totale de ces déchets plastiques aura pour la première fois augmenté de $50$\,\% par rapport à sa valeur de 2017. + \begin{enumerate} + \item Recopier et compléter l'algorithme ci-dessous pour que la variable $N$ contienne la réponse au problème posé. + + \begin{center} + \begin{tabularx}{0.4\linewidth}{|X|}\hline + $N = 2017$\\ + $U = 18$ \\ + $S = 300 + U$ \\ + while $S < 450$: \\ + \hspace{1cm} $N = \ldots$\\ + \hspace{1cm} $U = \ldots$\\ + \hspace{1cm} $S = \ldots$\\ + \hline + \end{tabularx} + \end{center} + \item Que contiennent les variables $S$, $U$ et $N$ après exécution de cet algorithme ? + + Interpréter les résultats dans le contexte de l'exercice. + \end{enumerate} + \end{enumerate} +\end{exercise} + +\begin{solution} + \begin{enumerate} + \item Une augmentation de $25\,\%$ revient à multiplier la quantité par $1.25$. La suite est donc bien géométrique. Son premier terme est $u_0 = 18$ et sa raison est $q = 1.25$ + \item + \[ + u_1 = u_0 * 1.25 = 22.5 + \] + \[ + u_2 = u_0 * 1.25^2 = 28.125 + \] + \item + \[ + u_n = u_0 \times q^n = 18 \times 1.25^n + \] + \item On calcule la quantité totale déversée entre 2017 et 2030. + \[ + \sum_{n = 0}^{13} u_n = u_0 \times \frac{1-q^{13}}{1-q} = 18 \times \frac{1 - 1.25^{13}}{1 - 1.25} = 1237.67 + \] + On en déduit la quantité totale de déchets en 2030 + \[ + 300 + 1237.67 = 1537.67 + \] + \item + \begin{enumerate} + \item ~ + \begin{center} + \begin{tabularx}{0.4\linewidth}{|X|}\hline + $N \gets 2017$\\ + $U \gets 18$ \\ + $S \gets 300 + U$ \\ + Tant que $S < 450$ \\ + \hspace{1cm} $N \gets N + 1$\\ + \hspace{1cm} $U \gets U * 1.25$\\ + \hspace{1cm} $S \gets S + u$\\ + Fin Tant que\\\hline + \end{tabularx} + \end{center} + \item \textit{Pas de correction automatisé} + \end{enumerate} + \end{enumerate} +\end{solution} + +\end{document} + +%%% Local Variables: +%%% mode: latex +%%% TeX-master: "master" +%%% End: diff --git a/TST/DS/DS_21_04_07/TST1/corr_11_210407_DS8.tex b/TST/DS/DS_21_04_07/TST1/corr_11_210407_DS8.tex new file mode 100644 index 0000000..a121a83 --- /dev/null +++ b/TST/DS/DS_21_04_07/TST1/corr_11_210407_DS8.tex @@ -0,0 +1,262 @@ +\documentclass[a4paper,10pt]{article} +\usepackage{myXsim} + +% Title Page +\title{DS8 \hfill FERREIRA Léo} +\tribe{TST} +\date{\hfillÀ render pour le Mercredi 7 avril} + +\xsimsetup{ + solution/print = true +} + +\begin{document} +\maketitle + +\begin{exercise}[subtitle={Automatismes}] + \textit{Toutes les questions de cette exercice sont indépendantes et peuvent être répondus séparément} + \begin{enumerate} + \item De janvier à septembre, une quantité a augmenté de $10\,\%$. Faire un schéma pour représenter la situation puis calculer le taux d'évolution moyen mensuel. + \item Une quantité augmente de $10\,\%$ par ans. En 2020, elle est de 123\euro. Quelle était sa valeur en 2019? Faire un schéma pour représenter la situation. + \item Déterminer l'équation de la droite \\ + \begin{tikzpicture}[xscale=0.8, yscale=0.5] + \tkzInit[xmin=-5,xmax=5,xstep=1, + ymin=-5,ymax=5,ystep=1] + \tkzGrid + \tkzAxeXY + \tkzFct[domain=-5:5,color=red,very thick]% + {1.5*\x -3}; + \end{tikzpicture} + \item Résoudre l'équation $5 \times 0.79^x = 30$ + \end{enumerate} +\end{exercise} + +\begin{solution} + \begin{enumerate} + \item On veut partager cette évolution en 8 évolutions. + \[ + \left(1 + \frac{10}{100}\right)^{\frac{1}{8}} = 1.012 + \] + Donc le taux d'évolution moyen est + \[ + t_m = 1.012 - 1 = 0.01200000000000001 + \] + \item Coefficient multiplicateur pour revenir en arrière + \[ + CM = (1 + \frac{10}{100})^{-1} = 0.9091 + \] + On en déduit la quantité en 2019 + \[ + 123 * 0.9091 = 111.8193 + \] + \item L'équation de la droite est + \[ + y = 1.5 x -3 + \] + \item Il faut penser à faire la division à par $5$ avant d'utiliser le log car sinon, on ne peut pas utiliser la formule $\log(a^n) = n\times \log(a)$. + + \[x = \frac{\log(6.0)}{\log(0.79)}\] + \end{enumerate} +\end{solution} + +\begin{exercise}[subtitle={Restaurant}] + Un \emph{food truck}, ouvert le midi et le soir, propose deux types de formules : + + \setlength\parindent{10mm} + \begin{itemize} + \item la formule \emph{Burger} ; + \item la formule \emph{Wok}. + \end{itemize} + \setlength\parindent{0mm} + + \medskip + + Le gérant a remarqué que 18\,\% de ses ventes ont lieu le midi. Le quart des ventes du midi correspondent à la formule \emph{Burger}, alors que 37\,\% des ventes du soir correspondent à la formule \emph{Wok}. + + Le gérant se constitue un fichier en notant, pour chaque vente, la formule choisie et le moment de cette vente (midi ou soir). + + On prélève une fiche de façon équiprobable. On définit les quatre évènements suivants: + + \begin{enumerate} + \item $M$ : \og la fiche correspond à une vente du midi\fg{} ; + \item $S$ : \og la fiche correspond à une vente du soir\fg {}; + \item $W$ : \og la fiche correspond à une formule \emph{Wok} \fg{} ; + \item $B$ : \og la fiche correspond à une formule \emph{Burger} \fg. + \end{enumerate} + \setlength\parindent{0mm} + + \medskip + + \begin{enumerate} + \item Recopier puis compléter l'arbre pondéré + + \begin{center} + \begin{tikzpicture}[sloped] + \node {.} + child {node {$M$} + child {node {$W$} + edge from parent + node[above] {...} + } + child {node {$B$} + edge from parent + node[above] {...} + } + edge from parent + node[above] {...} + } + child[missing] {} + child { node {$S$} + child {node {$W$} + edge from parent + node[above] {...} + } + child {node {$B$} + edge from parent + node[above] {...} + } + edge from parent + node[above] {...} + } ; + \end{tikzpicture} + \end{center} + + \item Calculer la probabilité de l'évènement $M \cap W$. Interpréter ce résultat dans le contexte de l'exercice. + \item Montrer que la probabilité que la fiche choisie corresponde à une formule \emph{Burger} est égale à $0.5616$. + \item On a prélevé une fiche correspondant à la formule \emph{Burger}. Quelle est la probabilité, arrondie au millième, que la vente ait eu lieu le soir? + \end{enumerate} +\end{exercise} + +\begin{solution} + \begin{enumerate} + \item + \begin{center} + \begin{tikzpicture}[sloped] + \node {.} + child {node {$M$} + child {node {$W$} + edge from parent + node[above] {$0.75$} + } + child {node {$B$} + edge from parent + node[above] {$0.25$} + } + edge from parent + node[above] {$0.18$} + } + child[missing] {} + child { node {$S$} + child {node {$W$} + edge from parent + node[above] {$0.37$} + } + child {node {$B$} + edge from parent + node[above] {$0.63$} + } + edge from parent + node[above] {$0.82$} + } ; + \end{tikzpicture} + \end{center} + \item On calcule la probabilité que la vente soit un wok et ait eu lieu à midi + \[ P(M\cap W) = P(M) \times P_M(W) = 0.18 \times 0.75 = 0.135 \] + \item Probabilité que la vente soit un burger. + \[ + P(B) = P(M\cap B) + P(S\cap B) = 0.18 \times 0.75 + 0.82 \times 0.37 = 0.5616 + \] + \item On cherche à calculer la quantité $P_B(S)$. Pour cela on utilise la formule de Bayes + \[ + P_B(S) = \frac{P(B\cap S)}{P(B)} = \frac{P_S(B) \times P(S)}{P(B)} = \frac{0.63\times 0.82}{0.5616} = 0.9198717948717948 \approx 0.92 + \] + \end{enumerate} +\end{solution} + +\begin{exercise}[subtitle={Continent plastique}] + \textit{Les quantités évoqués dans cette exercice sont générés au hasard et sont donc complètement farfelus.} + \medskip + Le \og continent de plastique\fg{} est la plus grande des plaques de déchets plastiques évoluant sur les océans. Elle occupe actuellement dans l'océan Pacifique une surface dont l'aire est évaluée à plus de $1,6$ million de km$^2$, entre Hawaï et la Californie. + + En 2017, des scientifiques ont estimé qu'il y avait $3$ millions de tonnes de déchets plastiques qui était déversé chaque année dans les océans et que cette quantité augmentait de $19\n\%$ par chaque année. + + On modélise l'évolution de la masse de ces déchets plastiques déversée chaque année, si rien n'est fait pour la réduire, par une suite géométrique $\left(u_n\right)$. L'arrondi au centième du terme $u_n$ représente la masse de ces déchets déversée chaque année, exprimée en million de tonnes, pour l'année $(2017 + n)$. + + \medskip + + \begin{enumerate} + \item Expliquer pourquoi la suite $u_n$ est géométrique? + \item Calculer $u_1$ et $u_2$. + \item Exprimer $u_n$ en fonction de $n$. + \item Au début de l'année 2017, il y avait $300$ millions de tonnes de déchets plastique. Calculer la quantité totale de déchets plastiques en 2030. + \item On souhaite déterminer en quelle année la masse totale de ces déchets plastiques aura pour la première fois augmenté de $50$\,\% par rapport à sa valeur de 2017. + \begin{enumerate} + \item Recopier et compléter l'algorithme ci-dessous pour que la variable $N$ contienne la réponse au problème posé. + + \begin{center} + \begin{tabularx}{0.4\linewidth}{|X|}\hline + $N = 2017$\\ + $U = 3$ \\ + $S = 300 + U$ \\ + while $S < 450$: \\ + \hspace{1cm} $N = \ldots$\\ + \hspace{1cm} $U = \ldots$\\ + \hspace{1cm} $S = \ldots$\\ + \hline + \end{tabularx} + \end{center} + \item Que contiennent les variables $S$, $U$ et $N$ après exécution de cet algorithme ? + + Interpréter les résultats dans le contexte de l'exercice. + \end{enumerate} + \end{enumerate} +\end{exercise} + +\begin{solution} + \begin{enumerate} + \item Une augmentation de $19\,\%$ revient à multiplier la quantité par $1.19$. La suite est donc bien géométrique. Son premier terme est $u_0 = 3$ et sa raison est $q = 1.19$ + \item + \[ + u_1 = u_0 * 1.19 = 3.57 + \] + \[ + u_2 = u_0 * 1.19^2 = 4.2483 + \] + \item + \[ + u_n = u_0 \times q^n = 3 \times 1.19^n + \] + \item On calcule la quantité totale déversée entre 2017 et 2030. + \[ + \sum_{n = 0}^{13} u_n = u_0 \times \frac{1-q^{13}}{1-q} = 3 \times \frac{1 - 1.19^{13}}{1 - 1.19} = 135.73 + \] + On en déduit la quantité totale de déchets en 2030 + \[ + 300 + 135.73 = 435.73 + \] + \item + \begin{enumerate} + \item ~ + \begin{center} + \begin{tabularx}{0.4\linewidth}{|X|}\hline + $N \gets 2017$\\ + $U \gets 3$ \\ + $S \gets 300 + U$ \\ + Tant que $S < 450$ \\ + \hspace{1cm} $N \gets N + 1$\\ + \hspace{1cm} $U \gets U * 1.19$\\ + \hspace{1cm} $S \gets S + u$\\ + Fin Tant que\\\hline + \end{tabularx} + \end{center} + \item \textit{Pas de correction automatisé} + \end{enumerate} + \end{enumerate} +\end{solution} + +\end{document} + +%%% Local Variables: +%%% mode: latex +%%% TeX-master: "master" +%%% End: diff --git a/TST/DS/DS_21_04_07/TST1/corr_12_210407_DS8.tex b/TST/DS/DS_21_04_07/TST1/corr_12_210407_DS8.tex new file mode 100644 index 0000000..f8e1879 --- /dev/null +++ b/TST/DS/DS_21_04_07/TST1/corr_12_210407_DS8.tex @@ -0,0 +1,262 @@ +\documentclass[a4paper,10pt]{article} +\usepackage{myXsim} + +% Title Page +\title{DS8 \hfill FILALI Zakaria} +\tribe{TST} +\date{\hfillÀ render pour le Mercredi 7 avril} + +\xsimsetup{ + solution/print = true +} + +\begin{document} +\maketitle + +\begin{exercise}[subtitle={Automatismes}] + \textit{Toutes les questions de cette exercice sont indépendantes et peuvent être répondus séparément} + \begin{enumerate} + \item De janvier à septembre, une quantité a augmenté de $19\,\%$. Faire un schéma pour représenter la situation puis calculer le taux d'évolution moyen mensuel. + \item Une quantité augmente de $19\,\%$ par ans. En 2020, elle est de 122\euro. Quelle était sa valeur en 2019? Faire un schéma pour représenter la situation. + \item Déterminer l'équation de la droite \\ + \begin{tikzpicture}[xscale=0.8, yscale=0.5] + \tkzInit[xmin=-5,xmax=5,xstep=1, + ymin=-5,ymax=5,ystep=1] + \tkzGrid + \tkzAxeXY + \tkzFct[domain=-5:5,color=red,very thick]% + {1.0*\x -2}; + \end{tikzpicture} + \item Résoudre l'équation $8 \times 0.25^x = 25$ + \end{enumerate} +\end{exercise} + +\begin{solution} + \begin{enumerate} + \item On veut partager cette évolution en 8 évolutions. + \[ + \left(1 + \frac{19}{100}\right)^{\frac{1}{8}} = 1.022 + \] + Donc le taux d'évolution moyen est + \[ + t_m = 1.022 - 1 = 0.02200000000000002 + \] + \item Coefficient multiplicateur pour revenir en arrière + \[ + CM = (1 + \frac{19}{100})^{-1} = 0.8403 + \] + On en déduit la quantité en 2019 + \[ + 122 * 0.8403 = 102.51660000000001 + \] + \item L'équation de la droite est + \[ + y = 1.0 x -2 + \] + \item Il faut penser à faire la division à par $8$ avant d'utiliser le log car sinon, on ne peut pas utiliser la formule $\log(a^n) = n\times \log(a)$. + + \[x = \frac{\log(3.12)}{\log(0.25)}\] + \end{enumerate} +\end{solution} + +\begin{exercise}[subtitle={Restaurant}] + Un \emph{food truck}, ouvert le midi et le soir, propose deux types de formules : + + \setlength\parindent{10mm} + \begin{itemize} + \item la formule \emph{Burger} ; + \item la formule \emph{Wok}. + \end{itemize} + \setlength\parindent{0mm} + + \medskip + + Le gérant a remarqué que 47\,\% de ses ventes ont lieu le midi. Le quart des ventes du midi correspondent à la formule \emph{Burger}, alors que 69\,\% des ventes du soir correspondent à la formule \emph{Wok}. + + Le gérant se constitue un fichier en notant, pour chaque vente, la formule choisie et le moment de cette vente (midi ou soir). + + On prélève une fiche de façon équiprobable. On définit les quatre évènements suivants: + + \begin{enumerate} + \item $M$ : \og la fiche correspond à une vente du midi\fg{} ; + \item $S$ : \og la fiche correspond à une vente du soir\fg {}; + \item $W$ : \og la fiche correspond à une formule \emph{Wok} \fg{} ; + \item $B$ : \og la fiche correspond à une formule \emph{Burger} \fg. + \end{enumerate} + \setlength\parindent{0mm} + + \medskip + + \begin{enumerate} + \item Recopier puis compléter l'arbre pondéré + + \begin{center} + \begin{tikzpicture}[sloped] + \node {.} + child {node {$M$} + child {node {$W$} + edge from parent + node[above] {...} + } + child {node {$B$} + edge from parent + node[above] {...} + } + edge from parent + node[above] {...} + } + child[missing] {} + child { node {$S$} + child {node {$W$} + edge from parent + node[above] {...} + } + child {node {$B$} + edge from parent + node[above] {...} + } + edge from parent + node[above] {...} + } ; + \end{tikzpicture} + \end{center} + + \item Calculer la probabilité de l'évènement $M \cap W$. Interpréter ce résultat dans le contexte de l'exercice. + \item Montrer que la probabilité que la fiche choisie corresponde à une formule \emph{Burger} est égale à $0.2818$. + \item On a prélevé une fiche correspondant à la formule \emph{Burger}. Quelle est la probabilité, arrondie au millième, que la vente ait eu lieu le soir? + \end{enumerate} +\end{exercise} + +\begin{solution} + \begin{enumerate} + \item + \begin{center} + \begin{tikzpicture}[sloped] + \node {.} + child {node {$M$} + child {node {$W$} + edge from parent + node[above] {$0.75$} + } + child {node {$B$} + edge from parent + node[above] {$0.25$} + } + edge from parent + node[above] {$0.47$} + } + child[missing] {} + child { node {$S$} + child {node {$W$} + edge from parent + node[above] {$0.69$} + } + child {node {$B$} + edge from parent + node[above] {$0.31$} + } + edge from parent + node[above] {$0.53$} + } ; + \end{tikzpicture} + \end{center} + \item On calcule la probabilité que la vente soit un wok et ait eu lieu à midi + \[ P(M\cap W) = P(M) \times P_M(W) = 0.47 \times 0.75 = 0.3525 \] + \item Probabilité que la vente soit un burger. + \[ + P(B) = P(M\cap B) + P(S\cap B) = 0.47 \times 0.75 + 0.53 \times 0.69 = 0.2818 + \] + \item On cherche à calculer la quantité $P_B(S)$. Pour cela on utilise la formule de Bayes + \[ + P_B(S) = \frac{P(B\cap S)}{P(B)} = \frac{P_S(B) \times P(S)}{P(B)} = \frac{0.31\times 0.53}{0.2818} = 0.5830376153300213 \approx 0.583 + \] + \end{enumerate} +\end{solution} + +\begin{exercise}[subtitle={Continent plastique}] + \textit{Les quantités évoqués dans cette exercice sont générés au hasard et sont donc complètement farfelus.} + \medskip + Le \og continent de plastique\fg{} est la plus grande des plaques de déchets plastiques évoluant sur les océans. Elle occupe actuellement dans l'océan Pacifique une surface dont l'aire est évaluée à plus de $1,6$ million de km$^2$, entre Hawaï et la Californie. + + En 2017, des scientifiques ont estimé qu'il y avait $11$ millions de tonnes de déchets plastiques qui était déversé chaque année dans les océans et que cette quantité augmentait de $16\n\%$ par chaque année. + + On modélise l'évolution de la masse de ces déchets plastiques déversée chaque année, si rien n'est fait pour la réduire, par une suite géométrique $\left(u_n\right)$. L'arrondi au centième du terme $u_n$ représente la masse de ces déchets déversée chaque année, exprimée en million de tonnes, pour l'année $(2017 + n)$. + + \medskip + + \begin{enumerate} + \item Expliquer pourquoi la suite $u_n$ est géométrique? + \item Calculer $u_1$ et $u_2$. + \item Exprimer $u_n$ en fonction de $n$. + \item Au début de l'année 2017, il y avait $300$ millions de tonnes de déchets plastique. Calculer la quantité totale de déchets plastiques en 2030. + \item On souhaite déterminer en quelle année la masse totale de ces déchets plastiques aura pour la première fois augmenté de $50$\,\% par rapport à sa valeur de 2017. + \begin{enumerate} + \item Recopier et compléter l'algorithme ci-dessous pour que la variable $N$ contienne la réponse au problème posé. + + \begin{center} + \begin{tabularx}{0.4\linewidth}{|X|}\hline + $N = 2017$\\ + $U = 11$ \\ + $S = 300 + U$ \\ + while $S < 450$: \\ + \hspace{1cm} $N = \ldots$\\ + \hspace{1cm} $U = \ldots$\\ + \hspace{1cm} $S = \ldots$\\ + \hline + \end{tabularx} + \end{center} + \item Que contiennent les variables $S$, $U$ et $N$ après exécution de cet algorithme ? + + Interpréter les résultats dans le contexte de l'exercice. + \end{enumerate} + \end{enumerate} +\end{exercise} + +\begin{solution} + \begin{enumerate} + \item Une augmentation de $16\,\%$ revient à multiplier la quantité par $1.16$. La suite est donc bien géométrique. Son premier terme est $u_0 = 11$ et sa raison est $q = 1.16$ + \item + \[ + u_1 = u_0 * 1.16 = 12.76 + \] + \[ + u_2 = u_0 * 1.16^2 = 14.8016 + \] + \item + \[ + u_n = u_0 \times q^n = 11 \times 1.16^n + \] + \item On calcule la quantité totale déversée entre 2017 et 2030. + \[ + \sum_{n = 0}^{13} u_n = u_0 \times \frac{1-q^{13}}{1-q} = 11 \times \frac{1 - 1.16^{13}}{1 - 1.16} = 404.65 + \] + On en déduit la quantité totale de déchets en 2030 + \[ + 300 + 404.65 = 704.65 + \] + \item + \begin{enumerate} + \item ~ + \begin{center} + \begin{tabularx}{0.4\linewidth}{|X|}\hline + $N \gets 2017$\\ + $U \gets 11$ \\ + $S \gets 300 + U$ \\ + Tant que $S < 450$ \\ + \hspace{1cm} $N \gets N + 1$\\ + \hspace{1cm} $U \gets U * 1.16$\\ + \hspace{1cm} $S \gets S + u$\\ + Fin Tant que\\\hline + \end{tabularx} + \end{center} + \item \textit{Pas de correction automatisé} + \end{enumerate} + \end{enumerate} +\end{solution} + +\end{document} + +%%% Local Variables: +%%% mode: latex +%%% TeX-master: "master" +%%% End: diff --git a/TST/DS/DS_21_04_07/TST1/corr_13_210407_DS8.tex b/TST/DS/DS_21_04_07/TST1/corr_13_210407_DS8.tex new file mode 100644 index 0000000..487c44a --- /dev/null +++ b/TST/DS/DS_21_04_07/TST1/corr_13_210407_DS8.tex @@ -0,0 +1,262 @@ +\documentclass[a4paper,10pt]{article} +\usepackage{myXsim} + +% Title Page +\title{DS8 \hfill FOIGNY Romain} +\tribe{TST} +\date{\hfillÀ render pour le Mercredi 7 avril} + +\xsimsetup{ + solution/print = true +} + +\begin{document} +\maketitle + +\begin{exercise}[subtitle={Automatismes}] + \textit{Toutes les questions de cette exercice sont indépendantes et peuvent être répondus séparément} + \begin{enumerate} + \item De janvier à septembre, une quantité a augmenté de $22\,\%$. Faire un schéma pour représenter la situation puis calculer le taux d'évolution moyen mensuel. + \item Une quantité augmente de $22\,\%$ par ans. En 2020, elle est de 125\euro. Quelle était sa valeur en 2019? Faire un schéma pour représenter la situation. + \item Déterminer l'équation de la droite \\ + \begin{tikzpicture}[xscale=0.8, yscale=0.5] + \tkzInit[xmin=-5,xmax=5,xstep=1, + ymin=-5,ymax=5,ystep=1] + \tkzGrid + \tkzAxeXY + \tkzFct[domain=-5:5,color=red,very thick]% + {2.6666666666666665*\x -4}; + \end{tikzpicture} + \item Résoudre l'équation $9 \times 0.35^x = 44$ + \end{enumerate} +\end{exercise} + +\begin{solution} + \begin{enumerate} + \item On veut partager cette évolution en 8 évolutions. + \[ + \left(1 + \frac{22}{100}\right)^{\frac{1}{8}} = 1.0252 + \] + Donc le taux d'évolution moyen est + \[ + t_m = 1.0252 - 1 = 0.02519999999999989 + \] + \item Coefficient multiplicateur pour revenir en arrière + \[ + CM = (1 + \frac{22}{100})^{-1} = 0.8197 + \] + On en déduit la quantité en 2019 + \[ + 125 * 0.8197 = 102.46249999999999 + \] + \item L'équation de la droite est + \[ + y = 2.6666666666666665 x -4 + \] + \item Il faut penser à faire la division à par $9$ avant d'utiliser le log car sinon, on ne peut pas utiliser la formule $\log(a^n) = n\times \log(a)$. + + \[x = \frac{\log(4.89)}{\log(0.35)}\] + \end{enumerate} +\end{solution} + +\begin{exercise}[subtitle={Restaurant}] + Un \emph{food truck}, ouvert le midi et le soir, propose deux types de formules : + + \setlength\parindent{10mm} + \begin{itemize} + \item la formule \emph{Burger} ; + \item la formule \emph{Wok}. + \end{itemize} + \setlength\parindent{0mm} + + \medskip + + Le gérant a remarqué que 14\,\% de ses ventes ont lieu le midi. Le quart des ventes du midi correspondent à la formule \emph{Burger}, alors que 17\,\% des ventes du soir correspondent à la formule \emph{Wok}. + + Le gérant se constitue un fichier en notant, pour chaque vente, la formule choisie et le moment de cette vente (midi ou soir). + + On prélève une fiche de façon équiprobable. On définit les quatre évènements suivants: + + \begin{enumerate} + \item $M$ : \og la fiche correspond à une vente du midi\fg{} ; + \item $S$ : \og la fiche correspond à une vente du soir\fg {}; + \item $W$ : \og la fiche correspond à une formule \emph{Wok} \fg{} ; + \item $B$ : \og la fiche correspond à une formule \emph{Burger} \fg. + \end{enumerate} + \setlength\parindent{0mm} + + \medskip + + \begin{enumerate} + \item Recopier puis compléter l'arbre pondéré + + \begin{center} + \begin{tikzpicture}[sloped] + \node {.} + child {node {$M$} + child {node {$W$} + edge from parent + node[above] {...} + } + child {node {$B$} + edge from parent + node[above] {...} + } + edge from parent + node[above] {...} + } + child[missing] {} + child { node {$S$} + child {node {$W$} + edge from parent + node[above] {...} + } + child {node {$B$} + edge from parent + node[above] {...} + } + edge from parent + node[above] {...} + } ; + \end{tikzpicture} + \end{center} + + \item Calculer la probabilité de l'évènement $M \cap W$. Interpréter ce résultat dans le contexte de l'exercice. + \item Montrer que la probabilité que la fiche choisie corresponde à une formule \emph{Burger} est égale à $0.7488$. + \item On a prélevé une fiche correspondant à la formule \emph{Burger}. Quelle est la probabilité, arrondie au millième, que la vente ait eu lieu le soir? + \end{enumerate} +\end{exercise} + +\begin{solution} + \begin{enumerate} + \item + \begin{center} + \begin{tikzpicture}[sloped] + \node {.} + child {node {$M$} + child {node {$W$} + edge from parent + node[above] {$0.75$} + } + child {node {$B$} + edge from parent + node[above] {$0.25$} + } + edge from parent + node[above] {$0.14$} + } + child[missing] {} + child { node {$S$} + child {node {$W$} + edge from parent + node[above] {$0.17$} + } + child {node {$B$} + edge from parent + node[above] {$0.83$} + } + edge from parent + node[above] {$0.86$} + } ; + \end{tikzpicture} + \end{center} + \item On calcule la probabilité que la vente soit un wok et ait eu lieu à midi + \[ P(M\cap W) = P(M) \times P_M(W) = 0.14 \times 0.75 = 0.105 \] + \item Probabilité que la vente soit un burger. + \[ + P(B) = P(M\cap B) + P(S\cap B) = 0.14 \times 0.75 + 0.86 \times 0.17 = 0.7488 + \] + \item On cherche à calculer la quantité $P_B(S)$. Pour cela on utilise la formule de Bayes + \[ + P_B(S) = \frac{P(B\cap S)}{P(B)} = \frac{P_S(B) \times P(S)}{P(B)} = \frac{0.83\times 0.86}{0.7488} = 0.953258547008547 \approx 0.953 + \] + \end{enumerate} +\end{solution} + +\begin{exercise}[subtitle={Continent plastique}] + \textit{Les quantités évoqués dans cette exercice sont générés au hasard et sont donc complètement farfelus.} + \medskip + Le \og continent de plastique\fg{} est la plus grande des plaques de déchets plastiques évoluant sur les océans. Elle occupe actuellement dans l'océan Pacifique une surface dont l'aire est évaluée à plus de $1,6$ million de km$^2$, entre Hawaï et la Californie. + + En 2017, des scientifiques ont estimé qu'il y avait $11$ millions de tonnes de déchets plastiques qui était déversé chaque année dans les océans et que cette quantité augmentait de $12\n\%$ par chaque année. + + On modélise l'évolution de la masse de ces déchets plastiques déversée chaque année, si rien n'est fait pour la réduire, par une suite géométrique $\left(u_n\right)$. L'arrondi au centième du terme $u_n$ représente la masse de ces déchets déversée chaque année, exprimée en million de tonnes, pour l'année $(2017 + n)$. + + \medskip + + \begin{enumerate} + \item Expliquer pourquoi la suite $u_n$ est géométrique? + \item Calculer $u_1$ et $u_2$. + \item Exprimer $u_n$ en fonction de $n$. + \item Au début de l'année 2017, il y avait $300$ millions de tonnes de déchets plastique. Calculer la quantité totale de déchets plastiques en 2030. + \item On souhaite déterminer en quelle année la masse totale de ces déchets plastiques aura pour la première fois augmenté de $50$\,\% par rapport à sa valeur de 2017. + \begin{enumerate} + \item Recopier et compléter l'algorithme ci-dessous pour que la variable $N$ contienne la réponse au problème posé. + + \begin{center} + \begin{tabularx}{0.4\linewidth}{|X|}\hline + $N = 2017$\\ + $U = 11$ \\ + $S = 300 + U$ \\ + while $S < 450$: \\ + \hspace{1cm} $N = \ldots$\\ + \hspace{1cm} $U = \ldots$\\ + \hspace{1cm} $S = \ldots$\\ + \hline + \end{tabularx} + \end{center} + \item Que contiennent les variables $S$, $U$ et $N$ après exécution de cet algorithme ? + + Interpréter les résultats dans le contexte de l'exercice. + \end{enumerate} + \end{enumerate} +\end{exercise} + +\begin{solution} + \begin{enumerate} + \item Une augmentation de $12\,\%$ revient à multiplier la quantité par $1.12$. La suite est donc bien géométrique. Son premier terme est $u_0 = 11$ et sa raison est $q = 1.12$ + \item + \[ + u_1 = u_0 * 1.12 = 12.32 + \] + \[ + u_2 = u_0 * 1.12^2 = 13.7984 + \] + \item + \[ + u_n = u_0 \times q^n = 11 \times 1.12^n + \] + \item On calcule la quantité totale déversée entre 2017 et 2030. + \[ + \sum_{n = 0}^{13} u_n = u_0 \times \frac{1-q^{13}}{1-q} = 11 \times \frac{1 - 1.12^{13}}{1 - 1.12} = 308.32 + \] + On en déduit la quantité totale de déchets en 2030 + \[ + 300 + 308.32 = 608.3199999999999 + \] + \item + \begin{enumerate} + \item ~ + \begin{center} + \begin{tabularx}{0.4\linewidth}{|X|}\hline + $N \gets 2017$\\ + $U \gets 11$ \\ + $S \gets 300 + U$ \\ + Tant que $S < 450$ \\ + \hspace{1cm} $N \gets N + 1$\\ + \hspace{1cm} $U \gets U * 1.12$\\ + \hspace{1cm} $S \gets S + u$\\ + Fin Tant que\\\hline + \end{tabularx} + \end{center} + \item \textit{Pas de correction automatisé} + \end{enumerate} + \end{enumerate} +\end{solution} + +\end{document} + +%%% Local Variables: +%%% mode: latex +%%% TeX-master: "master" +%%% End: diff --git a/TST/DS/DS_21_04_07/TST1/corr_14_210407_DS8.tex b/TST/DS/DS_21_04_07/TST1/corr_14_210407_DS8.tex new file mode 100644 index 0000000..52bdb89 --- /dev/null +++ b/TST/DS/DS_21_04_07/TST1/corr_14_210407_DS8.tex @@ -0,0 +1,262 @@ +\documentclass[a4paper,10pt]{article} +\usepackage{myXsim} + +% Title Page +\title{DS8 \hfill HIPOLITO DA SILVA Andréa} +\tribe{TST} +\date{\hfillÀ render pour le Mercredi 7 avril} + +\xsimsetup{ + solution/print = true +} + +\begin{document} +\maketitle + +\begin{exercise}[subtitle={Automatismes}] + \textit{Toutes les questions de cette exercice sont indépendantes et peuvent être répondus séparément} + \begin{enumerate} + \item De janvier à septembre, une quantité a augmenté de $23\,\%$. Faire un schéma pour représenter la situation puis calculer le taux d'évolution moyen mensuel. + \item Une quantité augmente de $23\,\%$ par ans. En 2020, elle est de 149\euro. Quelle était sa valeur en 2019? Faire un schéma pour représenter la situation. + \item Déterminer l'équation de la droite \\ + \begin{tikzpicture}[xscale=0.8, yscale=0.5] + \tkzInit[xmin=-5,xmax=5,xstep=1, + ymin=-5,ymax=5,ystep=1] + \tkzGrid + \tkzAxeXY + \tkzFct[domain=-5:5,color=red,very thick]% + {0.6666666666666666*\x -1}; + \end{tikzpicture} + \item Résoudre l'équation $7 \times 0.38^x = 21$ + \end{enumerate} +\end{exercise} + +\begin{solution} + \begin{enumerate} + \item On veut partager cette évolution en 8 évolutions. + \[ + \left(1 + \frac{23}{100}\right)^{\frac{1}{8}} = 1.0262 + \] + Donc le taux d'évolution moyen est + \[ + t_m = 1.0262 - 1 = 0.0262 + \] + \item Coefficient multiplicateur pour revenir en arrière + \[ + CM = (1 + \frac{23}{100})^{-1} = 0.813 + \] + On en déduit la quantité en 2019 + \[ + 149 * 0.813 = 121.13699999999999 + \] + \item L'équation de la droite est + \[ + y = 0.6666666666666666 x -1 + \] + \item Il faut penser à faire la division à par $7$ avant d'utiliser le log car sinon, on ne peut pas utiliser la formule $\log(a^n) = n\times \log(a)$. + + \[x = \frac{\log(3.0)}{\log(0.38)}\] + \end{enumerate} +\end{solution} + +\begin{exercise}[subtitle={Restaurant}] + Un \emph{food truck}, ouvert le midi et le soir, propose deux types de formules : + + \setlength\parindent{10mm} + \begin{itemize} + \item la formule \emph{Burger} ; + \item la formule \emph{Wok}. + \end{itemize} + \setlength\parindent{0mm} + + \medskip + + Le gérant a remarqué que 35\,\% de ses ventes ont lieu le midi. Le quart des ventes du midi correspondent à la formule \emph{Burger}, alors que 13\,\% des ventes du soir correspondent à la formule \emph{Wok}. + + Le gérant se constitue un fichier en notant, pour chaque vente, la formule choisie et le moment de cette vente (midi ou soir). + + On prélève une fiche de façon équiprobable. On définit les quatre évènements suivants: + + \begin{enumerate} + \item $M$ : \og la fiche correspond à une vente du midi\fg{} ; + \item $S$ : \og la fiche correspond à une vente du soir\fg {}; + \item $W$ : \og la fiche correspond à une formule \emph{Wok} \fg{} ; + \item $B$ : \og la fiche correspond à une formule \emph{Burger} \fg. + \end{enumerate} + \setlength\parindent{0mm} + + \medskip + + \begin{enumerate} + \item Recopier puis compléter l'arbre pondéré + + \begin{center} + \begin{tikzpicture}[sloped] + \node {.} + child {node {$M$} + child {node {$W$} + edge from parent + node[above] {...} + } + child {node {$B$} + edge from parent + node[above] {...} + } + edge from parent + node[above] {...} + } + child[missing] {} + child { node {$S$} + child {node {$W$} + edge from parent + node[above] {...} + } + child {node {$B$} + edge from parent + node[above] {...} + } + edge from parent + node[above] {...} + } ; + \end{tikzpicture} + \end{center} + + \item Calculer la probabilité de l'évènement $M \cap W$. Interpréter ce résultat dans le contexte de l'exercice. + \item Montrer que la probabilité que la fiche choisie corresponde à une formule \emph{Burger} est égale à $0.653$. + \item On a prélevé une fiche correspondant à la formule \emph{Burger}. Quelle est la probabilité, arrondie au millième, que la vente ait eu lieu le soir? + \end{enumerate} +\end{exercise} + +\begin{solution} + \begin{enumerate} + \item + \begin{center} + \begin{tikzpicture}[sloped] + \node {.} + child {node {$M$} + child {node {$W$} + edge from parent + node[above] {$0.75$} + } + child {node {$B$} + edge from parent + node[above] {$0.25$} + } + edge from parent + node[above] {$0.35$} + } + child[missing] {} + child { node {$S$} + child {node {$W$} + edge from parent + node[above] {$0.13$} + } + child {node {$B$} + edge from parent + node[above] {$0.87$} + } + edge from parent + node[above] {$0.65$} + } ; + \end{tikzpicture} + \end{center} + \item On calcule la probabilité que la vente soit un wok et ait eu lieu à midi + \[ P(M\cap W) = P(M) \times P_M(W) = 0.35 \times 0.75 = 0.2625 \] + \item Probabilité que la vente soit un burger. + \[ + P(B) = P(M\cap B) + P(S\cap B) = 0.35 \times 0.75 + 0.65 \times 0.13 = 0.653 + \] + \item On cherche à calculer la quantité $P_B(S)$. Pour cela on utilise la formule de Bayes + \[ + P_B(S) = \frac{P(B\cap S)}{P(B)} = \frac{P_S(B) \times P(S)}{P(B)} = \frac{0.87\times 0.65}{0.653} = 0.8660030627871362 \approx 0.866 + \] + \end{enumerate} +\end{solution} + +\begin{exercise}[subtitle={Continent plastique}] + \textit{Les quantités évoqués dans cette exercice sont générés au hasard et sont donc complètement farfelus.} + \medskip + Le \og continent de plastique\fg{} est la plus grande des plaques de déchets plastiques évoluant sur les océans. Elle occupe actuellement dans l'océan Pacifique une surface dont l'aire est évaluée à plus de $1,6$ million de km$^2$, entre Hawaï et la Californie. + + En 2017, des scientifiques ont estimé qu'il y avait $4$ millions de tonnes de déchets plastiques qui était déversé chaque année dans les océans et que cette quantité augmentait de $23\n\%$ par chaque année. + + On modélise l'évolution de la masse de ces déchets plastiques déversée chaque année, si rien n'est fait pour la réduire, par une suite géométrique $\left(u_n\right)$. L'arrondi au centième du terme $u_n$ représente la masse de ces déchets déversée chaque année, exprimée en million de tonnes, pour l'année $(2017 + n)$. + + \medskip + + \begin{enumerate} + \item Expliquer pourquoi la suite $u_n$ est géométrique? + \item Calculer $u_1$ et $u_2$. + \item Exprimer $u_n$ en fonction de $n$. + \item Au début de l'année 2017, il y avait $300$ millions de tonnes de déchets plastique. Calculer la quantité totale de déchets plastiques en 2030. + \item On souhaite déterminer en quelle année la masse totale de ces déchets plastiques aura pour la première fois augmenté de $50$\,\% par rapport à sa valeur de 2017. + \begin{enumerate} + \item Recopier et compléter l'algorithme ci-dessous pour que la variable $N$ contienne la réponse au problème posé. + + \begin{center} + \begin{tabularx}{0.4\linewidth}{|X|}\hline + $N = 2017$\\ + $U = 4$ \\ + $S = 300 + U$ \\ + while $S < 450$: \\ + \hspace{1cm} $N = \ldots$\\ + \hspace{1cm} $U = \ldots$\\ + \hspace{1cm} $S = \ldots$\\ + \hline + \end{tabularx} + \end{center} + \item Que contiennent les variables $S$, $U$ et $N$ après exécution de cet algorithme ? + + Interpréter les résultats dans le contexte de l'exercice. + \end{enumerate} + \end{enumerate} +\end{exercise} + +\begin{solution} + \begin{enumerate} + \item Une augmentation de $23\,\%$ revient à multiplier la quantité par $1.23$. La suite est donc bien géométrique. Son premier terme est $u_0 = 4$ et sa raison est $q = 1.23$ + \item + \[ + u_1 = u_0 * 1.23 = 4.92 + \] + \[ + u_2 = u_0 * 1.23^2 = 6.0516 + \] + \item + \[ + u_n = u_0 \times q^n = 4 \times 1.23^n + \] + \item On calcule la quantité totale déversée entre 2017 et 2030. + \[ + \sum_{n = 0}^{13} u_n = u_0 \times \frac{1-q^{13}}{1-q} = 4 \times \frac{1 - 1.23^{13}}{1 - 1.23} = 239.12 + \] + On en déduit la quantité totale de déchets en 2030 + \[ + 300 + 239.12 = 539.12 + \] + \item + \begin{enumerate} + \item ~ + \begin{center} + \begin{tabularx}{0.4\linewidth}{|X|}\hline + $N \gets 2017$\\ + $U \gets 4$ \\ + $S \gets 300 + U$ \\ + Tant que $S < 450$ \\ + \hspace{1cm} $N \gets N + 1$\\ + \hspace{1cm} $U \gets U * 1.23$\\ + \hspace{1cm} $S \gets S + u$\\ + Fin Tant que\\\hline + \end{tabularx} + \end{center} + \item \textit{Pas de correction automatisé} + \end{enumerate} + \end{enumerate} +\end{solution} + +\end{document} + +%%% Local Variables: +%%% mode: latex +%%% TeX-master: "master" +%%% End: diff --git a/TST/DS/DS_21_04_07/TST1/corr_15_210407_DS8.tex b/TST/DS/DS_21_04_07/TST1/corr_15_210407_DS8.tex new file mode 100644 index 0000000..79186f9 --- /dev/null +++ b/TST/DS/DS_21_04_07/TST1/corr_15_210407_DS8.tex @@ -0,0 +1,262 @@ +\documentclass[a4paper,10pt]{article} +\usepackage{myXsim} + +% Title Page +\title{DS8 \hfill HUMBERT Rayan} +\tribe{TST} +\date{\hfillÀ render pour le Mercredi 7 avril} + +\xsimsetup{ + solution/print = true +} + +\begin{document} +\maketitle + +\begin{exercise}[subtitle={Automatismes}] + \textit{Toutes les questions de cette exercice sont indépendantes et peuvent être répondus séparément} + \begin{enumerate} + \item De janvier à septembre, une quantité a augmenté de $17\,\%$. Faire un schéma pour représenter la situation puis calculer le taux d'évolution moyen mensuel. + \item Une quantité augmente de $17\,\%$ par ans. En 2020, elle est de 149\euro. Quelle était sa valeur en 2019? Faire un schéma pour représenter la situation. + \item Déterminer l'équation de la droite \\ + \begin{tikzpicture}[xscale=0.8, yscale=0.5] + \tkzInit[xmin=-5,xmax=5,xstep=1, + ymin=-5,ymax=5,ystep=1] + \tkzGrid + \tkzAxeXY + \tkzFct[domain=-5:5,color=red,very thick]% + {0.5*\x -1}; + \end{tikzpicture} + \item Résoudre l'équation $5 \times 0.02^x = 46$ + \end{enumerate} +\end{exercise} + +\begin{solution} + \begin{enumerate} + \item On veut partager cette évolution en 8 évolutions. + \[ + \left(1 + \frac{17}{100}\right)^{\frac{1}{8}} = 1.0198 + \] + Donc le taux d'évolution moyen est + \[ + t_m = 1.0198 - 1 = 0.01980000000000004 + \] + \item Coefficient multiplicateur pour revenir en arrière + \[ + CM = (1 + \frac{17}{100})^{-1} = 0.8547 + \] + On en déduit la quantité en 2019 + \[ + 149 * 0.8547 = 127.3503 + \] + \item L'équation de la droite est + \[ + y = 0.5 x -1 + \] + \item Il faut penser à faire la division à par $5$ avant d'utiliser le log car sinon, on ne peut pas utiliser la formule $\log(a^n) = n\times \log(a)$. + + \[x = \frac{\log(9.2)}{\log(0.02)}\] + \end{enumerate} +\end{solution} + +\begin{exercise}[subtitle={Restaurant}] + Un \emph{food truck}, ouvert le midi et le soir, propose deux types de formules : + + \setlength\parindent{10mm} + \begin{itemize} + \item la formule \emph{Burger} ; + \item la formule \emph{Wok}. + \end{itemize} + \setlength\parindent{0mm} + + \medskip + + Le gérant a remarqué que 65\,\% de ses ventes ont lieu le midi. Le quart des ventes du midi correspondent à la formule \emph{Burger}, alors que 93\,\% des ventes du soir correspondent à la formule \emph{Wok}. + + Le gérant se constitue un fichier en notant, pour chaque vente, la formule choisie et le moment de cette vente (midi ou soir). + + On prélève une fiche de façon équiprobable. On définit les quatre évènements suivants: + + \begin{enumerate} + \item $M$ : \og la fiche correspond à une vente du midi\fg{} ; + \item $S$ : \og la fiche correspond à une vente du soir\fg {}; + \item $W$ : \og la fiche correspond à une formule \emph{Wok} \fg{} ; + \item $B$ : \og la fiche correspond à une formule \emph{Burger} \fg. + \end{enumerate} + \setlength\parindent{0mm} + + \medskip + + \begin{enumerate} + \item Recopier puis compléter l'arbre pondéré + + \begin{center} + \begin{tikzpicture}[sloped] + \node {.} + child {node {$M$} + child {node {$W$} + edge from parent + node[above] {...} + } + child {node {$B$} + edge from parent + node[above] {...} + } + edge from parent + node[above] {...} + } + child[missing] {} + child { node {$S$} + child {node {$W$} + edge from parent + node[above] {...} + } + child {node {$B$} + edge from parent + node[above] {...} + } + edge from parent + node[above] {...} + } ; + \end{tikzpicture} + \end{center} + + \item Calculer la probabilité de l'évènement $M \cap W$. Interpréter ce résultat dans le contexte de l'exercice. + \item Montrer que la probabilité que la fiche choisie corresponde à une formule \emph{Burger} est égale à $0.187$. + \item On a prélevé une fiche correspondant à la formule \emph{Burger}. Quelle est la probabilité, arrondie au millième, que la vente ait eu lieu le soir? + \end{enumerate} +\end{exercise} + +\begin{solution} + \begin{enumerate} + \item + \begin{center} + \begin{tikzpicture}[sloped] + \node {.} + child {node {$M$} + child {node {$W$} + edge from parent + node[above] {$0.75$} + } + child {node {$B$} + edge from parent + node[above] {$0.25$} + } + edge from parent + node[above] {$0.65$} + } + child[missing] {} + child { node {$S$} + child {node {$W$} + edge from parent + node[above] {$0.93$} + } + child {node {$B$} + edge from parent + node[above] {$0.07$} + } + edge from parent + node[above] {$0.35$} + } ; + \end{tikzpicture} + \end{center} + \item On calcule la probabilité que la vente soit un wok et ait eu lieu à midi + \[ P(M\cap W) = P(M) \times P_M(W) = 0.65 \times 0.75 = 0.4875 \] + \item Probabilité que la vente soit un burger. + \[ + P(B) = P(M\cap B) + P(S\cap B) = 0.65 \times 0.75 + 0.35 \times 0.93 = 0.187 + \] + \item On cherche à calculer la quantité $P_B(S)$. Pour cela on utilise la formule de Bayes + \[ + P_B(S) = \frac{P(B\cap S)}{P(B)} = \frac{P_S(B) \times P(S)}{P(B)} = \frac{0.07\times 0.35}{0.187} = 0.13101604278074866 \approx 0.131 + \] + \end{enumerate} +\end{solution} + +\begin{exercise}[subtitle={Continent plastique}] + \textit{Les quantités évoqués dans cette exercice sont générés au hasard et sont donc complètement farfelus.} + \medskip + Le \og continent de plastique\fg{} est la plus grande des plaques de déchets plastiques évoluant sur les océans. Elle occupe actuellement dans l'océan Pacifique une surface dont l'aire est évaluée à plus de $1,6$ million de km$^2$, entre Hawaï et la Californie. + + En 2017, des scientifiques ont estimé qu'il y avait $15$ millions de tonnes de déchets plastiques qui était déversé chaque année dans les océans et que cette quantité augmentait de $12\n\%$ par chaque année. + + On modélise l'évolution de la masse de ces déchets plastiques déversée chaque année, si rien n'est fait pour la réduire, par une suite géométrique $\left(u_n\right)$. L'arrondi au centième du terme $u_n$ représente la masse de ces déchets déversée chaque année, exprimée en million de tonnes, pour l'année $(2017 + n)$. + + \medskip + + \begin{enumerate} + \item Expliquer pourquoi la suite $u_n$ est géométrique? + \item Calculer $u_1$ et $u_2$. + \item Exprimer $u_n$ en fonction de $n$. + \item Au début de l'année 2017, il y avait $300$ millions de tonnes de déchets plastique. Calculer la quantité totale de déchets plastiques en 2030. + \item On souhaite déterminer en quelle année la masse totale de ces déchets plastiques aura pour la première fois augmenté de $50$\,\% par rapport à sa valeur de 2017. + \begin{enumerate} + \item Recopier et compléter l'algorithme ci-dessous pour que la variable $N$ contienne la réponse au problème posé. + + \begin{center} + \begin{tabularx}{0.4\linewidth}{|X|}\hline + $N = 2017$\\ + $U = 15$ \\ + $S = 300 + U$ \\ + while $S < 450$: \\ + \hspace{1cm} $N = \ldots$\\ + \hspace{1cm} $U = \ldots$\\ + \hspace{1cm} $S = \ldots$\\ + \hline + \end{tabularx} + \end{center} + \item Que contiennent les variables $S$, $U$ et $N$ après exécution de cet algorithme ? + + Interpréter les résultats dans le contexte de l'exercice. + \end{enumerate} + \end{enumerate} +\end{exercise} + +\begin{solution} + \begin{enumerate} + \item Une augmentation de $12\,\%$ revient à multiplier la quantité par $1.12$. La suite est donc bien géométrique. Son premier terme est $u_0 = 15$ et sa raison est $q = 1.12$ + \item + \[ + u_1 = u_0 * 1.12 = 16.8 + \] + \[ + u_2 = u_0 * 1.12^2 = 18.816 + \] + \item + \[ + u_n = u_0 \times q^n = 15 \times 1.12^n + \] + \item On calcule la quantité totale déversée entre 2017 et 2030. + \[ + \sum_{n = 0}^{13} u_n = u_0 \times \frac{1-q^{13}}{1-q} = 15 \times \frac{1 - 1.12^{13}}{1 - 1.12} = 420.44 + \] + On en déduit la quantité totale de déchets en 2030 + \[ + 300 + 420.44 = 720.44 + \] + \item + \begin{enumerate} + \item ~ + \begin{center} + \begin{tabularx}{0.4\linewidth}{|X|}\hline + $N \gets 2017$\\ + $U \gets 15$ \\ + $S \gets 300 + U$ \\ + Tant que $S < 450$ \\ + \hspace{1cm} $N \gets N + 1$\\ + \hspace{1cm} $U \gets U * 1.12$\\ + \hspace{1cm} $S \gets S + u$\\ + Fin Tant que\\\hline + \end{tabularx} + \end{center} + \item \textit{Pas de correction automatisé} + \end{enumerate} + \end{enumerate} +\end{solution} + +\end{document} + +%%% Local Variables: +%%% mode: latex +%%% TeX-master: "master" +%%% End: diff --git a/TST/DS/DS_21_04_07/TST1/corr_16_210407_DS8.tex b/TST/DS/DS_21_04_07/TST1/corr_16_210407_DS8.tex new file mode 100644 index 0000000..9aa4436 --- /dev/null +++ b/TST/DS/DS_21_04_07/TST1/corr_16_210407_DS8.tex @@ -0,0 +1,262 @@ +\documentclass[a4paper,10pt]{article} +\usepackage{myXsim} + +% Title Page +\title{DS8 \hfill MASSON Grace} +\tribe{TST} +\date{\hfillÀ render pour le Mercredi 7 avril} + +\xsimsetup{ + solution/print = true +} + +\begin{document} +\maketitle + +\begin{exercise}[subtitle={Automatismes}] + \textit{Toutes les questions de cette exercice sont indépendantes et peuvent être répondus séparément} + \begin{enumerate} + \item De janvier à septembre, une quantité a augmenté de $24\,\%$. Faire un schéma pour représenter la situation puis calculer le taux d'évolution moyen mensuel. + \item Une quantité augmente de $24\,\%$ par ans. En 2020, elle est de 145\euro. Quelle était sa valeur en 2019? Faire un schéma pour représenter la situation. + \item Déterminer l'équation de la droite \\ + \begin{tikzpicture}[xscale=0.8, yscale=0.5] + \tkzInit[xmin=-5,xmax=5,xstep=1, + ymin=-5,ymax=5,ystep=1] + \tkzGrid + \tkzAxeXY + \tkzFct[domain=-5:5,color=red,very thick]% + {3.0*\x -3}; + \end{tikzpicture} + \item Résoudre l'équation $5 \times 0.53^x = 18$ + \end{enumerate} +\end{exercise} + +\begin{solution} + \begin{enumerate} + \item On veut partager cette évolution en 8 évolutions. + \[ + \left(1 + \frac{24}{100}\right)^{\frac{1}{8}} = 1.0273 + \] + Donc le taux d'évolution moyen est + \[ + t_m = 1.0273 - 1 = 0.027300000000000102 + \] + \item Coefficient multiplicateur pour revenir en arrière + \[ + CM = (1 + \frac{24}{100})^{-1} = 0.8065 + \] + On en déduit la quantité en 2019 + \[ + 145 * 0.8065 = 116.9425 + \] + \item L'équation de la droite est + \[ + y = 3.0 x -3 + \] + \item Il faut penser à faire la division à par $5$ avant d'utiliser le log car sinon, on ne peut pas utiliser la formule $\log(a^n) = n\times \log(a)$. + + \[x = \frac{\log(3.6)}{\log(0.53)}\] + \end{enumerate} +\end{solution} + +\begin{exercise}[subtitle={Restaurant}] + Un \emph{food truck}, ouvert le midi et le soir, propose deux types de formules : + + \setlength\parindent{10mm} + \begin{itemize} + \item la formule \emph{Burger} ; + \item la formule \emph{Wok}. + \end{itemize} + \setlength\parindent{0mm} + + \medskip + + Le gérant a remarqué que 73\,\% de ses ventes ont lieu le midi. Le quart des ventes du midi correspondent à la formule \emph{Burger}, alors que 56\,\% des ventes du soir correspondent à la formule \emph{Wok}. + + Le gérant se constitue un fichier en notant, pour chaque vente, la formule choisie et le moment de cette vente (midi ou soir). + + On prélève une fiche de façon équiprobable. On définit les quatre évènements suivants: + + \begin{enumerate} + \item $M$ : \og la fiche correspond à une vente du midi\fg{} ; + \item $S$ : \og la fiche correspond à une vente du soir\fg {}; + \item $W$ : \og la fiche correspond à une formule \emph{Wok} \fg{} ; + \item $B$ : \og la fiche correspond à une formule \emph{Burger} \fg. + \end{enumerate} + \setlength\parindent{0mm} + + \medskip + + \begin{enumerate} + \item Recopier puis compléter l'arbre pondéré + + \begin{center} + \begin{tikzpicture}[sloped] + \node {.} + child {node {$M$} + child {node {$W$} + edge from parent + node[above] {...} + } + child {node {$B$} + edge from parent + node[above] {...} + } + edge from parent + node[above] {...} + } + child[missing] {} + child { node {$S$} + child {node {$W$} + edge from parent + node[above] {...} + } + child {node {$B$} + edge from parent + node[above] {...} + } + edge from parent + node[above] {...} + } ; + \end{tikzpicture} + \end{center} + + \item Calculer la probabilité de l'évènement $M \cap W$. Interpréter ce résultat dans le contexte de l'exercice. + \item Montrer que la probabilité que la fiche choisie corresponde à une formule \emph{Burger} est égale à $0.2986$. + \item On a prélevé une fiche correspondant à la formule \emph{Burger}. Quelle est la probabilité, arrondie au millième, que la vente ait eu lieu le soir? + \end{enumerate} +\end{exercise} + +\begin{solution} + \begin{enumerate} + \item + \begin{center} + \begin{tikzpicture}[sloped] + \node {.} + child {node {$M$} + child {node {$W$} + edge from parent + node[above] {$0.75$} + } + child {node {$B$} + edge from parent + node[above] {$0.25$} + } + edge from parent + node[above] {$0.73$} + } + child[missing] {} + child { node {$S$} + child {node {$W$} + edge from parent + node[above] {$0.57$} + } + child {node {$B$} + edge from parent + node[above] {$0.43$} + } + edge from parent + node[above] {$0.27$} + } ; + \end{tikzpicture} + \end{center} + \item On calcule la probabilité que la vente soit un wok et ait eu lieu à midi + \[ P(M\cap W) = P(M) \times P_M(W) = 0.73 \times 0.75 = 0.5475 \] + \item Probabilité que la vente soit un burger. + \[ + P(B) = P(M\cap B) + P(S\cap B) = 0.73 \times 0.75 + 0.27 \times 0.57 = 0.2986 + \] + \item On cherche à calculer la quantité $P_B(S)$. Pour cela on utilise la formule de Bayes + \[ + P_B(S) = \frac{P(B\cap S)}{P(B)} = \frac{P_S(B) \times P(S)}{P(B)} = \frac{0.43\times 0.27}{0.2986} = 0.38881446751507037 \approx 0.389 + \] + \end{enumerate} +\end{solution} + +\begin{exercise}[subtitle={Continent plastique}] + \textit{Les quantités évoqués dans cette exercice sont générés au hasard et sont donc complètement farfelus.} + \medskip + Le \og continent de plastique\fg{} est la plus grande des plaques de déchets plastiques évoluant sur les océans. Elle occupe actuellement dans l'océan Pacifique une surface dont l'aire est évaluée à plus de $1,6$ million de km$^2$, entre Hawaï et la Californie. + + En 2017, des scientifiques ont estimé qu'il y avait $4$ millions de tonnes de déchets plastiques qui était déversé chaque année dans les océans et que cette quantité augmentait de $19\n\%$ par chaque année. + + On modélise l'évolution de la masse de ces déchets plastiques déversée chaque année, si rien n'est fait pour la réduire, par une suite géométrique $\left(u_n\right)$. L'arrondi au centième du terme $u_n$ représente la masse de ces déchets déversée chaque année, exprimée en million de tonnes, pour l'année $(2017 + n)$. + + \medskip + + \begin{enumerate} + \item Expliquer pourquoi la suite $u_n$ est géométrique? + \item Calculer $u_1$ et $u_2$. + \item Exprimer $u_n$ en fonction de $n$. + \item Au début de l'année 2017, il y avait $300$ millions de tonnes de déchets plastique. Calculer la quantité totale de déchets plastiques en 2030. + \item On souhaite déterminer en quelle année la masse totale de ces déchets plastiques aura pour la première fois augmenté de $50$\,\% par rapport à sa valeur de 2017. + \begin{enumerate} + \item Recopier et compléter l'algorithme ci-dessous pour que la variable $N$ contienne la réponse au problème posé. + + \begin{center} + \begin{tabularx}{0.4\linewidth}{|X|}\hline + $N = 2017$\\ + $U = 4$ \\ + $S = 300 + U$ \\ + while $S < 450$: \\ + \hspace{1cm} $N = \ldots$\\ + \hspace{1cm} $U = \ldots$\\ + \hspace{1cm} $S = \ldots$\\ + \hline + \end{tabularx} + \end{center} + \item Que contiennent les variables $S$, $U$ et $N$ après exécution de cet algorithme ? + + Interpréter les résultats dans le contexte de l'exercice. + \end{enumerate} + \end{enumerate} +\end{exercise} + +\begin{solution} + \begin{enumerate} + \item Une augmentation de $19\,\%$ revient à multiplier la quantité par $1.19$. La suite est donc bien géométrique. Son premier terme est $u_0 = 4$ et sa raison est $q = 1.19$ + \item + \[ + u_1 = u_0 * 1.19 = 4.76 + \] + \[ + u_2 = u_0 * 1.19^2 = 5.6644 + \] + \item + \[ + u_n = u_0 \times q^n = 4 \times 1.19^n + \] + \item On calcule la quantité totale déversée entre 2017 et 2030. + \[ + \sum_{n = 0}^{13} u_n = u_0 \times \frac{1-q^{13}}{1-q} = 4 \times \frac{1 - 1.19^{13}}{1 - 1.19} = 180.98 + \] + On en déduit la quantité totale de déchets en 2030 + \[ + 300 + 180.98 = 480.98 + \] + \item + \begin{enumerate} + \item ~ + \begin{center} + \begin{tabularx}{0.4\linewidth}{|X|}\hline + $N \gets 2017$\\ + $U \gets 4$ \\ + $S \gets 300 + U$ \\ + Tant que $S < 450$ \\ + \hspace{1cm} $N \gets N + 1$\\ + \hspace{1cm} $U \gets U * 1.19$\\ + \hspace{1cm} $S \gets S + u$\\ + Fin Tant que\\\hline + \end{tabularx} + \end{center} + \item \textit{Pas de correction automatisé} + \end{enumerate} + \end{enumerate} +\end{solution} + +\end{document} + +%%% Local Variables: +%%% mode: latex +%%% TeX-master: "master" +%%% End: diff --git a/TST/DS/DS_21_04_07/TST1/corr_17_210407_DS8.tex b/TST/DS/DS_21_04_07/TST1/corr_17_210407_DS8.tex new file mode 100644 index 0000000..25d8450 --- /dev/null +++ b/TST/DS/DS_21_04_07/TST1/corr_17_210407_DS8.tex @@ -0,0 +1,262 @@ +\documentclass[a4paper,10pt]{article} +\usepackage{myXsim} + +% Title Page +\title{DS8 \hfill MOKHTARI Nissrine} +\tribe{TST} +\date{\hfillÀ render pour le Mercredi 7 avril} + +\xsimsetup{ + solution/print = true +} + +\begin{document} +\maketitle + +\begin{exercise}[subtitle={Automatismes}] + \textit{Toutes les questions de cette exercice sont indépendantes et peuvent être répondus séparément} + \begin{enumerate} + \item De janvier à septembre, une quantité a augmenté de $11\,\%$. Faire un schéma pour représenter la situation puis calculer le taux d'évolution moyen mensuel. + \item Une quantité augmente de $11\,\%$ par ans. En 2020, elle est de 138\euro. Quelle était sa valeur en 2019? Faire un schéma pour représenter la situation. + \item Déterminer l'équation de la droite \\ + \begin{tikzpicture}[xscale=0.8, yscale=0.5] + \tkzInit[xmin=-5,xmax=5,xstep=1, + ymin=-5,ymax=5,ystep=1] + \tkzGrid + \tkzAxeXY + \tkzFct[domain=-5:5,color=red,very thick]% + {2.0*\x -3}; + \end{tikzpicture} + \item Résoudre l'équation $8 \times 0.17^x = 35$ + \end{enumerate} +\end{exercise} + +\begin{solution} + \begin{enumerate} + \item On veut partager cette évolution en 8 évolutions. + \[ + \left(1 + \frac{11}{100}\right)^{\frac{1}{8}} = 1.0131 + \] + Donc le taux d'évolution moyen est + \[ + t_m = 1.0131 - 1 = 0.01309999999999989 + \] + \item Coefficient multiplicateur pour revenir en arrière + \[ + CM = (1 + \frac{11}{100})^{-1} = 0.9009 + \] + On en déduit la quantité en 2019 + \[ + 138 * 0.9009 = 124.3242 + \] + \item L'équation de la droite est + \[ + y = 2.0 x -3 + \] + \item Il faut penser à faire la division à par $8$ avant d'utiliser le log car sinon, on ne peut pas utiliser la formule $\log(a^n) = n\times \log(a)$. + + \[x = \frac{\log(4.38)}{\log(0.17)}\] + \end{enumerate} +\end{solution} + +\begin{exercise}[subtitle={Restaurant}] + Un \emph{food truck}, ouvert le midi et le soir, propose deux types de formules : + + \setlength\parindent{10mm} + \begin{itemize} + \item la formule \emph{Burger} ; + \item la formule \emph{Wok}. + \end{itemize} + \setlength\parindent{0mm} + + \medskip + + Le gérant a remarqué que 62\,\% de ses ventes ont lieu le midi. Le quart des ventes du midi correspondent à la formule \emph{Burger}, alors que 28\,\% des ventes du soir correspondent à la formule \emph{Wok}. + + Le gérant se constitue un fichier en notant, pour chaque vente, la formule choisie et le moment de cette vente (midi ou soir). + + On prélève une fiche de façon équiprobable. On définit les quatre évènements suivants: + + \begin{enumerate} + \item $M$ : \og la fiche correspond à une vente du midi\fg{} ; + \item $S$ : \og la fiche correspond à une vente du soir\fg {}; + \item $W$ : \og la fiche correspond à une formule \emph{Wok} \fg{} ; + \item $B$ : \og la fiche correspond à une formule \emph{Burger} \fg. + \end{enumerate} + \setlength\parindent{0mm} + + \medskip + + \begin{enumerate} + \item Recopier puis compléter l'arbre pondéré + + \begin{center} + \begin{tikzpicture}[sloped] + \node {.} + child {node {$M$} + child {node {$W$} + edge from parent + node[above] {...} + } + child {node {$B$} + edge from parent + node[above] {...} + } + edge from parent + node[above] {...} + } + child[missing] {} + child { node {$S$} + child {node {$W$} + edge from parent + node[above] {...} + } + child {node {$B$} + edge from parent + node[above] {...} + } + edge from parent + node[above] {...} + } ; + \end{tikzpicture} + \end{center} + + \item Calculer la probabilité de l'évènement $M \cap W$. Interpréter ce résultat dans le contexte de l'exercice. + \item Montrer que la probabilité que la fiche choisie corresponde à une formule \emph{Burger} est égale à $0.4248$. + \item On a prélevé une fiche correspondant à la formule \emph{Burger}. Quelle est la probabilité, arrondie au millième, que la vente ait eu lieu le soir? + \end{enumerate} +\end{exercise} + +\begin{solution} + \begin{enumerate} + \item + \begin{center} + \begin{tikzpicture}[sloped] + \node {.} + child {node {$M$} + child {node {$W$} + edge from parent + node[above] {$0.75$} + } + child {node {$B$} + edge from parent + node[above] {$0.25$} + } + edge from parent + node[above] {$0.62$} + } + child[missing] {} + child { node {$S$} + child {node {$W$} + edge from parent + node[above] {$0.29$} + } + child {node {$B$} + edge from parent + node[above] {$0.71$} + } + edge from parent + node[above] {$0.38$} + } ; + \end{tikzpicture} + \end{center} + \item On calcule la probabilité que la vente soit un wok et ait eu lieu à midi + \[ P(M\cap W) = P(M) \times P_M(W) = 0.62 \times 0.75 = 0.465 \] + \item Probabilité que la vente soit un burger. + \[ + P(B) = P(M\cap B) + P(S\cap B) = 0.62 \times 0.75 + 0.38 \times 0.29 = 0.4248 + \] + \item On cherche à calculer la quantité $P_B(S)$. Pour cela on utilise la formule de Bayes + \[ + P_B(S) = \frac{P(B\cap S)}{P(B)} = \frac{P_S(B) \times P(S)}{P(B)} = \frac{0.71\times 0.38}{0.4248} = 0.6351224105461393 \approx 0.635 + \] + \end{enumerate} +\end{solution} + +\begin{exercise}[subtitle={Continent plastique}] + \textit{Les quantités évoqués dans cette exercice sont générés au hasard et sont donc complètement farfelus.} + \medskip + Le \og continent de plastique\fg{} est la plus grande des plaques de déchets plastiques évoluant sur les océans. Elle occupe actuellement dans l'océan Pacifique une surface dont l'aire est évaluée à plus de $1,6$ million de km$^2$, entre Hawaï et la Californie. + + En 2017, des scientifiques ont estimé qu'il y avait $4$ millions de tonnes de déchets plastiques qui était déversé chaque année dans les océans et que cette quantité augmentait de $30\n\%$ par chaque année. + + On modélise l'évolution de la masse de ces déchets plastiques déversée chaque année, si rien n'est fait pour la réduire, par une suite géométrique $\left(u_n\right)$. L'arrondi au centième du terme $u_n$ représente la masse de ces déchets déversée chaque année, exprimée en million de tonnes, pour l'année $(2017 + n)$. + + \medskip + + \begin{enumerate} + \item Expliquer pourquoi la suite $u_n$ est géométrique? + \item Calculer $u_1$ et $u_2$. + \item Exprimer $u_n$ en fonction de $n$. + \item Au début de l'année 2017, il y avait $300$ millions de tonnes de déchets plastique. Calculer la quantité totale de déchets plastiques en 2030. + \item On souhaite déterminer en quelle année la masse totale de ces déchets plastiques aura pour la première fois augmenté de $50$\,\% par rapport à sa valeur de 2017. + \begin{enumerate} + \item Recopier et compléter l'algorithme ci-dessous pour que la variable $N$ contienne la réponse au problème posé. + + \begin{center} + \begin{tabularx}{0.4\linewidth}{|X|}\hline + $N = 2017$\\ + $U = 4$ \\ + $S = 300 + U$ \\ + while $S < 450$: \\ + \hspace{1cm} $N = \ldots$\\ + \hspace{1cm} $U = \ldots$\\ + \hspace{1cm} $S = \ldots$\\ + \hline + \end{tabularx} + \end{center} + \item Que contiennent les variables $S$, $U$ et $N$ après exécution de cet algorithme ? + + Interpréter les résultats dans le contexte de l'exercice. + \end{enumerate} + \end{enumerate} +\end{exercise} + +\begin{solution} + \begin{enumerate} + \item Une augmentation de $30\,\%$ revient à multiplier la quantité par $1.3$. La suite est donc bien géométrique. Son premier terme est $u_0 = 4$ et sa raison est $q = 1.3$ + \item + \[ + u_1 = u_0 * 1.3 = 5.2 + \] + \[ + u_2 = u_0 * 1.3^2 = 6.76 + \] + \item + \[ + u_n = u_0 \times q^n = 4 \times 1.3^n + \] + \item On calcule la quantité totale déversée entre 2017 et 2030. + \[ + \sum_{n = 0}^{13} u_n = u_0 \times \frac{1-q^{13}}{1-q} = 4 \times \frac{1 - 1.3^{13}}{1 - 1.3} = 390.5 + \] + On en déduit la quantité totale de déchets en 2030 + \[ + 300 + 390.5 = 690.5 + \] + \item + \begin{enumerate} + \item ~ + \begin{center} + \begin{tabularx}{0.4\linewidth}{|X|}\hline + $N \gets 2017$\\ + $U \gets 4$ \\ + $S \gets 300 + U$ \\ + Tant que $S < 450$ \\ + \hspace{1cm} $N \gets N + 1$\\ + \hspace{1cm} $U \gets U * 1.3$\\ + \hspace{1cm} $S \gets S + u$\\ + Fin Tant que\\\hline + \end{tabularx} + \end{center} + \item \textit{Pas de correction automatisé} + \end{enumerate} + \end{enumerate} +\end{solution} + +\end{document} + +%%% Local Variables: +%%% mode: latex +%%% TeX-master: "master" +%%% End: diff --git a/TST/DS/DS_21_04_07/TST1/corr_18_210407_DS8.tex b/TST/DS/DS_21_04_07/TST1/corr_18_210407_DS8.tex new file mode 100644 index 0000000..c556d25 --- /dev/null +++ b/TST/DS/DS_21_04_07/TST1/corr_18_210407_DS8.tex @@ -0,0 +1,262 @@ +\documentclass[a4paper,10pt]{article} +\usepackage{myXsim} + +% Title Page +\title{DS8 \hfill MOUFAQ Amine} +\tribe{TST} +\date{\hfillÀ render pour le Mercredi 7 avril} + +\xsimsetup{ + solution/print = true +} + +\begin{document} +\maketitle + +\begin{exercise}[subtitle={Automatismes}] + \textit{Toutes les questions de cette exercice sont indépendantes et peuvent être répondus séparément} + \begin{enumerate} + \item De janvier à septembre, une quantité a augmenté de $16\,\%$. Faire un schéma pour représenter la situation puis calculer le taux d'évolution moyen mensuel. + \item Une quantité augmente de $16\,\%$ par ans. En 2020, elle est de 115\euro. Quelle était sa valeur en 2019? Faire un schéma pour représenter la situation. + \item Déterminer l'équation de la droite \\ + \begin{tikzpicture}[xscale=0.8, yscale=0.5] + \tkzInit[xmin=-5,xmax=5,xstep=1, + ymin=-5,ymax=5,ystep=1] + \tkzGrid + \tkzAxeXY + \tkzFct[domain=-5:5,color=red,very thick]% + {0.5*\x -1}; + \end{tikzpicture} + \item Résoudre l'équation $6 \times 0.24^x = 41$ + \end{enumerate} +\end{exercise} + +\begin{solution} + \begin{enumerate} + \item On veut partager cette évolution en 8 évolutions. + \[ + \left(1 + \frac{16}{100}\right)^{\frac{1}{8}} = 1.0187 + \] + Donc le taux d'évolution moyen est + \[ + t_m = 1.0187 - 1 = 0.01869999999999994 + \] + \item Coefficient multiplicateur pour revenir en arrière + \[ + CM = (1 + \frac{16}{100})^{-1} = 0.8621 + \] + On en déduit la quantité en 2019 + \[ + 115 * 0.8621 = 99.1415 + \] + \item L'équation de la droite est + \[ + y = 0.5 x -1 + \] + \item Il faut penser à faire la division à par $6$ avant d'utiliser le log car sinon, on ne peut pas utiliser la formule $\log(a^n) = n\times \log(a)$. + + \[x = \frac{\log(6.83)}{\log(0.24)}\] + \end{enumerate} +\end{solution} + +\begin{exercise}[subtitle={Restaurant}] + Un \emph{food truck}, ouvert le midi et le soir, propose deux types de formules : + + \setlength\parindent{10mm} + \begin{itemize} + \item la formule \emph{Burger} ; + \item la formule \emph{Wok}. + \end{itemize} + \setlength\parindent{0mm} + + \medskip + + Le gérant a remarqué que 78\,\% de ses ventes ont lieu le midi. Le quart des ventes du midi correspondent à la formule \emph{Burger}, alors que 37\,\% des ventes du soir correspondent à la formule \emph{Wok}. + + Le gérant se constitue un fichier en notant, pour chaque vente, la formule choisie et le moment de cette vente (midi ou soir). + + On prélève une fiche de façon équiprobable. On définit les quatre évènements suivants: + + \begin{enumerate} + \item $M$ : \og la fiche correspond à une vente du midi\fg{} ; + \item $S$ : \og la fiche correspond à une vente du soir\fg {}; + \item $W$ : \og la fiche correspond à une formule \emph{Wok} \fg{} ; + \item $B$ : \og la fiche correspond à une formule \emph{Burger} \fg. + \end{enumerate} + \setlength\parindent{0mm} + + \medskip + + \begin{enumerate} + \item Recopier puis compléter l'arbre pondéré + + \begin{center} + \begin{tikzpicture}[sloped] + \node {.} + child {node {$M$} + child {node {$W$} + edge from parent + node[above] {...} + } + child {node {$B$} + edge from parent + node[above] {...} + } + edge from parent + node[above] {...} + } + child[missing] {} + child { node {$S$} + child {node {$W$} + edge from parent + node[above] {...} + } + child {node {$B$} + edge from parent + node[above] {...} + } + edge from parent + node[above] {...} + } ; + \end{tikzpicture} + \end{center} + + \item Calculer la probabilité de l'évènement $M \cap W$. Interpréter ce résultat dans le contexte de l'exercice. + \item Montrer que la probabilité que la fiche choisie corresponde à une formule \emph{Burger} est égale à $0.3336$. + \item On a prélevé une fiche correspondant à la formule \emph{Burger}. Quelle est la probabilité, arrondie au millième, que la vente ait eu lieu le soir? + \end{enumerate} +\end{exercise} + +\begin{solution} + \begin{enumerate} + \item + \begin{center} + \begin{tikzpicture}[sloped] + \node {.} + child {node {$M$} + child {node {$W$} + edge from parent + node[above] {$0.75$} + } + child {node {$B$} + edge from parent + node[above] {$0.25$} + } + edge from parent + node[above] {$0.78$} + } + child[missing] {} + child { node {$S$} + child {node {$W$} + edge from parent + node[above] {$0.37$} + } + child {node {$B$} + edge from parent + node[above] {$0.63$} + } + edge from parent + node[above] {$0.22$} + } ; + \end{tikzpicture} + \end{center} + \item On calcule la probabilité que la vente soit un wok et ait eu lieu à midi + \[ P(M\cap W) = P(M) \times P_M(W) = 0.78 \times 0.75 = 0.585 \] + \item Probabilité que la vente soit un burger. + \[ + P(B) = P(M\cap B) + P(S\cap B) = 0.78 \times 0.75 + 0.22 \times 0.37 = 0.3336 + \] + \item On cherche à calculer la quantité $P_B(S)$. Pour cela on utilise la formule de Bayes + \[ + P_B(S) = \frac{P(B\cap S)}{P(B)} = \frac{P_S(B) \times P(S)}{P(B)} = \frac{0.63\times 0.22}{0.3336} = 0.41546762589928055 \approx 0.415 + \] + \end{enumerate} +\end{solution} + +\begin{exercise}[subtitle={Continent plastique}] + \textit{Les quantités évoqués dans cette exercice sont générés au hasard et sont donc complètement farfelus.} + \medskip + Le \og continent de plastique\fg{} est la plus grande des plaques de déchets plastiques évoluant sur les océans. Elle occupe actuellement dans l'océan Pacifique une surface dont l'aire est évaluée à plus de $1,6$ million de km$^2$, entre Hawaï et la Californie. + + En 2017, des scientifiques ont estimé qu'il y avait $10$ millions de tonnes de déchets plastiques qui était déversé chaque année dans les océans et que cette quantité augmentait de $24\n\%$ par chaque année. + + On modélise l'évolution de la masse de ces déchets plastiques déversée chaque année, si rien n'est fait pour la réduire, par une suite géométrique $\left(u_n\right)$. L'arrondi au centième du terme $u_n$ représente la masse de ces déchets déversée chaque année, exprimée en million de tonnes, pour l'année $(2017 + n)$. + + \medskip + + \begin{enumerate} + \item Expliquer pourquoi la suite $u_n$ est géométrique? + \item Calculer $u_1$ et $u_2$. + \item Exprimer $u_n$ en fonction de $n$. + \item Au début de l'année 2017, il y avait $300$ millions de tonnes de déchets plastique. Calculer la quantité totale de déchets plastiques en 2030. + \item On souhaite déterminer en quelle année la masse totale de ces déchets plastiques aura pour la première fois augmenté de $50$\,\% par rapport à sa valeur de 2017. + \begin{enumerate} + \item Recopier et compléter l'algorithme ci-dessous pour que la variable $N$ contienne la réponse au problème posé. + + \begin{center} + \begin{tabularx}{0.4\linewidth}{|X|}\hline + $N = 2017$\\ + $U = 10$ \\ + $S = 300 + U$ \\ + while $S < 450$: \\ + \hspace{1cm} $N = \ldots$\\ + \hspace{1cm} $U = \ldots$\\ + \hspace{1cm} $S = \ldots$\\ + \hline + \end{tabularx} + \end{center} + \item Que contiennent les variables $S$, $U$ et $N$ après exécution de cet algorithme ? + + Interpréter les résultats dans le contexte de l'exercice. + \end{enumerate} + \end{enumerate} +\end{exercise} + +\begin{solution} + \begin{enumerate} + \item Une augmentation de $24\,\%$ revient à multiplier la quantité par $1.24$. La suite est donc bien géométrique. Son premier terme est $u_0 = 10$ et sa raison est $q = 1.24$ + \item + \[ + u_1 = u_0 * 1.24 = 12.4 + \] + \[ + u_2 = u_0 * 1.24^2 = 15.376 + \] + \item + \[ + u_n = u_0 \times q^n = 10 \times 1.24^n + \] + \item On calcule la quantité totale déversée entre 2017 et 2030. + \[ + \sum_{n = 0}^{13} u_n = u_0 \times \frac{1-q^{13}}{1-q} = 10 \times \frac{1 - 1.24^{13}}{1 - 1.24} = 641.1 + \] + On en déduit la quantité totale de déchets en 2030 + \[ + 300 + 641.1 = 941.1 + \] + \item + \begin{enumerate} + \item ~ + \begin{center} + \begin{tabularx}{0.4\linewidth}{|X|}\hline + $N \gets 2017$\\ + $U \gets 10$ \\ + $S \gets 300 + U$ \\ + Tant que $S < 450$ \\ + \hspace{1cm} $N \gets N + 1$\\ + \hspace{1cm} $U \gets U * 1.24$\\ + \hspace{1cm} $S \gets S + u$\\ + Fin Tant que\\\hline + \end{tabularx} + \end{center} + \item \textit{Pas de correction automatisé} + \end{enumerate} + \end{enumerate} +\end{solution} + +\end{document} + +%%% Local Variables: +%%% mode: latex +%%% TeX-master: "master" +%%% End: diff --git a/TST/DS/DS_21_04_07/TST1/corr_19_210407_DS8.tex b/TST/DS/DS_21_04_07/TST1/corr_19_210407_DS8.tex new file mode 100644 index 0000000..58a14ef --- /dev/null +++ b/TST/DS/DS_21_04_07/TST1/corr_19_210407_DS8.tex @@ -0,0 +1,262 @@ +\documentclass[a4paper,10pt]{article} +\usepackage{myXsim} + +% Title Page +\title{DS8 \hfill ONAL Yakub} +\tribe{TST} +\date{\hfillÀ render pour le Mercredi 7 avril} + +\xsimsetup{ + solution/print = true +} + +\begin{document} +\maketitle + +\begin{exercise}[subtitle={Automatismes}] + \textit{Toutes les questions de cette exercice sont indépendantes et peuvent être répondus séparément} + \begin{enumerate} + \item De janvier à septembre, une quantité a augmenté de $27\,\%$. Faire un schéma pour représenter la situation puis calculer le taux d'évolution moyen mensuel. + \item Une quantité augmente de $27\,\%$ par ans. En 2020, elle est de 132\euro. Quelle était sa valeur en 2019? Faire un schéma pour représenter la situation. + \item Déterminer l'équation de la droite \\ + \begin{tikzpicture}[xscale=0.8, yscale=0.5] + \tkzInit[xmin=-5,xmax=5,xstep=1, + ymin=-5,ymax=5,ystep=1] + \tkzGrid + \tkzAxeXY + \tkzFct[domain=-5:5,color=red,very thick]% + {1.0*\x -1}; + \end{tikzpicture} + \item Résoudre l'équation $10 \times 0.18^x = 7$ + \end{enumerate} +\end{exercise} + +\begin{solution} + \begin{enumerate} + \item On veut partager cette évolution en 8 évolutions. + \[ + \left(1 + \frac{27}{100}\right)^{\frac{1}{8}} = 1.0303 + \] + Donc le taux d'évolution moyen est + \[ + t_m = 1.0303 - 1 = 0.030299999999999994 + \] + \item Coefficient multiplicateur pour revenir en arrière + \[ + CM = (1 + \frac{27}{100})^{-1} = 0.7874 + \] + On en déduit la quantité en 2019 + \[ + 132 * 0.7874 = 103.9368 + \] + \item L'équation de la droite est + \[ + y = 1.0 x -1 + \] + \item Il faut penser à faire la division à par $10$ avant d'utiliser le log car sinon, on ne peut pas utiliser la formule $\log(a^n) = n\times \log(a)$. + + \[x = \frac{\log(0.7)}{\log(0.18)}\] + \end{enumerate} +\end{solution} + +\begin{exercise}[subtitle={Restaurant}] + Un \emph{food truck}, ouvert le midi et le soir, propose deux types de formules : + + \setlength\parindent{10mm} + \begin{itemize} + \item la formule \emph{Burger} ; + \item la formule \emph{Wok}. + \end{itemize} + \setlength\parindent{0mm} + + \medskip + + Le gérant a remarqué que 78\,\% de ses ventes ont lieu le midi. Le quart des ventes du midi correspondent à la formule \emph{Burger}, alors que 28\,\% des ventes du soir correspondent à la formule \emph{Wok}. + + Le gérant se constitue un fichier en notant, pour chaque vente, la formule choisie et le moment de cette vente (midi ou soir). + + On prélève une fiche de façon équiprobable. On définit les quatre évènements suivants: + + \begin{enumerate} + \item $M$ : \og la fiche correspond à une vente du midi\fg{} ; + \item $S$ : \og la fiche correspond à une vente du soir\fg {}; + \item $W$ : \og la fiche correspond à une formule \emph{Wok} \fg{} ; + \item $B$ : \og la fiche correspond à une formule \emph{Burger} \fg. + \end{enumerate} + \setlength\parindent{0mm} + + \medskip + + \begin{enumerate} + \item Recopier puis compléter l'arbre pondéré + + \begin{center} + \begin{tikzpicture}[sloped] + \node {.} + child {node {$M$} + child {node {$W$} + edge from parent + node[above] {...} + } + child {node {$B$} + edge from parent + node[above] {...} + } + edge from parent + node[above] {...} + } + child[missing] {} + child { node {$S$} + child {node {$W$} + edge from parent + node[above] {...} + } + child {node {$B$} + edge from parent + node[above] {...} + } + edge from parent + node[above] {...} + } ; + \end{tikzpicture} + \end{center} + + \item Calculer la probabilité de l'évènement $M \cap W$. Interpréter ce résultat dans le contexte de l'exercice. + \item Montrer que la probabilité que la fiche choisie corresponde à une formule \emph{Burger} est égale à $0.3534$. + \item On a prélevé une fiche correspondant à la formule \emph{Burger}. Quelle est la probabilité, arrondie au millième, que la vente ait eu lieu le soir? + \end{enumerate} +\end{exercise} + +\begin{solution} + \begin{enumerate} + \item + \begin{center} + \begin{tikzpicture}[sloped] + \node {.} + child {node {$M$} + child {node {$W$} + edge from parent + node[above] {$0.75$} + } + child {node {$B$} + edge from parent + node[above] {$0.25$} + } + edge from parent + node[above] {$0.78$} + } + child[missing] {} + child { node {$S$} + child {node {$W$} + edge from parent + node[above] {$0.28$} + } + child {node {$B$} + edge from parent + node[above] {$0.72$} + } + edge from parent + node[above] {$0.22$} + } ; + \end{tikzpicture} + \end{center} + \item On calcule la probabilité que la vente soit un wok et ait eu lieu à midi + \[ P(M\cap W) = P(M) \times P_M(W) = 0.78 \times 0.75 = 0.585 \] + \item Probabilité que la vente soit un burger. + \[ + P(B) = P(M\cap B) + P(S\cap B) = 0.78 \times 0.75 + 0.22 \times 0.28 = 0.3534 + \] + \item On cherche à calculer la quantité $P_B(S)$. Pour cela on utilise la formule de Bayes + \[ + P_B(S) = \frac{P(B\cap S)}{P(B)} = \frac{P_S(B) \times P(S)}{P(B)} = \frac{0.72\times 0.22}{0.3534} = 0.4482173174872665 \approx 0.448 + \] + \end{enumerate} +\end{solution} + +\begin{exercise}[subtitle={Continent plastique}] + \textit{Les quantités évoqués dans cette exercice sont générés au hasard et sont donc complètement farfelus.} + \medskip + Le \og continent de plastique\fg{} est la plus grande des plaques de déchets plastiques évoluant sur les océans. Elle occupe actuellement dans l'océan Pacifique une surface dont l'aire est évaluée à plus de $1,6$ million de km$^2$, entre Hawaï et la Californie. + + En 2017, des scientifiques ont estimé qu'il y avait $3$ millions de tonnes de déchets plastiques qui était déversé chaque année dans les océans et que cette quantité augmentait de $21\n\%$ par chaque année. + + On modélise l'évolution de la masse de ces déchets plastiques déversée chaque année, si rien n'est fait pour la réduire, par une suite géométrique $\left(u_n\right)$. L'arrondi au centième du terme $u_n$ représente la masse de ces déchets déversée chaque année, exprimée en million de tonnes, pour l'année $(2017 + n)$. + + \medskip + + \begin{enumerate} + \item Expliquer pourquoi la suite $u_n$ est géométrique? + \item Calculer $u_1$ et $u_2$. + \item Exprimer $u_n$ en fonction de $n$. + \item Au début de l'année 2017, il y avait $300$ millions de tonnes de déchets plastique. Calculer la quantité totale de déchets plastiques en 2030. + \item On souhaite déterminer en quelle année la masse totale de ces déchets plastiques aura pour la première fois augmenté de $50$\,\% par rapport à sa valeur de 2017. + \begin{enumerate} + \item Recopier et compléter l'algorithme ci-dessous pour que la variable $N$ contienne la réponse au problème posé. + + \begin{center} + \begin{tabularx}{0.4\linewidth}{|X|}\hline + $N = 2017$\\ + $U = 3$ \\ + $S = 300 + U$ \\ + while $S < 450$: \\ + \hspace{1cm} $N = \ldots$\\ + \hspace{1cm} $U = \ldots$\\ + \hspace{1cm} $S = \ldots$\\ + \hline + \end{tabularx} + \end{center} + \item Que contiennent les variables $S$, $U$ et $N$ après exécution de cet algorithme ? + + Interpréter les résultats dans le contexte de l'exercice. + \end{enumerate} + \end{enumerate} +\end{exercise} + +\begin{solution} + \begin{enumerate} + \item Une augmentation de $21\,\%$ revient à multiplier la quantité par $1.21$. La suite est donc bien géométrique. Son premier terme est $u_0 = 3$ et sa raison est $q = 1.21$ + \item + \[ + u_1 = u_0 * 1.21 = 3.63 + \] + \[ + u_2 = u_0 * 1.21^2 = 4.3923 + \] + \item + \[ + u_n = u_0 \times q^n = 3 \times 1.21^n + \] + \item On calcule la quantité totale déversée entre 2017 et 2030. + \[ + \sum_{n = 0}^{13} u_n = u_0 \times \frac{1-q^{13}}{1-q} = 3 \times \frac{1 - 1.21^{13}}{1 - 1.21} = 155.97 + \] + On en déduit la quantité totale de déchets en 2030 + \[ + 300 + 155.97 = 455.97 + \] + \item + \begin{enumerate} + \item ~ + \begin{center} + \begin{tabularx}{0.4\linewidth}{|X|}\hline + $N \gets 2017$\\ + $U \gets 3$ \\ + $S \gets 300 + U$ \\ + Tant que $S < 450$ \\ + \hspace{1cm} $N \gets N + 1$\\ + \hspace{1cm} $U \gets U * 1.21$\\ + \hspace{1cm} $S \gets S + u$\\ + Fin Tant que\\\hline + \end{tabularx} + \end{center} + \item \textit{Pas de correction automatisé} + \end{enumerate} + \end{enumerate} +\end{solution} + +\end{document} + +%%% Local Variables: +%%% mode: latex +%%% TeX-master: "master" +%%% End: diff --git a/TST/DS/DS_21_04_07/TST1/corr_20_210407_DS8.tex b/TST/DS/DS_21_04_07/TST1/corr_20_210407_DS8.tex new file mode 100644 index 0000000..f2bbbac --- /dev/null +++ b/TST/DS/DS_21_04_07/TST1/corr_20_210407_DS8.tex @@ -0,0 +1,262 @@ +\documentclass[a4paper,10pt]{article} +\usepackage{myXsim} + +% Title Page +\title{DS8 \hfill SORIANO Laura} +\tribe{TST} +\date{\hfillÀ render pour le Mercredi 7 avril} + +\xsimsetup{ + solution/print = true +} + +\begin{document} +\maketitle + +\begin{exercise}[subtitle={Automatismes}] + \textit{Toutes les questions de cette exercice sont indépendantes et peuvent être répondus séparément} + \begin{enumerate} + \item De janvier à septembre, une quantité a augmenté de $26\,\%$. Faire un schéma pour représenter la situation puis calculer le taux d'évolution moyen mensuel. + \item Une quantité augmente de $26\,\%$ par ans. En 2020, elle est de 122\euro. Quelle était sa valeur en 2019? Faire un schéma pour représenter la situation. + \item Déterminer l'équation de la droite \\ + \begin{tikzpicture}[xscale=0.8, yscale=0.5] + \tkzInit[xmin=-5,xmax=5,xstep=1, + ymin=-5,ymax=5,ystep=1] + \tkzGrid + \tkzAxeXY + \tkzFct[domain=-5:5,color=red,very thick]% + {3.0*\x -3}; + \end{tikzpicture} + \item Résoudre l'équation $2 \times 0.02^x = 10$ + \end{enumerate} +\end{exercise} + +\begin{solution} + \begin{enumerate} + \item On veut partager cette évolution en 8 évolutions. + \[ + \left(1 + \frac{26}{100}\right)^{\frac{1}{8}} = 1.0293 + \] + Donc le taux d'évolution moyen est + \[ + t_m = 1.0293 - 1 = 0.029300000000000104 + \] + \item Coefficient multiplicateur pour revenir en arrière + \[ + CM = (1 + \frac{26}{100})^{-1} = 0.7937 + \] + On en déduit la quantité en 2019 + \[ + 122 * 0.7937 = 96.8314 + \] + \item L'équation de la droite est + \[ + y = 3.0 x -3 + \] + \item Il faut penser à faire la division à par $2$ avant d'utiliser le log car sinon, on ne peut pas utiliser la formule $\log(a^n) = n\times \log(a)$. + + \[x = \frac{\log(5.0)}{\log(0.02)}\] + \end{enumerate} +\end{solution} + +\begin{exercise}[subtitle={Restaurant}] + Un \emph{food truck}, ouvert le midi et le soir, propose deux types de formules : + + \setlength\parindent{10mm} + \begin{itemize} + \item la formule \emph{Burger} ; + \item la formule \emph{Wok}. + \end{itemize} + \setlength\parindent{0mm} + + \medskip + + Le gérant a remarqué que 0\,\% de ses ventes ont lieu le midi. Le quart des ventes du midi correspondent à la formule \emph{Burger}, alors que 21\,\% des ventes du soir correspondent à la formule \emph{Wok}. + + Le gérant se constitue un fichier en notant, pour chaque vente, la formule choisie et le moment de cette vente (midi ou soir). + + On prélève une fiche de façon équiprobable. On définit les quatre évènements suivants: + + \begin{enumerate} + \item $M$ : \og la fiche correspond à une vente du midi\fg{} ; + \item $S$ : \og la fiche correspond à une vente du soir\fg {}; + \item $W$ : \og la fiche correspond à une formule \emph{Wok} \fg{} ; + \item $B$ : \og la fiche correspond à une formule \emph{Burger} \fg. + \end{enumerate} + \setlength\parindent{0mm} + + \medskip + + \begin{enumerate} + \item Recopier puis compléter l'arbre pondéré + + \begin{center} + \begin{tikzpicture}[sloped] + \node {.} + child {node {$M$} + child {node {$W$} + edge from parent + node[above] {...} + } + child {node {$B$} + edge from parent + node[above] {...} + } + edge from parent + node[above] {...} + } + child[missing] {} + child { node {$S$} + child {node {$W$} + edge from parent + node[above] {...} + } + child {node {$B$} + edge from parent + node[above] {...} + } + edge from parent + node[above] {...} + } ; + \end{tikzpicture} + \end{center} + + \item Calculer la probabilité de l'évènement $M \cap W$. Interpréter ce résultat dans le contexte de l'exercice. + \item Montrer que la probabilité que la fiche choisie corresponde à une formule \emph{Burger} est égale à $0.79$. + \item On a prélevé une fiche correspondant à la formule \emph{Burger}. Quelle est la probabilité, arrondie au millième, que la vente ait eu lieu le soir? + \end{enumerate} +\end{exercise} + +\begin{solution} + \begin{enumerate} + \item + \begin{center} + \begin{tikzpicture}[sloped] + \node {.} + child {node {$M$} + child {node {$W$} + edge from parent + node[above] {$0.75$} + } + child {node {$B$} + edge from parent + node[above] {$0.25$} + } + edge from parent + node[above] {$0.0$} + } + child[missing] {} + child { node {$S$} + child {node {$W$} + edge from parent + node[above] {$0.21$} + } + child {node {$B$} + edge from parent + node[above] {$0.79$} + } + edge from parent + node[above] {$1.0$} + } ; + \end{tikzpicture} + \end{center} + \item On calcule la probabilité que la vente soit un wok et ait eu lieu à midi + \[ P(M\cap W) = P(M) \times P_M(W) = 0.0 \times 0.75 = 0.0 \] + \item Probabilité que la vente soit un burger. + \[ + P(B) = P(M\cap B) + P(S\cap B) = 0.0 \times 0.75 + 1.0 \times 0.21 = 0.79 + \] + \item On cherche à calculer la quantité $P_B(S)$. Pour cela on utilise la formule de Bayes + \[ + P_B(S) = \frac{P(B\cap S)}{P(B)} = \frac{P_S(B) \times P(S)}{P(B)} = \frac{0.79\times 1.0}{0.79} = 1.0 \approx 1.0 + \] + \end{enumerate} +\end{solution} + +\begin{exercise}[subtitle={Continent plastique}] + \textit{Les quantités évoqués dans cette exercice sont générés au hasard et sont donc complètement farfelus.} + \medskip + Le \og continent de plastique\fg{} est la plus grande des plaques de déchets plastiques évoluant sur les océans. Elle occupe actuellement dans l'océan Pacifique une surface dont l'aire est évaluée à plus de $1,6$ million de km$^2$, entre Hawaï et la Californie. + + En 2017, des scientifiques ont estimé qu'il y avait $11$ millions de tonnes de déchets plastiques qui était déversé chaque année dans les océans et que cette quantité augmentait de $16\n\%$ par chaque année. + + On modélise l'évolution de la masse de ces déchets plastiques déversée chaque année, si rien n'est fait pour la réduire, par une suite géométrique $\left(u_n\right)$. L'arrondi au centième du terme $u_n$ représente la masse de ces déchets déversée chaque année, exprimée en million de tonnes, pour l'année $(2017 + n)$. + + \medskip + + \begin{enumerate} + \item Expliquer pourquoi la suite $u_n$ est géométrique? + \item Calculer $u_1$ et $u_2$. + \item Exprimer $u_n$ en fonction de $n$. + \item Au début de l'année 2017, il y avait $300$ millions de tonnes de déchets plastique. Calculer la quantité totale de déchets plastiques en 2030. + \item On souhaite déterminer en quelle année la masse totale de ces déchets plastiques aura pour la première fois augmenté de $50$\,\% par rapport à sa valeur de 2017. + \begin{enumerate} + \item Recopier et compléter l'algorithme ci-dessous pour que la variable $N$ contienne la réponse au problème posé. + + \begin{center} + \begin{tabularx}{0.4\linewidth}{|X|}\hline + $N = 2017$\\ + $U = 11$ \\ + $S = 300 + U$ \\ + while $S < 450$: \\ + \hspace{1cm} $N = \ldots$\\ + \hspace{1cm} $U = \ldots$\\ + \hspace{1cm} $S = \ldots$\\ + \hline + \end{tabularx} + \end{center} + \item Que contiennent les variables $S$, $U$ et $N$ après exécution de cet algorithme ? + + Interpréter les résultats dans le contexte de l'exercice. + \end{enumerate} + \end{enumerate} +\end{exercise} + +\begin{solution} + \begin{enumerate} + \item Une augmentation de $16\,\%$ revient à multiplier la quantité par $1.16$. La suite est donc bien géométrique. Son premier terme est $u_0 = 11$ et sa raison est $q = 1.16$ + \item + \[ + u_1 = u_0 * 1.16 = 12.76 + \] + \[ + u_2 = u_0 * 1.16^2 = 14.8016 + \] + \item + \[ + u_n = u_0 \times q^n = 11 \times 1.16^n + \] + \item On calcule la quantité totale déversée entre 2017 et 2030. + \[ + \sum_{n = 0}^{13} u_n = u_0 \times \frac{1-q^{13}}{1-q} = 11 \times \frac{1 - 1.16^{13}}{1 - 1.16} = 404.65 + \] + On en déduit la quantité totale de déchets en 2030 + \[ + 300 + 404.65 = 704.65 + \] + \item + \begin{enumerate} + \item ~ + \begin{center} + \begin{tabularx}{0.4\linewidth}{|X|}\hline + $N \gets 2017$\\ + $U \gets 11$ \\ + $S \gets 300 + U$ \\ + Tant que $S < 450$ \\ + \hspace{1cm} $N \gets N + 1$\\ + \hspace{1cm} $U \gets U * 1.16$\\ + \hspace{1cm} $S \gets S + u$\\ + Fin Tant que\\\hline + \end{tabularx} + \end{center} + \item \textit{Pas de correction automatisé} + \end{enumerate} + \end{enumerate} +\end{solution} + +\end{document} + +%%% Local Variables: +%%% mode: latex +%%% TeX-master: "master" +%%% End: diff --git a/TST/DS/DS_21_04_07/TST1/corr_21_210407_DS8.tex b/TST/DS/DS_21_04_07/TST1/corr_21_210407_DS8.tex new file mode 100644 index 0000000..c20e53f --- /dev/null +++ b/TST/DS/DS_21_04_07/TST1/corr_21_210407_DS8.tex @@ -0,0 +1,262 @@ +\documentclass[a4paper,10pt]{article} +\usepackage{myXsim} + +% Title Page +\title{DS8 \hfill VECCHIO Léa} +\tribe{TST} +\date{\hfillÀ render pour le Mercredi 7 avril} + +\xsimsetup{ + solution/print = true +} + +\begin{document} +\maketitle + +\begin{exercise}[subtitle={Automatismes}] + \textit{Toutes les questions de cette exercice sont indépendantes et peuvent être répondus séparément} + \begin{enumerate} + \item De janvier à septembre, une quantité a augmenté de $28\,\%$. Faire un schéma pour représenter la situation puis calculer le taux d'évolution moyen mensuel. + \item Une quantité augmente de $28\,\%$ par ans. En 2020, elle est de 113\euro. Quelle était sa valeur en 2019? Faire un schéma pour représenter la situation. + \item Déterminer l'équation de la droite \\ + \begin{tikzpicture}[xscale=0.8, yscale=0.5] + \tkzInit[xmin=-5,xmax=5,xstep=1, + ymin=-5,ymax=5,ystep=1] + \tkzGrid + \tkzAxeXY + \tkzFct[domain=-5:5,color=red,very thick]% + {2.0*\x -2}; + \end{tikzpicture} + \item Résoudre l'équation $8 \times 0.84^x = 26$ + \end{enumerate} +\end{exercise} + +\begin{solution} + \begin{enumerate} + \item On veut partager cette évolution en 8 évolutions. + \[ + \left(1 + \frac{28}{100}\right)^{\frac{1}{8}} = 1.0313 + \] + Donc le taux d'évolution moyen est + \[ + t_m = 1.0313 - 1 = 0.031300000000000106 + \] + \item Coefficient multiplicateur pour revenir en arrière + \[ + CM = (1 + \frac{28}{100})^{-1} = 0.7812 + \] + On en déduit la quantité en 2019 + \[ + 113 * 0.7812 = 88.2756 + \] + \item L'équation de la droite est + \[ + y = 2.0 x -2 + \] + \item Il faut penser à faire la division à par $8$ avant d'utiliser le log car sinon, on ne peut pas utiliser la formule $\log(a^n) = n\times \log(a)$. + + \[x = \frac{\log(3.25)}{\log(0.84)}\] + \end{enumerate} +\end{solution} + +\begin{exercise}[subtitle={Restaurant}] + Un \emph{food truck}, ouvert le midi et le soir, propose deux types de formules : + + \setlength\parindent{10mm} + \begin{itemize} + \item la formule \emph{Burger} ; + \item la formule \emph{Wok}. + \end{itemize} + \setlength\parindent{0mm} + + \medskip + + Le gérant a remarqué que 18\,\% de ses ventes ont lieu le midi. Le quart des ventes du midi correspondent à la formule \emph{Burger}, alors que 16\,\% des ventes du soir correspondent à la formule \emph{Wok}. + + Le gérant se constitue un fichier en notant, pour chaque vente, la formule choisie et le moment de cette vente (midi ou soir). + + On prélève une fiche de façon équiprobable. On définit les quatre évènements suivants: + + \begin{enumerate} + \item $M$ : \og la fiche correspond à une vente du midi\fg{} ; + \item $S$ : \og la fiche correspond à une vente du soir\fg {}; + \item $W$ : \og la fiche correspond à une formule \emph{Wok} \fg{} ; + \item $B$ : \og la fiche correspond à une formule \emph{Burger} \fg. + \end{enumerate} + \setlength\parindent{0mm} + + \medskip + + \begin{enumerate} + \item Recopier puis compléter l'arbre pondéré + + \begin{center} + \begin{tikzpicture}[sloped] + \node {.} + child {node {$M$} + child {node {$W$} + edge from parent + node[above] {...} + } + child {node {$B$} + edge from parent + node[above] {...} + } + edge from parent + node[above] {...} + } + child[missing] {} + child { node {$S$} + child {node {$W$} + edge from parent + node[above] {...} + } + child {node {$B$} + edge from parent + node[above] {...} + } + edge from parent + node[above] {...} + } ; + \end{tikzpicture} + \end{center} + + \item Calculer la probabilité de l'évènement $M \cap W$. Interpréter ce résultat dans le contexte de l'exercice. + \item Montrer que la probabilité que la fiche choisie corresponde à une formule \emph{Burger} est égale à $0.7338$. + \item On a prélevé une fiche correspondant à la formule \emph{Burger}. Quelle est la probabilité, arrondie au millième, que la vente ait eu lieu le soir? + \end{enumerate} +\end{exercise} + +\begin{solution} + \begin{enumerate} + \item + \begin{center} + \begin{tikzpicture}[sloped] + \node {.} + child {node {$M$} + child {node {$W$} + edge from parent + node[above] {$0.75$} + } + child {node {$B$} + edge from parent + node[above] {$0.25$} + } + edge from parent + node[above] {$0.18$} + } + child[missing] {} + child { node {$S$} + child {node {$W$} + edge from parent + node[above] {$0.16$} + } + child {node {$B$} + edge from parent + node[above] {$0.84$} + } + edge from parent + node[above] {$0.82$} + } ; + \end{tikzpicture} + \end{center} + \item On calcule la probabilité que la vente soit un wok et ait eu lieu à midi + \[ P(M\cap W) = P(M) \times P_M(W) = 0.18 \times 0.75 = 0.135 \] + \item Probabilité que la vente soit un burger. + \[ + P(B) = P(M\cap B) + P(S\cap B) = 0.18 \times 0.75 + 0.82 \times 0.16 = 0.7338 + \] + \item On cherche à calculer la quantité $P_B(S)$. Pour cela on utilise la formule de Bayes + \[ + P_B(S) = \frac{P(B\cap S)}{P(B)} = \frac{P_S(B) \times P(S)}{P(B)} = \frac{0.84\times 0.82}{0.7338} = 0.9386753883892068 \approx 0.939 + \] + \end{enumerate} +\end{solution} + +\begin{exercise}[subtitle={Continent plastique}] + \textit{Les quantités évoqués dans cette exercice sont générés au hasard et sont donc complètement farfelus.} + \medskip + Le \og continent de plastique\fg{} est la plus grande des plaques de déchets plastiques évoluant sur les océans. Elle occupe actuellement dans l'océan Pacifique une surface dont l'aire est évaluée à plus de $1,6$ million de km$^2$, entre Hawaï et la Californie. + + En 2017, des scientifiques ont estimé qu'il y avait $2$ millions de tonnes de déchets plastiques qui était déversé chaque année dans les océans et que cette quantité augmentait de $24\n\%$ par chaque année. + + On modélise l'évolution de la masse de ces déchets plastiques déversée chaque année, si rien n'est fait pour la réduire, par une suite géométrique $\left(u_n\right)$. L'arrondi au centième du terme $u_n$ représente la masse de ces déchets déversée chaque année, exprimée en million de tonnes, pour l'année $(2017 + n)$. + + \medskip + + \begin{enumerate} + \item Expliquer pourquoi la suite $u_n$ est géométrique? + \item Calculer $u_1$ et $u_2$. + \item Exprimer $u_n$ en fonction de $n$. + \item Au début de l'année 2017, il y avait $300$ millions de tonnes de déchets plastique. Calculer la quantité totale de déchets plastiques en 2030. + \item On souhaite déterminer en quelle année la masse totale de ces déchets plastiques aura pour la première fois augmenté de $50$\,\% par rapport à sa valeur de 2017. + \begin{enumerate} + \item Recopier et compléter l'algorithme ci-dessous pour que la variable $N$ contienne la réponse au problème posé. + + \begin{center} + \begin{tabularx}{0.4\linewidth}{|X|}\hline + $N = 2017$\\ + $U = 2$ \\ + $S = 300 + U$ \\ + while $S < 450$: \\ + \hspace{1cm} $N = \ldots$\\ + \hspace{1cm} $U = \ldots$\\ + \hspace{1cm} $S = \ldots$\\ + \hline + \end{tabularx} + \end{center} + \item Que contiennent les variables $S$, $U$ et $N$ après exécution de cet algorithme ? + + Interpréter les résultats dans le contexte de l'exercice. + \end{enumerate} + \end{enumerate} +\end{exercise} + +\begin{solution} + \begin{enumerate} + \item Une augmentation de $24\,\%$ revient à multiplier la quantité par $1.24$. La suite est donc bien géométrique. Son premier terme est $u_0 = 2$ et sa raison est $q = 1.24$ + \item + \[ + u_1 = u_0 * 1.24 = 2.48 + \] + \[ + u_2 = u_0 * 1.24^2 = 3.0752 + \] + \item + \[ + u_n = u_0 \times q^n = 2 \times 1.24^n + \] + \item On calcule la quantité totale déversée entre 2017 et 2030. + \[ + \sum_{n = 0}^{13} u_n = u_0 \times \frac{1-q^{13}}{1-q} = 2 \times \frac{1 - 1.24^{13}}{1 - 1.24} = 128.22 + \] + On en déduit la quantité totale de déchets en 2030 + \[ + 300 + 128.22 = 428.22 + \] + \item + \begin{enumerate} + \item ~ + \begin{center} + \begin{tabularx}{0.4\linewidth}{|X|}\hline + $N \gets 2017$\\ + $U \gets 2$ \\ + $S \gets 300 + U$ \\ + Tant que $S < 450$ \\ + \hspace{1cm} $N \gets N + 1$\\ + \hspace{1cm} $U \gets U * 1.24$\\ + \hspace{1cm} $S \gets S + u$\\ + Fin Tant que\\\hline + \end{tabularx} + \end{center} + \item \textit{Pas de correction automatisé} + \end{enumerate} + \end{enumerate} +\end{solution} + +\end{document} + +%%% Local Variables: +%%% mode: latex +%%% TeX-master: "master" +%%% End: diff --git a/TST/DS/DS_21_04_07/TST1/corr_all_210407_DS8.pdf b/TST/DS/DS_21_04_07/TST1/corr_all_210407_DS8.pdf new file mode 100644 index 0000000..b256be6 Binary files /dev/null and b/TST/DS/DS_21_04_07/TST1/corr_all_210407_DS8.pdf differ diff --git a/TST/DS/DS_21_04_07/TST3/01_210407_DS8.tex b/TST/DS/DS_21_04_07/TST3/01_210407_DS8.tex new file mode 100644 index 0000000..0e2f7e1 --- /dev/null +++ b/TST/DS/DS_21_04_07/TST3/01_210407_DS8.tex @@ -0,0 +1,262 @@ +\documentclass[a4paper,10pt]{article} +\usepackage{myXsim} + +% Title Page +\title{DS8 \hfill BELARBI Samira} +\tribe{TST} +\date{\hfillÀ render pour le Mercredi 7 avril} + +\xsimsetup{ + solution/print = false +} + +\begin{document} +\maketitle + +\begin{exercise}[subtitle={Automatismes}] + \textit{Toutes les questions de cette exercice sont indépendantes et peuvent être répondus séparément} + \begin{enumerate} + \item De janvier à septembre, une quantité a augmenté de $21\,\%$. Faire un schéma pour représenter la situation puis calculer le taux d'évolution moyen mensuel. + \item Une quantité augmente de $21\,\%$ par ans. En 2020, elle est de 143\euro. Quelle était sa valeur en 2019? Faire un schéma pour représenter la situation. + \item Déterminer l'équation de la droite \\ + \begin{tikzpicture}[xscale=0.8, yscale=0.5] + \tkzInit[xmin=-5,xmax=5,xstep=1, + ymin=-5,ymax=5,ystep=1] + \tkzGrid + \tkzAxeXY + \tkzFct[domain=-5:5,color=red,very thick]% + {2.6666666666666665*\x -4}; + \end{tikzpicture} + \item Résoudre l'équation $8 \times 0.76^x = 26$ + \end{enumerate} +\end{exercise} + +\begin{solution} + \begin{enumerate} + \item On veut partager cette évolution en 8 évolutions. + \[ + \left(1 + \frac{21}{100}\right)^{\frac{1}{8}} = 1.0241 + \] + Donc le taux d'évolution moyen est + \[ + t_m = 1.0241 - 1 = 0.02410000000000001 + \] + \item Coefficient multiplicateur pour revenir en arrière + \[ + CM = (1 + \frac{21}{100})^{-1} = 0.8264 + \] + On en déduit la quantité en 2019 + \[ + 143 * 0.8264 = 118.1752 + \] + \item L'équation de la droite est + \[ + y = 2.6666666666666665 x -4 + \] + \item Il faut penser à faire la division à par $8$ avant d'utiliser le log car sinon, on ne peut pas utiliser la formule $\log(a^n) = n\times \log(a)$. + + \[x = \frac{\log(3.25)}{\log(0.76)}\] + \end{enumerate} +\end{solution} + +\begin{exercise}[subtitle={Restaurant}] + Un \emph{food truck}, ouvert le midi et le soir, propose deux types de formules : + + \setlength\parindent{10mm} + \begin{itemize} + \item la formule \emph{Burger} ; + \item la formule \emph{Wok}. + \end{itemize} + \setlength\parindent{0mm} + + \medskip + + Le gérant a remarqué que 21\,\% de ses ventes ont lieu le midi. Le quart des ventes du midi correspondent à la formule \emph{Burger}, alors que 27\,\% des ventes du soir correspondent à la formule \emph{Wok}. + + Le gérant se constitue un fichier en notant, pour chaque vente, la formule choisie et le moment de cette vente (midi ou soir). + + On prélève une fiche de façon équiprobable. On définit les quatre évènements suivants: + + \begin{enumerate} + \item $M$ : \og la fiche correspond à une vente du midi\fg{} ; + \item $S$ : \og la fiche correspond à une vente du soir\fg {}; + \item $W$ : \og la fiche correspond à une formule \emph{Wok} \fg{} ; + \item $B$ : \og la fiche correspond à une formule \emph{Burger} \fg. + \end{enumerate} + \setlength\parindent{0mm} + + \medskip + + \begin{enumerate} + \item Recopier puis compléter l'arbre pondéré + + \begin{center} + \begin{tikzpicture}[sloped] + \node {.} + child {node {$M$} + child {node {$W$} + edge from parent + node[above] {...} + } + child {node {$B$} + edge from parent + node[above] {...} + } + edge from parent + node[above] {...} + } + child[missing] {} + child { node {$S$} + child {node {$W$} + edge from parent + node[above] {...} + } + child {node {$B$} + edge from parent + node[above] {...} + } + edge from parent + node[above] {...} + } ; + \end{tikzpicture} + \end{center} + + \item Calculer la probabilité de l'évènement $M \cap W$. Interpréter ce résultat dans le contexte de l'exercice. + \item Montrer que la probabilité que la fiche choisie corresponde à une formule \emph{Burger} est égale à $0.6292$. + \item On a prélevé une fiche correspondant à la formule \emph{Burger}. Quelle est la probabilité, arrondie au millième, que la vente ait eu lieu le soir? + \end{enumerate} +\end{exercise} + +\begin{solution} + \begin{enumerate} + \item + \begin{center} + \begin{tikzpicture}[sloped] + \node {.} + child {node {$M$} + child {node {$W$} + edge from parent + node[above] {$0.75$} + } + child {node {$B$} + edge from parent + node[above] {$0.25$} + } + edge from parent + node[above] {$0.21$} + } + child[missing] {} + child { node {$S$} + child {node {$W$} + edge from parent + node[above] {$0.27$} + } + child {node {$B$} + edge from parent + node[above] {$0.73$} + } + edge from parent + node[above] {$0.79$} + } ; + \end{tikzpicture} + \end{center} + \item On calcule la probabilité que la vente soit un wok et ait eu lieu à midi + \[ P(M\cap W) = P(M) \times P_M(W) = 0.21 \times 0.75 = 0.1575 \] + \item Probabilité que la vente soit un burger. + \[ + P(B) = P(M\cap B) + P(S\cap B) = 0.21 \times 0.75 + 0.79 \times 0.27 = 0.6292 + \] + \item On cherche à calculer la quantité $P_B(S)$. Pour cela on utilise la formule de Bayes + \[ + P_B(S) = \frac{P(B\cap S)}{P(B)} = \frac{P_S(B) \times P(S)}{P(B)} = \frac{0.73\times 0.79}{0.6292} = 0.9165607120152575 \approx 0.917 + \] + \end{enumerate} +\end{solution} + +\begin{exercise}[subtitle={Continent plastique}] + \textit{Les quantités évoqués dans cette exercice sont générés au hasard et sont donc complètement farfelus.} + \medskip + Le \og continent de plastique\fg{} est la plus grande des plaques de déchets plastiques évoluant sur les océans. Elle occupe actuellement dans l'océan Pacifique une surface dont l'aire est évaluée à plus de $1,6$ million de km$^2$, entre Hawaï et la Californie. + + En 2017, des scientifiques ont estimé qu'il y avait $20$ millions de tonnes de déchets plastiques qui était déversé chaque année dans les océans et que cette quantité augmentait de $28\n\%$ par chaque année. + + On modélise l'évolution de la masse de ces déchets plastiques déversée chaque année, si rien n'est fait pour la réduire, par une suite géométrique $\left(u_n\right)$. L'arrondi au centième du terme $u_n$ représente la masse de ces déchets déversée chaque année, exprimée en million de tonnes, pour l'année $(2017 + n)$. + + \medskip + + \begin{enumerate} + \item Expliquer pourquoi la suite $u_n$ est géométrique? + \item Calculer $u_1$ et $u_2$. + \item Exprimer $u_n$ en fonction de $n$. + \item Au début de l'année 2017, il y avait $300$ millions de tonnes de déchets plastique. Calculer la quantité totale de déchets plastiques en 2030. + \item On souhaite déterminer en quelle année la masse totale de ces déchets plastiques aura pour la première fois augmenté de $50$\,\% par rapport à sa valeur de 2017. + \begin{enumerate} + \item Recopier et compléter l'algorithme ci-dessous pour que la variable $N$ contienne la réponse au problème posé. + + \begin{center} + \begin{tabularx}{0.4\linewidth}{|X|}\hline + $N = 2017$\\ + $U = 20$ \\ + $S = 300 + U$ \\ + while $S < 450$: \\ + \hspace{1cm} $N = \ldots$\\ + \hspace{1cm} $U = \ldots$\\ + \hspace{1cm} $S = \ldots$\\ + \hline + \end{tabularx} + \end{center} + \item Que contiennent les variables $S$, $U$ et $N$ après exécution de cet algorithme ? + + Interpréter les résultats dans le contexte de l'exercice. + \end{enumerate} + \end{enumerate} +\end{exercise} + +\begin{solution} + \begin{enumerate} + \item Une augmentation de $28\,\%$ revient à multiplier la quantité par $1.28$. La suite est donc bien géométrique. Son premier terme est $u_0 = 20$ et sa raison est $q = 1.28$ + \item + \[ + u_1 = u_0 * 1.28 = 25.6 + \] + \[ + u_2 = u_0 * 1.28^2 = 32.768 + \] + \item + \[ + u_n = u_0 \times q^n = 20 \times 1.28^n + \] + \item On calcule la quantité totale déversée entre 2017 et 2030. + \[ + \sum_{n = 0}^{13} u_n = u_0 \times \frac{1-q^{13}}{1-q} = 20 \times \frac{1 - 1.28^{13}}{1 - 1.28} = 1697.06 + \] + On en déduit la quantité totale de déchets en 2030 + \[ + 300 + 1697.06 = 1997.06 + \] + \item + \begin{enumerate} + \item ~ + \begin{center} + \begin{tabularx}{0.4\linewidth}{|X|}\hline + $N \gets 2017$\\ + $U \gets 20$ \\ + $S \gets 300 + U$ \\ + Tant que $S < 450$ \\ + \hspace{1cm} $N \gets N + 1$\\ + \hspace{1cm} $U \gets U * 1.28$\\ + \hspace{1cm} $S \gets S + u$\\ + Fin Tant que\\\hline + \end{tabularx} + \end{center} + \item \textit{Pas de correction automatisé} + \end{enumerate} + \end{enumerate} +\end{solution} + +\end{document} + +%%% Local Variables: +%%% mode: latex +%%% TeX-master: "master" +%%% End: diff --git a/TST/DS/DS_21_04_07/TST3/02_210407_DS8.tex b/TST/DS/DS_21_04_07/TST3/02_210407_DS8.tex new file mode 100644 index 0000000..713c28b --- /dev/null +++ b/TST/DS/DS_21_04_07/TST3/02_210407_DS8.tex @@ -0,0 +1,262 @@ +\documentclass[a4paper,10pt]{article} +\usepackage{myXsim} + +% Title Page +\title{DS8 \hfill BERTAN Ufuk} +\tribe{TST} +\date{\hfillÀ render pour le Mercredi 7 avril} + +\xsimsetup{ + solution/print = false +} + +\begin{document} +\maketitle + +\begin{exercise}[subtitle={Automatismes}] + \textit{Toutes les questions de cette exercice sont indépendantes et peuvent être répondus séparément} + \begin{enumerate} + \item De janvier à septembre, une quantité a augmenté de $22\,\%$. Faire un schéma pour représenter la situation puis calculer le taux d'évolution moyen mensuel. + \item Une quantité augmente de $22\,\%$ par ans. En 2020, elle est de 134\euro. Quelle était sa valeur en 2019? Faire un schéma pour représenter la situation. + \item Déterminer l'équation de la droite \\ + \begin{tikzpicture}[xscale=0.8, yscale=0.5] + \tkzInit[xmin=-5,xmax=5,xstep=1, + ymin=-5,ymax=5,ystep=1] + \tkzGrid + \tkzAxeXY + \tkzFct[domain=-5:5,color=red,very thick]% + {2.0*\x -2}; + \end{tikzpicture} + \item Résoudre l'équation $5 \times 0.99^x = 21$ + \end{enumerate} +\end{exercise} + +\begin{solution} + \begin{enumerate} + \item On veut partager cette évolution en 8 évolutions. + \[ + \left(1 + \frac{22}{100}\right)^{\frac{1}{8}} = 1.0252 + \] + Donc le taux d'évolution moyen est + \[ + t_m = 1.0252 - 1 = 0.02519999999999989 + \] + \item Coefficient multiplicateur pour revenir en arrière + \[ + CM = (1 + \frac{22}{100})^{-1} = 0.8197 + \] + On en déduit la quantité en 2019 + \[ + 134 * 0.8197 = 109.8398 + \] + \item L'équation de la droite est + \[ + y = 2.0 x -2 + \] + \item Il faut penser à faire la division à par $5$ avant d'utiliser le log car sinon, on ne peut pas utiliser la formule $\log(a^n) = n\times \log(a)$. + + \[x = \frac{\log(4.2)}{\log(0.99)}\] + \end{enumerate} +\end{solution} + +\begin{exercise}[subtitle={Restaurant}] + Un \emph{food truck}, ouvert le midi et le soir, propose deux types de formules : + + \setlength\parindent{10mm} + \begin{itemize} + \item la formule \emph{Burger} ; + \item la formule \emph{Wok}. + \end{itemize} + \setlength\parindent{0mm} + + \medskip + + Le gérant a remarqué que 64\,\% de ses ventes ont lieu le midi. Le quart des ventes du midi correspondent à la formule \emph{Burger}, alors que 100\,\% des ventes du soir correspondent à la formule \emph{Wok}. + + Le gérant se constitue un fichier en notant, pour chaque vente, la formule choisie et le moment de cette vente (midi ou soir). + + On prélève une fiche de façon équiprobable. On définit les quatre évènements suivants: + + \begin{enumerate} + \item $M$ : \og la fiche correspond à une vente du midi\fg{} ; + \item $S$ : \og la fiche correspond à une vente du soir\fg {}; + \item $W$ : \og la fiche correspond à une formule \emph{Wok} \fg{} ; + \item $B$ : \og la fiche correspond à une formule \emph{Burger} \fg. + \end{enumerate} + \setlength\parindent{0mm} + + \medskip + + \begin{enumerate} + \item Recopier puis compléter l'arbre pondéré + + \begin{center} + \begin{tikzpicture}[sloped] + \node {.} + child {node {$M$} + child {node {$W$} + edge from parent + node[above] {...} + } + child {node {$B$} + edge from parent + node[above] {...} + } + edge from parent + node[above] {...} + } + child[missing] {} + child { node {$S$} + child {node {$W$} + edge from parent + node[above] {...} + } + child {node {$B$} + edge from parent + node[above] {...} + } + edge from parent + node[above] {...} + } ; + \end{tikzpicture} + \end{center} + + \item Calculer la probabilité de l'évènement $M \cap W$. Interpréter ce résultat dans le contexte de l'exercice. + \item Montrer que la probabilité que la fiche choisie corresponde à une formule \emph{Burger} est égale à $0.16$. + \item On a prélevé une fiche correspondant à la formule \emph{Burger}. Quelle est la probabilité, arrondie au millième, que la vente ait eu lieu le soir? + \end{enumerate} +\end{exercise} + +\begin{solution} + \begin{enumerate} + \item + \begin{center} + \begin{tikzpicture}[sloped] + \node {.} + child {node {$M$} + child {node {$W$} + edge from parent + node[above] {$0.75$} + } + child {node {$B$} + edge from parent + node[above] {$0.25$} + } + edge from parent + node[above] {$0.64$} + } + child[missing] {} + child { node {$S$} + child {node {$W$} + edge from parent + node[above] {$1.0$} + } + child {node {$B$} + edge from parent + node[above] {$0.0$} + } + edge from parent + node[above] {$0.36$} + } ; + \end{tikzpicture} + \end{center} + \item On calcule la probabilité que la vente soit un wok et ait eu lieu à midi + \[ P(M\cap W) = P(M) \times P_M(W) = 0.64 \times 0.75 = 0.48 \] + \item Probabilité que la vente soit un burger. + \[ + P(B) = P(M\cap B) + P(S\cap B) = 0.64 \times 0.75 + 0.36 \times 1.0 = 0.16 + \] + \item On cherche à calculer la quantité $P_B(S)$. Pour cela on utilise la formule de Bayes + \[ + P_B(S) = \frac{P(B\cap S)}{P(B)} = \frac{P_S(B) \times P(S)}{P(B)} = \frac{0.0\times 0.36}{0.16} = 0.0 \approx 0.0 + \] + \end{enumerate} +\end{solution} + +\begin{exercise}[subtitle={Continent plastique}] + \textit{Les quantités évoqués dans cette exercice sont générés au hasard et sont donc complètement farfelus.} + \medskip + Le \og continent de plastique\fg{} est la plus grande des plaques de déchets plastiques évoluant sur les océans. Elle occupe actuellement dans l'océan Pacifique une surface dont l'aire est évaluée à plus de $1,6$ million de km$^2$, entre Hawaï et la Californie. + + En 2017, des scientifiques ont estimé qu'il y avait $2$ millions de tonnes de déchets plastiques qui était déversé chaque année dans les océans et que cette quantité augmentait de $27\n\%$ par chaque année. + + On modélise l'évolution de la masse de ces déchets plastiques déversée chaque année, si rien n'est fait pour la réduire, par une suite géométrique $\left(u_n\right)$. L'arrondi au centième du terme $u_n$ représente la masse de ces déchets déversée chaque année, exprimée en million de tonnes, pour l'année $(2017 + n)$. + + \medskip + + \begin{enumerate} + \item Expliquer pourquoi la suite $u_n$ est géométrique? + \item Calculer $u_1$ et $u_2$. + \item Exprimer $u_n$ en fonction de $n$. + \item Au début de l'année 2017, il y avait $300$ millions de tonnes de déchets plastique. Calculer la quantité totale de déchets plastiques en 2030. + \item On souhaite déterminer en quelle année la masse totale de ces déchets plastiques aura pour la première fois augmenté de $50$\,\% par rapport à sa valeur de 2017. + \begin{enumerate} + \item Recopier et compléter l'algorithme ci-dessous pour que la variable $N$ contienne la réponse au problème posé. + + \begin{center} + \begin{tabularx}{0.4\linewidth}{|X|}\hline + $N = 2017$\\ + $U = 2$ \\ + $S = 300 + U$ \\ + while $S < 450$: \\ + \hspace{1cm} $N = \ldots$\\ + \hspace{1cm} $U = \ldots$\\ + \hspace{1cm} $S = \ldots$\\ + \hline + \end{tabularx} + \end{center} + \item Que contiennent les variables $S$, $U$ et $N$ après exécution de cet algorithme ? + + Interpréter les résultats dans le contexte de l'exercice. + \end{enumerate} + \end{enumerate} +\end{exercise} + +\begin{solution} + \begin{enumerate} + \item Une augmentation de $27\,\%$ revient à multiplier la quantité par $1.27$. La suite est donc bien géométrique. Son premier terme est $u_0 = 2$ et sa raison est $q = 1.27$ + \item + \[ + u_1 = u_0 * 1.27 = 2.54 + \] + \[ + u_2 = u_0 * 1.27^2 = 3.2258 + \] + \item + \[ + u_n = u_0 \times q^n = 2 \times 1.27^n + \] + \item On calcule la quantité totale déversée entre 2017 et 2030. + \[ + \sum_{n = 0}^{13} u_n = u_0 \times \frac{1-q^{13}}{1-q} = 2 \times \frac{1 - 1.27^{13}}{1 - 1.27} = 158.21 + \] + On en déduit la quantité totale de déchets en 2030 + \[ + 300 + 158.21 = 458.21000000000004 + \] + \item + \begin{enumerate} + \item ~ + \begin{center} + \begin{tabularx}{0.4\linewidth}{|X|}\hline + $N \gets 2017$\\ + $U \gets 2$ \\ + $S \gets 300 + U$ \\ + Tant que $S < 450$ \\ + \hspace{1cm} $N \gets N + 1$\\ + \hspace{1cm} $U \gets U * 1.27$\\ + \hspace{1cm} $S \gets S + u$\\ + Fin Tant que\\\hline + \end{tabularx} + \end{center} + \item \textit{Pas de correction automatisé} + \end{enumerate} + \end{enumerate} +\end{solution} + +\end{document} + +%%% Local Variables: +%%% mode: latex +%%% TeX-master: "master" +%%% End: diff --git a/TST/DS/DS_21_04_07/TST3/03_210407_DS8.tex b/TST/DS/DS_21_04_07/TST3/03_210407_DS8.tex new file mode 100644 index 0000000..85f4f63 --- /dev/null +++ b/TST/DS/DS_21_04_07/TST3/03_210407_DS8.tex @@ -0,0 +1,262 @@ +\documentclass[a4paper,10pt]{article} +\usepackage{myXsim} + +% Title Page +\title{DS8 \hfill BOUALIA Bilel} +\tribe{TST} +\date{\hfillÀ render pour le Mercredi 7 avril} + +\xsimsetup{ + solution/print = false +} + +\begin{document} +\maketitle + +\begin{exercise}[subtitle={Automatismes}] + \textit{Toutes les questions de cette exercice sont indépendantes et peuvent être répondus séparément} + \begin{enumerate} + \item De janvier à septembre, une quantité a augmenté de $17\,\%$. Faire un schéma pour représenter la situation puis calculer le taux d'évolution moyen mensuel. + \item Une quantité augmente de $17\,\%$ par ans. En 2020, elle est de 135\euro. Quelle était sa valeur en 2019? Faire un schéma pour représenter la situation. + \item Déterminer l'équation de la droite \\ + \begin{tikzpicture}[xscale=0.8, yscale=0.5] + \tkzInit[xmin=-5,xmax=5,xstep=1, + ymin=-5,ymax=5,ystep=1] + \tkzGrid + \tkzAxeXY + \tkzFct[domain=-5:5,color=red,very thick]% + {2.0*\x -4}; + \end{tikzpicture} + \item Résoudre l'équation $3 \times 0.2^x = 2$ + \end{enumerate} +\end{exercise} + +\begin{solution} + \begin{enumerate} + \item On veut partager cette évolution en 8 évolutions. + \[ + \left(1 + \frac{17}{100}\right)^{\frac{1}{8}} = 1.0198 + \] + Donc le taux d'évolution moyen est + \[ + t_m = 1.0198 - 1 = 0.01980000000000004 + \] + \item Coefficient multiplicateur pour revenir en arrière + \[ + CM = (1 + \frac{17}{100})^{-1} = 0.8547 + \] + On en déduit la quantité en 2019 + \[ + 135 * 0.8547 = 115.3845 + \] + \item L'équation de la droite est + \[ + y = 2.0 x -4 + \] + \item Il faut penser à faire la division à par $3$ avant d'utiliser le log car sinon, on ne peut pas utiliser la formule $\log(a^n) = n\times \log(a)$. + + \[x = \frac{\log(0.67)}{\log(0.2)}\] + \end{enumerate} +\end{solution} + +\begin{exercise}[subtitle={Restaurant}] + Un \emph{food truck}, ouvert le midi et le soir, propose deux types de formules : + + \setlength\parindent{10mm} + \begin{itemize} + \item la formule \emph{Burger} ; + \item la formule \emph{Wok}. + \end{itemize} + \setlength\parindent{0mm} + + \medskip + + Le gérant a remarqué que 89\,\% de ses ventes ont lieu le midi. Le quart des ventes du midi correspondent à la formule \emph{Burger}, alors que 12\,\% des ventes du soir correspondent à la formule \emph{Wok}. + + Le gérant se constitue un fichier en notant, pour chaque vente, la formule choisie et le moment de cette vente (midi ou soir). + + On prélève une fiche de façon équiprobable. On définit les quatre évènements suivants: + + \begin{enumerate} + \item $M$ : \og la fiche correspond à une vente du midi\fg{} ; + \item $S$ : \og la fiche correspond à une vente du soir\fg {}; + \item $W$ : \og la fiche correspond à une formule \emph{Wok} \fg{} ; + \item $B$ : \og la fiche correspond à une formule \emph{Burger} \fg. + \end{enumerate} + \setlength\parindent{0mm} + + \medskip + + \begin{enumerate} + \item Recopier puis compléter l'arbre pondéré + + \begin{center} + \begin{tikzpicture}[sloped] + \node {.} + child {node {$M$} + child {node {$W$} + edge from parent + node[above] {...} + } + child {node {$B$} + edge from parent + node[above] {...} + } + edge from parent + node[above] {...} + } + child[missing] {} + child { node {$S$} + child {node {$W$} + edge from parent + node[above] {...} + } + child {node {$B$} + edge from parent + node[above] {...} + } + edge from parent + node[above] {...} + } ; + \end{tikzpicture} + \end{center} + + \item Calculer la probabilité de l'évènement $M \cap W$. Interpréter ce résultat dans le contexte de l'exercice. + \item Montrer que la probabilité que la fiche choisie corresponde à une formule \emph{Burger} est égale à $0.3193$. + \item On a prélevé une fiche correspondant à la formule \emph{Burger}. Quelle est la probabilité, arrondie au millième, que la vente ait eu lieu le soir? + \end{enumerate} +\end{exercise} + +\begin{solution} + \begin{enumerate} + \item + \begin{center} + \begin{tikzpicture}[sloped] + \node {.} + child {node {$M$} + child {node {$W$} + edge from parent + node[above] {$0.75$} + } + child {node {$B$} + edge from parent + node[above] {$0.25$} + } + edge from parent + node[above] {$0.89$} + } + child[missing] {} + child { node {$S$} + child {node {$W$} + edge from parent + node[above] {$0.12$} + } + child {node {$B$} + edge from parent + node[above] {$0.88$} + } + edge from parent + node[above] {$0.11$} + } ; + \end{tikzpicture} + \end{center} + \item On calcule la probabilité que la vente soit un wok et ait eu lieu à midi + \[ P(M\cap W) = P(M) \times P_M(W) = 0.89 \times 0.75 = 0.6675 \] + \item Probabilité que la vente soit un burger. + \[ + P(B) = P(M\cap B) + P(S\cap B) = 0.89 \times 0.75 + 0.11 \times 0.12 = 0.3193 + \] + \item On cherche à calculer la quantité $P_B(S)$. Pour cela on utilise la formule de Bayes + \[ + P_B(S) = \frac{P(B\cap S)}{P(B)} = \frac{P_S(B) \times P(S)}{P(B)} = \frac{0.88\times 0.11}{0.3193} = 0.303163169433135 \approx 0.303 + \] + \end{enumerate} +\end{solution} + +\begin{exercise}[subtitle={Continent plastique}] + \textit{Les quantités évoqués dans cette exercice sont générés au hasard et sont donc complètement farfelus.} + \medskip + Le \og continent de plastique\fg{} est la plus grande des plaques de déchets plastiques évoluant sur les océans. Elle occupe actuellement dans l'océan Pacifique une surface dont l'aire est évaluée à plus de $1,6$ million de km$^2$, entre Hawaï et la Californie. + + En 2017, des scientifiques ont estimé qu'il y avait $10$ millions de tonnes de déchets plastiques qui était déversé chaque année dans les océans et que cette quantité augmentait de $27\n\%$ par chaque année. + + On modélise l'évolution de la masse de ces déchets plastiques déversée chaque année, si rien n'est fait pour la réduire, par une suite géométrique $\left(u_n\right)$. L'arrondi au centième du terme $u_n$ représente la masse de ces déchets déversée chaque année, exprimée en million de tonnes, pour l'année $(2017 + n)$. + + \medskip + + \begin{enumerate} + \item Expliquer pourquoi la suite $u_n$ est géométrique? + \item Calculer $u_1$ et $u_2$. + \item Exprimer $u_n$ en fonction de $n$. + \item Au début de l'année 2017, il y avait $300$ millions de tonnes de déchets plastique. Calculer la quantité totale de déchets plastiques en 2030. + \item On souhaite déterminer en quelle année la masse totale de ces déchets plastiques aura pour la première fois augmenté de $50$\,\% par rapport à sa valeur de 2017. + \begin{enumerate} + \item Recopier et compléter l'algorithme ci-dessous pour que la variable $N$ contienne la réponse au problème posé. + + \begin{center} + \begin{tabularx}{0.4\linewidth}{|X|}\hline + $N = 2017$\\ + $U = 10$ \\ + $S = 300 + U$ \\ + while $S < 450$: \\ + \hspace{1cm} $N = \ldots$\\ + \hspace{1cm} $U = \ldots$\\ + \hspace{1cm} $S = \ldots$\\ + \hline + \end{tabularx} + \end{center} + \item Que contiennent les variables $S$, $U$ et $N$ après exécution de cet algorithme ? + + Interpréter les résultats dans le contexte de l'exercice. + \end{enumerate} + \end{enumerate} +\end{exercise} + +\begin{solution} + \begin{enumerate} + \item Une augmentation de $27\,\%$ revient à multiplier la quantité par $1.27$. La suite est donc bien géométrique. Son premier terme est $u_0 = 10$ et sa raison est $q = 1.27$ + \item + \[ + u_1 = u_0 * 1.27 = 12.7 + \] + \[ + u_2 = u_0 * 1.27^2 = 16.129 + \] + \item + \[ + u_n = u_0 \times q^n = 10 \times 1.27^n + \] + \item On calcule la quantité totale déversée entre 2017 et 2030. + \[ + \sum_{n = 0}^{13} u_n = u_0 \times \frac{1-q^{13}}{1-q} = 10 \times \frac{1 - 1.27^{13}}{1 - 1.27} = 791.07 + \] + On en déduit la quantité totale de déchets en 2030 + \[ + 300 + 791.07 = 1091.0700000000002 + \] + \item + \begin{enumerate} + \item ~ + \begin{center} + \begin{tabularx}{0.4\linewidth}{|X|}\hline + $N \gets 2017$\\ + $U \gets 10$ \\ + $S \gets 300 + U$ \\ + Tant que $S < 450$ \\ + \hspace{1cm} $N \gets N + 1$\\ + \hspace{1cm} $U \gets U * 1.27$\\ + \hspace{1cm} $S \gets S + u$\\ + Fin Tant que\\\hline + \end{tabularx} + \end{center} + \item \textit{Pas de correction automatisé} + \end{enumerate} + \end{enumerate} +\end{solution} + +\end{document} + +%%% Local Variables: +%%% mode: latex +%%% TeX-master: "master" +%%% End: diff --git a/TST/DS/DS_21_04_07/TST3/04_210407_DS8.tex b/TST/DS/DS_21_04_07/TST3/04_210407_DS8.tex new file mode 100644 index 0000000..6631ab2 --- /dev/null +++ b/TST/DS/DS_21_04_07/TST3/04_210407_DS8.tex @@ -0,0 +1,262 @@ +\documentclass[a4paper,10pt]{article} +\usepackage{myXsim} + +% Title Page +\title{DS8 \hfill BOUCHOUX Kevin} +\tribe{TST} +\date{\hfillÀ render pour le Mercredi 7 avril} + +\xsimsetup{ + solution/print = false +} + +\begin{document} +\maketitle + +\begin{exercise}[subtitle={Automatismes}] + \textit{Toutes les questions de cette exercice sont indépendantes et peuvent être répondus séparément} + \begin{enumerate} + \item De janvier à septembre, une quantité a augmenté de $10\,\%$. Faire un schéma pour représenter la situation puis calculer le taux d'évolution moyen mensuel. + \item Une quantité augmente de $10\,\%$ par ans. En 2020, elle est de 118\euro. Quelle était sa valeur en 2019? Faire un schéma pour représenter la situation. + \item Déterminer l'équation de la droite \\ + \begin{tikzpicture}[xscale=0.8, yscale=0.5] + \tkzInit[xmin=-5,xmax=5,xstep=1, + ymin=-5,ymax=5,ystep=1] + \tkzGrid + \tkzAxeXY + \tkzFct[domain=-5:5,color=red,very thick]% + {3.0*\x -3}; + \end{tikzpicture} + \item Résoudre l'équation $2 \times 0.93^x = 44$ + \end{enumerate} +\end{exercise} + +\begin{solution} + \begin{enumerate} + \item On veut partager cette évolution en 8 évolutions. + \[ + \left(1 + \frac{10}{100}\right)^{\frac{1}{8}} = 1.012 + \] + Donc le taux d'évolution moyen est + \[ + t_m = 1.012 - 1 = 0.01200000000000001 + \] + \item Coefficient multiplicateur pour revenir en arrière + \[ + CM = (1 + \frac{10}{100})^{-1} = 0.9091 + \] + On en déduit la quantité en 2019 + \[ + 118 * 0.9091 = 107.27380000000001 + \] + \item L'équation de la droite est + \[ + y = 3.0 x -3 + \] + \item Il faut penser à faire la division à par $2$ avant d'utiliser le log car sinon, on ne peut pas utiliser la formule $\log(a^n) = n\times \log(a)$. + + \[x = \frac{\log(22.0)}{\log(0.93)}\] + \end{enumerate} +\end{solution} + +\begin{exercise}[subtitle={Restaurant}] + Un \emph{food truck}, ouvert le midi et le soir, propose deux types de formules : + + \setlength\parindent{10mm} + \begin{itemize} + \item la formule \emph{Burger} ; + \item la formule \emph{Wok}. + \end{itemize} + \setlength\parindent{0mm} + + \medskip + + Le gérant a remarqué que 83\,\% de ses ventes ont lieu le midi. Le quart des ventes du midi correspondent à la formule \emph{Burger}, alors que 26\,\% des ventes du soir correspondent à la formule \emph{Wok}. + + Le gérant se constitue un fichier en notant, pour chaque vente, la formule choisie et le moment de cette vente (midi ou soir). + + On prélève une fiche de façon équiprobable. On définit les quatre évènements suivants: + + \begin{enumerate} + \item $M$ : \og la fiche correspond à une vente du midi\fg{} ; + \item $S$ : \og la fiche correspond à une vente du soir\fg {}; + \item $W$ : \og la fiche correspond à une formule \emph{Wok} \fg{} ; + \item $B$ : \og la fiche correspond à une formule \emph{Burger} \fg. + \end{enumerate} + \setlength\parindent{0mm} + + \medskip + + \begin{enumerate} + \item Recopier puis compléter l'arbre pondéré + + \begin{center} + \begin{tikzpicture}[sloped] + \node {.} + child {node {$M$} + child {node {$W$} + edge from parent + node[above] {...} + } + child {node {$B$} + edge from parent + node[above] {...} + } + edge from parent + node[above] {...} + } + child[missing] {} + child { node {$S$} + child {node {$W$} + edge from parent + node[above] {...} + } + child {node {$B$} + edge from parent + node[above] {...} + } + edge from parent + node[above] {...} + } ; + \end{tikzpicture} + \end{center} + + \item Calculer la probabilité de l'évènement $M \cap W$. Interpréter ce résultat dans le contexte de l'exercice. + \item Montrer que la probabilité que la fiche choisie corresponde à une formule \emph{Burger} est égale à $0.3333$. + \item On a prélevé une fiche correspondant à la formule \emph{Burger}. Quelle est la probabilité, arrondie au millième, que la vente ait eu lieu le soir? + \end{enumerate} +\end{exercise} + +\begin{solution} + \begin{enumerate} + \item + \begin{center} + \begin{tikzpicture}[sloped] + \node {.} + child {node {$M$} + child {node {$W$} + edge from parent + node[above] {$0.75$} + } + child {node {$B$} + edge from parent + node[above] {$0.25$} + } + edge from parent + node[above] {$0.83$} + } + child[missing] {} + child { node {$S$} + child {node {$W$} + edge from parent + node[above] {$0.26$} + } + child {node {$B$} + edge from parent + node[above] {$0.74$} + } + edge from parent + node[above] {$0.17$} + } ; + \end{tikzpicture} + \end{center} + \item On calcule la probabilité que la vente soit un wok et ait eu lieu à midi + \[ P(M\cap W) = P(M) \times P_M(W) = 0.83 \times 0.75 = 0.6225 \] + \item Probabilité que la vente soit un burger. + \[ + P(B) = P(M\cap B) + P(S\cap B) = 0.83 \times 0.75 + 0.17 \times 0.26 = 0.3333 + \] + \item On cherche à calculer la quantité $P_B(S)$. Pour cela on utilise la formule de Bayes + \[ + P_B(S) = \frac{P(B\cap S)}{P(B)} = \frac{P_S(B) \times P(S)}{P(B)} = \frac{0.74\times 0.17}{0.3333} = 0.37743774377437744 \approx 0.377 + \] + \end{enumerate} +\end{solution} + +\begin{exercise}[subtitle={Continent plastique}] + \textit{Les quantités évoqués dans cette exercice sont générés au hasard et sont donc complètement farfelus.} + \medskip + Le \og continent de plastique\fg{} est la plus grande des plaques de déchets plastiques évoluant sur les océans. Elle occupe actuellement dans l'océan Pacifique une surface dont l'aire est évaluée à plus de $1,6$ million de km$^2$, entre Hawaï et la Californie. + + En 2017, des scientifiques ont estimé qu'il y avait $18$ millions de tonnes de déchets plastiques qui était déversé chaque année dans les océans et que cette quantité augmentait de $14\n\%$ par chaque année. + + On modélise l'évolution de la masse de ces déchets plastiques déversée chaque année, si rien n'est fait pour la réduire, par une suite géométrique $\left(u_n\right)$. L'arrondi au centième du terme $u_n$ représente la masse de ces déchets déversée chaque année, exprimée en million de tonnes, pour l'année $(2017 + n)$. + + \medskip + + \begin{enumerate} + \item Expliquer pourquoi la suite $u_n$ est géométrique? + \item Calculer $u_1$ et $u_2$. + \item Exprimer $u_n$ en fonction de $n$. + \item Au début de l'année 2017, il y avait $300$ millions de tonnes de déchets plastique. Calculer la quantité totale de déchets plastiques en 2030. + \item On souhaite déterminer en quelle année la masse totale de ces déchets plastiques aura pour la première fois augmenté de $50$\,\% par rapport à sa valeur de 2017. + \begin{enumerate} + \item Recopier et compléter l'algorithme ci-dessous pour que la variable $N$ contienne la réponse au problème posé. + + \begin{center} + \begin{tabularx}{0.4\linewidth}{|X|}\hline + $N = 2017$\\ + $U = 18$ \\ + $S = 300 + U$ \\ + while $S < 450$: \\ + \hspace{1cm} $N = \ldots$\\ + \hspace{1cm} $U = \ldots$\\ + \hspace{1cm} $S = \ldots$\\ + \hline + \end{tabularx} + \end{center} + \item Que contiennent les variables $S$, $U$ et $N$ après exécution de cet algorithme ? + + Interpréter les résultats dans le contexte de l'exercice. + \end{enumerate} + \end{enumerate} +\end{exercise} + +\begin{solution} + \begin{enumerate} + \item Une augmentation de $14\,\%$ revient à multiplier la quantité par $1.1400000000000001$. La suite est donc bien géométrique. Son premier terme est $u_0 = 18$ et sa raison est $q = 1.1400000000000001$ + \item + \[ + u_1 = u_0 * 1.1400000000000001 = 20.520000000000003 + \] + \[ + u_2 = u_0 * 1.1400000000000001^2 = 23.3928 + \] + \item + \[ + u_n = u_0 \times q^n = 18 \times 1.1400000000000001^n + \] + \item On calcule la quantité totale déversée entre 2017 et 2030. + \[ + \sum_{n = 0}^{13} u_n = u_0 \times \frac{1-q^{13}}{1-q} = 18 \times \frac{1 - 1.1400000000000001^{13}}{1 - 1.1400000000000001} = 577.6 + \] + On en déduit la quantité totale de déchets en 2030 + \[ + 300 + 577.6 = 877.6 + \] + \item + \begin{enumerate} + \item ~ + \begin{center} + \begin{tabularx}{0.4\linewidth}{|X|}\hline + $N \gets 2017$\\ + $U \gets 18$ \\ + $S \gets 300 + U$ \\ + Tant que $S < 450$ \\ + \hspace{1cm} $N \gets N + 1$\\ + \hspace{1cm} $U \gets U * 1.1400000000000001$\\ + \hspace{1cm} $S \gets S + u$\\ + Fin Tant que\\\hline + \end{tabularx} + \end{center} + \item \textit{Pas de correction automatisé} + \end{enumerate} + \end{enumerate} +\end{solution} + +\end{document} + +%%% Local Variables: +%%% mode: latex +%%% TeX-master: "master" +%%% End: diff --git a/TST/DS/DS_21_04_07/TST3/05_210407_DS8.tex b/TST/DS/DS_21_04_07/TST3/05_210407_DS8.tex new file mode 100644 index 0000000..dba00d3 --- /dev/null +++ b/TST/DS/DS_21_04_07/TST3/05_210407_DS8.tex @@ -0,0 +1,262 @@ +\documentclass[a4paper,10pt]{article} +\usepackage{myXsim} + +% Title Page +\title{DS8 \hfill BUDIN Nathan} +\tribe{TST} +\date{\hfillÀ render pour le Mercredi 7 avril} + +\xsimsetup{ + solution/print = false +} + +\begin{document} +\maketitle + +\begin{exercise}[subtitle={Automatismes}] + \textit{Toutes les questions de cette exercice sont indépendantes et peuvent être répondus séparément} + \begin{enumerate} + \item De janvier à septembre, une quantité a augmenté de $16\,\%$. Faire un schéma pour représenter la situation puis calculer le taux d'évolution moyen mensuel. + \item Une quantité augmente de $16\,\%$ par ans. En 2020, elle est de 112\euro. Quelle était sa valeur en 2019? Faire un schéma pour représenter la situation. + \item Déterminer l'équation de la droite \\ + \begin{tikzpicture}[xscale=0.8, yscale=0.5] + \tkzInit[xmin=-5,xmax=5,xstep=1, + ymin=-5,ymax=5,ystep=1] + \tkzGrid + \tkzAxeXY + \tkzFct[domain=-5:5,color=red,very thick]% + {1.0*\x -2}; + \end{tikzpicture} + \item Résoudre l'équation $7 \times 0.22^x = 49$ + \end{enumerate} +\end{exercise} + +\begin{solution} + \begin{enumerate} + \item On veut partager cette évolution en 8 évolutions. + \[ + \left(1 + \frac{16}{100}\right)^{\frac{1}{8}} = 1.0187 + \] + Donc le taux d'évolution moyen est + \[ + t_m = 1.0187 - 1 = 0.01869999999999994 + \] + \item Coefficient multiplicateur pour revenir en arrière + \[ + CM = (1 + \frac{16}{100})^{-1} = 0.8621 + \] + On en déduit la quantité en 2019 + \[ + 112 * 0.8621 = 96.5552 + \] + \item L'équation de la droite est + \[ + y = 1.0 x -2 + \] + \item Il faut penser à faire la division à par $7$ avant d'utiliser le log car sinon, on ne peut pas utiliser la formule $\log(a^n) = n\times \log(a)$. + + \[x = \frac{\log(7.0)}{\log(0.22)}\] + \end{enumerate} +\end{solution} + +\begin{exercise}[subtitle={Restaurant}] + Un \emph{food truck}, ouvert le midi et le soir, propose deux types de formules : + + \setlength\parindent{10mm} + \begin{itemize} + \item la formule \emph{Burger} ; + \item la formule \emph{Wok}. + \end{itemize} + \setlength\parindent{0mm} + + \medskip + + Le gérant a remarqué que 75\,\% de ses ventes ont lieu le midi. Le quart des ventes du midi correspondent à la formule \emph{Burger}, alors que 71\,\% des ventes du soir correspondent à la formule \emph{Wok}. + + Le gérant se constitue un fichier en notant, pour chaque vente, la formule choisie et le moment de cette vente (midi ou soir). + + On prélève une fiche de façon équiprobable. On définit les quatre évènements suivants: + + \begin{enumerate} + \item $M$ : \og la fiche correspond à une vente du midi\fg{} ; + \item $S$ : \og la fiche correspond à une vente du soir\fg {}; + \item $W$ : \og la fiche correspond à une formule \emph{Wok} \fg{} ; + \item $B$ : \og la fiche correspond à une formule \emph{Burger} \fg. + \end{enumerate} + \setlength\parindent{0mm} + + \medskip + + \begin{enumerate} + \item Recopier puis compléter l'arbre pondéré + + \begin{center} + \begin{tikzpicture}[sloped] + \node {.} + child {node {$M$} + child {node {$W$} + edge from parent + node[above] {...} + } + child {node {$B$} + edge from parent + node[above] {...} + } + edge from parent + node[above] {...} + } + child[missing] {} + child { node {$S$} + child {node {$W$} + edge from parent + node[above] {...} + } + child {node {$B$} + edge from parent + node[above] {...} + } + edge from parent + node[above] {...} + } ; + \end{tikzpicture} + \end{center} + + \item Calculer la probabilité de l'évènement $M \cap W$. Interpréter ce résultat dans le contexte de l'exercice. + \item Montrer que la probabilité que la fiche choisie corresponde à une formule \emph{Burger} est égale à $0.26$. + \item On a prélevé une fiche correspondant à la formule \emph{Burger}. Quelle est la probabilité, arrondie au millième, que la vente ait eu lieu le soir? + \end{enumerate} +\end{exercise} + +\begin{solution} + \begin{enumerate} + \item + \begin{center} + \begin{tikzpicture}[sloped] + \node {.} + child {node {$M$} + child {node {$W$} + edge from parent + node[above] {$0.75$} + } + child {node {$B$} + edge from parent + node[above] {$0.25$} + } + edge from parent + node[above] {$0.75$} + } + child[missing] {} + child { node {$S$} + child {node {$W$} + edge from parent + node[above] {$0.71$} + } + child {node {$B$} + edge from parent + node[above] {$0.29$} + } + edge from parent + node[above] {$0.25$} + } ; + \end{tikzpicture} + \end{center} + \item On calcule la probabilité que la vente soit un wok et ait eu lieu à midi + \[ P(M\cap W) = P(M) \times P_M(W) = 0.75 \times 0.75 = 0.5625 \] + \item Probabilité que la vente soit un burger. + \[ + P(B) = P(M\cap B) + P(S\cap B) = 0.75 \times 0.75 + 0.25 \times 0.71 = 0.26 + \] + \item On cherche à calculer la quantité $P_B(S)$. Pour cela on utilise la formule de Bayes + \[ + P_B(S) = \frac{P(B\cap S)}{P(B)} = \frac{P_S(B) \times P(S)}{P(B)} = \frac{0.29\times 0.25}{0.26} = 0.2788461538461538 \approx 0.279 + \] + \end{enumerate} +\end{solution} + +\begin{exercise}[subtitle={Continent plastique}] + \textit{Les quantités évoqués dans cette exercice sont générés au hasard et sont donc complètement farfelus.} + \medskip + Le \og continent de plastique\fg{} est la plus grande des plaques de déchets plastiques évoluant sur les océans. Elle occupe actuellement dans l'océan Pacifique une surface dont l'aire est évaluée à plus de $1,6$ million de km$^2$, entre Hawaï et la Californie. + + En 2017, des scientifiques ont estimé qu'il y avait $12$ millions de tonnes de déchets plastiques qui était déversé chaque année dans les océans et que cette quantité augmentait de $25\n\%$ par chaque année. + + On modélise l'évolution de la masse de ces déchets plastiques déversée chaque année, si rien n'est fait pour la réduire, par une suite géométrique $\left(u_n\right)$. L'arrondi au centième du terme $u_n$ représente la masse de ces déchets déversée chaque année, exprimée en million de tonnes, pour l'année $(2017 + n)$. + + \medskip + + \begin{enumerate} + \item Expliquer pourquoi la suite $u_n$ est géométrique? + \item Calculer $u_1$ et $u_2$. + \item Exprimer $u_n$ en fonction de $n$. + \item Au début de l'année 2017, il y avait $300$ millions de tonnes de déchets plastique. Calculer la quantité totale de déchets plastiques en 2030. + \item On souhaite déterminer en quelle année la masse totale de ces déchets plastiques aura pour la première fois augmenté de $50$\,\% par rapport à sa valeur de 2017. + \begin{enumerate} + \item Recopier et compléter l'algorithme ci-dessous pour que la variable $N$ contienne la réponse au problème posé. + + \begin{center} + \begin{tabularx}{0.4\linewidth}{|X|}\hline + $N = 2017$\\ + $U = 12$ \\ + $S = 300 + U$ \\ + while $S < 450$: \\ + \hspace{1cm} $N = \ldots$\\ + \hspace{1cm} $U = \ldots$\\ + \hspace{1cm} $S = \ldots$\\ + \hline + \end{tabularx} + \end{center} + \item Que contiennent les variables $S$, $U$ et $N$ après exécution de cet algorithme ? + + Interpréter les résultats dans le contexte de l'exercice. + \end{enumerate} + \end{enumerate} +\end{exercise} + +\begin{solution} + \begin{enumerate} + \item Une augmentation de $25\,\%$ revient à multiplier la quantité par $1.25$. La suite est donc bien géométrique. Son premier terme est $u_0 = 12$ et sa raison est $q = 1.25$ + \item + \[ + u_1 = u_0 * 1.25 = 15.0 + \] + \[ + u_2 = u_0 * 1.25^2 = 18.75 + \] + \item + \[ + u_n = u_0 \times q^n = 12 \times 1.25^n + \] + \item On calcule la quantité totale déversée entre 2017 et 2030. + \[ + \sum_{n = 0}^{13} u_n = u_0 \times \frac{1-q^{13}}{1-q} = 12 \times \frac{1 - 1.25^{13}}{1 - 1.25} = 825.11 + \] + On en déduit la quantité totale de déchets en 2030 + \[ + 300 + 825.11 = 1125.1100000000001 + \] + \item + \begin{enumerate} + \item ~ + \begin{center} + \begin{tabularx}{0.4\linewidth}{|X|}\hline + $N \gets 2017$\\ + $U \gets 12$ \\ + $S \gets 300 + U$ \\ + Tant que $S < 450$ \\ + \hspace{1cm} $N \gets N + 1$\\ + \hspace{1cm} $U \gets U * 1.25$\\ + \hspace{1cm} $S \gets S + u$\\ + Fin Tant que\\\hline + \end{tabularx} + \end{center} + \item \textit{Pas de correction automatisé} + \end{enumerate} + \end{enumerate} +\end{solution} + +\end{document} + +%%% Local Variables: +%%% mode: latex +%%% TeX-master: "master" +%%% End: diff --git a/TST/DS/DS_21_04_07/TST3/06_210407_DS8.tex b/TST/DS/DS_21_04_07/TST3/06_210407_DS8.tex new file mode 100644 index 0000000..4503b69 --- /dev/null +++ b/TST/DS/DS_21_04_07/TST3/06_210407_DS8.tex @@ -0,0 +1,262 @@ +\documentclass[a4paper,10pt]{article} +\usepackage{myXsim} + +% Title Page +\title{DS8 \hfill DARICHE Kaïs} +\tribe{TST} +\date{\hfillÀ render pour le Mercredi 7 avril} + +\xsimsetup{ + solution/print = false +} + +\begin{document} +\maketitle + +\begin{exercise}[subtitle={Automatismes}] + \textit{Toutes les questions de cette exercice sont indépendantes et peuvent être répondus séparément} + \begin{enumerate} + \item De janvier à septembre, une quantité a augmenté de $27\,\%$. Faire un schéma pour représenter la situation puis calculer le taux d'évolution moyen mensuel. + \item Une quantité augmente de $27\,\%$ par ans. En 2020, elle est de 143\euro. Quelle était sa valeur en 2019? Faire un schéma pour représenter la situation. + \item Déterminer l'équation de la droite \\ + \begin{tikzpicture}[xscale=0.8, yscale=0.5] + \tkzInit[xmin=-5,xmax=5,xstep=1, + ymin=-5,ymax=5,ystep=1] + \tkzGrid + \tkzAxeXY + \tkzFct[domain=-5:5,color=red,very thick]% + {3.0*\x -3}; + \end{tikzpicture} + \item Résoudre l'équation $7 \times 0.19^x = 21$ + \end{enumerate} +\end{exercise} + +\begin{solution} + \begin{enumerate} + \item On veut partager cette évolution en 8 évolutions. + \[ + \left(1 + \frac{27}{100}\right)^{\frac{1}{8}} = 1.0303 + \] + Donc le taux d'évolution moyen est + \[ + t_m = 1.0303 - 1 = 0.030299999999999994 + \] + \item Coefficient multiplicateur pour revenir en arrière + \[ + CM = (1 + \frac{27}{100})^{-1} = 0.7874 + \] + On en déduit la quantité en 2019 + \[ + 143 * 0.7874 = 112.59819999999999 + \] + \item L'équation de la droite est + \[ + y = 3.0 x -3 + \] + \item Il faut penser à faire la division à par $7$ avant d'utiliser le log car sinon, on ne peut pas utiliser la formule $\log(a^n) = n\times \log(a)$. + + \[x = \frac{\log(3.0)}{\log(0.19)}\] + \end{enumerate} +\end{solution} + +\begin{exercise}[subtitle={Restaurant}] + Un \emph{food truck}, ouvert le midi et le soir, propose deux types de formules : + + \setlength\parindent{10mm} + \begin{itemize} + \item la formule \emph{Burger} ; + \item la formule \emph{Wok}. + \end{itemize} + \setlength\parindent{0mm} + + \medskip + + Le gérant a remarqué que 15\,\% de ses ventes ont lieu le midi. Le quart des ventes du midi correspondent à la formule \emph{Burger}, alors que 13\,\% des ventes du soir correspondent à la formule \emph{Wok}. + + Le gérant se constitue un fichier en notant, pour chaque vente, la formule choisie et le moment de cette vente (midi ou soir). + + On prélève une fiche de façon équiprobable. On définit les quatre évènements suivants: + + \begin{enumerate} + \item $M$ : \og la fiche correspond à une vente du midi\fg{} ; + \item $S$ : \og la fiche correspond à une vente du soir\fg {}; + \item $W$ : \og la fiche correspond à une formule \emph{Wok} \fg{} ; + \item $B$ : \og la fiche correspond à une formule \emph{Burger} \fg. + \end{enumerate} + \setlength\parindent{0mm} + + \medskip + + \begin{enumerate} + \item Recopier puis compléter l'arbre pondéré + + \begin{center} + \begin{tikzpicture}[sloped] + \node {.} + child {node {$M$} + child {node {$W$} + edge from parent + node[above] {...} + } + child {node {$B$} + edge from parent + node[above] {...} + } + edge from parent + node[above] {...} + } + child[missing] {} + child { node {$S$} + child {node {$W$} + edge from parent + node[above] {...} + } + child {node {$B$} + edge from parent + node[above] {...} + } + edge from parent + node[above] {...} + } ; + \end{tikzpicture} + \end{center} + + \item Calculer la probabilité de l'évènement $M \cap W$. Interpréter ce résultat dans le contexte de l'exercice. + \item Montrer que la probabilité que la fiche choisie corresponde à une formule \emph{Burger} est égale à $0.777$. + \item On a prélevé une fiche correspondant à la formule \emph{Burger}. Quelle est la probabilité, arrondie au millième, que la vente ait eu lieu le soir? + \end{enumerate} +\end{exercise} + +\begin{solution} + \begin{enumerate} + \item + \begin{center} + \begin{tikzpicture}[sloped] + \node {.} + child {node {$M$} + child {node {$W$} + edge from parent + node[above] {$0.75$} + } + child {node {$B$} + edge from parent + node[above] {$0.25$} + } + edge from parent + node[above] {$0.15$} + } + child[missing] {} + child { node {$S$} + child {node {$W$} + edge from parent + node[above] {$0.13$} + } + child {node {$B$} + edge from parent + node[above] {$0.87$} + } + edge from parent + node[above] {$0.85$} + } ; + \end{tikzpicture} + \end{center} + \item On calcule la probabilité que la vente soit un wok et ait eu lieu à midi + \[ P(M\cap W) = P(M) \times P_M(W) = 0.15 \times 0.75 = 0.1125 \] + \item Probabilité que la vente soit un burger. + \[ + P(B) = P(M\cap B) + P(S\cap B) = 0.15 \times 0.75 + 0.85 \times 0.13 = 0.777 + \] + \item On cherche à calculer la quantité $P_B(S)$. Pour cela on utilise la formule de Bayes + \[ + P_B(S) = \frac{P(B\cap S)}{P(B)} = \frac{P_S(B) \times P(S)}{P(B)} = \frac{0.87\times 0.85}{0.777} = 0.9517374517374516 \approx 0.952 + \] + \end{enumerate} +\end{solution} + +\begin{exercise}[subtitle={Continent plastique}] + \textit{Les quantités évoqués dans cette exercice sont générés au hasard et sont donc complètement farfelus.} + \medskip + Le \og continent de plastique\fg{} est la plus grande des plaques de déchets plastiques évoluant sur les océans. Elle occupe actuellement dans l'océan Pacifique une surface dont l'aire est évaluée à plus de $1,6$ million de km$^2$, entre Hawaï et la Californie. + + En 2017, des scientifiques ont estimé qu'il y avait $2$ millions de tonnes de déchets plastiques qui était déversé chaque année dans les océans et que cette quantité augmentait de $30\n\%$ par chaque année. + + On modélise l'évolution de la masse de ces déchets plastiques déversée chaque année, si rien n'est fait pour la réduire, par une suite géométrique $\left(u_n\right)$. L'arrondi au centième du terme $u_n$ représente la masse de ces déchets déversée chaque année, exprimée en million de tonnes, pour l'année $(2017 + n)$. + + \medskip + + \begin{enumerate} + \item Expliquer pourquoi la suite $u_n$ est géométrique? + \item Calculer $u_1$ et $u_2$. + \item Exprimer $u_n$ en fonction de $n$. + \item Au début de l'année 2017, il y avait $300$ millions de tonnes de déchets plastique. Calculer la quantité totale de déchets plastiques en 2030. + \item On souhaite déterminer en quelle année la masse totale de ces déchets plastiques aura pour la première fois augmenté de $50$\,\% par rapport à sa valeur de 2017. + \begin{enumerate} + \item Recopier et compléter l'algorithme ci-dessous pour que la variable $N$ contienne la réponse au problème posé. + + \begin{center} + \begin{tabularx}{0.4\linewidth}{|X|}\hline + $N = 2017$\\ + $U = 2$ \\ + $S = 300 + U$ \\ + while $S < 450$: \\ + \hspace{1cm} $N = \ldots$\\ + \hspace{1cm} $U = \ldots$\\ + \hspace{1cm} $S = \ldots$\\ + \hline + \end{tabularx} + \end{center} + \item Que contiennent les variables $S$, $U$ et $N$ après exécution de cet algorithme ? + + Interpréter les résultats dans le contexte de l'exercice. + \end{enumerate} + \end{enumerate} +\end{exercise} + +\begin{solution} + \begin{enumerate} + \item Une augmentation de $30\,\%$ revient à multiplier la quantité par $1.3$. La suite est donc bien géométrique. Son premier terme est $u_0 = 2$ et sa raison est $q = 1.3$ + \item + \[ + u_1 = u_0 * 1.3 = 2.6 + \] + \[ + u_2 = u_0 * 1.3^2 = 3.38 + \] + \item + \[ + u_n = u_0 \times q^n = 2 \times 1.3^n + \] + \item On calcule la quantité totale déversée entre 2017 et 2030. + \[ + \sum_{n = 0}^{13} u_n = u_0 \times \frac{1-q^{13}}{1-q} = 2 \times \frac{1 - 1.3^{13}}{1 - 1.3} = 195.25 + \] + On en déduit la quantité totale de déchets en 2030 + \[ + 300 + 195.25 = 495.25 + \] + \item + \begin{enumerate} + \item ~ + \begin{center} + \begin{tabularx}{0.4\linewidth}{|X|}\hline + $N \gets 2017$\\ + $U \gets 2$ \\ + $S \gets 300 + U$ \\ + Tant que $S < 450$ \\ + \hspace{1cm} $N \gets N + 1$\\ + \hspace{1cm} $U \gets U * 1.3$\\ + \hspace{1cm} $S \gets S + u$\\ + Fin Tant que\\\hline + \end{tabularx} + \end{center} + \item \textit{Pas de correction automatisé} + \end{enumerate} + \end{enumerate} +\end{solution} + +\end{document} + +%%% Local Variables: +%%% mode: latex +%%% TeX-master: "master" +%%% End: diff --git a/TST/DS/DS_21_04_07/TST3/07_210407_DS8.tex b/TST/DS/DS_21_04_07/TST3/07_210407_DS8.tex new file mode 100644 index 0000000..2b6b175 --- /dev/null +++ b/TST/DS/DS_21_04_07/TST3/07_210407_DS8.tex @@ -0,0 +1,262 @@ +\documentclass[a4paper,10pt]{article} +\usepackage{myXsim} + +% Title Page +\title{DS8 \hfill DEBRAS Noémie} +\tribe{TST} +\date{\hfillÀ render pour le Mercredi 7 avril} + +\xsimsetup{ + solution/print = false +} + +\begin{document} +\maketitle + +\begin{exercise}[subtitle={Automatismes}] + \textit{Toutes les questions de cette exercice sont indépendantes et peuvent être répondus séparément} + \begin{enumerate} + \item De janvier à septembre, une quantité a augmenté de $24\,\%$. Faire un schéma pour représenter la situation puis calculer le taux d'évolution moyen mensuel. + \item Une quantité augmente de $24\,\%$ par ans. En 2020, elle est de 110\euro. Quelle était sa valeur en 2019? Faire un schéma pour représenter la situation. + \item Déterminer l'équation de la droite \\ + \begin{tikzpicture}[xscale=0.8, yscale=0.5] + \tkzInit[xmin=-5,xmax=5,xstep=1, + ymin=-5,ymax=5,ystep=1] + \tkzGrid + \tkzAxeXY + \tkzFct[domain=-5:5,color=red,very thick]% + {0.6666666666666666*\x -1}; + \end{tikzpicture} + \item Résoudre l'équation $8 \times 0.49^x = 9$ + \end{enumerate} +\end{exercise} + +\begin{solution} + \begin{enumerate} + \item On veut partager cette évolution en 8 évolutions. + \[ + \left(1 + \frac{24}{100}\right)^{\frac{1}{8}} = 1.0273 + \] + Donc le taux d'évolution moyen est + \[ + t_m = 1.0273 - 1 = 0.027300000000000102 + \] + \item Coefficient multiplicateur pour revenir en arrière + \[ + CM = (1 + \frac{24}{100})^{-1} = 0.8065 + \] + On en déduit la quantité en 2019 + \[ + 110 * 0.8065 = 88.715 + \] + \item L'équation de la droite est + \[ + y = 0.6666666666666666 x -1 + \] + \item Il faut penser à faire la division à par $8$ avant d'utiliser le log car sinon, on ne peut pas utiliser la formule $\log(a^n) = n\times \log(a)$. + + \[x = \frac{\log(1.12)}{\log(0.49)}\] + \end{enumerate} +\end{solution} + +\begin{exercise}[subtitle={Restaurant}] + Un \emph{food truck}, ouvert le midi et le soir, propose deux types de formules : + + \setlength\parindent{10mm} + \begin{itemize} + \item la formule \emph{Burger} ; + \item la formule \emph{Wok}. + \end{itemize} + \setlength\parindent{0mm} + + \medskip + + Le gérant a remarqué que 49\,\% de ses ventes ont lieu le midi. Le quart des ventes du midi correspondent à la formule \emph{Burger}, alors que 61\,\% des ventes du soir correspondent à la formule \emph{Wok}. + + Le gérant se constitue un fichier en notant, pour chaque vente, la formule choisie et le moment de cette vente (midi ou soir). + + On prélève une fiche de façon équiprobable. On définit les quatre évènements suivants: + + \begin{enumerate} + \item $M$ : \og la fiche correspond à une vente du midi\fg{} ; + \item $S$ : \og la fiche correspond à une vente du soir\fg {}; + \item $W$ : \og la fiche correspond à une formule \emph{Wok} \fg{} ; + \item $B$ : \og la fiche correspond à une formule \emph{Burger} \fg. + \end{enumerate} + \setlength\parindent{0mm} + + \medskip + + \begin{enumerate} + \item Recopier puis compléter l'arbre pondéré + + \begin{center} + \begin{tikzpicture}[sloped] + \node {.} + child {node {$M$} + child {node {$W$} + edge from parent + node[above] {...} + } + child {node {$B$} + edge from parent + node[above] {...} + } + edge from parent + node[above] {...} + } + child[missing] {} + child { node {$S$} + child {node {$W$} + edge from parent + node[above] {...} + } + child {node {$B$} + edge from parent + node[above] {...} + } + edge from parent + node[above] {...} + } ; + \end{tikzpicture} + \end{center} + + \item Calculer la probabilité de l'évènement $M \cap W$. Interpréter ce résultat dans le contexte de l'exercice. + \item Montrer que la probabilité que la fiche choisie corresponde à une formule \emph{Burger} est égale à $0.3214$. + \item On a prélevé une fiche correspondant à la formule \emph{Burger}. Quelle est la probabilité, arrondie au millième, que la vente ait eu lieu le soir? + \end{enumerate} +\end{exercise} + +\begin{solution} + \begin{enumerate} + \item + \begin{center} + \begin{tikzpicture}[sloped] + \node {.} + child {node {$M$} + child {node {$W$} + edge from parent + node[above] {$0.75$} + } + child {node {$B$} + edge from parent + node[above] {$0.25$} + } + edge from parent + node[above] {$0.49$} + } + child[missing] {} + child { node {$S$} + child {node {$W$} + edge from parent + node[above] {$0.61$} + } + child {node {$B$} + edge from parent + node[above] {$0.39$} + } + edge from parent + node[above] {$0.51$} + } ; + \end{tikzpicture} + \end{center} + \item On calcule la probabilité que la vente soit un wok et ait eu lieu à midi + \[ P(M\cap W) = P(M) \times P_M(W) = 0.49 \times 0.75 = 0.3675 \] + \item Probabilité que la vente soit un burger. + \[ + P(B) = P(M\cap B) + P(S\cap B) = 0.49 \times 0.75 + 0.51 \times 0.61 = 0.3214 + \] + \item On cherche à calculer la quantité $P_B(S)$. Pour cela on utilise la formule de Bayes + \[ + P_B(S) = \frac{P(B\cap S)}{P(B)} = \frac{P_S(B) \times P(S)}{P(B)} = \frac{0.39\times 0.51}{0.3214} = 0.618855009334163 \approx 0.619 + \] + \end{enumerate} +\end{solution} + +\begin{exercise}[subtitle={Continent plastique}] + \textit{Les quantités évoqués dans cette exercice sont générés au hasard et sont donc complètement farfelus.} + \medskip + Le \og continent de plastique\fg{} est la plus grande des plaques de déchets plastiques évoluant sur les océans. Elle occupe actuellement dans l'océan Pacifique une surface dont l'aire est évaluée à plus de $1,6$ million de km$^2$, entre Hawaï et la Californie. + + En 2017, des scientifiques ont estimé qu'il y avait $15$ millions de tonnes de déchets plastiques qui était déversé chaque année dans les océans et que cette quantité augmentait de $21\n\%$ par chaque année. + + On modélise l'évolution de la masse de ces déchets plastiques déversée chaque année, si rien n'est fait pour la réduire, par une suite géométrique $\left(u_n\right)$. L'arrondi au centième du terme $u_n$ représente la masse de ces déchets déversée chaque année, exprimée en million de tonnes, pour l'année $(2017 + n)$. + + \medskip + + \begin{enumerate} + \item Expliquer pourquoi la suite $u_n$ est géométrique? + \item Calculer $u_1$ et $u_2$. + \item Exprimer $u_n$ en fonction de $n$. + \item Au début de l'année 2017, il y avait $300$ millions de tonnes de déchets plastique. Calculer la quantité totale de déchets plastiques en 2030. + \item On souhaite déterminer en quelle année la masse totale de ces déchets plastiques aura pour la première fois augmenté de $50$\,\% par rapport à sa valeur de 2017. + \begin{enumerate} + \item Recopier et compléter l'algorithme ci-dessous pour que la variable $N$ contienne la réponse au problème posé. + + \begin{center} + \begin{tabularx}{0.4\linewidth}{|X|}\hline + $N = 2017$\\ + $U = 15$ \\ + $S = 300 + U$ \\ + while $S < 450$: \\ + \hspace{1cm} $N = \ldots$\\ + \hspace{1cm} $U = \ldots$\\ + \hspace{1cm} $S = \ldots$\\ + \hline + \end{tabularx} + \end{center} + \item Que contiennent les variables $S$, $U$ et $N$ après exécution de cet algorithme ? + + Interpréter les résultats dans le contexte de l'exercice. + \end{enumerate} + \end{enumerate} +\end{exercise} + +\begin{solution} + \begin{enumerate} + \item Une augmentation de $21\,\%$ revient à multiplier la quantité par $1.21$. La suite est donc bien géométrique. Son premier terme est $u_0 = 15$ et sa raison est $q = 1.21$ + \item + \[ + u_1 = u_0 * 1.21 = 18.15 + \] + \[ + u_2 = u_0 * 1.21^2 = 21.9615 + \] + \item + \[ + u_n = u_0 \times q^n = 15 \times 1.21^n + \] + \item On calcule la quantité totale déversée entre 2017 et 2030. + \[ + \sum_{n = 0}^{13} u_n = u_0 \times \frac{1-q^{13}}{1-q} = 15 \times \frac{1 - 1.21^{13}}{1 - 1.21} = 779.87 + \] + On en déduit la quantité totale de déchets en 2030 + \[ + 300 + 779.87 = 1079.87 + \] + \item + \begin{enumerate} + \item ~ + \begin{center} + \begin{tabularx}{0.4\linewidth}{|X|}\hline + $N \gets 2017$\\ + $U \gets 15$ \\ + $S \gets 300 + U$ \\ + Tant que $S < 450$ \\ + \hspace{1cm} $N \gets N + 1$\\ + \hspace{1cm} $U \gets U * 1.21$\\ + \hspace{1cm} $S \gets S + u$\\ + Fin Tant que\\\hline + \end{tabularx} + \end{center} + \item \textit{Pas de correction automatisé} + \end{enumerate} + \end{enumerate} +\end{solution} + +\end{document} + +%%% Local Variables: +%%% mode: latex +%%% TeX-master: "master" +%%% End: diff --git a/TST/DS/DS_21_04_07/TST3/08_210407_DS8.tex b/TST/DS/DS_21_04_07/TST3/08_210407_DS8.tex new file mode 100644 index 0000000..e2d71c4 --- /dev/null +++ b/TST/DS/DS_21_04_07/TST3/08_210407_DS8.tex @@ -0,0 +1,262 @@ +\documentclass[a4paper,10pt]{article} +\usepackage{myXsim} + +% Title Page +\title{DS8 \hfill GERMAIN Anaïs} +\tribe{TST} +\date{\hfillÀ render pour le Mercredi 7 avril} + +\xsimsetup{ + solution/print = false +} + +\begin{document} +\maketitle + +\begin{exercise}[subtitle={Automatismes}] + \textit{Toutes les questions de cette exercice sont indépendantes et peuvent être répondus séparément} + \begin{enumerate} + \item De janvier à septembre, une quantité a augmenté de $27\,\%$. Faire un schéma pour représenter la situation puis calculer le taux d'évolution moyen mensuel. + \item Une quantité augmente de $27\,\%$ par ans. En 2020, elle est de 132\euro. Quelle était sa valeur en 2019? Faire un schéma pour représenter la situation. + \item Déterminer l'équation de la droite \\ + \begin{tikzpicture}[xscale=0.8, yscale=0.5] + \tkzInit[xmin=-5,xmax=5,xstep=1, + ymin=-5,ymax=5,ystep=1] + \tkzGrid + \tkzAxeXY + \tkzFct[domain=-5:5,color=red,very thick]% + {2.0*\x -4}; + \end{tikzpicture} + \item Résoudre l'équation $7 \times 0.05^x = 18$ + \end{enumerate} +\end{exercise} + +\begin{solution} + \begin{enumerate} + \item On veut partager cette évolution en 8 évolutions. + \[ + \left(1 + \frac{27}{100}\right)^{\frac{1}{8}} = 1.0303 + \] + Donc le taux d'évolution moyen est + \[ + t_m = 1.0303 - 1 = 0.030299999999999994 + \] + \item Coefficient multiplicateur pour revenir en arrière + \[ + CM = (1 + \frac{27}{100})^{-1} = 0.7874 + \] + On en déduit la quantité en 2019 + \[ + 132 * 0.7874 = 103.9368 + \] + \item L'équation de la droite est + \[ + y = 2.0 x -4 + \] + \item Il faut penser à faire la division à par $7$ avant d'utiliser le log car sinon, on ne peut pas utiliser la formule $\log(a^n) = n\times \log(a)$. + + \[x = \frac{\log(2.57)}{\log(0.05)}\] + \end{enumerate} +\end{solution} + +\begin{exercise}[subtitle={Restaurant}] + Un \emph{food truck}, ouvert le midi et le soir, propose deux types de formules : + + \setlength\parindent{10mm} + \begin{itemize} + \item la formule \emph{Burger} ; + \item la formule \emph{Wok}. + \end{itemize} + \setlength\parindent{0mm} + + \medskip + + Le gérant a remarqué que 38\,\% de ses ventes ont lieu le midi. Le quart des ventes du midi correspondent à la formule \emph{Burger}, alors que 63\,\% des ventes du soir correspondent à la formule \emph{Wok}. + + Le gérant se constitue un fichier en notant, pour chaque vente, la formule choisie et le moment de cette vente (midi ou soir). + + On prélève une fiche de façon équiprobable. On définit les quatre évènements suivants: + + \begin{enumerate} + \item $M$ : \og la fiche correspond à une vente du midi\fg{} ; + \item $S$ : \og la fiche correspond à une vente du soir\fg {}; + \item $W$ : \og la fiche correspond à une formule \emph{Wok} \fg{} ; + \item $B$ : \og la fiche correspond à une formule \emph{Burger} \fg. + \end{enumerate} + \setlength\parindent{0mm} + + \medskip + + \begin{enumerate} + \item Recopier puis compléter l'arbre pondéré + + \begin{center} + \begin{tikzpicture}[sloped] + \node {.} + child {node {$M$} + child {node {$W$} + edge from parent + node[above] {...} + } + child {node {$B$} + edge from parent + node[above] {...} + } + edge from parent + node[above] {...} + } + child[missing] {} + child { node {$S$} + child {node {$W$} + edge from parent + node[above] {...} + } + child {node {$B$} + edge from parent + node[above] {...} + } + edge from parent + node[above] {...} + } ; + \end{tikzpicture} + \end{center} + + \item Calculer la probabilité de l'évènement $M \cap W$. Interpréter ce résultat dans le contexte de l'exercice. + \item Montrer que la probabilité que la fiche choisie corresponde à une formule \emph{Burger} est égale à $0.3244$. + \item On a prélevé une fiche correspondant à la formule \emph{Burger}. Quelle est la probabilité, arrondie au millième, que la vente ait eu lieu le soir? + \end{enumerate} +\end{exercise} + +\begin{solution} + \begin{enumerate} + \item + \begin{center} + \begin{tikzpicture}[sloped] + \node {.} + child {node {$M$} + child {node {$W$} + edge from parent + node[above] {$0.75$} + } + child {node {$B$} + edge from parent + node[above] {$0.25$} + } + edge from parent + node[above] {$0.38$} + } + child[missing] {} + child { node {$S$} + child {node {$W$} + edge from parent + node[above] {$0.63$} + } + child {node {$B$} + edge from parent + node[above] {$0.37$} + } + edge from parent + node[above] {$0.62$} + } ; + \end{tikzpicture} + \end{center} + \item On calcule la probabilité que la vente soit un wok et ait eu lieu à midi + \[ P(M\cap W) = P(M) \times P_M(W) = 0.38 \times 0.75 = 0.285 \] + \item Probabilité que la vente soit un burger. + \[ + P(B) = P(M\cap B) + P(S\cap B) = 0.38 \times 0.75 + 0.62 \times 0.63 = 0.3244 + \] + \item On cherche à calculer la quantité $P_B(S)$. Pour cela on utilise la formule de Bayes + \[ + P_B(S) = \frac{P(B\cap S)}{P(B)} = \frac{P_S(B) \times P(S)}{P(B)} = \frac{0.37\times 0.62}{0.3244} = 0.7071516646115905 \approx 0.707 + \] + \end{enumerate} +\end{solution} + +\begin{exercise}[subtitle={Continent plastique}] + \textit{Les quantités évoqués dans cette exercice sont générés au hasard et sont donc complètement farfelus.} + \medskip + Le \og continent de plastique\fg{} est la plus grande des plaques de déchets plastiques évoluant sur les océans. Elle occupe actuellement dans l'océan Pacifique une surface dont l'aire est évaluée à plus de $1,6$ million de km$^2$, entre Hawaï et la Californie. + + En 2017, des scientifiques ont estimé qu'il y avait $18$ millions de tonnes de déchets plastiques qui était déversé chaque année dans les océans et que cette quantité augmentait de $28\n\%$ par chaque année. + + On modélise l'évolution de la masse de ces déchets plastiques déversée chaque année, si rien n'est fait pour la réduire, par une suite géométrique $\left(u_n\right)$. L'arrondi au centième du terme $u_n$ représente la masse de ces déchets déversée chaque année, exprimée en million de tonnes, pour l'année $(2017 + n)$. + + \medskip + + \begin{enumerate} + \item Expliquer pourquoi la suite $u_n$ est géométrique? + \item Calculer $u_1$ et $u_2$. + \item Exprimer $u_n$ en fonction de $n$. + \item Au début de l'année 2017, il y avait $300$ millions de tonnes de déchets plastique. Calculer la quantité totale de déchets plastiques en 2030. + \item On souhaite déterminer en quelle année la masse totale de ces déchets plastiques aura pour la première fois augmenté de $50$\,\% par rapport à sa valeur de 2017. + \begin{enumerate} + \item Recopier et compléter l'algorithme ci-dessous pour que la variable $N$ contienne la réponse au problème posé. + + \begin{center} + \begin{tabularx}{0.4\linewidth}{|X|}\hline + $N = 2017$\\ + $U = 18$ \\ + $S = 300 + U$ \\ + while $S < 450$: \\ + \hspace{1cm} $N = \ldots$\\ + \hspace{1cm} $U = \ldots$\\ + \hspace{1cm} $S = \ldots$\\ + \hline + \end{tabularx} + \end{center} + \item Que contiennent les variables $S$, $U$ et $N$ après exécution de cet algorithme ? + + Interpréter les résultats dans le contexte de l'exercice. + \end{enumerate} + \end{enumerate} +\end{exercise} + +\begin{solution} + \begin{enumerate} + \item Une augmentation de $28\,\%$ revient à multiplier la quantité par $1.28$. La suite est donc bien géométrique. Son premier terme est $u_0 = 18$ et sa raison est $q = 1.28$ + \item + \[ + u_1 = u_0 * 1.28 = 23.04 + \] + \[ + u_2 = u_0 * 1.28^2 = 29.4912 + \] + \item + \[ + u_n = u_0 \times q^n = 18 \times 1.28^n + \] + \item On calcule la quantité totale déversée entre 2017 et 2030. + \[ + \sum_{n = 0}^{13} u_n = u_0 \times \frac{1-q^{13}}{1-q} = 18 \times \frac{1 - 1.28^{13}}{1 - 1.28} = 1527.35 + \] + On en déduit la quantité totale de déchets en 2030 + \[ + 300 + 1527.35 = 1827.35 + \] + \item + \begin{enumerate} + \item ~ + \begin{center} + \begin{tabularx}{0.4\linewidth}{|X|}\hline + $N \gets 2017$\\ + $U \gets 18$ \\ + $S \gets 300 + U$ \\ + Tant que $S < 450$ \\ + \hspace{1cm} $N \gets N + 1$\\ + \hspace{1cm} $U \gets U * 1.28$\\ + \hspace{1cm} $S \gets S + u$\\ + Fin Tant que\\\hline + \end{tabularx} + \end{center} + \item \textit{Pas de correction automatisé} + \end{enumerate} + \end{enumerate} +\end{solution} + +\end{document} + +%%% Local Variables: +%%% mode: latex +%%% TeX-master: "master" +%%% End: diff --git a/TST/DS/DS_21_04_07/TST3/09_210407_DS8.tex b/TST/DS/DS_21_04_07/TST3/09_210407_DS8.tex new file mode 100644 index 0000000..9dc5718 --- /dev/null +++ b/TST/DS/DS_21_04_07/TST3/09_210407_DS8.tex @@ -0,0 +1,262 @@ +\documentclass[a4paper,10pt]{article} +\usepackage{myXsim} + +% Title Page +\title{DS8 \hfill HADJRAS Mohcine} +\tribe{TST} +\date{\hfillÀ render pour le Mercredi 7 avril} + +\xsimsetup{ + solution/print = false +} + +\begin{document} +\maketitle + +\begin{exercise}[subtitle={Automatismes}] + \textit{Toutes les questions de cette exercice sont indépendantes et peuvent être répondus séparément} + \begin{enumerate} + \item De janvier à septembre, une quantité a augmenté de $19\,\%$. Faire un schéma pour représenter la situation puis calculer le taux d'évolution moyen mensuel. + \item Une quantité augmente de $19\,\%$ par ans. En 2020, elle est de 113\euro. Quelle était sa valeur en 2019? Faire un schéma pour représenter la situation. + \item Déterminer l'équation de la droite \\ + \begin{tikzpicture}[xscale=0.8, yscale=0.5] + \tkzInit[xmin=-5,xmax=5,xstep=1, + ymin=-5,ymax=5,ystep=1] + \tkzGrid + \tkzAxeXY + \tkzFct[domain=-5:5,color=red,very thick]% + {0.5*\x -1}; + \end{tikzpicture} + \item Résoudre l'équation $4 \times 0.54^x = 6$ + \end{enumerate} +\end{exercise} + +\begin{solution} + \begin{enumerate} + \item On veut partager cette évolution en 8 évolutions. + \[ + \left(1 + \frac{19}{100}\right)^{\frac{1}{8}} = 1.022 + \] + Donc le taux d'évolution moyen est + \[ + t_m = 1.022 - 1 = 0.02200000000000002 + \] + \item Coefficient multiplicateur pour revenir en arrière + \[ + CM = (1 + \frac{19}{100})^{-1} = 0.8403 + \] + On en déduit la quantité en 2019 + \[ + 113 * 0.8403 = 94.9539 + \] + \item L'équation de la droite est + \[ + y = 0.5 x -1 + \] + \item Il faut penser à faire la division à par $4$ avant d'utiliser le log car sinon, on ne peut pas utiliser la formule $\log(a^n) = n\times \log(a)$. + + \[x = \frac{\log(1.5)}{\log(0.54)}\] + \end{enumerate} +\end{solution} + +\begin{exercise}[subtitle={Restaurant}] + Un \emph{food truck}, ouvert le midi et le soir, propose deux types de formules : + + \setlength\parindent{10mm} + \begin{itemize} + \item la formule \emph{Burger} ; + \item la formule \emph{Wok}. + \end{itemize} + \setlength\parindent{0mm} + + \medskip + + Le gérant a remarqué que 68\,\% de ses ventes ont lieu le midi. Le quart des ventes du midi correspondent à la formule \emph{Burger}, alors que 43\,\% des ventes du soir correspondent à la formule \emph{Wok}. + + Le gérant se constitue un fichier en notant, pour chaque vente, la formule choisie et le moment de cette vente (midi ou soir). + + On prélève une fiche de façon équiprobable. On définit les quatre évènements suivants: + + \begin{enumerate} + \item $M$ : \og la fiche correspond à une vente du midi\fg{} ; + \item $S$ : \og la fiche correspond à une vente du soir\fg {}; + \item $W$ : \og la fiche correspond à une formule \emph{Wok} \fg{} ; + \item $B$ : \og la fiche correspond à une formule \emph{Burger} \fg. + \end{enumerate} + \setlength\parindent{0mm} + + \medskip + + \begin{enumerate} + \item Recopier puis compléter l'arbre pondéré + + \begin{center} + \begin{tikzpicture}[sloped] + \node {.} + child {node {$M$} + child {node {$W$} + edge from parent + node[above] {...} + } + child {node {$B$} + edge from parent + node[above] {...} + } + edge from parent + node[above] {...} + } + child[missing] {} + child { node {$S$} + child {node {$W$} + edge from parent + node[above] {...} + } + child {node {$B$} + edge from parent + node[above] {...} + } + edge from parent + node[above] {...} + } ; + \end{tikzpicture} + \end{center} + + \item Calculer la probabilité de l'évènement $M \cap W$. Interpréter ce résultat dans le contexte de l'exercice. + \item Montrer que la probabilité que la fiche choisie corresponde à une formule \emph{Burger} est égale à $0.3524$. + \item On a prélevé une fiche correspondant à la formule \emph{Burger}. Quelle est la probabilité, arrondie au millième, que la vente ait eu lieu le soir? + \end{enumerate} +\end{exercise} + +\begin{solution} + \begin{enumerate} + \item + \begin{center} + \begin{tikzpicture}[sloped] + \node {.} + child {node {$M$} + child {node {$W$} + edge from parent + node[above] {$0.75$} + } + child {node {$B$} + edge from parent + node[above] {$0.25$} + } + edge from parent + node[above] {$0.68$} + } + child[missing] {} + child { node {$S$} + child {node {$W$} + edge from parent + node[above] {$0.43$} + } + child {node {$B$} + edge from parent + node[above] {$0.57$} + } + edge from parent + node[above] {$0.32$} + } ; + \end{tikzpicture} + \end{center} + \item On calcule la probabilité que la vente soit un wok et ait eu lieu à midi + \[ P(M\cap W) = P(M) \times P_M(W) = 0.68 \times 0.75 = 0.51 \] + \item Probabilité que la vente soit un burger. + \[ + P(B) = P(M\cap B) + P(S\cap B) = 0.68 \times 0.75 + 0.32 \times 0.43 = 0.3524 + \] + \item On cherche à calculer la quantité $P_B(S)$. Pour cela on utilise la formule de Bayes + \[ + P_B(S) = \frac{P(B\cap S)}{P(B)} = \frac{P_S(B) \times P(S)}{P(B)} = \frac{0.57\times 0.32}{0.3524} = 0.5175936435868331 \approx 0.518 + \] + \end{enumerate} +\end{solution} + +\begin{exercise}[subtitle={Continent plastique}] + \textit{Les quantités évoqués dans cette exercice sont générés au hasard et sont donc complètement farfelus.} + \medskip + Le \og continent de plastique\fg{} est la plus grande des plaques de déchets plastiques évoluant sur les océans. Elle occupe actuellement dans l'océan Pacifique une surface dont l'aire est évaluée à plus de $1,6$ million de km$^2$, entre Hawaï et la Californie. + + En 2017, des scientifiques ont estimé qu'il y avait $4$ millions de tonnes de déchets plastiques qui était déversé chaque année dans les océans et que cette quantité augmentait de $10\n\%$ par chaque année. + + On modélise l'évolution de la masse de ces déchets plastiques déversée chaque année, si rien n'est fait pour la réduire, par une suite géométrique $\left(u_n\right)$. L'arrondi au centième du terme $u_n$ représente la masse de ces déchets déversée chaque année, exprimée en million de tonnes, pour l'année $(2017 + n)$. + + \medskip + + \begin{enumerate} + \item Expliquer pourquoi la suite $u_n$ est géométrique? + \item Calculer $u_1$ et $u_2$. + \item Exprimer $u_n$ en fonction de $n$. + \item Au début de l'année 2017, il y avait $300$ millions de tonnes de déchets plastique. Calculer la quantité totale de déchets plastiques en 2030. + \item On souhaite déterminer en quelle année la masse totale de ces déchets plastiques aura pour la première fois augmenté de $50$\,\% par rapport à sa valeur de 2017. + \begin{enumerate} + \item Recopier et compléter l'algorithme ci-dessous pour que la variable $N$ contienne la réponse au problème posé. + + \begin{center} + \begin{tabularx}{0.4\linewidth}{|X|}\hline + $N = 2017$\\ + $U = 4$ \\ + $S = 300 + U$ \\ + while $S < 450$: \\ + \hspace{1cm} $N = \ldots$\\ + \hspace{1cm} $U = \ldots$\\ + \hspace{1cm} $S = \ldots$\\ + \hline + \end{tabularx} + \end{center} + \item Que contiennent les variables $S$, $U$ et $N$ après exécution de cet algorithme ? + + Interpréter les résultats dans le contexte de l'exercice. + \end{enumerate} + \end{enumerate} +\end{exercise} + +\begin{solution} + \begin{enumerate} + \item Une augmentation de $10\,\%$ revient à multiplier la quantité par $1.1$. La suite est donc bien géométrique. Son premier terme est $u_0 = 4$ et sa raison est $q = 1.1$ + \item + \[ + u_1 = u_0 * 1.1 = 4.4 + \] + \[ + u_2 = u_0 * 1.1^2 = 4.84 + \] + \item + \[ + u_n = u_0 \times q^n = 4 \times 1.1^n + \] + \item On calcule la quantité totale déversée entre 2017 et 2030. + \[ + \sum_{n = 0}^{13} u_n = u_0 \times \frac{1-q^{13}}{1-q} = 4 \times \frac{1 - 1.1^{13}}{1 - 1.1} = 98.09 + \] + On en déduit la quantité totale de déchets en 2030 + \[ + 300 + 98.09 = 398.09000000000003 + \] + \item + \begin{enumerate} + \item ~ + \begin{center} + \begin{tabularx}{0.4\linewidth}{|X|}\hline + $N \gets 2017$\\ + $U \gets 4$ \\ + $S \gets 300 + U$ \\ + Tant que $S < 450$ \\ + \hspace{1cm} $N \gets N + 1$\\ + \hspace{1cm} $U \gets U * 1.1$\\ + \hspace{1cm} $S \gets S + u$\\ + Fin Tant que\\\hline + \end{tabularx} + \end{center} + \item \textit{Pas de correction automatisé} + \end{enumerate} + \end{enumerate} +\end{solution} + +\end{document} + +%%% Local Variables: +%%% mode: latex +%%% TeX-master: "master" +%%% End: diff --git a/TST/DS/DS_21_04_07/TST3/10_210407_DS8.tex b/TST/DS/DS_21_04_07/TST3/10_210407_DS8.tex new file mode 100644 index 0000000..4add16a --- /dev/null +++ b/TST/DS/DS_21_04_07/TST3/10_210407_DS8.tex @@ -0,0 +1,262 @@ +\documentclass[a4paper,10pt]{article} +\usepackage{myXsim} + +% Title Page +\title{DS8 \hfill HENRIST Maxime} +\tribe{TST} +\date{\hfillÀ render pour le Mercredi 7 avril} + +\xsimsetup{ + solution/print = false +} + +\begin{document} +\maketitle + +\begin{exercise}[subtitle={Automatismes}] + \textit{Toutes les questions de cette exercice sont indépendantes et peuvent être répondus séparément} + \begin{enumerate} + \item De janvier à septembre, une quantité a augmenté de $19\,\%$. Faire un schéma pour représenter la situation puis calculer le taux d'évolution moyen mensuel. + \item Une quantité augmente de $19\,\%$ par ans. En 2020, elle est de 133\euro. Quelle était sa valeur en 2019? Faire un schéma pour représenter la situation. + \item Déterminer l'équation de la droite \\ + \begin{tikzpicture}[xscale=0.8, yscale=0.5] + \tkzInit[xmin=-5,xmax=5,xstep=1, + ymin=-5,ymax=5,ystep=1] + \tkzGrid + \tkzAxeXY + \tkzFct[domain=-5:5,color=red,very thick]% + {1.0*\x -2}; + \end{tikzpicture} + \item Résoudre l'équation $7 \times 0.82^x = 19$ + \end{enumerate} +\end{exercise} + +\begin{solution} + \begin{enumerate} + \item On veut partager cette évolution en 8 évolutions. + \[ + \left(1 + \frac{19}{100}\right)^{\frac{1}{8}} = 1.022 + \] + Donc le taux d'évolution moyen est + \[ + t_m = 1.022 - 1 = 0.02200000000000002 + \] + \item Coefficient multiplicateur pour revenir en arrière + \[ + CM = (1 + \frac{19}{100})^{-1} = 0.8403 + \] + On en déduit la quantité en 2019 + \[ + 133 * 0.8403 = 111.7599 + \] + \item L'équation de la droite est + \[ + y = 1.0 x -2 + \] + \item Il faut penser à faire la division à par $7$ avant d'utiliser le log car sinon, on ne peut pas utiliser la formule $\log(a^n) = n\times \log(a)$. + + \[x = \frac{\log(2.71)}{\log(0.82)}\] + \end{enumerate} +\end{solution} + +\begin{exercise}[subtitle={Restaurant}] + Un \emph{food truck}, ouvert le midi et le soir, propose deux types de formules : + + \setlength\parindent{10mm} + \begin{itemize} + \item la formule \emph{Burger} ; + \item la formule \emph{Wok}. + \end{itemize} + \setlength\parindent{0mm} + + \medskip + + Le gérant a remarqué que 20\,\% de ses ventes ont lieu le midi. Le quart des ventes du midi correspondent à la formule \emph{Burger}, alors que 56\,\% des ventes du soir correspondent à la formule \emph{Wok}. + + Le gérant se constitue un fichier en notant, pour chaque vente, la formule choisie et le moment de cette vente (midi ou soir). + + On prélève une fiche de façon équiprobable. On définit les quatre évènements suivants: + + \begin{enumerate} + \item $M$ : \og la fiche correspond à une vente du midi\fg{} ; + \item $S$ : \og la fiche correspond à une vente du soir\fg {}; + \item $W$ : \og la fiche correspond à une formule \emph{Wok} \fg{} ; + \item $B$ : \og la fiche correspond à une formule \emph{Burger} \fg. + \end{enumerate} + \setlength\parindent{0mm} + + \medskip + + \begin{enumerate} + \item Recopier puis compléter l'arbre pondéré + + \begin{center} + \begin{tikzpicture}[sloped] + \node {.} + child {node {$M$} + child {node {$W$} + edge from parent + node[above] {...} + } + child {node {$B$} + edge from parent + node[above] {...} + } + edge from parent + node[above] {...} + } + child[missing] {} + child { node {$S$} + child {node {$W$} + edge from parent + node[above] {...} + } + child {node {$B$} + edge from parent + node[above] {...} + } + edge from parent + node[above] {...} + } ; + \end{tikzpicture} + \end{center} + + \item Calculer la probabilité de l'évènement $M \cap W$. Interpréter ce résultat dans le contexte de l'exercice. + \item Montrer que la probabilité que la fiche choisie corresponde à une formule \emph{Burger} est égale à $0.394$. + \item On a prélevé une fiche correspondant à la formule \emph{Burger}. Quelle est la probabilité, arrondie au millième, que la vente ait eu lieu le soir? + \end{enumerate} +\end{exercise} + +\begin{solution} + \begin{enumerate} + \item + \begin{center} + \begin{tikzpicture}[sloped] + \node {.} + child {node {$M$} + child {node {$W$} + edge from parent + node[above] {$0.75$} + } + child {node {$B$} + edge from parent + node[above] {$0.25$} + } + edge from parent + node[above] {$0.2$} + } + child[missing] {} + child { node {$S$} + child {node {$W$} + edge from parent + node[above] {$0.57$} + } + child {node {$B$} + edge from parent + node[above] {$0.43$} + } + edge from parent + node[above] {$0.8$} + } ; + \end{tikzpicture} + \end{center} + \item On calcule la probabilité que la vente soit un wok et ait eu lieu à midi + \[ P(M\cap W) = P(M) \times P_M(W) = 0.2 \times 0.75 = 0.15 \] + \item Probabilité que la vente soit un burger. + \[ + P(B) = P(M\cap B) + P(S\cap B) = 0.2 \times 0.75 + 0.8 \times 0.57 = 0.394 + \] + \item On cherche à calculer la quantité $P_B(S)$. Pour cela on utilise la formule de Bayes + \[ + P_B(S) = \frac{P(B\cap S)}{P(B)} = \frac{P_S(B) \times P(S)}{P(B)} = \frac{0.43\times 0.8}{0.394} = 0.8730964467005077 \approx 0.873 + \] + \end{enumerate} +\end{solution} + +\begin{exercise}[subtitle={Continent plastique}] + \textit{Les quantités évoqués dans cette exercice sont générés au hasard et sont donc complètement farfelus.} + \medskip + Le \og continent de plastique\fg{} est la plus grande des plaques de déchets plastiques évoluant sur les océans. Elle occupe actuellement dans l'océan Pacifique une surface dont l'aire est évaluée à plus de $1,6$ million de km$^2$, entre Hawaï et la Californie. + + En 2017, des scientifiques ont estimé qu'il y avait $5$ millions de tonnes de déchets plastiques qui était déversé chaque année dans les océans et que cette quantité augmentait de $16\n\%$ par chaque année. + + On modélise l'évolution de la masse de ces déchets plastiques déversée chaque année, si rien n'est fait pour la réduire, par une suite géométrique $\left(u_n\right)$. L'arrondi au centième du terme $u_n$ représente la masse de ces déchets déversée chaque année, exprimée en million de tonnes, pour l'année $(2017 + n)$. + + \medskip + + \begin{enumerate} + \item Expliquer pourquoi la suite $u_n$ est géométrique? + \item Calculer $u_1$ et $u_2$. + \item Exprimer $u_n$ en fonction de $n$. + \item Au début de l'année 2017, il y avait $300$ millions de tonnes de déchets plastique. Calculer la quantité totale de déchets plastiques en 2030. + \item On souhaite déterminer en quelle année la masse totale de ces déchets plastiques aura pour la première fois augmenté de $50$\,\% par rapport à sa valeur de 2017. + \begin{enumerate} + \item Recopier et compléter l'algorithme ci-dessous pour que la variable $N$ contienne la réponse au problème posé. + + \begin{center} + \begin{tabularx}{0.4\linewidth}{|X|}\hline + $N = 2017$\\ + $U = 5$ \\ + $S = 300 + U$ \\ + while $S < 450$: \\ + \hspace{1cm} $N = \ldots$\\ + \hspace{1cm} $U = \ldots$\\ + \hspace{1cm} $S = \ldots$\\ + \hline + \end{tabularx} + \end{center} + \item Que contiennent les variables $S$, $U$ et $N$ après exécution de cet algorithme ? + + Interpréter les résultats dans le contexte de l'exercice. + \end{enumerate} + \end{enumerate} +\end{exercise} + +\begin{solution} + \begin{enumerate} + \item Une augmentation de $16\,\%$ revient à multiplier la quantité par $1.16$. La suite est donc bien géométrique. Son premier terme est $u_0 = 5$ et sa raison est $q = 1.16$ + \item + \[ + u_1 = u_0 * 1.16 = 5.8 + \] + \[ + u_2 = u_0 * 1.16^2 = 6.728 + \] + \item + \[ + u_n = u_0 \times q^n = 5 \times 1.16^n + \] + \item On calcule la quantité totale déversée entre 2017 et 2030. + \[ + \sum_{n = 0}^{13} u_n = u_0 \times \frac{1-q^{13}}{1-q} = 5 \times \frac{1 - 1.16^{13}}{1 - 1.16} = 183.93 + \] + On en déduit la quantité totale de déchets en 2030 + \[ + 300 + 183.93 = 483.93 + \] + \item + \begin{enumerate} + \item ~ + \begin{center} + \begin{tabularx}{0.4\linewidth}{|X|}\hline + $N \gets 2017$\\ + $U \gets 5$ \\ + $S \gets 300 + U$ \\ + Tant que $S < 450$ \\ + \hspace{1cm} $N \gets N + 1$\\ + \hspace{1cm} $U \gets U * 1.16$\\ + \hspace{1cm} $S \gets S + u$\\ + Fin Tant que\\\hline + \end{tabularx} + \end{center} + \item \textit{Pas de correction automatisé} + \end{enumerate} + \end{enumerate} +\end{solution} + +\end{document} + +%%% Local Variables: +%%% mode: latex +%%% TeX-master: "master" +%%% End: diff --git a/TST/DS/DS_21_04_07/TST3/11_210407_DS8.tex b/TST/DS/DS_21_04_07/TST3/11_210407_DS8.tex new file mode 100644 index 0000000..a2f366f --- /dev/null +++ b/TST/DS/DS_21_04_07/TST3/11_210407_DS8.tex @@ -0,0 +1,262 @@ +\documentclass[a4paper,10pt]{article} +\usepackage{myXsim} + +% Title Page +\title{DS8 \hfill INFANTES Antoine} +\tribe{TST} +\date{\hfillÀ render pour le Mercredi 7 avril} + +\xsimsetup{ + solution/print = false +} + +\begin{document} +\maketitle + +\begin{exercise}[subtitle={Automatismes}] + \textit{Toutes les questions de cette exercice sont indépendantes et peuvent être répondus séparément} + \begin{enumerate} + \item De janvier à septembre, une quantité a augmenté de $18\,\%$. Faire un schéma pour représenter la situation puis calculer le taux d'évolution moyen mensuel. + \item Une quantité augmente de $18\,\%$ par ans. En 2020, elle est de 130\euro. Quelle était sa valeur en 2019? Faire un schéma pour représenter la situation. + \item Déterminer l'équation de la droite \\ + \begin{tikzpicture}[xscale=0.8, yscale=0.5] + \tkzInit[xmin=-5,xmax=5,xstep=1, + ymin=-5,ymax=5,ystep=1] + \tkzGrid + \tkzAxeXY + \tkzFct[domain=-5:5,color=red,very thick]% + {2.6666666666666665*\x -4}; + \end{tikzpicture} + \item Résoudre l'équation $3 \times 0.31^x = 15$ + \end{enumerate} +\end{exercise} + +\begin{solution} + \begin{enumerate} + \item On veut partager cette évolution en 8 évolutions. + \[ + \left(1 + \frac{18}{100}\right)^{\frac{1}{8}} = 1.0209 + \] + Donc le taux d'évolution moyen est + \[ + t_m = 1.0209 - 1 = 0.02089999999999992 + \] + \item Coefficient multiplicateur pour revenir en arrière + \[ + CM = (1 + \frac{18}{100})^{-1} = 0.8475 + \] + On en déduit la quantité en 2019 + \[ + 130 * 0.8475 = 110.175 + \] + \item L'équation de la droite est + \[ + y = 2.6666666666666665 x -4 + \] + \item Il faut penser à faire la division à par $3$ avant d'utiliser le log car sinon, on ne peut pas utiliser la formule $\log(a^n) = n\times \log(a)$. + + \[x = \frac{\log(5.0)}{\log(0.31)}\] + \end{enumerate} +\end{solution} + +\begin{exercise}[subtitle={Restaurant}] + Un \emph{food truck}, ouvert le midi et le soir, propose deux types de formules : + + \setlength\parindent{10mm} + \begin{itemize} + \item la formule \emph{Burger} ; + \item la formule \emph{Wok}. + \end{itemize} + \setlength\parindent{0mm} + + \medskip + + Le gérant a remarqué que 33\,\% de ses ventes ont lieu le midi. Le quart des ventes du midi correspondent à la formule \emph{Burger}, alors que 13\,\% des ventes du soir correspondent à la formule \emph{Wok}. + + Le gérant se constitue un fichier en notant, pour chaque vente, la formule choisie et le moment de cette vente (midi ou soir). + + On prélève une fiche de façon équiprobable. On définit les quatre évènements suivants: + + \begin{enumerate} + \item $M$ : \og la fiche correspond à une vente du midi\fg{} ; + \item $S$ : \og la fiche correspond à une vente du soir\fg {}; + \item $W$ : \og la fiche correspond à une formule \emph{Wok} \fg{} ; + \item $B$ : \og la fiche correspond à une formule \emph{Burger} \fg. + \end{enumerate} + \setlength\parindent{0mm} + + \medskip + + \begin{enumerate} + \item Recopier puis compléter l'arbre pondéré + + \begin{center} + \begin{tikzpicture}[sloped] + \node {.} + child {node {$M$} + child {node {$W$} + edge from parent + node[above] {...} + } + child {node {$B$} + edge from parent + node[above] {...} + } + edge from parent + node[above] {...} + } + child[missing] {} + child { node {$S$} + child {node {$W$} + edge from parent + node[above] {...} + } + child {node {$B$} + edge from parent + node[above] {...} + } + edge from parent + node[above] {...} + } ; + \end{tikzpicture} + \end{center} + + \item Calculer la probabilité de l'évènement $M \cap W$. Interpréter ce résultat dans le contexte de l'exercice. + \item Montrer que la probabilité que la fiche choisie corresponde à une formule \emph{Burger} est égale à $0.6654$. + \item On a prélevé une fiche correspondant à la formule \emph{Burger}. Quelle est la probabilité, arrondie au millième, que la vente ait eu lieu le soir? + \end{enumerate} +\end{exercise} + +\begin{solution} + \begin{enumerate} + \item + \begin{center} + \begin{tikzpicture}[sloped] + \node {.} + child {node {$M$} + child {node {$W$} + edge from parent + node[above] {$0.75$} + } + child {node {$B$} + edge from parent + node[above] {$0.25$} + } + edge from parent + node[above] {$0.33$} + } + child[missing] {} + child { node {$S$} + child {node {$W$} + edge from parent + node[above] {$0.13$} + } + child {node {$B$} + edge from parent + node[above] {$0.87$} + } + edge from parent + node[above] {$0.67$} + } ; + \end{tikzpicture} + \end{center} + \item On calcule la probabilité que la vente soit un wok et ait eu lieu à midi + \[ P(M\cap W) = P(M) \times P_M(W) = 0.33 \times 0.75 = 0.2475 \] + \item Probabilité que la vente soit un burger. + \[ + P(B) = P(M\cap B) + P(S\cap B) = 0.33 \times 0.75 + 0.67 \times 0.13 = 0.6654 + \] + \item On cherche à calculer la quantité $P_B(S)$. Pour cela on utilise la formule de Bayes + \[ + P_B(S) = \frac{P(B\cap S)}{P(B)} = \frac{P_S(B) \times P(S)}{P(B)} = \frac{0.87\times 0.67}{0.6654} = 0.8760144274120831 \approx 0.876 + \] + \end{enumerate} +\end{solution} + +\begin{exercise}[subtitle={Continent plastique}] + \textit{Les quantités évoqués dans cette exercice sont générés au hasard et sont donc complètement farfelus.} + \medskip + Le \og continent de plastique\fg{} est la plus grande des plaques de déchets plastiques évoluant sur les océans. Elle occupe actuellement dans l'océan Pacifique une surface dont l'aire est évaluée à plus de $1,6$ million de km$^2$, entre Hawaï et la Californie. + + En 2017, des scientifiques ont estimé qu'il y avait $12$ millions de tonnes de déchets plastiques qui était déversé chaque année dans les océans et que cette quantité augmentait de $26\n\%$ par chaque année. + + On modélise l'évolution de la masse de ces déchets plastiques déversée chaque année, si rien n'est fait pour la réduire, par une suite géométrique $\left(u_n\right)$. L'arrondi au centième du terme $u_n$ représente la masse de ces déchets déversée chaque année, exprimée en million de tonnes, pour l'année $(2017 + n)$. + + \medskip + + \begin{enumerate} + \item Expliquer pourquoi la suite $u_n$ est géométrique? + \item Calculer $u_1$ et $u_2$. + \item Exprimer $u_n$ en fonction de $n$. + \item Au début de l'année 2017, il y avait $300$ millions de tonnes de déchets plastique. Calculer la quantité totale de déchets plastiques en 2030. + \item On souhaite déterminer en quelle année la masse totale de ces déchets plastiques aura pour la première fois augmenté de $50$\,\% par rapport à sa valeur de 2017. + \begin{enumerate} + \item Recopier et compléter l'algorithme ci-dessous pour que la variable $N$ contienne la réponse au problème posé. + + \begin{center} + \begin{tabularx}{0.4\linewidth}{|X|}\hline + $N = 2017$\\ + $U = 12$ \\ + $S = 300 + U$ \\ + while $S < 450$: \\ + \hspace{1cm} $N = \ldots$\\ + \hspace{1cm} $U = \ldots$\\ + \hspace{1cm} $S = \ldots$\\ + \hline + \end{tabularx} + \end{center} + \item Que contiennent les variables $S$, $U$ et $N$ après exécution de cet algorithme ? + + Interpréter les résultats dans le contexte de l'exercice. + \end{enumerate} + \end{enumerate} +\end{exercise} + +\begin{solution} + \begin{enumerate} + \item Une augmentation de $26\,\%$ revient à multiplier la quantité par $1.26$. La suite est donc bien géométrique. Son premier terme est $u_0 = 12$ et sa raison est $q = 1.26$ + \item + \[ + u_1 = u_0 * 1.26 = 15.120000000000001 + \] + \[ + u_2 = u_0 * 1.26^2 = 19.0512 + \] + \item + \[ + u_n = u_0 \times q^n = 12 \times 1.26^n + \] + \item On calcule la quantité totale déversée entre 2017 et 2030. + \[ + \sum_{n = 0}^{13} u_n = u_0 \times \frac{1-q^{13}}{1-q} = 12 \times \frac{1 - 1.26^{13}}{1 - 1.26} = 885.01 + \] + On en déduit la quantité totale de déchets en 2030 + \[ + 300 + 885.01 = 1185.01 + \] + \item + \begin{enumerate} + \item ~ + \begin{center} + \begin{tabularx}{0.4\linewidth}{|X|}\hline + $N \gets 2017$\\ + $U \gets 12$ \\ + $S \gets 300 + U$ \\ + Tant que $S < 450$ \\ + \hspace{1cm} $N \gets N + 1$\\ + \hspace{1cm} $U \gets U * 1.26$\\ + \hspace{1cm} $S \gets S + u$\\ + Fin Tant que\\\hline + \end{tabularx} + \end{center} + \item \textit{Pas de correction automatisé} + \end{enumerate} + \end{enumerate} +\end{solution} + +\end{document} + +%%% Local Variables: +%%% mode: latex +%%% TeX-master: "master" +%%% End: diff --git a/TST/DS/DS_21_04_07/TST3/12_210407_DS8.tex b/TST/DS/DS_21_04_07/TST3/12_210407_DS8.tex new file mode 100644 index 0000000..aeda1ed --- /dev/null +++ b/TST/DS/DS_21_04_07/TST3/12_210407_DS8.tex @@ -0,0 +1,262 @@ +\documentclass[a4paper,10pt]{article} +\usepackage{myXsim} + +% Title Page +\title{DS8 \hfill MAGRO Robin} +\tribe{TST} +\date{\hfillÀ render pour le Mercredi 7 avril} + +\xsimsetup{ + solution/print = false +} + +\begin{document} +\maketitle + +\begin{exercise}[subtitle={Automatismes}] + \textit{Toutes les questions de cette exercice sont indépendantes et peuvent être répondus séparément} + \begin{enumerate} + \item De janvier à septembre, une quantité a augmenté de $11\,\%$. Faire un schéma pour représenter la situation puis calculer le taux d'évolution moyen mensuel. + \item Une quantité augmente de $11\,\%$ par ans. En 2020, elle est de 143\euro. Quelle était sa valeur en 2019? Faire un schéma pour représenter la situation. + \item Déterminer l'équation de la droite \\ + \begin{tikzpicture}[xscale=0.8, yscale=0.5] + \tkzInit[xmin=-5,xmax=5,xstep=1, + ymin=-5,ymax=5,ystep=1] + \tkzGrid + \tkzAxeXY + \tkzFct[domain=-5:5,color=red,very thick]% + {3.0*\x -3}; + \end{tikzpicture} + \item Résoudre l'équation $5 \times 0.12^x = 40$ + \end{enumerate} +\end{exercise} + +\begin{solution} + \begin{enumerate} + \item On veut partager cette évolution en 8 évolutions. + \[ + \left(1 + \frac{11}{100}\right)^{\frac{1}{8}} = 1.0131 + \] + Donc le taux d'évolution moyen est + \[ + t_m = 1.0131 - 1 = 0.01309999999999989 + \] + \item Coefficient multiplicateur pour revenir en arrière + \[ + CM = (1 + \frac{11}{100})^{-1} = 0.9009 + \] + On en déduit la quantité en 2019 + \[ + 143 * 0.9009 = 128.8287 + \] + \item L'équation de la droite est + \[ + y = 3.0 x -3 + \] + \item Il faut penser à faire la division à par $5$ avant d'utiliser le log car sinon, on ne peut pas utiliser la formule $\log(a^n) = n\times \log(a)$. + + \[x = \frac{\log(8.0)}{\log(0.12)}\] + \end{enumerate} +\end{solution} + +\begin{exercise}[subtitle={Restaurant}] + Un \emph{food truck}, ouvert le midi et le soir, propose deux types de formules : + + \setlength\parindent{10mm} + \begin{itemize} + \item la formule \emph{Burger} ; + \item la formule \emph{Wok}. + \end{itemize} + \setlength\parindent{0mm} + + \medskip + + Le gérant a remarqué que 37\,\% de ses ventes ont lieu le midi. Le quart des ventes du midi correspondent à la formule \emph{Burger}, alors que 32\,\% des ventes du soir correspondent à la formule \emph{Wok}. + + Le gérant se constitue un fichier en notant, pour chaque vente, la formule choisie et le moment de cette vente (midi ou soir). + + On prélève une fiche de façon équiprobable. On définit les quatre évènements suivants: + + \begin{enumerate} + \item $M$ : \og la fiche correspond à une vente du midi\fg{} ; + \item $S$ : \og la fiche correspond à une vente du soir\fg {}; + \item $W$ : \og la fiche correspond à une formule \emph{Wok} \fg{} ; + \item $B$ : \og la fiche correspond à une formule \emph{Burger} \fg. + \end{enumerate} + \setlength\parindent{0mm} + + \medskip + + \begin{enumerate} + \item Recopier puis compléter l'arbre pondéré + + \begin{center} + \begin{tikzpicture}[sloped] + \node {.} + child {node {$M$} + child {node {$W$} + edge from parent + node[above] {...} + } + child {node {$B$} + edge from parent + node[above] {...} + } + edge from parent + node[above] {...} + } + child[missing] {} + child { node {$S$} + child {node {$W$} + edge from parent + node[above] {...} + } + child {node {$B$} + edge from parent + node[above] {...} + } + edge from parent + node[above] {...} + } ; + \end{tikzpicture} + \end{center} + + \item Calculer la probabilité de l'évènement $M \cap W$. Interpréter ce résultat dans le contexte de l'exercice. + \item Montrer que la probabilité que la fiche choisie corresponde à une formule \emph{Burger} est égale à $0.5209$. + \item On a prélevé une fiche correspondant à la formule \emph{Burger}. Quelle est la probabilité, arrondie au millième, que la vente ait eu lieu le soir? + \end{enumerate} +\end{exercise} + +\begin{solution} + \begin{enumerate} + \item + \begin{center} + \begin{tikzpicture}[sloped] + \node {.} + child {node {$M$} + child {node {$W$} + edge from parent + node[above] {$0.75$} + } + child {node {$B$} + edge from parent + node[above] {$0.25$} + } + edge from parent + node[above] {$0.37$} + } + child[missing] {} + child { node {$S$} + child {node {$W$} + edge from parent + node[above] {$0.32$} + } + child {node {$B$} + edge from parent + node[above] {$0.68$} + } + edge from parent + node[above] {$0.63$} + } ; + \end{tikzpicture} + \end{center} + \item On calcule la probabilité que la vente soit un wok et ait eu lieu à midi + \[ P(M\cap W) = P(M) \times P_M(W) = 0.37 \times 0.75 = 0.2775 \] + \item Probabilité que la vente soit un burger. + \[ + P(B) = P(M\cap B) + P(S\cap B) = 0.37 \times 0.75 + 0.63 \times 0.32 = 0.5209 + \] + \item On cherche à calculer la quantité $P_B(S)$. Pour cela on utilise la formule de Bayes + \[ + P_B(S) = \frac{P(B\cap S)}{P(B)} = \frac{P_S(B) \times P(S)}{P(B)} = \frac{0.68\times 0.63}{0.5209} = 0.8224227298905741 \approx 0.822 + \] + \end{enumerate} +\end{solution} + +\begin{exercise}[subtitle={Continent plastique}] + \textit{Les quantités évoqués dans cette exercice sont générés au hasard et sont donc complètement farfelus.} + \medskip + Le \og continent de plastique\fg{} est la plus grande des plaques de déchets plastiques évoluant sur les océans. Elle occupe actuellement dans l'océan Pacifique une surface dont l'aire est évaluée à plus de $1,6$ million de km$^2$, entre Hawaï et la Californie. + + En 2017, des scientifiques ont estimé qu'il y avait $16$ millions de tonnes de déchets plastiques qui était déversé chaque année dans les océans et que cette quantité augmentait de $17\n\%$ par chaque année. + + On modélise l'évolution de la masse de ces déchets plastiques déversée chaque année, si rien n'est fait pour la réduire, par une suite géométrique $\left(u_n\right)$. L'arrondi au centième du terme $u_n$ représente la masse de ces déchets déversée chaque année, exprimée en million de tonnes, pour l'année $(2017 + n)$. + + \medskip + + \begin{enumerate} + \item Expliquer pourquoi la suite $u_n$ est géométrique? + \item Calculer $u_1$ et $u_2$. + \item Exprimer $u_n$ en fonction de $n$. + \item Au début de l'année 2017, il y avait $300$ millions de tonnes de déchets plastique. Calculer la quantité totale de déchets plastiques en 2030. + \item On souhaite déterminer en quelle année la masse totale de ces déchets plastiques aura pour la première fois augmenté de $50$\,\% par rapport à sa valeur de 2017. + \begin{enumerate} + \item Recopier et compléter l'algorithme ci-dessous pour que la variable $N$ contienne la réponse au problème posé. + + \begin{center} + \begin{tabularx}{0.4\linewidth}{|X|}\hline + $N = 2017$\\ + $U = 16$ \\ + $S = 300 + U$ \\ + while $S < 450$: \\ + \hspace{1cm} $N = \ldots$\\ + \hspace{1cm} $U = \ldots$\\ + \hspace{1cm} $S = \ldots$\\ + \hline + \end{tabularx} + \end{center} + \item Que contiennent les variables $S$, $U$ et $N$ après exécution de cet algorithme ? + + Interpréter les résultats dans le contexte de l'exercice. + \end{enumerate} + \end{enumerate} +\end{exercise} + +\begin{solution} + \begin{enumerate} + \item Une augmentation de $17\,\%$ revient à multiplier la quantité par $1.17$. La suite est donc bien géométrique. Son premier terme est $u_0 = 16$ et sa raison est $q = 1.17$ + \item + \[ + u_1 = u_0 * 1.17 = 18.72 + \] + \[ + u_2 = u_0 * 1.17^2 = 21.9024 + \] + \item + \[ + u_n = u_0 \times q^n = 16 \times 1.17^n + \] + \item On calcule la quantité totale déversée entre 2017 et 2030. + \[ + \sum_{n = 0}^{13} u_n = u_0 \times \frac{1-q^{13}}{1-q} = 16 \times \frac{1 - 1.17^{13}}{1 - 1.17} = 630.46 + \] + On en déduit la quantité totale de déchets en 2030 + \[ + 300 + 630.46 = 930.46 + \] + \item + \begin{enumerate} + \item ~ + \begin{center} + \begin{tabularx}{0.4\linewidth}{|X|}\hline + $N \gets 2017$\\ + $U \gets 16$ \\ + $S \gets 300 + U$ \\ + Tant que $S < 450$ \\ + \hspace{1cm} $N \gets N + 1$\\ + \hspace{1cm} $U \gets U * 1.17$\\ + \hspace{1cm} $S \gets S + u$\\ + Fin Tant que\\\hline + \end{tabularx} + \end{center} + \item \textit{Pas de correction automatisé} + \end{enumerate} + \end{enumerate} +\end{solution} + +\end{document} + +%%% Local Variables: +%%% mode: latex +%%% TeX-master: "master" +%%% End: diff --git a/TST/DS/DS_21_04_07/TST3/13_210407_DS8.tex b/TST/DS/DS_21_04_07/TST3/13_210407_DS8.tex new file mode 100644 index 0000000..5b68176 --- /dev/null +++ b/TST/DS/DS_21_04_07/TST3/13_210407_DS8.tex @@ -0,0 +1,262 @@ +\documentclass[a4paper,10pt]{article} +\usepackage{myXsim} + +% Title Page +\title{DS8 \hfill MORFIN Chloé} +\tribe{TST} +\date{\hfillÀ render pour le Mercredi 7 avril} + +\xsimsetup{ + solution/print = false +} + +\begin{document} +\maketitle + +\begin{exercise}[subtitle={Automatismes}] + \textit{Toutes les questions de cette exercice sont indépendantes et peuvent être répondus séparément} + \begin{enumerate} + \item De janvier à septembre, une quantité a augmenté de $18\,\%$. Faire un schéma pour représenter la situation puis calculer le taux d'évolution moyen mensuel. + \item Une quantité augmente de $18\,\%$ par ans. En 2020, elle est de 119\euro. Quelle était sa valeur en 2019? Faire un schéma pour représenter la situation. + \item Déterminer l'équation de la droite \\ + \begin{tikzpicture}[xscale=0.8, yscale=0.5] + \tkzInit[xmin=-5,xmax=5,xstep=1, + ymin=-5,ymax=5,ystep=1] + \tkzGrid + \tkzAxeXY + \tkzFct[domain=-5:5,color=red,very thick]% + {4.0*\x -4}; + \end{tikzpicture} + \item Résoudre l'équation $3 \times 1.0^x = 32$ + \end{enumerate} +\end{exercise} + +\begin{solution} + \begin{enumerate} + \item On veut partager cette évolution en 8 évolutions. + \[ + \left(1 + \frac{18}{100}\right)^{\frac{1}{8}} = 1.0209 + \] + Donc le taux d'évolution moyen est + \[ + t_m = 1.0209 - 1 = 0.02089999999999992 + \] + \item Coefficient multiplicateur pour revenir en arrière + \[ + CM = (1 + \frac{18}{100})^{-1} = 0.8475 + \] + On en déduit la quantité en 2019 + \[ + 119 * 0.8475 = 100.8525 + \] + \item L'équation de la droite est + \[ + y = 4.0 x -4 + \] + \item Il faut penser à faire la division à par $3$ avant d'utiliser le log car sinon, on ne peut pas utiliser la formule $\log(a^n) = n\times \log(a)$. + + \[x = \frac{\log(10.67)}{\log(1.0)}\] + \end{enumerate} +\end{solution} + +\begin{exercise}[subtitle={Restaurant}] + Un \emph{food truck}, ouvert le midi et le soir, propose deux types de formules : + + \setlength\parindent{10mm} + \begin{itemize} + \item la formule \emph{Burger} ; + \item la formule \emph{Wok}. + \end{itemize} + \setlength\parindent{0mm} + + \medskip + + Le gérant a remarqué que 24\,\% de ses ventes ont lieu le midi. Le quart des ventes du midi correspondent à la formule \emph{Burger}, alors que 57\,\% des ventes du soir correspondent à la formule \emph{Wok}. + + Le gérant se constitue un fichier en notant, pour chaque vente, la formule choisie et le moment de cette vente (midi ou soir). + + On prélève une fiche de façon équiprobable. On définit les quatre évènements suivants: + + \begin{enumerate} + \item $M$ : \og la fiche correspond à une vente du midi\fg{} ; + \item $S$ : \og la fiche correspond à une vente du soir\fg {}; + \item $W$ : \og la fiche correspond à une formule \emph{Wok} \fg{} ; + \item $B$ : \og la fiche correspond à une formule \emph{Burger} \fg. + \end{enumerate} + \setlength\parindent{0mm} + + \medskip + + \begin{enumerate} + \item Recopier puis compléter l'arbre pondéré + + \begin{center} + \begin{tikzpicture}[sloped] + \node {.} + child {node {$M$} + child {node {$W$} + edge from parent + node[above] {...} + } + child {node {$B$} + edge from parent + node[above] {...} + } + edge from parent + node[above] {...} + } + child[missing] {} + child { node {$S$} + child {node {$W$} + edge from parent + node[above] {...} + } + child {node {$B$} + edge from parent + node[above] {...} + } + edge from parent + node[above] {...} + } ; + \end{tikzpicture} + \end{center} + + \item Calculer la probabilité de l'évènement $M \cap W$. Interpréter ce résultat dans le contexte de l'exercice. + \item Montrer que la probabilité que la fiche choisie corresponde à une formule \emph{Burger} est égale à $0.3792$. + \item On a prélevé une fiche correspondant à la formule \emph{Burger}. Quelle est la probabilité, arrondie au millième, que la vente ait eu lieu le soir? + \end{enumerate} +\end{exercise} + +\begin{solution} + \begin{enumerate} + \item + \begin{center} + \begin{tikzpicture}[sloped] + \node {.} + child {node {$M$} + child {node {$W$} + edge from parent + node[above] {$0.75$} + } + child {node {$B$} + edge from parent + node[above] {$0.25$} + } + edge from parent + node[above] {$0.24$} + } + child[missing] {} + child { node {$S$} + child {node {$W$} + edge from parent + node[above] {$0.58$} + } + child {node {$B$} + edge from parent + node[above] {$0.42$} + } + edge from parent + node[above] {$0.76$} + } ; + \end{tikzpicture} + \end{center} + \item On calcule la probabilité que la vente soit un wok et ait eu lieu à midi + \[ P(M\cap W) = P(M) \times P_M(W) = 0.24 \times 0.75 = 0.18 \] + \item Probabilité que la vente soit un burger. + \[ + P(B) = P(M\cap B) + P(S\cap B) = 0.24 \times 0.75 + 0.76 \times 0.58 = 0.3792 + \] + \item On cherche à calculer la quantité $P_B(S)$. Pour cela on utilise la formule de Bayes + \[ + P_B(S) = \frac{P(B\cap S)}{P(B)} = \frac{P_S(B) \times P(S)}{P(B)} = \frac{0.42\times 0.76}{0.3792} = 0.8417721518987342 \approx 0.842 + \] + \end{enumerate} +\end{solution} + +\begin{exercise}[subtitle={Continent plastique}] + \textit{Les quantités évoqués dans cette exercice sont générés au hasard et sont donc complètement farfelus.} + \medskip + Le \og continent de plastique\fg{} est la plus grande des plaques de déchets plastiques évoluant sur les océans. Elle occupe actuellement dans l'océan Pacifique une surface dont l'aire est évaluée à plus de $1,6$ million de km$^2$, entre Hawaï et la Californie. + + En 2017, des scientifiques ont estimé qu'il y avait $17$ millions de tonnes de déchets plastiques qui était déversé chaque année dans les océans et que cette quantité augmentait de $16\n\%$ par chaque année. + + On modélise l'évolution de la masse de ces déchets plastiques déversée chaque année, si rien n'est fait pour la réduire, par une suite géométrique $\left(u_n\right)$. L'arrondi au centième du terme $u_n$ représente la masse de ces déchets déversée chaque année, exprimée en million de tonnes, pour l'année $(2017 + n)$. + + \medskip + + \begin{enumerate} + \item Expliquer pourquoi la suite $u_n$ est géométrique? + \item Calculer $u_1$ et $u_2$. + \item Exprimer $u_n$ en fonction de $n$. + \item Au début de l'année 2017, il y avait $300$ millions de tonnes de déchets plastique. Calculer la quantité totale de déchets plastiques en 2030. + \item On souhaite déterminer en quelle année la masse totale de ces déchets plastiques aura pour la première fois augmenté de $50$\,\% par rapport à sa valeur de 2017. + \begin{enumerate} + \item Recopier et compléter l'algorithme ci-dessous pour que la variable $N$ contienne la réponse au problème posé. + + \begin{center} + \begin{tabularx}{0.4\linewidth}{|X|}\hline + $N = 2017$\\ + $U = 17$ \\ + $S = 300 + U$ \\ + while $S < 450$: \\ + \hspace{1cm} $N = \ldots$\\ + \hspace{1cm} $U = \ldots$\\ + \hspace{1cm} $S = \ldots$\\ + \hline + \end{tabularx} + \end{center} + \item Que contiennent les variables $S$, $U$ et $N$ après exécution de cet algorithme ? + + Interpréter les résultats dans le contexte de l'exercice. + \end{enumerate} + \end{enumerate} +\end{exercise} + +\begin{solution} + \begin{enumerate} + \item Une augmentation de $16\,\%$ revient à multiplier la quantité par $1.16$. La suite est donc bien géométrique. Son premier terme est $u_0 = 17$ et sa raison est $q = 1.16$ + \item + \[ + u_1 = u_0 * 1.16 = 19.72 + \] + \[ + u_2 = u_0 * 1.16^2 = 22.8752 + \] + \item + \[ + u_n = u_0 \times q^n = 17 \times 1.16^n + \] + \item On calcule la quantité totale déversée entre 2017 et 2030. + \[ + \sum_{n = 0}^{13} u_n = u_0 \times \frac{1-q^{13}}{1-q} = 17 \times \frac{1 - 1.16^{13}}{1 - 1.16} = 625.37 + \] + On en déduit la quantité totale de déchets en 2030 + \[ + 300 + 625.37 = 925.37 + \] + \item + \begin{enumerate} + \item ~ + \begin{center} + \begin{tabularx}{0.4\linewidth}{|X|}\hline + $N \gets 2017$\\ + $U \gets 17$ \\ + $S \gets 300 + U$ \\ + Tant que $S < 450$ \\ + \hspace{1cm} $N \gets N + 1$\\ + \hspace{1cm} $U \gets U * 1.16$\\ + \hspace{1cm} $S \gets S + u$\\ + Fin Tant que\\\hline + \end{tabularx} + \end{center} + \item \textit{Pas de correction automatisé} + \end{enumerate} + \end{enumerate} +\end{solution} + +\end{document} + +%%% Local Variables: +%%% mode: latex +%%% TeX-master: "master" +%%% End: diff --git a/TST/DS/DS_21_04_07/TST3/14_210407_DS8.tex b/TST/DS/DS_21_04_07/TST3/14_210407_DS8.tex new file mode 100644 index 0000000..dfa2919 --- /dev/null +++ b/TST/DS/DS_21_04_07/TST3/14_210407_DS8.tex @@ -0,0 +1,262 @@ +\documentclass[a4paper,10pt]{article} +\usepackage{myXsim} + +% Title Page +\title{DS8 \hfill PERES RAMALHO Emeric} +\tribe{TST} +\date{\hfillÀ render pour le Mercredi 7 avril} + +\xsimsetup{ + solution/print = false +} + +\begin{document} +\maketitle + +\begin{exercise}[subtitle={Automatismes}] + \textit{Toutes les questions de cette exercice sont indépendantes et peuvent être répondus séparément} + \begin{enumerate} + \item De janvier à septembre, une quantité a augmenté de $23\,\%$. Faire un schéma pour représenter la situation puis calculer le taux d'évolution moyen mensuel. + \item Une quantité augmente de $23\,\%$ par ans. En 2020, elle est de 118\euro. Quelle était sa valeur en 2019? Faire un schéma pour représenter la situation. + \item Déterminer l'équation de la droite \\ + \begin{tikzpicture}[xscale=0.8, yscale=0.5] + \tkzInit[xmin=-5,xmax=5,xstep=1, + ymin=-5,ymax=5,ystep=1] + \tkzGrid + \tkzAxeXY + \tkzFct[domain=-5:5,color=red,very thick]% + {1.3333333333333333*\x -2}; + \end{tikzpicture} + \item Résoudre l'équation $5 \times 0.46^x = 2$ + \end{enumerate} +\end{exercise} + +\begin{solution} + \begin{enumerate} + \item On veut partager cette évolution en 8 évolutions. + \[ + \left(1 + \frac{23}{100}\right)^{\frac{1}{8}} = 1.0262 + \] + Donc le taux d'évolution moyen est + \[ + t_m = 1.0262 - 1 = 0.0262 + \] + \item Coefficient multiplicateur pour revenir en arrière + \[ + CM = (1 + \frac{23}{100})^{-1} = 0.813 + \] + On en déduit la quantité en 2019 + \[ + 118 * 0.813 = 95.934 + \] + \item L'équation de la droite est + \[ + y = 1.3333333333333333 x -2 + \] + \item Il faut penser à faire la division à par $5$ avant d'utiliser le log car sinon, on ne peut pas utiliser la formule $\log(a^n) = n\times \log(a)$. + + \[x = \frac{\log(0.4)}{\log(0.46)}\] + \end{enumerate} +\end{solution} + +\begin{exercise}[subtitle={Restaurant}] + Un \emph{food truck}, ouvert le midi et le soir, propose deux types de formules : + + \setlength\parindent{10mm} + \begin{itemize} + \item la formule \emph{Burger} ; + \item la formule \emph{Wok}. + \end{itemize} + \setlength\parindent{0mm} + + \medskip + + Le gérant a remarqué que 11\,\% de ses ventes ont lieu le midi. Le quart des ventes du midi correspondent à la formule \emph{Burger}, alors que 18\,\% des ventes du soir correspondent à la formule \emph{Wok}. + + Le gérant se constitue un fichier en notant, pour chaque vente, la formule choisie et le moment de cette vente (midi ou soir). + + On prélève une fiche de façon équiprobable. On définit les quatre évènements suivants: + + \begin{enumerate} + \item $M$ : \og la fiche correspond à une vente du midi\fg{} ; + \item $S$ : \og la fiche correspond à une vente du soir\fg {}; + \item $W$ : \og la fiche correspond à une formule \emph{Wok} \fg{} ; + \item $B$ : \og la fiche correspond à une formule \emph{Burger} \fg. + \end{enumerate} + \setlength\parindent{0mm} + + \medskip + + \begin{enumerate} + \item Recopier puis compléter l'arbre pondéré + + \begin{center} + \begin{tikzpicture}[sloped] + \node {.} + child {node {$M$} + child {node {$W$} + edge from parent + node[above] {...} + } + child {node {$B$} + edge from parent + node[above] {...} + } + edge from parent + node[above] {...} + } + child[missing] {} + child { node {$S$} + child {node {$W$} + edge from parent + node[above] {...} + } + child {node {$B$} + edge from parent + node[above] {...} + } + edge from parent + node[above] {...} + } ; + \end{tikzpicture} + \end{center} + + \item Calculer la probabilité de l'évènement $M \cap W$. Interpréter ce résultat dans le contexte de l'exercice. + \item Montrer que la probabilité que la fiche choisie corresponde à une formule \emph{Burger} est égale à $0.7573$. + \item On a prélevé une fiche correspondant à la formule \emph{Burger}. Quelle est la probabilité, arrondie au millième, que la vente ait eu lieu le soir? + \end{enumerate} +\end{exercise} + +\begin{solution} + \begin{enumerate} + \item + \begin{center} + \begin{tikzpicture}[sloped] + \node {.} + child {node {$M$} + child {node {$W$} + edge from parent + node[above] {$0.75$} + } + child {node {$B$} + edge from parent + node[above] {$0.25$} + } + edge from parent + node[above] {$0.11$} + } + child[missing] {} + child { node {$S$} + child {node {$W$} + edge from parent + node[above] {$0.18$} + } + child {node {$B$} + edge from parent + node[above] {$0.82$} + } + edge from parent + node[above] {$0.89$} + } ; + \end{tikzpicture} + \end{center} + \item On calcule la probabilité que la vente soit un wok et ait eu lieu à midi + \[ P(M\cap W) = P(M) \times P_M(W) = 0.11 \times 0.75 = 0.0825 \] + \item Probabilité que la vente soit un burger. + \[ + P(B) = P(M\cap B) + P(S\cap B) = 0.11 \times 0.75 + 0.89 \times 0.18 = 0.7573 + \] + \item On cherche à calculer la quantité $P_B(S)$. Pour cela on utilise la formule de Bayes + \[ + P_B(S) = \frac{P(B\cap S)}{P(B)} = \frac{P_S(B) \times P(S)}{P(B)} = \frac{0.82\times 0.89}{0.7573} = 0.9636867819886439 \approx 0.964 + \] + \end{enumerate} +\end{solution} + +\begin{exercise}[subtitle={Continent plastique}] + \textit{Les quantités évoqués dans cette exercice sont générés au hasard et sont donc complètement farfelus.} + \medskip + Le \og continent de plastique\fg{} est la plus grande des plaques de déchets plastiques évoluant sur les océans. Elle occupe actuellement dans l'océan Pacifique une surface dont l'aire est évaluée à plus de $1,6$ million de km$^2$, entre Hawaï et la Californie. + + En 2017, des scientifiques ont estimé qu'il y avait $19$ millions de tonnes de déchets plastiques qui était déversé chaque année dans les océans et que cette quantité augmentait de $22\n\%$ par chaque année. + + On modélise l'évolution de la masse de ces déchets plastiques déversée chaque année, si rien n'est fait pour la réduire, par une suite géométrique $\left(u_n\right)$. L'arrondi au centième du terme $u_n$ représente la masse de ces déchets déversée chaque année, exprimée en million de tonnes, pour l'année $(2017 + n)$. + + \medskip + + \begin{enumerate} + \item Expliquer pourquoi la suite $u_n$ est géométrique? + \item Calculer $u_1$ et $u_2$. + \item Exprimer $u_n$ en fonction de $n$. + \item Au début de l'année 2017, il y avait $300$ millions de tonnes de déchets plastique. Calculer la quantité totale de déchets plastiques en 2030. + \item On souhaite déterminer en quelle année la masse totale de ces déchets plastiques aura pour la première fois augmenté de $50$\,\% par rapport à sa valeur de 2017. + \begin{enumerate} + \item Recopier et compléter l'algorithme ci-dessous pour que la variable $N$ contienne la réponse au problème posé. + + \begin{center} + \begin{tabularx}{0.4\linewidth}{|X|}\hline + $N = 2017$\\ + $U = 19$ \\ + $S = 300 + U$ \\ + while $S < 450$: \\ + \hspace{1cm} $N = \ldots$\\ + \hspace{1cm} $U = \ldots$\\ + \hspace{1cm} $S = \ldots$\\ + \hline + \end{tabularx} + \end{center} + \item Que contiennent les variables $S$, $U$ et $N$ après exécution de cet algorithme ? + + Interpréter les résultats dans le contexte de l'exercice. + \end{enumerate} + \end{enumerate} +\end{exercise} + +\begin{solution} + \begin{enumerate} + \item Une augmentation de $22\,\%$ revient à multiplier la quantité par $1.22$. La suite est donc bien géométrique. Son premier terme est $u_0 = 19$ et sa raison est $q = 1.22$ + \item + \[ + u_1 = u_0 * 1.22 = 23.18 + \] + \[ + u_2 = u_0 * 1.22^2 = 28.2796 + \] + \item + \[ + u_n = u_0 \times q^n = 19 \times 1.22^n + \] + \item On calcule la quantité totale déversée entre 2017 et 2030. + \[ + \sum_{n = 0}^{13} u_n = u_0 \times \frac{1-q^{13}}{1-q} = 19 \times \frac{1 - 1.22^{13}}{1 - 1.22} = 1059.17 + \] + On en déduit la quantité totale de déchets en 2030 + \[ + 300 + 1059.17 = 1359.17 + \] + \item + \begin{enumerate} + \item ~ + \begin{center} + \begin{tabularx}{0.4\linewidth}{|X|}\hline + $N \gets 2017$\\ + $U \gets 19$ \\ + $S \gets 300 + U$ \\ + Tant que $S < 450$ \\ + \hspace{1cm} $N \gets N + 1$\\ + \hspace{1cm} $U \gets U * 1.22$\\ + \hspace{1cm} $S \gets S + u$\\ + Fin Tant que\\\hline + \end{tabularx} + \end{center} + \item \textit{Pas de correction automatisé} + \end{enumerate} + \end{enumerate} +\end{solution} + +\end{document} + +%%% Local Variables: +%%% mode: latex +%%% TeX-master: "master" +%%% End: diff --git a/TST/DS/DS_21_04_07/TST3/15_210407_DS8.tex b/TST/DS/DS_21_04_07/TST3/15_210407_DS8.tex new file mode 100644 index 0000000..1e2aa8a --- /dev/null +++ b/TST/DS/DS_21_04_07/TST3/15_210407_DS8.tex @@ -0,0 +1,262 @@ +\documentclass[a4paper,10pt]{article} +\usepackage{myXsim} + +% Title Page +\title{DS8 \hfill RADOUAA Saleh} +\tribe{TST} +\date{\hfillÀ render pour le Mercredi 7 avril} + +\xsimsetup{ + solution/print = false +} + +\begin{document} +\maketitle + +\begin{exercise}[subtitle={Automatismes}] + \textit{Toutes les questions de cette exercice sont indépendantes et peuvent être répondus séparément} + \begin{enumerate} + \item De janvier à septembre, une quantité a augmenté de $27\,\%$. Faire un schéma pour représenter la situation puis calculer le taux d'évolution moyen mensuel. + \item Une quantité augmente de $27\,\%$ par ans. En 2020, elle est de 141\euro. Quelle était sa valeur en 2019? Faire un schéma pour représenter la situation. + \item Déterminer l'équation de la droite \\ + \begin{tikzpicture}[xscale=0.8, yscale=0.5] + \tkzInit[xmin=-5,xmax=5,xstep=1, + ymin=-5,ymax=5,ystep=1] + \tkzGrid + \tkzAxeXY + \tkzFct[domain=-5:5,color=red,very thick]% + {2.0*\x -4}; + \end{tikzpicture} + \item Résoudre l'équation $2 \times 0.88^x = 8$ + \end{enumerate} +\end{exercise} + +\begin{solution} + \begin{enumerate} + \item On veut partager cette évolution en 8 évolutions. + \[ + \left(1 + \frac{27}{100}\right)^{\frac{1}{8}} = 1.0303 + \] + Donc le taux d'évolution moyen est + \[ + t_m = 1.0303 - 1 = 0.030299999999999994 + \] + \item Coefficient multiplicateur pour revenir en arrière + \[ + CM = (1 + \frac{27}{100})^{-1} = 0.7874 + \] + On en déduit la quantité en 2019 + \[ + 141 * 0.7874 = 111.0234 + \] + \item L'équation de la droite est + \[ + y = 2.0 x -4 + \] + \item Il faut penser à faire la division à par $2$ avant d'utiliser le log car sinon, on ne peut pas utiliser la formule $\log(a^n) = n\times \log(a)$. + + \[x = \frac{\log(4.0)}{\log(0.88)}\] + \end{enumerate} +\end{solution} + +\begin{exercise}[subtitle={Restaurant}] + Un \emph{food truck}, ouvert le midi et le soir, propose deux types de formules : + + \setlength\parindent{10mm} + \begin{itemize} + \item la formule \emph{Burger} ; + \item la formule \emph{Wok}. + \end{itemize} + \setlength\parindent{0mm} + + \medskip + + Le gérant a remarqué que 1\,\% de ses ventes ont lieu le midi. Le quart des ventes du midi correspondent à la formule \emph{Burger}, alors que 99\,\% des ventes du soir correspondent à la formule \emph{Wok}. + + Le gérant se constitue un fichier en notant, pour chaque vente, la formule choisie et le moment de cette vente (midi ou soir). + + On prélève une fiche de façon équiprobable. On définit les quatre évènements suivants: + + \begin{enumerate} + \item $M$ : \og la fiche correspond à une vente du midi\fg{} ; + \item $S$ : \og la fiche correspond à une vente du soir\fg {}; + \item $W$ : \og la fiche correspond à une formule \emph{Wok} \fg{} ; + \item $B$ : \og la fiche correspond à une formule \emph{Burger} \fg. + \end{enumerate} + \setlength\parindent{0mm} + + \medskip + + \begin{enumerate} + \item Recopier puis compléter l'arbre pondéré + + \begin{center} + \begin{tikzpicture}[sloped] + \node {.} + child {node {$M$} + child {node {$W$} + edge from parent + node[above] {...} + } + child {node {$B$} + edge from parent + node[above] {...} + } + edge from parent + node[above] {...} + } + child[missing] {} + child { node {$S$} + child {node {$W$} + edge from parent + node[above] {...} + } + child {node {$B$} + edge from parent + node[above] {...} + } + edge from parent + node[above] {...} + } ; + \end{tikzpicture} + \end{center} + + \item Calculer la probabilité de l'évènement $M \cap W$. Interpréter ce résultat dans le contexte de l'exercice. + \item Montrer que la probabilité que la fiche choisie corresponde à une formule \emph{Burger} est égale à $0.0124$. + \item On a prélevé une fiche correspondant à la formule \emph{Burger}. Quelle est la probabilité, arrondie au millième, que la vente ait eu lieu le soir? + \end{enumerate} +\end{exercise} + +\begin{solution} + \begin{enumerate} + \item + \begin{center} + \begin{tikzpicture}[sloped] + \node {.} + child {node {$M$} + child {node {$W$} + edge from parent + node[above] {$0.75$} + } + child {node {$B$} + edge from parent + node[above] {$0.25$} + } + edge from parent + node[above] {$0.01$} + } + child[missing] {} + child { node {$S$} + child {node {$W$} + edge from parent + node[above] {$0.99$} + } + child {node {$B$} + edge from parent + node[above] {$0.01$} + } + edge from parent + node[above] {$0.99$} + } ; + \end{tikzpicture} + \end{center} + \item On calcule la probabilité que la vente soit un wok et ait eu lieu à midi + \[ P(M\cap W) = P(M) \times P_M(W) = 0.01 \times 0.75 = 0.0075 \] + \item Probabilité que la vente soit un burger. + \[ + P(B) = P(M\cap B) + P(S\cap B) = 0.01 \times 0.75 + 0.99 \times 0.99 = 0.0124 + \] + \item On cherche à calculer la quantité $P_B(S)$. Pour cela on utilise la formule de Bayes + \[ + P_B(S) = \frac{P(B\cap S)}{P(B)} = \frac{P_S(B) \times P(S)}{P(B)} = \frac{0.01\times 0.99}{0.0124} = 0.7983870967741936 \approx 0.798 + \] + \end{enumerate} +\end{solution} + +\begin{exercise}[subtitle={Continent plastique}] + \textit{Les quantités évoqués dans cette exercice sont générés au hasard et sont donc complètement farfelus.} + \medskip + Le \og continent de plastique\fg{} est la plus grande des plaques de déchets plastiques évoluant sur les océans. Elle occupe actuellement dans l'océan Pacifique une surface dont l'aire est évaluée à plus de $1,6$ million de km$^2$, entre Hawaï et la Californie. + + En 2017, des scientifiques ont estimé qu'il y avait $5$ millions de tonnes de déchets plastiques qui était déversé chaque année dans les océans et que cette quantité augmentait de $17\n\%$ par chaque année. + + On modélise l'évolution de la masse de ces déchets plastiques déversée chaque année, si rien n'est fait pour la réduire, par une suite géométrique $\left(u_n\right)$. L'arrondi au centième du terme $u_n$ représente la masse de ces déchets déversée chaque année, exprimée en million de tonnes, pour l'année $(2017 + n)$. + + \medskip + + \begin{enumerate} + \item Expliquer pourquoi la suite $u_n$ est géométrique? + \item Calculer $u_1$ et $u_2$. + \item Exprimer $u_n$ en fonction de $n$. + \item Au début de l'année 2017, il y avait $300$ millions de tonnes de déchets plastique. Calculer la quantité totale de déchets plastiques en 2030. + \item On souhaite déterminer en quelle année la masse totale de ces déchets plastiques aura pour la première fois augmenté de $50$\,\% par rapport à sa valeur de 2017. + \begin{enumerate} + \item Recopier et compléter l'algorithme ci-dessous pour que la variable $N$ contienne la réponse au problème posé. + + \begin{center} + \begin{tabularx}{0.4\linewidth}{|X|}\hline + $N = 2017$\\ + $U = 5$ \\ + $S = 300 + U$ \\ + while $S < 450$: \\ + \hspace{1cm} $N = \ldots$\\ + \hspace{1cm} $U = \ldots$\\ + \hspace{1cm} $S = \ldots$\\ + \hline + \end{tabularx} + \end{center} + \item Que contiennent les variables $S$, $U$ et $N$ après exécution de cet algorithme ? + + Interpréter les résultats dans le contexte de l'exercice. + \end{enumerate} + \end{enumerate} +\end{exercise} + +\begin{solution} + \begin{enumerate} + \item Une augmentation de $17\,\%$ revient à multiplier la quantité par $1.17$. La suite est donc bien géométrique. Son premier terme est $u_0 = 5$ et sa raison est $q = 1.17$ + \item + \[ + u_1 = u_0 * 1.17 = 5.85 + \] + \[ + u_2 = u_0 * 1.17^2 = 6.8445 + \] + \item + \[ + u_n = u_0 \times q^n = 5 \times 1.17^n + \] + \item On calcule la quantité totale déversée entre 2017 et 2030. + \[ + \sum_{n = 0}^{13} u_n = u_0 \times \frac{1-q^{13}}{1-q} = 5 \times \frac{1 - 1.17^{13}}{1 - 1.17} = 197.02 + \] + On en déduit la quantité totale de déchets en 2030 + \[ + 300 + 197.02 = 497.02 + \] + \item + \begin{enumerate} + \item ~ + \begin{center} + \begin{tabularx}{0.4\linewidth}{|X|}\hline + $N \gets 2017$\\ + $U \gets 5$ \\ + $S \gets 300 + U$ \\ + Tant que $S < 450$ \\ + \hspace{1cm} $N \gets N + 1$\\ + \hspace{1cm} $U \gets U * 1.17$\\ + \hspace{1cm} $S \gets S + u$\\ + Fin Tant que\\\hline + \end{tabularx} + \end{center} + \item \textit{Pas de correction automatisé} + \end{enumerate} + \end{enumerate} +\end{solution} + +\end{document} + +%%% Local Variables: +%%% mode: latex +%%% TeX-master: "master" +%%% End: diff --git a/TST/DS/DS_21_04_07/TST3/16_210407_DS8.tex b/TST/DS/DS_21_04_07/TST3/16_210407_DS8.tex new file mode 100644 index 0000000..c1b4a00 --- /dev/null +++ b/TST/DS/DS_21_04_07/TST3/16_210407_DS8.tex @@ -0,0 +1,262 @@ +\documentclass[a4paper,10pt]{article} +\usepackage{myXsim} + +% Title Page +\title{DS8 \hfill TAY Ummuhan} +\tribe{TST} +\date{\hfillÀ render pour le Mercredi 7 avril} + +\xsimsetup{ + solution/print = false +} + +\begin{document} +\maketitle + +\begin{exercise}[subtitle={Automatismes}] + \textit{Toutes les questions de cette exercice sont indépendantes et peuvent être répondus séparément} + \begin{enumerate} + \item De janvier à septembre, une quantité a augmenté de $14\,\%$. Faire un schéma pour représenter la situation puis calculer le taux d'évolution moyen mensuel. + \item Une quantité augmente de $14\,\%$ par ans. En 2020, elle est de 116\euro. Quelle était sa valeur en 2019? Faire un schéma pour représenter la situation. + \item Déterminer l'équation de la droite \\ + \begin{tikzpicture}[xscale=0.8, yscale=0.5] + \tkzInit[xmin=-5,xmax=5,xstep=1, + ymin=-5,ymax=5,ystep=1] + \tkzGrid + \tkzAxeXY + \tkzFct[domain=-5:5,color=red,very thick]% + {1.3333333333333333*\x -2}; + \end{tikzpicture} + \item Résoudre l'équation $6 \times 0.14^x = 14$ + \end{enumerate} +\end{exercise} + +\begin{solution} + \begin{enumerate} + \item On veut partager cette évolution en 8 évolutions. + \[ + \left(1 + \frac{14}{100}\right)^{\frac{1}{8}} = 1.0165 + \] + Donc le taux d'évolution moyen est + \[ + t_m = 1.0165 - 1 = 0.01649999999999996 + \] + \item Coefficient multiplicateur pour revenir en arrière + \[ + CM = (1 + \frac{14}{100})^{-1} = 0.8772 + \] + On en déduit la quantité en 2019 + \[ + 116 * 0.8772 = 101.7552 + \] + \item L'équation de la droite est + \[ + y = 1.3333333333333333 x -2 + \] + \item Il faut penser à faire la division à par $6$ avant d'utiliser le log car sinon, on ne peut pas utiliser la formule $\log(a^n) = n\times \log(a)$. + + \[x = \frac{\log(2.33)}{\log(0.14)}\] + \end{enumerate} +\end{solution} + +\begin{exercise}[subtitle={Restaurant}] + Un \emph{food truck}, ouvert le midi et le soir, propose deux types de formules : + + \setlength\parindent{10mm} + \begin{itemize} + \item la formule \emph{Burger} ; + \item la formule \emph{Wok}. + \end{itemize} + \setlength\parindent{0mm} + + \medskip + + Le gérant a remarqué que 28\,\% de ses ventes ont lieu le midi. Le quart des ventes du midi correspondent à la formule \emph{Burger}, alors que 80\,\% des ventes du soir correspondent à la formule \emph{Wok}. + + Le gérant se constitue un fichier en notant, pour chaque vente, la formule choisie et le moment de cette vente (midi ou soir). + + On prélève une fiche de façon équiprobable. On définit les quatre évènements suivants: + + \begin{enumerate} + \item $M$ : \og la fiche correspond à une vente du midi\fg{} ; + \item $S$ : \og la fiche correspond à une vente du soir\fg {}; + \item $W$ : \og la fiche correspond à une formule \emph{Wok} \fg{} ; + \item $B$ : \og la fiche correspond à une formule \emph{Burger} \fg. + \end{enumerate} + \setlength\parindent{0mm} + + \medskip + + \begin{enumerate} + \item Recopier puis compléter l'arbre pondéré + + \begin{center} + \begin{tikzpicture}[sloped] + \node {.} + child {node {$M$} + child {node {$W$} + edge from parent + node[above] {...} + } + child {node {$B$} + edge from parent + node[above] {...} + } + edge from parent + node[above] {...} + } + child[missing] {} + child { node {$S$} + child {node {$W$} + edge from parent + node[above] {...} + } + child {node {$B$} + edge from parent + node[above] {...} + } + edge from parent + node[above] {...} + } ; + \end{tikzpicture} + \end{center} + + \item Calculer la probabilité de l'évènement $M \cap W$. Interpréter ce résultat dans le contexte de l'exercice. + \item Montrer que la probabilité que la fiche choisie corresponde à une formule \emph{Burger} est égale à $0.214$. + \item On a prélevé une fiche correspondant à la formule \emph{Burger}. Quelle est la probabilité, arrondie au millième, que la vente ait eu lieu le soir? + \end{enumerate} +\end{exercise} + +\begin{solution} + \begin{enumerate} + \item + \begin{center} + \begin{tikzpicture}[sloped] + \node {.} + child {node {$M$} + child {node {$W$} + edge from parent + node[above] {$0.75$} + } + child {node {$B$} + edge from parent + node[above] {$0.25$} + } + edge from parent + node[above] {$0.28$} + } + child[missing] {} + child { node {$S$} + child {node {$W$} + edge from parent + node[above] {$0.8$} + } + child {node {$B$} + edge from parent + node[above] {$0.2$} + } + edge from parent + node[above] {$0.72$} + } ; + \end{tikzpicture} + \end{center} + \item On calcule la probabilité que la vente soit un wok et ait eu lieu à midi + \[ P(M\cap W) = P(M) \times P_M(W) = 0.28 \times 0.75 = 0.21 \] + \item Probabilité que la vente soit un burger. + \[ + P(B) = P(M\cap B) + P(S\cap B) = 0.28 \times 0.75 + 0.72 \times 0.8 = 0.214 + \] + \item On cherche à calculer la quantité $P_B(S)$. Pour cela on utilise la formule de Bayes + \[ + P_B(S) = \frac{P(B\cap S)}{P(B)} = \frac{P_S(B) \times P(S)}{P(B)} = \frac{0.2\times 0.72}{0.214} = 0.6728971962616822 \approx 0.673 + \] + \end{enumerate} +\end{solution} + +\begin{exercise}[subtitle={Continent plastique}] + \textit{Les quantités évoqués dans cette exercice sont générés au hasard et sont donc complètement farfelus.} + \medskip + Le \og continent de plastique\fg{} est la plus grande des plaques de déchets plastiques évoluant sur les océans. Elle occupe actuellement dans l'océan Pacifique une surface dont l'aire est évaluée à plus de $1,6$ million de km$^2$, entre Hawaï et la Californie. + + En 2017, des scientifiques ont estimé qu'il y avait $7$ millions de tonnes de déchets plastiques qui était déversé chaque année dans les océans et que cette quantité augmentait de $26\n\%$ par chaque année. + + On modélise l'évolution de la masse de ces déchets plastiques déversée chaque année, si rien n'est fait pour la réduire, par une suite géométrique $\left(u_n\right)$. L'arrondi au centième du terme $u_n$ représente la masse de ces déchets déversée chaque année, exprimée en million de tonnes, pour l'année $(2017 + n)$. + + \medskip + + \begin{enumerate} + \item Expliquer pourquoi la suite $u_n$ est géométrique? + \item Calculer $u_1$ et $u_2$. + \item Exprimer $u_n$ en fonction de $n$. + \item Au début de l'année 2017, il y avait $300$ millions de tonnes de déchets plastique. Calculer la quantité totale de déchets plastiques en 2030. + \item On souhaite déterminer en quelle année la masse totale de ces déchets plastiques aura pour la première fois augmenté de $50$\,\% par rapport à sa valeur de 2017. + \begin{enumerate} + \item Recopier et compléter l'algorithme ci-dessous pour que la variable $N$ contienne la réponse au problème posé. + + \begin{center} + \begin{tabularx}{0.4\linewidth}{|X|}\hline + $N = 2017$\\ + $U = 7$ \\ + $S = 300 + U$ \\ + while $S < 450$: \\ + \hspace{1cm} $N = \ldots$\\ + \hspace{1cm} $U = \ldots$\\ + \hspace{1cm} $S = \ldots$\\ + \hline + \end{tabularx} + \end{center} + \item Que contiennent les variables $S$, $U$ et $N$ après exécution de cet algorithme ? + + Interpréter les résultats dans le contexte de l'exercice. + \end{enumerate} + \end{enumerate} +\end{exercise} + +\begin{solution} + \begin{enumerate} + \item Une augmentation de $26\,\%$ revient à multiplier la quantité par $1.26$. La suite est donc bien géométrique. Son premier terme est $u_0 = 7$ et sa raison est $q = 1.26$ + \item + \[ + u_1 = u_0 * 1.26 = 8.82 + \] + \[ + u_2 = u_0 * 1.26^2 = 11.1132 + \] + \item + \[ + u_n = u_0 \times q^n = 7 \times 1.26^n + \] + \item On calcule la quantité totale déversée entre 2017 et 2030. + \[ + \sum_{n = 0}^{13} u_n = u_0 \times \frac{1-q^{13}}{1-q} = 7 \times \frac{1 - 1.26^{13}}{1 - 1.26} = 516.25 + \] + On en déduit la quantité totale de déchets en 2030 + \[ + 300 + 516.25 = 816.25 + \] + \item + \begin{enumerate} + \item ~ + \begin{center} + \begin{tabularx}{0.4\linewidth}{|X|}\hline + $N \gets 2017$\\ + $U \gets 7$ \\ + $S \gets 300 + U$ \\ + Tant que $S < 450$ \\ + \hspace{1cm} $N \gets N + 1$\\ + \hspace{1cm} $U \gets U * 1.26$\\ + \hspace{1cm} $S \gets S + u$\\ + Fin Tant que\\\hline + \end{tabularx} + \end{center} + \item \textit{Pas de correction automatisé} + \end{enumerate} + \end{enumerate} +\end{solution} + +\end{document} + +%%% Local Variables: +%%% mode: latex +%%% TeX-master: "master" +%%% End: diff --git a/TST/DS/DS_21_04_07/TST3/17_210407_DS8.tex b/TST/DS/DS_21_04_07/TST3/17_210407_DS8.tex new file mode 100644 index 0000000..c684e17 --- /dev/null +++ b/TST/DS/DS_21_04_07/TST3/17_210407_DS8.tex @@ -0,0 +1,262 @@ +\documentclass[a4paper,10pt]{article} +\usepackage{myXsim} + +% Title Page +\title{DS8 \hfill VIALON-DUPERRON Victorien} +\tribe{TST} +\date{\hfillÀ render pour le Mercredi 7 avril} + +\xsimsetup{ + solution/print = false +} + +\begin{document} +\maketitle + +\begin{exercise}[subtitle={Automatismes}] + \textit{Toutes les questions de cette exercice sont indépendantes et peuvent être répondus séparément} + \begin{enumerate} + \item De janvier à septembre, une quantité a augmenté de $20\,\%$. Faire un schéma pour représenter la situation puis calculer le taux d'évolution moyen mensuel. + \item Une quantité augmente de $20\,\%$ par ans. En 2020, elle est de 137\euro. Quelle était sa valeur en 2019? Faire un schéma pour représenter la situation. + \item Déterminer l'équation de la droite \\ + \begin{tikzpicture}[xscale=0.8, yscale=0.5] + \tkzInit[xmin=-5,xmax=5,xstep=1, + ymin=-5,ymax=5,ystep=1] + \tkzGrid + \tkzAxeXY + \tkzFct[domain=-5:5,color=red,very thick]% + {2.6666666666666665*\x -4}; + \end{tikzpicture} + \item Résoudre l'équation $10 \times 0.26^x = 42$ + \end{enumerate} +\end{exercise} + +\begin{solution} + \begin{enumerate} + \item On veut partager cette évolution en 8 évolutions. + \[ + \left(1 + \frac{20}{100}\right)^{\frac{1}{8}} = 1.0231 + \] + Donc le taux d'évolution moyen est + \[ + t_m = 1.0231 - 1 = 0.0230999999999999 + \] + \item Coefficient multiplicateur pour revenir en arrière + \[ + CM = (1 + \frac{20}{100})^{-1} = 0.8333 + \] + On en déduit la quantité en 2019 + \[ + 137 * 0.8333 = 114.16210000000001 + \] + \item L'équation de la droite est + \[ + y = 2.6666666666666665 x -4 + \] + \item Il faut penser à faire la division à par $10$ avant d'utiliser le log car sinon, on ne peut pas utiliser la formule $\log(a^n) = n\times \log(a)$. + + \[x = \frac{\log(4.2)}{\log(0.26)}\] + \end{enumerate} +\end{solution} + +\begin{exercise}[subtitle={Restaurant}] + Un \emph{food truck}, ouvert le midi et le soir, propose deux types de formules : + + \setlength\parindent{10mm} + \begin{itemize} + \item la formule \emph{Burger} ; + \item la formule \emph{Wok}. + \end{itemize} + \setlength\parindent{0mm} + + \medskip + + Le gérant a remarqué que 87\,\% de ses ventes ont lieu le midi. Le quart des ventes du midi correspondent à la formule \emph{Burger}, alors que 47\,\% des ventes du soir correspondent à la formule \emph{Wok}. + + Le gérant se constitue un fichier en notant, pour chaque vente, la formule choisie et le moment de cette vente (midi ou soir). + + On prélève une fiche de façon équiprobable. On définit les quatre évènements suivants: + + \begin{enumerate} + \item $M$ : \og la fiche correspond à une vente du midi\fg{} ; + \item $S$ : \og la fiche correspond à une vente du soir\fg {}; + \item $W$ : \og la fiche correspond à une formule \emph{Wok} \fg{} ; + \item $B$ : \og la fiche correspond à une formule \emph{Burger} \fg. + \end{enumerate} + \setlength\parindent{0mm} + + \medskip + + \begin{enumerate} + \item Recopier puis compléter l'arbre pondéré + + \begin{center} + \begin{tikzpicture}[sloped] + \node {.} + child {node {$M$} + child {node {$W$} + edge from parent + node[above] {...} + } + child {node {$B$} + edge from parent + node[above] {...} + } + edge from parent + node[above] {...} + } + child[missing] {} + child { node {$S$} + child {node {$W$} + edge from parent + node[above] {...} + } + child {node {$B$} + edge from parent + node[above] {...} + } + edge from parent + node[above] {...} + } ; + \end{tikzpicture} + \end{center} + + \item Calculer la probabilité de l'évènement $M \cap W$. Interpréter ce résultat dans le contexte de l'exercice. + \item Montrer que la probabilité que la fiche choisie corresponde à une formule \emph{Burger} est égale à $0.2864$. + \item On a prélevé une fiche correspondant à la formule \emph{Burger}. Quelle est la probabilité, arrondie au millième, que la vente ait eu lieu le soir? + \end{enumerate} +\end{exercise} + +\begin{solution} + \begin{enumerate} + \item + \begin{center} + \begin{tikzpicture}[sloped] + \node {.} + child {node {$M$} + child {node {$W$} + edge from parent + node[above] {$0.75$} + } + child {node {$B$} + edge from parent + node[above] {$0.25$} + } + edge from parent + node[above] {$0.87$} + } + child[missing] {} + child { node {$S$} + child {node {$W$} + edge from parent + node[above] {$0.47$} + } + child {node {$B$} + edge from parent + node[above] {$0.53$} + } + edge from parent + node[above] {$0.13$} + } ; + \end{tikzpicture} + \end{center} + \item On calcule la probabilité que la vente soit un wok et ait eu lieu à midi + \[ P(M\cap W) = P(M) \times P_M(W) = 0.87 \times 0.75 = 0.6525 \] + \item Probabilité que la vente soit un burger. + \[ + P(B) = P(M\cap B) + P(S\cap B) = 0.87 \times 0.75 + 0.13 \times 0.47 = 0.2864 + \] + \item On cherche à calculer la quantité $P_B(S)$. Pour cela on utilise la formule de Bayes + \[ + P_B(S) = \frac{P(B\cap S)}{P(B)} = \frac{P_S(B) \times P(S)}{P(B)} = \frac{0.53\times 0.13}{0.2864} = 0.24057262569832405 \approx 0.241 + \] + \end{enumerate} +\end{solution} + +\begin{exercise}[subtitle={Continent plastique}] + \textit{Les quantités évoqués dans cette exercice sont générés au hasard et sont donc complètement farfelus.} + \medskip + Le \og continent de plastique\fg{} est la plus grande des plaques de déchets plastiques évoluant sur les océans. Elle occupe actuellement dans l'océan Pacifique une surface dont l'aire est évaluée à plus de $1,6$ million de km$^2$, entre Hawaï et la Californie. + + En 2017, des scientifiques ont estimé qu'il y avait $14$ millions de tonnes de déchets plastiques qui était déversé chaque année dans les océans et que cette quantité augmentait de $17\n\%$ par chaque année. + + On modélise l'évolution de la masse de ces déchets plastiques déversée chaque année, si rien n'est fait pour la réduire, par une suite géométrique $\left(u_n\right)$. L'arrondi au centième du terme $u_n$ représente la masse de ces déchets déversée chaque année, exprimée en million de tonnes, pour l'année $(2017 + n)$. + + \medskip + + \begin{enumerate} + \item Expliquer pourquoi la suite $u_n$ est géométrique? + \item Calculer $u_1$ et $u_2$. + \item Exprimer $u_n$ en fonction de $n$. + \item Au début de l'année 2017, il y avait $300$ millions de tonnes de déchets plastique. Calculer la quantité totale de déchets plastiques en 2030. + \item On souhaite déterminer en quelle année la masse totale de ces déchets plastiques aura pour la première fois augmenté de $50$\,\% par rapport à sa valeur de 2017. + \begin{enumerate} + \item Recopier et compléter l'algorithme ci-dessous pour que la variable $N$ contienne la réponse au problème posé. + + \begin{center} + \begin{tabularx}{0.4\linewidth}{|X|}\hline + $N = 2017$\\ + $U = 14$ \\ + $S = 300 + U$ \\ + while $S < 450$: \\ + \hspace{1cm} $N = \ldots$\\ + \hspace{1cm} $U = \ldots$\\ + \hspace{1cm} $S = \ldots$\\ + \hline + \end{tabularx} + \end{center} + \item Que contiennent les variables $S$, $U$ et $N$ après exécution de cet algorithme ? + + Interpréter les résultats dans le contexte de l'exercice. + \end{enumerate} + \end{enumerate} +\end{exercise} + +\begin{solution} + \begin{enumerate} + \item Une augmentation de $17\,\%$ revient à multiplier la quantité par $1.17$. La suite est donc bien géométrique. Son premier terme est $u_0 = 14$ et sa raison est $q = 1.17$ + \item + \[ + u_1 = u_0 * 1.17 = 16.38 + \] + \[ + u_2 = u_0 * 1.17^2 = 19.1646 + \] + \item + \[ + u_n = u_0 \times q^n = 14 \times 1.17^n + \] + \item On calcule la quantité totale déversée entre 2017 et 2030. + \[ + \sum_{n = 0}^{13} u_n = u_0 \times \frac{1-q^{13}}{1-q} = 14 \times \frac{1 - 1.17^{13}}{1 - 1.17} = 551.66 + \] + On en déduit la quantité totale de déchets en 2030 + \[ + 300 + 551.66 = 851.66 + \] + \item + \begin{enumerate} + \item ~ + \begin{center} + \begin{tabularx}{0.4\linewidth}{|X|}\hline + $N \gets 2017$\\ + $U \gets 14$ \\ + $S \gets 300 + U$ \\ + Tant que $S < 450$ \\ + \hspace{1cm} $N \gets N + 1$\\ + \hspace{1cm} $U \gets U * 1.17$\\ + \hspace{1cm} $S \gets S + u$\\ + Fin Tant que\\\hline + \end{tabularx} + \end{center} + \item \textit{Pas de correction automatisé} + \end{enumerate} + \end{enumerate} +\end{solution} + +\end{document} + +%%% Local Variables: +%%% mode: latex +%%% TeX-master: "master" +%%% End: diff --git a/TST/DS/DS_21_04_07/TST3/18_210407_DS8.tex b/TST/DS/DS_21_04_07/TST3/18_210407_DS8.tex new file mode 100644 index 0000000..8da3e4c --- /dev/null +++ b/TST/DS/DS_21_04_07/TST3/18_210407_DS8.tex @@ -0,0 +1,262 @@ +\documentclass[a4paper,10pt]{article} +\usepackage{myXsim} + +% Title Page +\title{DS8 \hfill ZENAGUI Yanis} +\tribe{TST} +\date{\hfillÀ render pour le Mercredi 7 avril} + +\xsimsetup{ + solution/print = false +} + +\begin{document} +\maketitle + +\begin{exercise}[subtitle={Automatismes}] + \textit{Toutes les questions de cette exercice sont indépendantes et peuvent être répondus séparément} + \begin{enumerate} + \item De janvier à septembre, une quantité a augmenté de $29\,\%$. Faire un schéma pour représenter la situation puis calculer le taux d'évolution moyen mensuel. + \item Une quantité augmente de $29\,\%$ par ans. En 2020, elle est de 142\euro. Quelle était sa valeur en 2019? Faire un schéma pour représenter la situation. + \item Déterminer l'équation de la droite \\ + \begin{tikzpicture}[xscale=0.8, yscale=0.5] + \tkzInit[xmin=-5,xmax=5,xstep=1, + ymin=-5,ymax=5,ystep=1] + \tkzGrid + \tkzAxeXY + \tkzFct[domain=-5:5,color=red,very thick]% + {2.0*\x -4}; + \end{tikzpicture} + \item Résoudre l'équation $6 \times 0.41^x = 34$ + \end{enumerate} +\end{exercise} + +\begin{solution} + \begin{enumerate} + \item On veut partager cette évolution en 8 évolutions. + \[ + \left(1 + \frac{29}{100}\right)^{\frac{1}{8}} = 1.0323 + \] + Donc le taux d'évolution moyen est + \[ + t_m = 1.0323 - 1 = 0.032299999999999995 + \] + \item Coefficient multiplicateur pour revenir en arrière + \[ + CM = (1 + \frac{29}{100})^{-1} = 0.7752 + \] + On en déduit la quantité en 2019 + \[ + 142 * 0.7752 = 110.0784 + \] + \item L'équation de la droite est + \[ + y = 2.0 x -4 + \] + \item Il faut penser à faire la division à par $6$ avant d'utiliser le log car sinon, on ne peut pas utiliser la formule $\log(a^n) = n\times \log(a)$. + + \[x = \frac{\log(5.67)}{\log(0.41)}\] + \end{enumerate} +\end{solution} + +\begin{exercise}[subtitle={Restaurant}] + Un \emph{food truck}, ouvert le midi et le soir, propose deux types de formules : + + \setlength\parindent{10mm} + \begin{itemize} + \item la formule \emph{Burger} ; + \item la formule \emph{Wok}. + \end{itemize} + \setlength\parindent{0mm} + + \medskip + + Le gérant a remarqué que 30\,\% de ses ventes ont lieu le midi. Le quart des ventes du midi correspondent à la formule \emph{Burger}, alors que 33\,\% des ventes du soir correspondent à la formule \emph{Wok}. + + Le gérant se constitue un fichier en notant, pour chaque vente, la formule choisie et le moment de cette vente (midi ou soir). + + On prélève une fiche de façon équiprobable. On définit les quatre évènements suivants: + + \begin{enumerate} + \item $M$ : \og la fiche correspond à une vente du midi\fg{} ; + \item $S$ : \og la fiche correspond à une vente du soir\fg {}; + \item $W$ : \og la fiche correspond à une formule \emph{Wok} \fg{} ; + \item $B$ : \og la fiche correspond à une formule \emph{Burger} \fg. + \end{enumerate} + \setlength\parindent{0mm} + + \medskip + + \begin{enumerate} + \item Recopier puis compléter l'arbre pondéré + + \begin{center} + \begin{tikzpicture}[sloped] + \node {.} + child {node {$M$} + child {node {$W$} + edge from parent + node[above] {...} + } + child {node {$B$} + edge from parent + node[above] {...} + } + edge from parent + node[above] {...} + } + child[missing] {} + child { node {$S$} + child {node {$W$} + edge from parent + node[above] {...} + } + child {node {$B$} + edge from parent + node[above] {...} + } + edge from parent + node[above] {...} + } ; + \end{tikzpicture} + \end{center} + + \item Calculer la probabilité de l'évènement $M \cap W$. Interpréter ce résultat dans le contexte de l'exercice. + \item Montrer que la probabilité que la fiche choisie corresponde à une formule \emph{Burger} est égale à $0.544$. + \item On a prélevé une fiche correspondant à la formule \emph{Burger}. Quelle est la probabilité, arrondie au millième, que la vente ait eu lieu le soir? + \end{enumerate} +\end{exercise} + +\begin{solution} + \begin{enumerate} + \item + \begin{center} + \begin{tikzpicture}[sloped] + \node {.} + child {node {$M$} + child {node {$W$} + edge from parent + node[above] {$0.75$} + } + child {node {$B$} + edge from parent + node[above] {$0.25$} + } + edge from parent + node[above] {$0.3$} + } + child[missing] {} + child { node {$S$} + child {node {$W$} + edge from parent + node[above] {$0.33$} + } + child {node {$B$} + edge from parent + node[above] {$0.67$} + } + edge from parent + node[above] {$0.7$} + } ; + \end{tikzpicture} + \end{center} + \item On calcule la probabilité que la vente soit un wok et ait eu lieu à midi + \[ P(M\cap W) = P(M) \times P_M(W) = 0.3 \times 0.75 = 0.225 \] + \item Probabilité que la vente soit un burger. + \[ + P(B) = P(M\cap B) + P(S\cap B) = 0.3 \times 0.75 + 0.7 \times 0.33 = 0.544 + \] + \item On cherche à calculer la quantité $P_B(S)$. Pour cela on utilise la formule de Bayes + \[ + P_B(S) = \frac{P(B\cap S)}{P(B)} = \frac{P_S(B) \times P(S)}{P(B)} = \frac{0.67\times 0.7}{0.544} = 0.8621323529411764 \approx 0.862 + \] + \end{enumerate} +\end{solution} + +\begin{exercise}[subtitle={Continent plastique}] + \textit{Les quantités évoqués dans cette exercice sont générés au hasard et sont donc complètement farfelus.} + \medskip + Le \og continent de plastique\fg{} est la plus grande des plaques de déchets plastiques évoluant sur les océans. Elle occupe actuellement dans l'océan Pacifique une surface dont l'aire est évaluée à plus de $1,6$ million de km$^2$, entre Hawaï et la Californie. + + En 2017, des scientifiques ont estimé qu'il y avait $4$ millions de tonnes de déchets plastiques qui était déversé chaque année dans les océans et que cette quantité augmentait de $18\n\%$ par chaque année. + + On modélise l'évolution de la masse de ces déchets plastiques déversée chaque année, si rien n'est fait pour la réduire, par une suite géométrique $\left(u_n\right)$. L'arrondi au centième du terme $u_n$ représente la masse de ces déchets déversée chaque année, exprimée en million de tonnes, pour l'année $(2017 + n)$. + + \medskip + + \begin{enumerate} + \item Expliquer pourquoi la suite $u_n$ est géométrique? + \item Calculer $u_1$ et $u_2$. + \item Exprimer $u_n$ en fonction de $n$. + \item Au début de l'année 2017, il y avait $300$ millions de tonnes de déchets plastique. Calculer la quantité totale de déchets plastiques en 2030. + \item On souhaite déterminer en quelle année la masse totale de ces déchets plastiques aura pour la première fois augmenté de $50$\,\% par rapport à sa valeur de 2017. + \begin{enumerate} + \item Recopier et compléter l'algorithme ci-dessous pour que la variable $N$ contienne la réponse au problème posé. + + \begin{center} + \begin{tabularx}{0.4\linewidth}{|X|}\hline + $N = 2017$\\ + $U = 4$ \\ + $S = 300 + U$ \\ + while $S < 450$: \\ + \hspace{1cm} $N = \ldots$\\ + \hspace{1cm} $U = \ldots$\\ + \hspace{1cm} $S = \ldots$\\ + \hline + \end{tabularx} + \end{center} + \item Que contiennent les variables $S$, $U$ et $N$ après exécution de cet algorithme ? + + Interpréter les résultats dans le contexte de l'exercice. + \end{enumerate} + \end{enumerate} +\end{exercise} + +\begin{solution} + \begin{enumerate} + \item Une augmentation de $18\,\%$ revient à multiplier la quantité par $1.18$. La suite est donc bien géométrique. Son premier terme est $u_0 = 4$ et sa raison est $q = 1.18$ + \item + \[ + u_1 = u_0 * 1.18 = 4.72 + \] + \[ + u_2 = u_0 * 1.18^2 = 5.5696 + \] + \item + \[ + u_n = u_0 \times q^n = 4 \times 1.18^n + \] + \item On calcule la quantité totale déversée entre 2017 et 2030. + \[ + \sum_{n = 0}^{13} u_n = u_0 \times \frac{1-q^{13}}{1-q} = 4 \times \frac{1 - 1.18^{13}}{1 - 1.18} = 168.87 + \] + On en déduit la quantité totale de déchets en 2030 + \[ + 300 + 168.87 = 468.87 + \] + \item + \begin{enumerate} + \item ~ + \begin{center} + \begin{tabularx}{0.4\linewidth}{|X|}\hline + $N \gets 2017$\\ + $U \gets 4$ \\ + $S \gets 300 + U$ \\ + Tant que $S < 450$ \\ + \hspace{1cm} $N \gets N + 1$\\ + \hspace{1cm} $U \gets U * 1.18$\\ + \hspace{1cm} $S \gets S + u$\\ + Fin Tant que\\\hline + \end{tabularx} + \end{center} + \item \textit{Pas de correction automatisé} + \end{enumerate} + \end{enumerate} +\end{solution} + +\end{document} + +%%% Local Variables: +%%% mode: latex +%%% TeX-master: "master" +%%% End: diff --git a/TST/DS/DS_21_04_07/TST3/19_210407_DS8.tex b/TST/DS/DS_21_04_07/TST3/19_210407_DS8.tex new file mode 100644 index 0000000..4f2a64e --- /dev/null +++ b/TST/DS/DS_21_04_07/TST3/19_210407_DS8.tex @@ -0,0 +1,262 @@ +\documentclass[a4paper,10pt]{article} +\usepackage{myXsim} + +% Title Page +\title{DS8 \hfill VIALON-DUPERRON Victorien} +\tribe{TST} +\date{\hfillÀ render pour le Mercredi 7 avril} + +\xsimsetup{ + solution/print = false +} + +\begin{document} +\maketitle + +\begin{exercise}[subtitle={Automatismes}] + \textit{Toutes les questions de cette exercice sont indépendantes et peuvent être répondus séparément} + \begin{enumerate} + \item De janvier à septembre, une quantité a augmenté de $10\,\%$. Faire un schéma pour représenter la situation puis calculer le taux d'évolution moyen mensuel. + \item Une quantité augmente de $10\,\%$ par ans. En 2020, elle est de 120\euro. Quelle était sa valeur en 2019? Faire un schéma pour représenter la situation. + \item Déterminer l'équation de la droite \\ + \begin{tikzpicture}[xscale=0.8, yscale=0.5] + \tkzInit[xmin=-5,xmax=5,xstep=1, + ymin=-5,ymax=5,ystep=1] + \tkzGrid + \tkzAxeXY + \tkzFct[domain=-5:5,color=red,very thick]% + {2.0*\x -3}; + \end{tikzpicture} + \item Résoudre l'équation $6 \times 0.69^x = 39$ + \end{enumerate} +\end{exercise} + +\begin{solution} + \begin{enumerate} + \item On veut partager cette évolution en 8 évolutions. + \[ + \left(1 + \frac{10}{100}\right)^{\frac{1}{8}} = 1.012 + \] + Donc le taux d'évolution moyen est + \[ + t_m = 1.012 - 1 = 0.01200000000000001 + \] + \item Coefficient multiplicateur pour revenir en arrière + \[ + CM = (1 + \frac{10}{100})^{-1} = 0.9091 + \] + On en déduit la quantité en 2019 + \[ + 120 * 0.9091 = 109.092 + \] + \item L'équation de la droite est + \[ + y = 2.0 x -3 + \] + \item Il faut penser à faire la division à par $6$ avant d'utiliser le log car sinon, on ne peut pas utiliser la formule $\log(a^n) = n\times \log(a)$. + + \[x = \frac{\log(6.5)}{\log(0.69)}\] + \end{enumerate} +\end{solution} + +\begin{exercise}[subtitle={Restaurant}] + Un \emph{food truck}, ouvert le midi et le soir, propose deux types de formules : + + \setlength\parindent{10mm} + \begin{itemize} + \item la formule \emph{Burger} ; + \item la formule \emph{Wok}. + \end{itemize} + \setlength\parindent{0mm} + + \medskip + + Le gérant a remarqué que 43\,\% de ses ventes ont lieu le midi. Le quart des ventes du midi correspondent à la formule \emph{Burger}, alors que 68\,\% des ventes du soir correspondent à la formule \emph{Wok}. + + Le gérant se constitue un fichier en notant, pour chaque vente, la formule choisie et le moment de cette vente (midi ou soir). + + On prélève une fiche de façon équiprobable. On définit les quatre évènements suivants: + + \begin{enumerate} + \item $M$ : \og la fiche correspond à une vente du midi\fg{} ; + \item $S$ : \og la fiche correspond à une vente du soir\fg {}; + \item $W$ : \og la fiche correspond à une formule \emph{Wok} \fg{} ; + \item $B$ : \og la fiche correspond à une formule \emph{Burger} \fg. + \end{enumerate} + \setlength\parindent{0mm} + + \medskip + + \begin{enumerate} + \item Recopier puis compléter l'arbre pondéré + + \begin{center} + \begin{tikzpicture}[sloped] + \node {.} + child {node {$M$} + child {node {$W$} + edge from parent + node[above] {...} + } + child {node {$B$} + edge from parent + node[above] {...} + } + edge from parent + node[above] {...} + } + child[missing] {} + child { node {$S$} + child {node {$W$} + edge from parent + node[above] {...} + } + child {node {$B$} + edge from parent + node[above] {...} + } + edge from parent + node[above] {...} + } ; + \end{tikzpicture} + \end{center} + + \item Calculer la probabilité de l'évènement $M \cap W$. Interpréter ce résultat dans le contexte de l'exercice. + \item Montrer que la probabilité que la fiche choisie corresponde à une formule \emph{Burger} est égale à $0.2899$. + \item On a prélevé une fiche correspondant à la formule \emph{Burger}. Quelle est la probabilité, arrondie au millième, que la vente ait eu lieu le soir? + \end{enumerate} +\end{exercise} + +\begin{solution} + \begin{enumerate} + \item + \begin{center} + \begin{tikzpicture}[sloped] + \node {.} + child {node {$M$} + child {node {$W$} + edge from parent + node[above] {$0.75$} + } + child {node {$B$} + edge from parent + node[above] {$0.25$} + } + edge from parent + node[above] {$0.43$} + } + child[missing] {} + child { node {$S$} + child {node {$W$} + edge from parent + node[above] {$0.68$} + } + child {node {$B$} + edge from parent + node[above] {$0.32$} + } + edge from parent + node[above] {$0.57$} + } ; + \end{tikzpicture} + \end{center} + \item On calcule la probabilité que la vente soit un wok et ait eu lieu à midi + \[ P(M\cap W) = P(M) \times P_M(W) = 0.43 \times 0.75 = 0.3225 \] + \item Probabilité que la vente soit un burger. + \[ + P(B) = P(M\cap B) + P(S\cap B) = 0.43 \times 0.75 + 0.57 \times 0.68 = 0.2899 + \] + \item On cherche à calculer la quantité $P_B(S)$. Pour cela on utilise la formule de Bayes + \[ + P_B(S) = \frac{P(B\cap S)}{P(B)} = \frac{P_S(B) \times P(S)}{P(B)} = \frac{0.32\times 0.57}{0.2899} = 0.6291824767161089 \approx 0.629 + \] + \end{enumerate} +\end{solution} + +\begin{exercise}[subtitle={Continent plastique}] + \textit{Les quantités évoqués dans cette exercice sont générés au hasard et sont donc complètement farfelus.} + \medskip + Le \og continent de plastique\fg{} est la plus grande des plaques de déchets plastiques évoluant sur les océans. Elle occupe actuellement dans l'océan Pacifique une surface dont l'aire est évaluée à plus de $1,6$ million de km$^2$, entre Hawaï et la Californie. + + En 2017, des scientifiques ont estimé qu'il y avait $7$ millions de tonnes de déchets plastiques qui était déversé chaque année dans les océans et que cette quantité augmentait de $11\n\%$ par chaque année. + + On modélise l'évolution de la masse de ces déchets plastiques déversée chaque année, si rien n'est fait pour la réduire, par une suite géométrique $\left(u_n\right)$. L'arrondi au centième du terme $u_n$ représente la masse de ces déchets déversée chaque année, exprimée en million de tonnes, pour l'année $(2017 + n)$. + + \medskip + + \begin{enumerate} + \item Expliquer pourquoi la suite $u_n$ est géométrique? + \item Calculer $u_1$ et $u_2$. + \item Exprimer $u_n$ en fonction de $n$. + \item Au début de l'année 2017, il y avait $300$ millions de tonnes de déchets plastique. Calculer la quantité totale de déchets plastiques en 2030. + \item On souhaite déterminer en quelle année la masse totale de ces déchets plastiques aura pour la première fois augmenté de $50$\,\% par rapport à sa valeur de 2017. + \begin{enumerate} + \item Recopier et compléter l'algorithme ci-dessous pour que la variable $N$ contienne la réponse au problème posé. + + \begin{center} + \begin{tabularx}{0.4\linewidth}{|X|}\hline + $N = 2017$\\ + $U = 7$ \\ + $S = 300 + U$ \\ + while $S < 450$: \\ + \hspace{1cm} $N = \ldots$\\ + \hspace{1cm} $U = \ldots$\\ + \hspace{1cm} $S = \ldots$\\ + \hline + \end{tabularx} + \end{center} + \item Que contiennent les variables $S$, $U$ et $N$ après exécution de cet algorithme ? + + Interpréter les résultats dans le contexte de l'exercice. + \end{enumerate} + \end{enumerate} +\end{exercise} + +\begin{solution} + \begin{enumerate} + \item Une augmentation de $11\,\%$ revient à multiplier la quantité par $1.11$. La suite est donc bien géométrique. Son premier terme est $u_0 = 7$ et sa raison est $q = 1.11$ + \item + \[ + u_1 = u_0 * 1.11 = 7.7700000000000005 + \] + \[ + u_2 = u_0 * 1.11^2 = 8.6247 + \] + \item + \[ + u_n = u_0 \times q^n = 7 \times 1.11^n + \] + \item On calcule la quantité totale déversée entre 2017 et 2030. + \[ + \sum_{n = 0}^{13} u_n = u_0 \times \frac{1-q^{13}}{1-q} = 7 \times \frac{1 - 1.11^{13}}{1 - 1.11} = 183.48 + \] + On en déduit la quantité totale de déchets en 2030 + \[ + 300 + 183.48 = 483.48 + \] + \item + \begin{enumerate} + \item ~ + \begin{center} + \begin{tabularx}{0.4\linewidth}{|X|}\hline + $N \gets 2017$\\ + $U \gets 7$ \\ + $S \gets 300 + U$ \\ + Tant que $S < 450$ \\ + \hspace{1cm} $N \gets N + 1$\\ + \hspace{1cm} $U \gets U * 1.11$\\ + \hspace{1cm} $S \gets S + u$\\ + Fin Tant que\\\hline + \end{tabularx} + \end{center} + \item \textit{Pas de correction automatisé} + \end{enumerate} + \end{enumerate} +\end{solution} + +\end{document} + +%%% Local Variables: +%%% mode: latex +%%% TeX-master: "master" +%%% End: diff --git a/TST/DS/DS_21_04_07/TST3/20_210407_DS8.tex b/TST/DS/DS_21_04_07/TST3/20_210407_DS8.tex new file mode 100644 index 0000000..fb794f4 --- /dev/null +++ b/TST/DS/DS_21_04_07/TST3/20_210407_DS8.tex @@ -0,0 +1,262 @@ +\documentclass[a4paper,10pt]{article} +\usepackage{myXsim} + +% Title Page +\title{DS8 \hfill ZENAGUI Yanis} +\tribe{TST} +\date{\hfillÀ render pour le Mercredi 7 avril} + +\xsimsetup{ + solution/print = false +} + +\begin{document} +\maketitle + +\begin{exercise}[subtitle={Automatismes}] + \textit{Toutes les questions de cette exercice sont indépendantes et peuvent être répondus séparément} + \begin{enumerate} + \item De janvier à septembre, une quantité a augmenté de $23\,\%$. Faire un schéma pour représenter la situation puis calculer le taux d'évolution moyen mensuel. + \item Une quantité augmente de $23\,\%$ par ans. En 2020, elle est de 135\euro. Quelle était sa valeur en 2019? Faire un schéma pour représenter la situation. + \item Déterminer l'équation de la droite \\ + \begin{tikzpicture}[xscale=0.8, yscale=0.5] + \tkzInit[xmin=-5,xmax=5,xstep=1, + ymin=-5,ymax=5,ystep=1] + \tkzGrid + \tkzAxeXY + \tkzFct[domain=-5:5,color=red,very thick]% + {0.5*\x -1}; + \end{tikzpicture} + \item Résoudre l'équation $3 \times 0.55^x = 39$ + \end{enumerate} +\end{exercise} + +\begin{solution} + \begin{enumerate} + \item On veut partager cette évolution en 8 évolutions. + \[ + \left(1 + \frac{23}{100}\right)^{\frac{1}{8}} = 1.0262 + \] + Donc le taux d'évolution moyen est + \[ + t_m = 1.0262 - 1 = 0.0262 + \] + \item Coefficient multiplicateur pour revenir en arrière + \[ + CM = (1 + \frac{23}{100})^{-1} = 0.813 + \] + On en déduit la quantité en 2019 + \[ + 135 * 0.813 = 109.755 + \] + \item L'équation de la droite est + \[ + y = 0.5 x -1 + \] + \item Il faut penser à faire la division à par $3$ avant d'utiliser le log car sinon, on ne peut pas utiliser la formule $\log(a^n) = n\times \log(a)$. + + \[x = \frac{\log(13.0)}{\log(0.55)}\] + \end{enumerate} +\end{solution} + +\begin{exercise}[subtitle={Restaurant}] + Un \emph{food truck}, ouvert le midi et le soir, propose deux types de formules : + + \setlength\parindent{10mm} + \begin{itemize} + \item la formule \emph{Burger} ; + \item la formule \emph{Wok}. + \end{itemize} + \setlength\parindent{0mm} + + \medskip + + Le gérant a remarqué que 9\,\% de ses ventes ont lieu le midi. Le quart des ventes du midi correspondent à la formule \emph{Burger}, alors que 11\,\% des ventes du soir correspondent à la formule \emph{Wok}. + + Le gérant se constitue un fichier en notant, pour chaque vente, la formule choisie et le moment de cette vente (midi ou soir). + + On prélève une fiche de façon équiprobable. On définit les quatre évènements suivants: + + \begin{enumerate} + \item $M$ : \og la fiche correspond à une vente du midi\fg{} ; + \item $S$ : \og la fiche correspond à une vente du soir\fg {}; + \item $W$ : \og la fiche correspond à une formule \emph{Wok} \fg{} ; + \item $B$ : \og la fiche correspond à une formule \emph{Burger} \fg. + \end{enumerate} + \setlength\parindent{0mm} + + \medskip + + \begin{enumerate} + \item Recopier puis compléter l'arbre pondéré + + \begin{center} + \begin{tikzpicture}[sloped] + \node {.} + child {node {$M$} + child {node {$W$} + edge from parent + node[above] {...} + } + child {node {$B$} + edge from parent + node[above] {...} + } + edge from parent + node[above] {...} + } + child[missing] {} + child { node {$S$} + child {node {$W$} + edge from parent + node[above] {...} + } + child {node {$B$} + edge from parent + node[above] {...} + } + edge from parent + node[above] {...} + } ; + \end{tikzpicture} + \end{center} + + \item Calculer la probabilité de l'évènement $M \cap W$. Interpréter ce résultat dans le contexte de l'exercice. + \item Montrer que la probabilité que la fiche choisie corresponde à une formule \emph{Burger} est égale à $0.8324$. + \item On a prélevé une fiche correspondant à la formule \emph{Burger}. Quelle est la probabilité, arrondie au millième, que la vente ait eu lieu le soir? + \end{enumerate} +\end{exercise} + +\begin{solution} + \begin{enumerate} + \item + \begin{center} + \begin{tikzpicture}[sloped] + \node {.} + child {node {$M$} + child {node {$W$} + edge from parent + node[above] {$0.75$} + } + child {node {$B$} + edge from parent + node[above] {$0.25$} + } + edge from parent + node[above] {$0.09$} + } + child[missing] {} + child { node {$S$} + child {node {$W$} + edge from parent + node[above] {$0.11$} + } + child {node {$B$} + edge from parent + node[above] {$0.89$} + } + edge from parent + node[above] {$0.91$} + } ; + \end{tikzpicture} + \end{center} + \item On calcule la probabilité que la vente soit un wok et ait eu lieu à midi + \[ P(M\cap W) = P(M) \times P_M(W) = 0.09 \times 0.75 = 0.0675 \] + \item Probabilité que la vente soit un burger. + \[ + P(B) = P(M\cap B) + P(S\cap B) = 0.09 \times 0.75 + 0.91 \times 0.11 = 0.8324 + \] + \item On cherche à calculer la quantité $P_B(S)$. Pour cela on utilise la formule de Bayes + \[ + P_B(S) = \frac{P(B\cap S)}{P(B)} = \frac{P_S(B) \times P(S)}{P(B)} = \frac{0.89\times 0.91}{0.8324} = 0.9729697260932244 \approx 0.973 + \] + \end{enumerate} +\end{solution} + +\begin{exercise}[subtitle={Continent plastique}] + \textit{Les quantités évoqués dans cette exercice sont générés au hasard et sont donc complètement farfelus.} + \medskip + Le \og continent de plastique\fg{} est la plus grande des plaques de déchets plastiques évoluant sur les océans. Elle occupe actuellement dans l'océan Pacifique une surface dont l'aire est évaluée à plus de $1,6$ million de km$^2$, entre Hawaï et la Californie. + + En 2017, des scientifiques ont estimé qu'il y avait $4$ millions de tonnes de déchets plastiques qui était déversé chaque année dans les océans et que cette quantité augmentait de $19\n\%$ par chaque année. + + On modélise l'évolution de la masse de ces déchets plastiques déversée chaque année, si rien n'est fait pour la réduire, par une suite géométrique $\left(u_n\right)$. L'arrondi au centième du terme $u_n$ représente la masse de ces déchets déversée chaque année, exprimée en million de tonnes, pour l'année $(2017 + n)$. + + \medskip + + \begin{enumerate} + \item Expliquer pourquoi la suite $u_n$ est géométrique? + \item Calculer $u_1$ et $u_2$. + \item Exprimer $u_n$ en fonction de $n$. + \item Au début de l'année 2017, il y avait $300$ millions de tonnes de déchets plastique. Calculer la quantité totale de déchets plastiques en 2030. + \item On souhaite déterminer en quelle année la masse totale de ces déchets plastiques aura pour la première fois augmenté de $50$\,\% par rapport à sa valeur de 2017. + \begin{enumerate} + \item Recopier et compléter l'algorithme ci-dessous pour que la variable $N$ contienne la réponse au problème posé. + + \begin{center} + \begin{tabularx}{0.4\linewidth}{|X|}\hline + $N = 2017$\\ + $U = 4$ \\ + $S = 300 + U$ \\ + while $S < 450$: \\ + \hspace{1cm} $N = \ldots$\\ + \hspace{1cm} $U = \ldots$\\ + \hspace{1cm} $S = \ldots$\\ + \hline + \end{tabularx} + \end{center} + \item Que contiennent les variables $S$, $U$ et $N$ après exécution de cet algorithme ? + + Interpréter les résultats dans le contexte de l'exercice. + \end{enumerate} + \end{enumerate} +\end{exercise} + +\begin{solution} + \begin{enumerate} + \item Une augmentation de $19\,\%$ revient à multiplier la quantité par $1.19$. La suite est donc bien géométrique. Son premier terme est $u_0 = 4$ et sa raison est $q = 1.19$ + \item + \[ + u_1 = u_0 * 1.19 = 4.76 + \] + \[ + u_2 = u_0 * 1.19^2 = 5.6644 + \] + \item + \[ + u_n = u_0 \times q^n = 4 \times 1.19^n + \] + \item On calcule la quantité totale déversée entre 2017 et 2030. + \[ + \sum_{n = 0}^{13} u_n = u_0 \times \frac{1-q^{13}}{1-q} = 4 \times \frac{1 - 1.19^{13}}{1 - 1.19} = 180.98 + \] + On en déduit la quantité totale de déchets en 2030 + \[ + 300 + 180.98 = 480.98 + \] + \item + \begin{enumerate} + \item ~ + \begin{center} + \begin{tabularx}{0.4\linewidth}{|X|}\hline + $N \gets 2017$\\ + $U \gets 4$ \\ + $S \gets 300 + U$ \\ + Tant que $S < 450$ \\ + \hspace{1cm} $N \gets N + 1$\\ + \hspace{1cm} $U \gets U * 1.19$\\ + \hspace{1cm} $S \gets S + u$\\ + Fin Tant que\\\hline + \end{tabularx} + \end{center} + \item \textit{Pas de correction automatisé} + \end{enumerate} + \end{enumerate} +\end{solution} + +\end{document} + +%%% Local Variables: +%%% mode: latex +%%% TeX-master: "master" +%%% End: diff --git a/TST/DS/DS_21_04_07/TST3/all_210407_DS8.pdf b/TST/DS/DS_21_04_07/TST3/all_210407_DS8.pdf new file mode 100644 index 0000000..02d521e Binary files /dev/null and b/TST/DS/DS_21_04_07/TST3/all_210407_DS8.pdf differ diff --git a/TST/DS/DS_21_04_07/TST3/corr_01_210407_DS8.tex b/TST/DS/DS_21_04_07/TST3/corr_01_210407_DS8.tex new file mode 100644 index 0000000..f65ed66 --- /dev/null +++ b/TST/DS/DS_21_04_07/TST3/corr_01_210407_DS8.tex @@ -0,0 +1,262 @@ +\documentclass[a4paper,10pt]{article} +\usepackage{myXsim} + +% Title Page +\title{DS8 \hfill BELARBI Samira} +\tribe{TST} +\date{\hfillÀ render pour le Mercredi 7 avril} + +\xsimsetup{ + solution/print = true +} + +\begin{document} +\maketitle + +\begin{exercise}[subtitle={Automatismes}] + \textit{Toutes les questions de cette exercice sont indépendantes et peuvent être répondus séparément} + \begin{enumerate} + \item De janvier à septembre, une quantité a augmenté de $21\,\%$. Faire un schéma pour représenter la situation puis calculer le taux d'évolution moyen mensuel. + \item Une quantité augmente de $21\,\%$ par ans. En 2020, elle est de 143\euro. Quelle était sa valeur en 2019? Faire un schéma pour représenter la situation. + \item Déterminer l'équation de la droite \\ + \begin{tikzpicture}[xscale=0.8, yscale=0.5] + \tkzInit[xmin=-5,xmax=5,xstep=1, + ymin=-5,ymax=5,ystep=1] + \tkzGrid + \tkzAxeXY + \tkzFct[domain=-5:5,color=red,very thick]% + {2.6666666666666665*\x -4}; + \end{tikzpicture} + \item Résoudre l'équation $8 \times 0.76^x = 26$ + \end{enumerate} +\end{exercise} + +\begin{solution} + \begin{enumerate} + \item On veut partager cette évolution en 8 évolutions. + \[ + \left(1 + \frac{21}{100}\right)^{\frac{1}{8}} = 1.0241 + \] + Donc le taux d'évolution moyen est + \[ + t_m = 1.0241 - 1 = 0.02410000000000001 + \] + \item Coefficient multiplicateur pour revenir en arrière + \[ + CM = (1 + \frac{21}{100})^{-1} = 0.8264 + \] + On en déduit la quantité en 2019 + \[ + 143 * 0.8264 = 118.1752 + \] + \item L'équation de la droite est + \[ + y = 2.6666666666666665 x -4 + \] + \item Il faut penser à faire la division à par $8$ avant d'utiliser le log car sinon, on ne peut pas utiliser la formule $\log(a^n) = n\times \log(a)$. + + \[x = \frac{\log(3.25)}{\log(0.76)}\] + \end{enumerate} +\end{solution} + +\begin{exercise}[subtitle={Restaurant}] + Un \emph{food truck}, ouvert le midi et le soir, propose deux types de formules : + + \setlength\parindent{10mm} + \begin{itemize} + \item la formule \emph{Burger} ; + \item la formule \emph{Wok}. + \end{itemize} + \setlength\parindent{0mm} + + \medskip + + Le gérant a remarqué que 21\,\% de ses ventes ont lieu le midi. Le quart des ventes du midi correspondent à la formule \emph{Burger}, alors que 27\,\% des ventes du soir correspondent à la formule \emph{Wok}. + + Le gérant se constitue un fichier en notant, pour chaque vente, la formule choisie et le moment de cette vente (midi ou soir). + + On prélève une fiche de façon équiprobable. On définit les quatre évènements suivants: + + \begin{enumerate} + \item $M$ : \og la fiche correspond à une vente du midi\fg{} ; + \item $S$ : \og la fiche correspond à une vente du soir\fg {}; + \item $W$ : \og la fiche correspond à une formule \emph{Wok} \fg{} ; + \item $B$ : \og la fiche correspond à une formule \emph{Burger} \fg. + \end{enumerate} + \setlength\parindent{0mm} + + \medskip + + \begin{enumerate} + \item Recopier puis compléter l'arbre pondéré + + \begin{center} + \begin{tikzpicture}[sloped] + \node {.} + child {node {$M$} + child {node {$W$} + edge from parent + node[above] {...} + } + child {node {$B$} + edge from parent + node[above] {...} + } + edge from parent + node[above] {...} + } + child[missing] {} + child { node {$S$} + child {node {$W$} + edge from parent + node[above] {...} + } + child {node {$B$} + edge from parent + node[above] {...} + } + edge from parent + node[above] {...} + } ; + \end{tikzpicture} + \end{center} + + \item Calculer la probabilité de l'évènement $M \cap W$. Interpréter ce résultat dans le contexte de l'exercice. + \item Montrer que la probabilité que la fiche choisie corresponde à une formule \emph{Burger} est égale à $0.6292$. + \item On a prélevé une fiche correspondant à la formule \emph{Burger}. Quelle est la probabilité, arrondie au millième, que la vente ait eu lieu le soir? + \end{enumerate} +\end{exercise} + +\begin{solution} + \begin{enumerate} + \item + \begin{center} + \begin{tikzpicture}[sloped] + \node {.} + child {node {$M$} + child {node {$W$} + edge from parent + node[above] {$0.75$} + } + child {node {$B$} + edge from parent + node[above] {$0.25$} + } + edge from parent + node[above] {$0.21$} + } + child[missing] {} + child { node {$S$} + child {node {$W$} + edge from parent + node[above] {$0.27$} + } + child {node {$B$} + edge from parent + node[above] {$0.73$} + } + edge from parent + node[above] {$0.79$} + } ; + \end{tikzpicture} + \end{center} + \item On calcule la probabilité que la vente soit un wok et ait eu lieu à midi + \[ P(M\cap W) = P(M) \times P_M(W) = 0.21 \times 0.75 = 0.1575 \] + \item Probabilité que la vente soit un burger. + \[ + P(B) = P(M\cap B) + P(S\cap B) = 0.21 \times 0.75 + 0.79 \times 0.27 = 0.6292 + \] + \item On cherche à calculer la quantité $P_B(S)$. Pour cela on utilise la formule de Bayes + \[ + P_B(S) = \frac{P(B\cap S)}{P(B)} = \frac{P_S(B) \times P(S)}{P(B)} = \frac{0.73\times 0.79}{0.6292} = 0.9165607120152575 \approx 0.917 + \] + \end{enumerate} +\end{solution} + +\begin{exercise}[subtitle={Continent plastique}] + \textit{Les quantités évoqués dans cette exercice sont générés au hasard et sont donc complètement farfelus.} + \medskip + Le \og continent de plastique\fg{} est la plus grande des plaques de déchets plastiques évoluant sur les océans. Elle occupe actuellement dans l'océan Pacifique une surface dont l'aire est évaluée à plus de $1,6$ million de km$^2$, entre Hawaï et la Californie. + + En 2017, des scientifiques ont estimé qu'il y avait $20$ millions de tonnes de déchets plastiques qui était déversé chaque année dans les océans et que cette quantité augmentait de $28\n\%$ par chaque année. + + On modélise l'évolution de la masse de ces déchets plastiques déversée chaque année, si rien n'est fait pour la réduire, par une suite géométrique $\left(u_n\right)$. L'arrondi au centième du terme $u_n$ représente la masse de ces déchets déversée chaque année, exprimée en million de tonnes, pour l'année $(2017 + n)$. + + \medskip + + \begin{enumerate} + \item Expliquer pourquoi la suite $u_n$ est géométrique? + \item Calculer $u_1$ et $u_2$. + \item Exprimer $u_n$ en fonction de $n$. + \item Au début de l'année 2017, il y avait $300$ millions de tonnes de déchets plastique. Calculer la quantité totale de déchets plastiques en 2030. + \item On souhaite déterminer en quelle année la masse totale de ces déchets plastiques aura pour la première fois augmenté de $50$\,\% par rapport à sa valeur de 2017. + \begin{enumerate} + \item Recopier et compléter l'algorithme ci-dessous pour que la variable $N$ contienne la réponse au problème posé. + + \begin{center} + \begin{tabularx}{0.4\linewidth}{|X|}\hline + $N = 2017$\\ + $U = 20$ \\ + $S = 300 + U$ \\ + while $S < 450$: \\ + \hspace{1cm} $N = \ldots$\\ + \hspace{1cm} $U = \ldots$\\ + \hspace{1cm} $S = \ldots$\\ + \hline + \end{tabularx} + \end{center} + \item Que contiennent les variables $S$, $U$ et $N$ après exécution de cet algorithme ? + + Interpréter les résultats dans le contexte de l'exercice. + \end{enumerate} + \end{enumerate} +\end{exercise} + +\begin{solution} + \begin{enumerate} + \item Une augmentation de $28\,\%$ revient à multiplier la quantité par $1.28$. La suite est donc bien géométrique. Son premier terme est $u_0 = 20$ et sa raison est $q = 1.28$ + \item + \[ + u_1 = u_0 * 1.28 = 25.6 + \] + \[ + u_2 = u_0 * 1.28^2 = 32.768 + \] + \item + \[ + u_n = u_0 \times q^n = 20 \times 1.28^n + \] + \item On calcule la quantité totale déversée entre 2017 et 2030. + \[ + \sum_{n = 0}^{13} u_n = u_0 \times \frac{1-q^{13}}{1-q} = 20 \times \frac{1 - 1.28^{13}}{1 - 1.28} = 1697.06 + \] + On en déduit la quantité totale de déchets en 2030 + \[ + 300 + 1697.06 = 1997.06 + \] + \item + \begin{enumerate} + \item ~ + \begin{center} + \begin{tabularx}{0.4\linewidth}{|X|}\hline + $N \gets 2017$\\ + $U \gets 20$ \\ + $S \gets 300 + U$ \\ + Tant que $S < 450$ \\ + \hspace{1cm} $N \gets N + 1$\\ + \hspace{1cm} $U \gets U * 1.28$\\ + \hspace{1cm} $S \gets S + u$\\ + Fin Tant que\\\hline + \end{tabularx} + \end{center} + \item \textit{Pas de correction automatisé} + \end{enumerate} + \end{enumerate} +\end{solution} + +\end{document} + +%%% Local Variables: +%%% mode: latex +%%% TeX-master: "master" +%%% End: diff --git a/TST/DS/DS_21_04_07/TST3/corr_02_210407_DS8.tex b/TST/DS/DS_21_04_07/TST3/corr_02_210407_DS8.tex new file mode 100644 index 0000000..f58594f --- /dev/null +++ b/TST/DS/DS_21_04_07/TST3/corr_02_210407_DS8.tex @@ -0,0 +1,262 @@ +\documentclass[a4paper,10pt]{article} +\usepackage{myXsim} + +% Title Page +\title{DS8 \hfill BERTAN Ufuk} +\tribe{TST} +\date{\hfillÀ render pour le Mercredi 7 avril} + +\xsimsetup{ + solution/print = true +} + +\begin{document} +\maketitle + +\begin{exercise}[subtitle={Automatismes}] + \textit{Toutes les questions de cette exercice sont indépendantes et peuvent être répondus séparément} + \begin{enumerate} + \item De janvier à septembre, une quantité a augmenté de $22\,\%$. Faire un schéma pour représenter la situation puis calculer le taux d'évolution moyen mensuel. + \item Une quantité augmente de $22\,\%$ par ans. En 2020, elle est de 134\euro. Quelle était sa valeur en 2019? Faire un schéma pour représenter la situation. + \item Déterminer l'équation de la droite \\ + \begin{tikzpicture}[xscale=0.8, yscale=0.5] + \tkzInit[xmin=-5,xmax=5,xstep=1, + ymin=-5,ymax=5,ystep=1] + \tkzGrid + \tkzAxeXY + \tkzFct[domain=-5:5,color=red,very thick]% + {2.0*\x -2}; + \end{tikzpicture} + \item Résoudre l'équation $5 \times 0.99^x = 21$ + \end{enumerate} +\end{exercise} + +\begin{solution} + \begin{enumerate} + \item On veut partager cette évolution en 8 évolutions. + \[ + \left(1 + \frac{22}{100}\right)^{\frac{1}{8}} = 1.0252 + \] + Donc le taux d'évolution moyen est + \[ + t_m = 1.0252 - 1 = 0.02519999999999989 + \] + \item Coefficient multiplicateur pour revenir en arrière + \[ + CM = (1 + \frac{22}{100})^{-1} = 0.8197 + \] + On en déduit la quantité en 2019 + \[ + 134 * 0.8197 = 109.8398 + \] + \item L'équation de la droite est + \[ + y = 2.0 x -2 + \] + \item Il faut penser à faire la division à par $5$ avant d'utiliser le log car sinon, on ne peut pas utiliser la formule $\log(a^n) = n\times \log(a)$. + + \[x = \frac{\log(4.2)}{\log(0.99)}\] + \end{enumerate} +\end{solution} + +\begin{exercise}[subtitle={Restaurant}] + Un \emph{food truck}, ouvert le midi et le soir, propose deux types de formules : + + \setlength\parindent{10mm} + \begin{itemize} + \item la formule \emph{Burger} ; + \item la formule \emph{Wok}. + \end{itemize} + \setlength\parindent{0mm} + + \medskip + + Le gérant a remarqué que 64\,\% de ses ventes ont lieu le midi. Le quart des ventes du midi correspondent à la formule \emph{Burger}, alors que 100\,\% des ventes du soir correspondent à la formule \emph{Wok}. + + Le gérant se constitue un fichier en notant, pour chaque vente, la formule choisie et le moment de cette vente (midi ou soir). + + On prélève une fiche de façon équiprobable. On définit les quatre évènements suivants: + + \begin{enumerate} + \item $M$ : \og la fiche correspond à une vente du midi\fg{} ; + \item $S$ : \og la fiche correspond à une vente du soir\fg {}; + \item $W$ : \og la fiche correspond à une formule \emph{Wok} \fg{} ; + \item $B$ : \og la fiche correspond à une formule \emph{Burger} \fg. + \end{enumerate} + \setlength\parindent{0mm} + + \medskip + + \begin{enumerate} + \item Recopier puis compléter l'arbre pondéré + + \begin{center} + \begin{tikzpicture}[sloped] + \node {.} + child {node {$M$} + child {node {$W$} + edge from parent + node[above] {...} + } + child {node {$B$} + edge from parent + node[above] {...} + } + edge from parent + node[above] {...} + } + child[missing] {} + child { node {$S$} + child {node {$W$} + edge from parent + node[above] {...} + } + child {node {$B$} + edge from parent + node[above] {...} + } + edge from parent + node[above] {...} + } ; + \end{tikzpicture} + \end{center} + + \item Calculer la probabilité de l'évènement $M \cap W$. Interpréter ce résultat dans le contexte de l'exercice. + \item Montrer que la probabilité que la fiche choisie corresponde à une formule \emph{Burger} est égale à $0.16$. + \item On a prélevé une fiche correspondant à la formule \emph{Burger}. Quelle est la probabilité, arrondie au millième, que la vente ait eu lieu le soir? + \end{enumerate} +\end{exercise} + +\begin{solution} + \begin{enumerate} + \item + \begin{center} + \begin{tikzpicture}[sloped] + \node {.} + child {node {$M$} + child {node {$W$} + edge from parent + node[above] {$0.75$} + } + child {node {$B$} + edge from parent + node[above] {$0.25$} + } + edge from parent + node[above] {$0.64$} + } + child[missing] {} + child { node {$S$} + child {node {$W$} + edge from parent + node[above] {$1.0$} + } + child {node {$B$} + edge from parent + node[above] {$0.0$} + } + edge from parent + node[above] {$0.36$} + } ; + \end{tikzpicture} + \end{center} + \item On calcule la probabilité que la vente soit un wok et ait eu lieu à midi + \[ P(M\cap W) = P(M) \times P_M(W) = 0.64 \times 0.75 = 0.48 \] + \item Probabilité que la vente soit un burger. + \[ + P(B) = P(M\cap B) + P(S\cap B) = 0.64 \times 0.75 + 0.36 \times 1.0 = 0.16 + \] + \item On cherche à calculer la quantité $P_B(S)$. Pour cela on utilise la formule de Bayes + \[ + P_B(S) = \frac{P(B\cap S)}{P(B)} = \frac{P_S(B) \times P(S)}{P(B)} = \frac{0.0\times 0.36}{0.16} = 0.0 \approx 0.0 + \] + \end{enumerate} +\end{solution} + +\begin{exercise}[subtitle={Continent plastique}] + \textit{Les quantités évoqués dans cette exercice sont générés au hasard et sont donc complètement farfelus.} + \medskip + Le \og continent de plastique\fg{} est la plus grande des plaques de déchets plastiques évoluant sur les océans. Elle occupe actuellement dans l'océan Pacifique une surface dont l'aire est évaluée à plus de $1,6$ million de km$^2$, entre Hawaï et la Californie. + + En 2017, des scientifiques ont estimé qu'il y avait $2$ millions de tonnes de déchets plastiques qui était déversé chaque année dans les océans et que cette quantité augmentait de $27\n\%$ par chaque année. + + On modélise l'évolution de la masse de ces déchets plastiques déversée chaque année, si rien n'est fait pour la réduire, par une suite géométrique $\left(u_n\right)$. L'arrondi au centième du terme $u_n$ représente la masse de ces déchets déversée chaque année, exprimée en million de tonnes, pour l'année $(2017 + n)$. + + \medskip + + \begin{enumerate} + \item Expliquer pourquoi la suite $u_n$ est géométrique? + \item Calculer $u_1$ et $u_2$. + \item Exprimer $u_n$ en fonction de $n$. + \item Au début de l'année 2017, il y avait $300$ millions de tonnes de déchets plastique. Calculer la quantité totale de déchets plastiques en 2030. + \item On souhaite déterminer en quelle année la masse totale de ces déchets plastiques aura pour la première fois augmenté de $50$\,\% par rapport à sa valeur de 2017. + \begin{enumerate} + \item Recopier et compléter l'algorithme ci-dessous pour que la variable $N$ contienne la réponse au problème posé. + + \begin{center} + \begin{tabularx}{0.4\linewidth}{|X|}\hline + $N = 2017$\\ + $U = 2$ \\ + $S = 300 + U$ \\ + while $S < 450$: \\ + \hspace{1cm} $N = \ldots$\\ + \hspace{1cm} $U = \ldots$\\ + \hspace{1cm} $S = \ldots$\\ + \hline + \end{tabularx} + \end{center} + \item Que contiennent les variables $S$, $U$ et $N$ après exécution de cet algorithme ? + + Interpréter les résultats dans le contexte de l'exercice. + \end{enumerate} + \end{enumerate} +\end{exercise} + +\begin{solution} + \begin{enumerate} + \item Une augmentation de $27\,\%$ revient à multiplier la quantité par $1.27$. La suite est donc bien géométrique. Son premier terme est $u_0 = 2$ et sa raison est $q = 1.27$ + \item + \[ + u_1 = u_0 * 1.27 = 2.54 + \] + \[ + u_2 = u_0 * 1.27^2 = 3.2258 + \] + \item + \[ + u_n = u_0 \times q^n = 2 \times 1.27^n + \] + \item On calcule la quantité totale déversée entre 2017 et 2030. + \[ + \sum_{n = 0}^{13} u_n = u_0 \times \frac{1-q^{13}}{1-q} = 2 \times \frac{1 - 1.27^{13}}{1 - 1.27} = 158.21 + \] + On en déduit la quantité totale de déchets en 2030 + \[ + 300 + 158.21 = 458.21000000000004 + \] + \item + \begin{enumerate} + \item ~ + \begin{center} + \begin{tabularx}{0.4\linewidth}{|X|}\hline + $N \gets 2017$\\ + $U \gets 2$ \\ + $S \gets 300 + U$ \\ + Tant que $S < 450$ \\ + \hspace{1cm} $N \gets N + 1$\\ + \hspace{1cm} $U \gets U * 1.27$\\ + \hspace{1cm} $S \gets S + u$\\ + Fin Tant que\\\hline + \end{tabularx} + \end{center} + \item \textit{Pas de correction automatisé} + \end{enumerate} + \end{enumerate} +\end{solution} + +\end{document} + +%%% Local Variables: +%%% mode: latex +%%% TeX-master: "master" +%%% End: diff --git a/TST/DS/DS_21_04_07/TST3/corr_03_210407_DS8.tex b/TST/DS/DS_21_04_07/TST3/corr_03_210407_DS8.tex new file mode 100644 index 0000000..7845778 --- /dev/null +++ b/TST/DS/DS_21_04_07/TST3/corr_03_210407_DS8.tex @@ -0,0 +1,262 @@ +\documentclass[a4paper,10pt]{article} +\usepackage{myXsim} + +% Title Page +\title{DS8 \hfill BOUALIA Bilel} +\tribe{TST} +\date{\hfillÀ render pour le Mercredi 7 avril} + +\xsimsetup{ + solution/print = true +} + +\begin{document} +\maketitle + +\begin{exercise}[subtitle={Automatismes}] + \textit{Toutes les questions de cette exercice sont indépendantes et peuvent être répondus séparément} + \begin{enumerate} + \item De janvier à septembre, une quantité a augmenté de $17\,\%$. Faire un schéma pour représenter la situation puis calculer le taux d'évolution moyen mensuel. + \item Une quantité augmente de $17\,\%$ par ans. En 2020, elle est de 135\euro. Quelle était sa valeur en 2019? Faire un schéma pour représenter la situation. + \item Déterminer l'équation de la droite \\ + \begin{tikzpicture}[xscale=0.8, yscale=0.5] + \tkzInit[xmin=-5,xmax=5,xstep=1, + ymin=-5,ymax=5,ystep=1] + \tkzGrid + \tkzAxeXY + \tkzFct[domain=-5:5,color=red,very thick]% + {2.0*\x -4}; + \end{tikzpicture} + \item Résoudre l'équation $3 \times 0.2^x = 2$ + \end{enumerate} +\end{exercise} + +\begin{solution} + \begin{enumerate} + \item On veut partager cette évolution en 8 évolutions. + \[ + \left(1 + \frac{17}{100}\right)^{\frac{1}{8}} = 1.0198 + \] + Donc le taux d'évolution moyen est + \[ + t_m = 1.0198 - 1 = 0.01980000000000004 + \] + \item Coefficient multiplicateur pour revenir en arrière + \[ + CM = (1 + \frac{17}{100})^{-1} = 0.8547 + \] + On en déduit la quantité en 2019 + \[ + 135 * 0.8547 = 115.3845 + \] + \item L'équation de la droite est + \[ + y = 2.0 x -4 + \] + \item Il faut penser à faire la division à par $3$ avant d'utiliser le log car sinon, on ne peut pas utiliser la formule $\log(a^n) = n\times \log(a)$. + + \[x = \frac{\log(0.67)}{\log(0.2)}\] + \end{enumerate} +\end{solution} + +\begin{exercise}[subtitle={Restaurant}] + Un \emph{food truck}, ouvert le midi et le soir, propose deux types de formules : + + \setlength\parindent{10mm} + \begin{itemize} + \item la formule \emph{Burger} ; + \item la formule \emph{Wok}. + \end{itemize} + \setlength\parindent{0mm} + + \medskip + + Le gérant a remarqué que 89\,\% de ses ventes ont lieu le midi. Le quart des ventes du midi correspondent à la formule \emph{Burger}, alors que 12\,\% des ventes du soir correspondent à la formule \emph{Wok}. + + Le gérant se constitue un fichier en notant, pour chaque vente, la formule choisie et le moment de cette vente (midi ou soir). + + On prélève une fiche de façon équiprobable. On définit les quatre évènements suivants: + + \begin{enumerate} + \item $M$ : \og la fiche correspond à une vente du midi\fg{} ; + \item $S$ : \og la fiche correspond à une vente du soir\fg {}; + \item $W$ : \og la fiche correspond à une formule \emph{Wok} \fg{} ; + \item $B$ : \og la fiche correspond à une formule \emph{Burger} \fg. + \end{enumerate} + \setlength\parindent{0mm} + + \medskip + + \begin{enumerate} + \item Recopier puis compléter l'arbre pondéré + + \begin{center} + \begin{tikzpicture}[sloped] + \node {.} + child {node {$M$} + child {node {$W$} + edge from parent + node[above] {...} + } + child {node {$B$} + edge from parent + node[above] {...} + } + edge from parent + node[above] {...} + } + child[missing] {} + child { node {$S$} + child {node {$W$} + edge from parent + node[above] {...} + } + child {node {$B$} + edge from parent + node[above] {...} + } + edge from parent + node[above] {...} + } ; + \end{tikzpicture} + \end{center} + + \item Calculer la probabilité de l'évènement $M \cap W$. Interpréter ce résultat dans le contexte de l'exercice. + \item Montrer que la probabilité que la fiche choisie corresponde à une formule \emph{Burger} est égale à $0.3193$. + \item On a prélevé une fiche correspondant à la formule \emph{Burger}. Quelle est la probabilité, arrondie au millième, que la vente ait eu lieu le soir? + \end{enumerate} +\end{exercise} + +\begin{solution} + \begin{enumerate} + \item + \begin{center} + \begin{tikzpicture}[sloped] + \node {.} + child {node {$M$} + child {node {$W$} + edge from parent + node[above] {$0.75$} + } + child {node {$B$} + edge from parent + node[above] {$0.25$} + } + edge from parent + node[above] {$0.89$} + } + child[missing] {} + child { node {$S$} + child {node {$W$} + edge from parent + node[above] {$0.12$} + } + child {node {$B$} + edge from parent + node[above] {$0.88$} + } + edge from parent + node[above] {$0.11$} + } ; + \end{tikzpicture} + \end{center} + \item On calcule la probabilité que la vente soit un wok et ait eu lieu à midi + \[ P(M\cap W) = P(M) \times P_M(W) = 0.89 \times 0.75 = 0.6675 \] + \item Probabilité que la vente soit un burger. + \[ + P(B) = P(M\cap B) + P(S\cap B) = 0.89 \times 0.75 + 0.11 \times 0.12 = 0.3193 + \] + \item On cherche à calculer la quantité $P_B(S)$. Pour cela on utilise la formule de Bayes + \[ + P_B(S) = \frac{P(B\cap S)}{P(B)} = \frac{P_S(B) \times P(S)}{P(B)} = \frac{0.88\times 0.11}{0.3193} = 0.303163169433135 \approx 0.303 + \] + \end{enumerate} +\end{solution} + +\begin{exercise}[subtitle={Continent plastique}] + \textit{Les quantités évoqués dans cette exercice sont générés au hasard et sont donc complètement farfelus.} + \medskip + Le \og continent de plastique\fg{} est la plus grande des plaques de déchets plastiques évoluant sur les océans. Elle occupe actuellement dans l'océan Pacifique une surface dont l'aire est évaluée à plus de $1,6$ million de km$^2$, entre Hawaï et la Californie. + + En 2017, des scientifiques ont estimé qu'il y avait $10$ millions de tonnes de déchets plastiques qui était déversé chaque année dans les océans et que cette quantité augmentait de $27\n\%$ par chaque année. + + On modélise l'évolution de la masse de ces déchets plastiques déversée chaque année, si rien n'est fait pour la réduire, par une suite géométrique $\left(u_n\right)$. L'arrondi au centième du terme $u_n$ représente la masse de ces déchets déversée chaque année, exprimée en million de tonnes, pour l'année $(2017 + n)$. + + \medskip + + \begin{enumerate} + \item Expliquer pourquoi la suite $u_n$ est géométrique? + \item Calculer $u_1$ et $u_2$. + \item Exprimer $u_n$ en fonction de $n$. + \item Au début de l'année 2017, il y avait $300$ millions de tonnes de déchets plastique. Calculer la quantité totale de déchets plastiques en 2030. + \item On souhaite déterminer en quelle année la masse totale de ces déchets plastiques aura pour la première fois augmenté de $50$\,\% par rapport à sa valeur de 2017. + \begin{enumerate} + \item Recopier et compléter l'algorithme ci-dessous pour que la variable $N$ contienne la réponse au problème posé. + + \begin{center} + \begin{tabularx}{0.4\linewidth}{|X|}\hline + $N = 2017$\\ + $U = 10$ \\ + $S = 300 + U$ \\ + while $S < 450$: \\ + \hspace{1cm} $N = \ldots$\\ + \hspace{1cm} $U = \ldots$\\ + \hspace{1cm} $S = \ldots$\\ + \hline + \end{tabularx} + \end{center} + \item Que contiennent les variables $S$, $U$ et $N$ après exécution de cet algorithme ? + + Interpréter les résultats dans le contexte de l'exercice. + \end{enumerate} + \end{enumerate} +\end{exercise} + +\begin{solution} + \begin{enumerate} + \item Une augmentation de $27\,\%$ revient à multiplier la quantité par $1.27$. La suite est donc bien géométrique. Son premier terme est $u_0 = 10$ et sa raison est $q = 1.27$ + \item + \[ + u_1 = u_0 * 1.27 = 12.7 + \] + \[ + u_2 = u_0 * 1.27^2 = 16.129 + \] + \item + \[ + u_n = u_0 \times q^n = 10 \times 1.27^n + \] + \item On calcule la quantité totale déversée entre 2017 et 2030. + \[ + \sum_{n = 0}^{13} u_n = u_0 \times \frac{1-q^{13}}{1-q} = 10 \times \frac{1 - 1.27^{13}}{1 - 1.27} = 791.07 + \] + On en déduit la quantité totale de déchets en 2030 + \[ + 300 + 791.07 = 1091.0700000000002 + \] + \item + \begin{enumerate} + \item ~ + \begin{center} + \begin{tabularx}{0.4\linewidth}{|X|}\hline + $N \gets 2017$\\ + $U \gets 10$ \\ + $S \gets 300 + U$ \\ + Tant que $S < 450$ \\ + \hspace{1cm} $N \gets N + 1$\\ + \hspace{1cm} $U \gets U * 1.27$\\ + \hspace{1cm} $S \gets S + u$\\ + Fin Tant que\\\hline + \end{tabularx} + \end{center} + \item \textit{Pas de correction automatisé} + \end{enumerate} + \end{enumerate} +\end{solution} + +\end{document} + +%%% Local Variables: +%%% mode: latex +%%% TeX-master: "master" +%%% End: diff --git a/TST/DS/DS_21_04_07/TST3/corr_04_210407_DS8.tex b/TST/DS/DS_21_04_07/TST3/corr_04_210407_DS8.tex new file mode 100644 index 0000000..13a5252 --- /dev/null +++ b/TST/DS/DS_21_04_07/TST3/corr_04_210407_DS8.tex @@ -0,0 +1,262 @@ +\documentclass[a4paper,10pt]{article} +\usepackage{myXsim} + +% Title Page +\title{DS8 \hfill BOUCHOUX Kevin} +\tribe{TST} +\date{\hfillÀ render pour le Mercredi 7 avril} + +\xsimsetup{ + solution/print = true +} + +\begin{document} +\maketitle + +\begin{exercise}[subtitle={Automatismes}] + \textit{Toutes les questions de cette exercice sont indépendantes et peuvent être répondus séparément} + \begin{enumerate} + \item De janvier à septembre, une quantité a augmenté de $10\,\%$. Faire un schéma pour représenter la situation puis calculer le taux d'évolution moyen mensuel. + \item Une quantité augmente de $10\,\%$ par ans. En 2020, elle est de 118\euro. Quelle était sa valeur en 2019? Faire un schéma pour représenter la situation. + \item Déterminer l'équation de la droite \\ + \begin{tikzpicture}[xscale=0.8, yscale=0.5] + \tkzInit[xmin=-5,xmax=5,xstep=1, + ymin=-5,ymax=5,ystep=1] + \tkzGrid + \tkzAxeXY + \tkzFct[domain=-5:5,color=red,very thick]% + {3.0*\x -3}; + \end{tikzpicture} + \item Résoudre l'équation $2 \times 0.93^x = 44$ + \end{enumerate} +\end{exercise} + +\begin{solution} + \begin{enumerate} + \item On veut partager cette évolution en 8 évolutions. + \[ + \left(1 + \frac{10}{100}\right)^{\frac{1}{8}} = 1.012 + \] + Donc le taux d'évolution moyen est + \[ + t_m = 1.012 - 1 = 0.01200000000000001 + \] + \item Coefficient multiplicateur pour revenir en arrière + \[ + CM = (1 + \frac{10}{100})^{-1} = 0.9091 + \] + On en déduit la quantité en 2019 + \[ + 118 * 0.9091 = 107.27380000000001 + \] + \item L'équation de la droite est + \[ + y = 3.0 x -3 + \] + \item Il faut penser à faire la division à par $2$ avant d'utiliser le log car sinon, on ne peut pas utiliser la formule $\log(a^n) = n\times \log(a)$. + + \[x = \frac{\log(22.0)}{\log(0.93)}\] + \end{enumerate} +\end{solution} + +\begin{exercise}[subtitle={Restaurant}] + Un \emph{food truck}, ouvert le midi et le soir, propose deux types de formules : + + \setlength\parindent{10mm} + \begin{itemize} + \item la formule \emph{Burger} ; + \item la formule \emph{Wok}. + \end{itemize} + \setlength\parindent{0mm} + + \medskip + + Le gérant a remarqué que 83\,\% de ses ventes ont lieu le midi. Le quart des ventes du midi correspondent à la formule \emph{Burger}, alors que 26\,\% des ventes du soir correspondent à la formule \emph{Wok}. + + Le gérant se constitue un fichier en notant, pour chaque vente, la formule choisie et le moment de cette vente (midi ou soir). + + On prélève une fiche de façon équiprobable. On définit les quatre évènements suivants: + + \begin{enumerate} + \item $M$ : \og la fiche correspond à une vente du midi\fg{} ; + \item $S$ : \og la fiche correspond à une vente du soir\fg {}; + \item $W$ : \og la fiche correspond à une formule \emph{Wok} \fg{} ; + \item $B$ : \og la fiche correspond à une formule \emph{Burger} \fg. + \end{enumerate} + \setlength\parindent{0mm} + + \medskip + + \begin{enumerate} + \item Recopier puis compléter l'arbre pondéré + + \begin{center} + \begin{tikzpicture}[sloped] + \node {.} + child {node {$M$} + child {node {$W$} + edge from parent + node[above] {...} + } + child {node {$B$} + edge from parent + node[above] {...} + } + edge from parent + node[above] {...} + } + child[missing] {} + child { node {$S$} + child {node {$W$} + edge from parent + node[above] {...} + } + child {node {$B$} + edge from parent + node[above] {...} + } + edge from parent + node[above] {...} + } ; + \end{tikzpicture} + \end{center} + + \item Calculer la probabilité de l'évènement $M \cap W$. Interpréter ce résultat dans le contexte de l'exercice. + \item Montrer que la probabilité que la fiche choisie corresponde à une formule \emph{Burger} est égale à $0.3333$. + \item On a prélevé une fiche correspondant à la formule \emph{Burger}. Quelle est la probabilité, arrondie au millième, que la vente ait eu lieu le soir? + \end{enumerate} +\end{exercise} + +\begin{solution} + \begin{enumerate} + \item + \begin{center} + \begin{tikzpicture}[sloped] + \node {.} + child {node {$M$} + child {node {$W$} + edge from parent + node[above] {$0.75$} + } + child {node {$B$} + edge from parent + node[above] {$0.25$} + } + edge from parent + node[above] {$0.83$} + } + child[missing] {} + child { node {$S$} + child {node {$W$} + edge from parent + node[above] {$0.26$} + } + child {node {$B$} + edge from parent + node[above] {$0.74$} + } + edge from parent + node[above] {$0.17$} + } ; + \end{tikzpicture} + \end{center} + \item On calcule la probabilité que la vente soit un wok et ait eu lieu à midi + \[ P(M\cap W) = P(M) \times P_M(W) = 0.83 \times 0.75 = 0.6225 \] + \item Probabilité que la vente soit un burger. + \[ + P(B) = P(M\cap B) + P(S\cap B) = 0.83 \times 0.75 + 0.17 \times 0.26 = 0.3333 + \] + \item On cherche à calculer la quantité $P_B(S)$. Pour cela on utilise la formule de Bayes + \[ + P_B(S) = \frac{P(B\cap S)}{P(B)} = \frac{P_S(B) \times P(S)}{P(B)} = \frac{0.74\times 0.17}{0.3333} = 0.37743774377437744 \approx 0.377 + \] + \end{enumerate} +\end{solution} + +\begin{exercise}[subtitle={Continent plastique}] + \textit{Les quantités évoqués dans cette exercice sont générés au hasard et sont donc complètement farfelus.} + \medskip + Le \og continent de plastique\fg{} est la plus grande des plaques de déchets plastiques évoluant sur les océans. Elle occupe actuellement dans l'océan Pacifique une surface dont l'aire est évaluée à plus de $1,6$ million de km$^2$, entre Hawaï et la Californie. + + En 2017, des scientifiques ont estimé qu'il y avait $18$ millions de tonnes de déchets plastiques qui était déversé chaque année dans les océans et que cette quantité augmentait de $14\n\%$ par chaque année. + + On modélise l'évolution de la masse de ces déchets plastiques déversée chaque année, si rien n'est fait pour la réduire, par une suite géométrique $\left(u_n\right)$. L'arrondi au centième du terme $u_n$ représente la masse de ces déchets déversée chaque année, exprimée en million de tonnes, pour l'année $(2017 + n)$. + + \medskip + + \begin{enumerate} + \item Expliquer pourquoi la suite $u_n$ est géométrique? + \item Calculer $u_1$ et $u_2$. + \item Exprimer $u_n$ en fonction de $n$. + \item Au début de l'année 2017, il y avait $300$ millions de tonnes de déchets plastique. Calculer la quantité totale de déchets plastiques en 2030. + \item On souhaite déterminer en quelle année la masse totale de ces déchets plastiques aura pour la première fois augmenté de $50$\,\% par rapport à sa valeur de 2017. + \begin{enumerate} + \item Recopier et compléter l'algorithme ci-dessous pour que la variable $N$ contienne la réponse au problème posé. + + \begin{center} + \begin{tabularx}{0.4\linewidth}{|X|}\hline + $N = 2017$\\ + $U = 18$ \\ + $S = 300 + U$ \\ + while $S < 450$: \\ + \hspace{1cm} $N = \ldots$\\ + \hspace{1cm} $U = \ldots$\\ + \hspace{1cm} $S = \ldots$\\ + \hline + \end{tabularx} + \end{center} + \item Que contiennent les variables $S$, $U$ et $N$ après exécution de cet algorithme ? + + Interpréter les résultats dans le contexte de l'exercice. + \end{enumerate} + \end{enumerate} +\end{exercise} + +\begin{solution} + \begin{enumerate} + \item Une augmentation de $14\,\%$ revient à multiplier la quantité par $1.1400000000000001$. La suite est donc bien géométrique. Son premier terme est $u_0 = 18$ et sa raison est $q = 1.1400000000000001$ + \item + \[ + u_1 = u_0 * 1.1400000000000001 = 20.520000000000003 + \] + \[ + u_2 = u_0 * 1.1400000000000001^2 = 23.3928 + \] + \item + \[ + u_n = u_0 \times q^n = 18 \times 1.1400000000000001^n + \] + \item On calcule la quantité totale déversée entre 2017 et 2030. + \[ + \sum_{n = 0}^{13} u_n = u_0 \times \frac{1-q^{13}}{1-q} = 18 \times \frac{1 - 1.1400000000000001^{13}}{1 - 1.1400000000000001} = 577.6 + \] + On en déduit la quantité totale de déchets en 2030 + \[ + 300 + 577.6 = 877.6 + \] + \item + \begin{enumerate} + \item ~ + \begin{center} + \begin{tabularx}{0.4\linewidth}{|X|}\hline + $N \gets 2017$\\ + $U \gets 18$ \\ + $S \gets 300 + U$ \\ + Tant que $S < 450$ \\ + \hspace{1cm} $N \gets N + 1$\\ + \hspace{1cm} $U \gets U * 1.1400000000000001$\\ + \hspace{1cm} $S \gets S + u$\\ + Fin Tant que\\\hline + \end{tabularx} + \end{center} + \item \textit{Pas de correction automatisé} + \end{enumerate} + \end{enumerate} +\end{solution} + +\end{document} + +%%% Local Variables: +%%% mode: latex +%%% TeX-master: "master" +%%% End: diff --git a/TST/DS/DS_21_04_07/TST3/corr_05_210407_DS8.tex b/TST/DS/DS_21_04_07/TST3/corr_05_210407_DS8.tex new file mode 100644 index 0000000..b1e1dab --- /dev/null +++ b/TST/DS/DS_21_04_07/TST3/corr_05_210407_DS8.tex @@ -0,0 +1,262 @@ +\documentclass[a4paper,10pt]{article} +\usepackage{myXsim} + +% Title Page +\title{DS8 \hfill BUDIN Nathan} +\tribe{TST} +\date{\hfillÀ render pour le Mercredi 7 avril} + +\xsimsetup{ + solution/print = true +} + +\begin{document} +\maketitle + +\begin{exercise}[subtitle={Automatismes}] + \textit{Toutes les questions de cette exercice sont indépendantes et peuvent être répondus séparément} + \begin{enumerate} + \item De janvier à septembre, une quantité a augmenté de $16\,\%$. Faire un schéma pour représenter la situation puis calculer le taux d'évolution moyen mensuel. + \item Une quantité augmente de $16\,\%$ par ans. En 2020, elle est de 112\euro. Quelle était sa valeur en 2019? Faire un schéma pour représenter la situation. + \item Déterminer l'équation de la droite \\ + \begin{tikzpicture}[xscale=0.8, yscale=0.5] + \tkzInit[xmin=-5,xmax=5,xstep=1, + ymin=-5,ymax=5,ystep=1] + \tkzGrid + \tkzAxeXY + \tkzFct[domain=-5:5,color=red,very thick]% + {1.0*\x -2}; + \end{tikzpicture} + \item Résoudre l'équation $7 \times 0.22^x = 49$ + \end{enumerate} +\end{exercise} + +\begin{solution} + \begin{enumerate} + \item On veut partager cette évolution en 8 évolutions. + \[ + \left(1 + \frac{16}{100}\right)^{\frac{1}{8}} = 1.0187 + \] + Donc le taux d'évolution moyen est + \[ + t_m = 1.0187 - 1 = 0.01869999999999994 + \] + \item Coefficient multiplicateur pour revenir en arrière + \[ + CM = (1 + \frac{16}{100})^{-1} = 0.8621 + \] + On en déduit la quantité en 2019 + \[ + 112 * 0.8621 = 96.5552 + \] + \item L'équation de la droite est + \[ + y = 1.0 x -2 + \] + \item Il faut penser à faire la division à par $7$ avant d'utiliser le log car sinon, on ne peut pas utiliser la formule $\log(a^n) = n\times \log(a)$. + + \[x = \frac{\log(7.0)}{\log(0.22)}\] + \end{enumerate} +\end{solution} + +\begin{exercise}[subtitle={Restaurant}] + Un \emph{food truck}, ouvert le midi et le soir, propose deux types de formules : + + \setlength\parindent{10mm} + \begin{itemize} + \item la formule \emph{Burger} ; + \item la formule \emph{Wok}. + \end{itemize} + \setlength\parindent{0mm} + + \medskip + + Le gérant a remarqué que 75\,\% de ses ventes ont lieu le midi. Le quart des ventes du midi correspondent à la formule \emph{Burger}, alors que 71\,\% des ventes du soir correspondent à la formule \emph{Wok}. + + Le gérant se constitue un fichier en notant, pour chaque vente, la formule choisie et le moment de cette vente (midi ou soir). + + On prélève une fiche de façon équiprobable. On définit les quatre évènements suivants: + + \begin{enumerate} + \item $M$ : \og la fiche correspond à une vente du midi\fg{} ; + \item $S$ : \og la fiche correspond à une vente du soir\fg {}; + \item $W$ : \og la fiche correspond à une formule \emph{Wok} \fg{} ; + \item $B$ : \og la fiche correspond à une formule \emph{Burger} \fg. + \end{enumerate} + \setlength\parindent{0mm} + + \medskip + + \begin{enumerate} + \item Recopier puis compléter l'arbre pondéré + + \begin{center} + \begin{tikzpicture}[sloped] + \node {.} + child {node {$M$} + child {node {$W$} + edge from parent + node[above] {...} + } + child {node {$B$} + edge from parent + node[above] {...} + } + edge from parent + node[above] {...} + } + child[missing] {} + child { node {$S$} + child {node {$W$} + edge from parent + node[above] {...} + } + child {node {$B$} + edge from parent + node[above] {...} + } + edge from parent + node[above] {...} + } ; + \end{tikzpicture} + \end{center} + + \item Calculer la probabilité de l'évènement $M \cap W$. Interpréter ce résultat dans le contexte de l'exercice. + \item Montrer que la probabilité que la fiche choisie corresponde à une formule \emph{Burger} est égale à $0.26$. + \item On a prélevé une fiche correspondant à la formule \emph{Burger}. Quelle est la probabilité, arrondie au millième, que la vente ait eu lieu le soir? + \end{enumerate} +\end{exercise} + +\begin{solution} + \begin{enumerate} + \item + \begin{center} + \begin{tikzpicture}[sloped] + \node {.} + child {node {$M$} + child {node {$W$} + edge from parent + node[above] {$0.75$} + } + child {node {$B$} + edge from parent + node[above] {$0.25$} + } + edge from parent + node[above] {$0.75$} + } + child[missing] {} + child { node {$S$} + child {node {$W$} + edge from parent + node[above] {$0.71$} + } + child {node {$B$} + edge from parent + node[above] {$0.29$} + } + edge from parent + node[above] {$0.25$} + } ; + \end{tikzpicture} + \end{center} + \item On calcule la probabilité que la vente soit un wok et ait eu lieu à midi + \[ P(M\cap W) = P(M) \times P_M(W) = 0.75 \times 0.75 = 0.5625 \] + \item Probabilité que la vente soit un burger. + \[ + P(B) = P(M\cap B) + P(S\cap B) = 0.75 \times 0.75 + 0.25 \times 0.71 = 0.26 + \] + \item On cherche à calculer la quantité $P_B(S)$. Pour cela on utilise la formule de Bayes + \[ + P_B(S) = \frac{P(B\cap S)}{P(B)} = \frac{P_S(B) \times P(S)}{P(B)} = \frac{0.29\times 0.25}{0.26} = 0.2788461538461538 \approx 0.279 + \] + \end{enumerate} +\end{solution} + +\begin{exercise}[subtitle={Continent plastique}] + \textit{Les quantités évoqués dans cette exercice sont générés au hasard et sont donc complètement farfelus.} + \medskip + Le \og continent de plastique\fg{} est la plus grande des plaques de déchets plastiques évoluant sur les océans. Elle occupe actuellement dans l'océan Pacifique une surface dont l'aire est évaluée à plus de $1,6$ million de km$^2$, entre Hawaï et la Californie. + + En 2017, des scientifiques ont estimé qu'il y avait $12$ millions de tonnes de déchets plastiques qui était déversé chaque année dans les océans et que cette quantité augmentait de $25\n\%$ par chaque année. + + On modélise l'évolution de la masse de ces déchets plastiques déversée chaque année, si rien n'est fait pour la réduire, par une suite géométrique $\left(u_n\right)$. L'arrondi au centième du terme $u_n$ représente la masse de ces déchets déversée chaque année, exprimée en million de tonnes, pour l'année $(2017 + n)$. + + \medskip + + \begin{enumerate} + \item Expliquer pourquoi la suite $u_n$ est géométrique? + \item Calculer $u_1$ et $u_2$. + \item Exprimer $u_n$ en fonction de $n$. + \item Au début de l'année 2017, il y avait $300$ millions de tonnes de déchets plastique. Calculer la quantité totale de déchets plastiques en 2030. + \item On souhaite déterminer en quelle année la masse totale de ces déchets plastiques aura pour la première fois augmenté de $50$\,\% par rapport à sa valeur de 2017. + \begin{enumerate} + \item Recopier et compléter l'algorithme ci-dessous pour que la variable $N$ contienne la réponse au problème posé. + + \begin{center} + \begin{tabularx}{0.4\linewidth}{|X|}\hline + $N = 2017$\\ + $U = 12$ \\ + $S = 300 + U$ \\ + while $S < 450$: \\ + \hspace{1cm} $N = \ldots$\\ + \hspace{1cm} $U = \ldots$\\ + \hspace{1cm} $S = \ldots$\\ + \hline + \end{tabularx} + \end{center} + \item Que contiennent les variables $S$, $U$ et $N$ après exécution de cet algorithme ? + + Interpréter les résultats dans le contexte de l'exercice. + \end{enumerate} + \end{enumerate} +\end{exercise} + +\begin{solution} + \begin{enumerate} + \item Une augmentation de $25\,\%$ revient à multiplier la quantité par $1.25$. La suite est donc bien géométrique. Son premier terme est $u_0 = 12$ et sa raison est $q = 1.25$ + \item + \[ + u_1 = u_0 * 1.25 = 15.0 + \] + \[ + u_2 = u_0 * 1.25^2 = 18.75 + \] + \item + \[ + u_n = u_0 \times q^n = 12 \times 1.25^n + \] + \item On calcule la quantité totale déversée entre 2017 et 2030. + \[ + \sum_{n = 0}^{13} u_n = u_0 \times \frac{1-q^{13}}{1-q} = 12 \times \frac{1 - 1.25^{13}}{1 - 1.25} = 825.11 + \] + On en déduit la quantité totale de déchets en 2030 + \[ + 300 + 825.11 = 1125.1100000000001 + \] + \item + \begin{enumerate} + \item ~ + \begin{center} + \begin{tabularx}{0.4\linewidth}{|X|}\hline + $N \gets 2017$\\ + $U \gets 12$ \\ + $S \gets 300 + U$ \\ + Tant que $S < 450$ \\ + \hspace{1cm} $N \gets N + 1$\\ + \hspace{1cm} $U \gets U * 1.25$\\ + \hspace{1cm} $S \gets S + u$\\ + Fin Tant que\\\hline + \end{tabularx} + \end{center} + \item \textit{Pas de correction automatisé} + \end{enumerate} + \end{enumerate} +\end{solution} + +\end{document} + +%%% Local Variables: +%%% mode: latex +%%% TeX-master: "master" +%%% End: diff --git a/TST/DS/DS_21_04_07/TST3/corr_06_210407_DS8.tex b/TST/DS/DS_21_04_07/TST3/corr_06_210407_DS8.tex new file mode 100644 index 0000000..1bb21d3 --- /dev/null +++ b/TST/DS/DS_21_04_07/TST3/corr_06_210407_DS8.tex @@ -0,0 +1,262 @@ +\documentclass[a4paper,10pt]{article} +\usepackage{myXsim} + +% Title Page +\title{DS8 \hfill DARICHE Kaïs} +\tribe{TST} +\date{\hfillÀ render pour le Mercredi 7 avril} + +\xsimsetup{ + solution/print = true +} + +\begin{document} +\maketitle + +\begin{exercise}[subtitle={Automatismes}] + \textit{Toutes les questions de cette exercice sont indépendantes et peuvent être répondus séparément} + \begin{enumerate} + \item De janvier à septembre, une quantité a augmenté de $27\,\%$. Faire un schéma pour représenter la situation puis calculer le taux d'évolution moyen mensuel. + \item Une quantité augmente de $27\,\%$ par ans. En 2020, elle est de 143\euro. Quelle était sa valeur en 2019? Faire un schéma pour représenter la situation. + \item Déterminer l'équation de la droite \\ + \begin{tikzpicture}[xscale=0.8, yscale=0.5] + \tkzInit[xmin=-5,xmax=5,xstep=1, + ymin=-5,ymax=5,ystep=1] + \tkzGrid + \tkzAxeXY + \tkzFct[domain=-5:5,color=red,very thick]% + {3.0*\x -3}; + \end{tikzpicture} + \item Résoudre l'équation $7 \times 0.19^x = 21$ + \end{enumerate} +\end{exercise} + +\begin{solution} + \begin{enumerate} + \item On veut partager cette évolution en 8 évolutions. + \[ + \left(1 + \frac{27}{100}\right)^{\frac{1}{8}} = 1.0303 + \] + Donc le taux d'évolution moyen est + \[ + t_m = 1.0303 - 1 = 0.030299999999999994 + \] + \item Coefficient multiplicateur pour revenir en arrière + \[ + CM = (1 + \frac{27}{100})^{-1} = 0.7874 + \] + On en déduit la quantité en 2019 + \[ + 143 * 0.7874 = 112.59819999999999 + \] + \item L'équation de la droite est + \[ + y = 3.0 x -3 + \] + \item Il faut penser à faire la division à par $7$ avant d'utiliser le log car sinon, on ne peut pas utiliser la formule $\log(a^n) = n\times \log(a)$. + + \[x = \frac{\log(3.0)}{\log(0.19)}\] + \end{enumerate} +\end{solution} + +\begin{exercise}[subtitle={Restaurant}] + Un \emph{food truck}, ouvert le midi et le soir, propose deux types de formules : + + \setlength\parindent{10mm} + \begin{itemize} + \item la formule \emph{Burger} ; + \item la formule \emph{Wok}. + \end{itemize} + \setlength\parindent{0mm} + + \medskip + + Le gérant a remarqué que 15\,\% de ses ventes ont lieu le midi. Le quart des ventes du midi correspondent à la formule \emph{Burger}, alors que 13\,\% des ventes du soir correspondent à la formule \emph{Wok}. + + Le gérant se constitue un fichier en notant, pour chaque vente, la formule choisie et le moment de cette vente (midi ou soir). + + On prélève une fiche de façon équiprobable. On définit les quatre évènements suivants: + + \begin{enumerate} + \item $M$ : \og la fiche correspond à une vente du midi\fg{} ; + \item $S$ : \og la fiche correspond à une vente du soir\fg {}; + \item $W$ : \og la fiche correspond à une formule \emph{Wok} \fg{} ; + \item $B$ : \og la fiche correspond à une formule \emph{Burger} \fg. + \end{enumerate} + \setlength\parindent{0mm} + + \medskip + + \begin{enumerate} + \item Recopier puis compléter l'arbre pondéré + + \begin{center} + \begin{tikzpicture}[sloped] + \node {.} + child {node {$M$} + child {node {$W$} + edge from parent + node[above] {...} + } + child {node {$B$} + edge from parent + node[above] {...} + } + edge from parent + node[above] {...} + } + child[missing] {} + child { node {$S$} + child {node {$W$} + edge from parent + node[above] {...} + } + child {node {$B$} + edge from parent + node[above] {...} + } + edge from parent + node[above] {...} + } ; + \end{tikzpicture} + \end{center} + + \item Calculer la probabilité de l'évènement $M \cap W$. Interpréter ce résultat dans le contexte de l'exercice. + \item Montrer que la probabilité que la fiche choisie corresponde à une formule \emph{Burger} est égale à $0.777$. + \item On a prélevé une fiche correspondant à la formule \emph{Burger}. Quelle est la probabilité, arrondie au millième, que la vente ait eu lieu le soir? + \end{enumerate} +\end{exercise} + +\begin{solution} + \begin{enumerate} + \item + \begin{center} + \begin{tikzpicture}[sloped] + \node {.} + child {node {$M$} + child {node {$W$} + edge from parent + node[above] {$0.75$} + } + child {node {$B$} + edge from parent + node[above] {$0.25$} + } + edge from parent + node[above] {$0.15$} + } + child[missing] {} + child { node {$S$} + child {node {$W$} + edge from parent + node[above] {$0.13$} + } + child {node {$B$} + edge from parent + node[above] {$0.87$} + } + edge from parent + node[above] {$0.85$} + } ; + \end{tikzpicture} + \end{center} + \item On calcule la probabilité que la vente soit un wok et ait eu lieu à midi + \[ P(M\cap W) = P(M) \times P_M(W) = 0.15 \times 0.75 = 0.1125 \] + \item Probabilité que la vente soit un burger. + \[ + P(B) = P(M\cap B) + P(S\cap B) = 0.15 \times 0.75 + 0.85 \times 0.13 = 0.777 + \] + \item On cherche à calculer la quantité $P_B(S)$. Pour cela on utilise la formule de Bayes + \[ + P_B(S) = \frac{P(B\cap S)}{P(B)} = \frac{P_S(B) \times P(S)}{P(B)} = \frac{0.87\times 0.85}{0.777} = 0.9517374517374516 \approx 0.952 + \] + \end{enumerate} +\end{solution} + +\begin{exercise}[subtitle={Continent plastique}] + \textit{Les quantités évoqués dans cette exercice sont générés au hasard et sont donc complètement farfelus.} + \medskip + Le \og continent de plastique\fg{} est la plus grande des plaques de déchets plastiques évoluant sur les océans. Elle occupe actuellement dans l'océan Pacifique une surface dont l'aire est évaluée à plus de $1,6$ million de km$^2$, entre Hawaï et la Californie. + + En 2017, des scientifiques ont estimé qu'il y avait $2$ millions de tonnes de déchets plastiques qui était déversé chaque année dans les océans et que cette quantité augmentait de $30\n\%$ par chaque année. + + On modélise l'évolution de la masse de ces déchets plastiques déversée chaque année, si rien n'est fait pour la réduire, par une suite géométrique $\left(u_n\right)$. L'arrondi au centième du terme $u_n$ représente la masse de ces déchets déversée chaque année, exprimée en million de tonnes, pour l'année $(2017 + n)$. + + \medskip + + \begin{enumerate} + \item Expliquer pourquoi la suite $u_n$ est géométrique? + \item Calculer $u_1$ et $u_2$. + \item Exprimer $u_n$ en fonction de $n$. + \item Au début de l'année 2017, il y avait $300$ millions de tonnes de déchets plastique. Calculer la quantité totale de déchets plastiques en 2030. + \item On souhaite déterminer en quelle année la masse totale de ces déchets plastiques aura pour la première fois augmenté de $50$\,\% par rapport à sa valeur de 2017. + \begin{enumerate} + \item Recopier et compléter l'algorithme ci-dessous pour que la variable $N$ contienne la réponse au problème posé. + + \begin{center} + \begin{tabularx}{0.4\linewidth}{|X|}\hline + $N = 2017$\\ + $U = 2$ \\ + $S = 300 + U$ \\ + while $S < 450$: \\ + \hspace{1cm} $N = \ldots$\\ + \hspace{1cm} $U = \ldots$\\ + \hspace{1cm} $S = \ldots$\\ + \hline + \end{tabularx} + \end{center} + \item Que contiennent les variables $S$, $U$ et $N$ après exécution de cet algorithme ? + + Interpréter les résultats dans le contexte de l'exercice. + \end{enumerate} + \end{enumerate} +\end{exercise} + +\begin{solution} + \begin{enumerate} + \item Une augmentation de $30\,\%$ revient à multiplier la quantité par $1.3$. La suite est donc bien géométrique. Son premier terme est $u_0 = 2$ et sa raison est $q = 1.3$ + \item + \[ + u_1 = u_0 * 1.3 = 2.6 + \] + \[ + u_2 = u_0 * 1.3^2 = 3.38 + \] + \item + \[ + u_n = u_0 \times q^n = 2 \times 1.3^n + \] + \item On calcule la quantité totale déversée entre 2017 et 2030. + \[ + \sum_{n = 0}^{13} u_n = u_0 \times \frac{1-q^{13}}{1-q} = 2 \times \frac{1 - 1.3^{13}}{1 - 1.3} = 195.25 + \] + On en déduit la quantité totale de déchets en 2030 + \[ + 300 + 195.25 = 495.25 + \] + \item + \begin{enumerate} + \item ~ + \begin{center} + \begin{tabularx}{0.4\linewidth}{|X|}\hline + $N \gets 2017$\\ + $U \gets 2$ \\ + $S \gets 300 + U$ \\ + Tant que $S < 450$ \\ + \hspace{1cm} $N \gets N + 1$\\ + \hspace{1cm} $U \gets U * 1.3$\\ + \hspace{1cm} $S \gets S + u$\\ + Fin Tant que\\\hline + \end{tabularx} + \end{center} + \item \textit{Pas de correction automatisé} + \end{enumerate} + \end{enumerate} +\end{solution} + +\end{document} + +%%% Local Variables: +%%% mode: latex +%%% TeX-master: "master" +%%% End: diff --git a/TST/DS/DS_21_04_07/TST3/corr_07_210407_DS8.tex b/TST/DS/DS_21_04_07/TST3/corr_07_210407_DS8.tex new file mode 100644 index 0000000..53c2b41 --- /dev/null +++ b/TST/DS/DS_21_04_07/TST3/corr_07_210407_DS8.tex @@ -0,0 +1,262 @@ +\documentclass[a4paper,10pt]{article} +\usepackage{myXsim} + +% Title Page +\title{DS8 \hfill DEBRAS Noémie} +\tribe{TST} +\date{\hfillÀ render pour le Mercredi 7 avril} + +\xsimsetup{ + solution/print = true +} + +\begin{document} +\maketitle + +\begin{exercise}[subtitle={Automatismes}] + \textit{Toutes les questions de cette exercice sont indépendantes et peuvent être répondus séparément} + \begin{enumerate} + \item De janvier à septembre, une quantité a augmenté de $24\,\%$. Faire un schéma pour représenter la situation puis calculer le taux d'évolution moyen mensuel. + \item Une quantité augmente de $24\,\%$ par ans. En 2020, elle est de 110\euro. Quelle était sa valeur en 2019? Faire un schéma pour représenter la situation. + \item Déterminer l'équation de la droite \\ + \begin{tikzpicture}[xscale=0.8, yscale=0.5] + \tkzInit[xmin=-5,xmax=5,xstep=1, + ymin=-5,ymax=5,ystep=1] + \tkzGrid + \tkzAxeXY + \tkzFct[domain=-5:5,color=red,very thick]% + {0.6666666666666666*\x -1}; + \end{tikzpicture} + \item Résoudre l'équation $8 \times 0.49^x = 9$ + \end{enumerate} +\end{exercise} + +\begin{solution} + \begin{enumerate} + \item On veut partager cette évolution en 8 évolutions. + \[ + \left(1 + \frac{24}{100}\right)^{\frac{1}{8}} = 1.0273 + \] + Donc le taux d'évolution moyen est + \[ + t_m = 1.0273 - 1 = 0.027300000000000102 + \] + \item Coefficient multiplicateur pour revenir en arrière + \[ + CM = (1 + \frac{24}{100})^{-1} = 0.8065 + \] + On en déduit la quantité en 2019 + \[ + 110 * 0.8065 = 88.715 + \] + \item L'équation de la droite est + \[ + y = 0.6666666666666666 x -1 + \] + \item Il faut penser à faire la division à par $8$ avant d'utiliser le log car sinon, on ne peut pas utiliser la formule $\log(a^n) = n\times \log(a)$. + + \[x = \frac{\log(1.12)}{\log(0.49)}\] + \end{enumerate} +\end{solution} + +\begin{exercise}[subtitle={Restaurant}] + Un \emph{food truck}, ouvert le midi et le soir, propose deux types de formules : + + \setlength\parindent{10mm} + \begin{itemize} + \item la formule \emph{Burger} ; + \item la formule \emph{Wok}. + \end{itemize} + \setlength\parindent{0mm} + + \medskip + + Le gérant a remarqué que 49\,\% de ses ventes ont lieu le midi. Le quart des ventes du midi correspondent à la formule \emph{Burger}, alors que 61\,\% des ventes du soir correspondent à la formule \emph{Wok}. + + Le gérant se constitue un fichier en notant, pour chaque vente, la formule choisie et le moment de cette vente (midi ou soir). + + On prélève une fiche de façon équiprobable. On définit les quatre évènements suivants: + + \begin{enumerate} + \item $M$ : \og la fiche correspond à une vente du midi\fg{} ; + \item $S$ : \og la fiche correspond à une vente du soir\fg {}; + \item $W$ : \og la fiche correspond à une formule \emph{Wok} \fg{} ; + \item $B$ : \og la fiche correspond à une formule \emph{Burger} \fg. + \end{enumerate} + \setlength\parindent{0mm} + + \medskip + + \begin{enumerate} + \item Recopier puis compléter l'arbre pondéré + + \begin{center} + \begin{tikzpicture}[sloped] + \node {.} + child {node {$M$} + child {node {$W$} + edge from parent + node[above] {...} + } + child {node {$B$} + edge from parent + node[above] {...} + } + edge from parent + node[above] {...} + } + child[missing] {} + child { node {$S$} + child {node {$W$} + edge from parent + node[above] {...} + } + child {node {$B$} + edge from parent + node[above] {...} + } + edge from parent + node[above] {...} + } ; + \end{tikzpicture} + \end{center} + + \item Calculer la probabilité de l'évènement $M \cap W$. Interpréter ce résultat dans le contexte de l'exercice. + \item Montrer que la probabilité que la fiche choisie corresponde à une formule \emph{Burger} est égale à $0.3214$. + \item On a prélevé une fiche correspondant à la formule \emph{Burger}. Quelle est la probabilité, arrondie au millième, que la vente ait eu lieu le soir? + \end{enumerate} +\end{exercise} + +\begin{solution} + \begin{enumerate} + \item + \begin{center} + \begin{tikzpicture}[sloped] + \node {.} + child {node {$M$} + child {node {$W$} + edge from parent + node[above] {$0.75$} + } + child {node {$B$} + edge from parent + node[above] {$0.25$} + } + edge from parent + node[above] {$0.49$} + } + child[missing] {} + child { node {$S$} + child {node {$W$} + edge from parent + node[above] {$0.61$} + } + child {node {$B$} + edge from parent + node[above] {$0.39$} + } + edge from parent + node[above] {$0.51$} + } ; + \end{tikzpicture} + \end{center} + \item On calcule la probabilité que la vente soit un wok et ait eu lieu à midi + \[ P(M\cap W) = P(M) \times P_M(W) = 0.49 \times 0.75 = 0.3675 \] + \item Probabilité que la vente soit un burger. + \[ + P(B) = P(M\cap B) + P(S\cap B) = 0.49 \times 0.75 + 0.51 \times 0.61 = 0.3214 + \] + \item On cherche à calculer la quantité $P_B(S)$. Pour cela on utilise la formule de Bayes + \[ + P_B(S) = \frac{P(B\cap S)}{P(B)} = \frac{P_S(B) \times P(S)}{P(B)} = \frac{0.39\times 0.51}{0.3214} = 0.618855009334163 \approx 0.619 + \] + \end{enumerate} +\end{solution} + +\begin{exercise}[subtitle={Continent plastique}] + \textit{Les quantités évoqués dans cette exercice sont générés au hasard et sont donc complètement farfelus.} + \medskip + Le \og continent de plastique\fg{} est la plus grande des plaques de déchets plastiques évoluant sur les océans. Elle occupe actuellement dans l'océan Pacifique une surface dont l'aire est évaluée à plus de $1,6$ million de km$^2$, entre Hawaï et la Californie. + + En 2017, des scientifiques ont estimé qu'il y avait $15$ millions de tonnes de déchets plastiques qui était déversé chaque année dans les océans et que cette quantité augmentait de $21\n\%$ par chaque année. + + On modélise l'évolution de la masse de ces déchets plastiques déversée chaque année, si rien n'est fait pour la réduire, par une suite géométrique $\left(u_n\right)$. L'arrondi au centième du terme $u_n$ représente la masse de ces déchets déversée chaque année, exprimée en million de tonnes, pour l'année $(2017 + n)$. + + \medskip + + \begin{enumerate} + \item Expliquer pourquoi la suite $u_n$ est géométrique? + \item Calculer $u_1$ et $u_2$. + \item Exprimer $u_n$ en fonction de $n$. + \item Au début de l'année 2017, il y avait $300$ millions de tonnes de déchets plastique. Calculer la quantité totale de déchets plastiques en 2030. + \item On souhaite déterminer en quelle année la masse totale de ces déchets plastiques aura pour la première fois augmenté de $50$\,\% par rapport à sa valeur de 2017. + \begin{enumerate} + \item Recopier et compléter l'algorithme ci-dessous pour que la variable $N$ contienne la réponse au problème posé. + + \begin{center} + \begin{tabularx}{0.4\linewidth}{|X|}\hline + $N = 2017$\\ + $U = 15$ \\ + $S = 300 + U$ \\ + while $S < 450$: \\ + \hspace{1cm} $N = \ldots$\\ + \hspace{1cm} $U = \ldots$\\ + \hspace{1cm} $S = \ldots$\\ + \hline + \end{tabularx} + \end{center} + \item Que contiennent les variables $S$, $U$ et $N$ après exécution de cet algorithme ? + + Interpréter les résultats dans le contexte de l'exercice. + \end{enumerate} + \end{enumerate} +\end{exercise} + +\begin{solution} + \begin{enumerate} + \item Une augmentation de $21\,\%$ revient à multiplier la quantité par $1.21$. La suite est donc bien géométrique. Son premier terme est $u_0 = 15$ et sa raison est $q = 1.21$ + \item + \[ + u_1 = u_0 * 1.21 = 18.15 + \] + \[ + u_2 = u_0 * 1.21^2 = 21.9615 + \] + \item + \[ + u_n = u_0 \times q^n = 15 \times 1.21^n + \] + \item On calcule la quantité totale déversée entre 2017 et 2030. + \[ + \sum_{n = 0}^{13} u_n = u_0 \times \frac{1-q^{13}}{1-q} = 15 \times \frac{1 - 1.21^{13}}{1 - 1.21} = 779.87 + \] + On en déduit la quantité totale de déchets en 2030 + \[ + 300 + 779.87 = 1079.87 + \] + \item + \begin{enumerate} + \item ~ + \begin{center} + \begin{tabularx}{0.4\linewidth}{|X|}\hline + $N \gets 2017$\\ + $U \gets 15$ \\ + $S \gets 300 + U$ \\ + Tant que $S < 450$ \\ + \hspace{1cm} $N \gets N + 1$\\ + \hspace{1cm} $U \gets U * 1.21$\\ + \hspace{1cm} $S \gets S + u$\\ + Fin Tant que\\\hline + \end{tabularx} + \end{center} + \item \textit{Pas de correction automatisé} + \end{enumerate} + \end{enumerate} +\end{solution} + +\end{document} + +%%% Local Variables: +%%% mode: latex +%%% TeX-master: "master" +%%% End: diff --git a/TST/DS/DS_21_04_07/TST3/corr_08_210407_DS8.tex b/TST/DS/DS_21_04_07/TST3/corr_08_210407_DS8.tex new file mode 100644 index 0000000..6f5200e --- /dev/null +++ b/TST/DS/DS_21_04_07/TST3/corr_08_210407_DS8.tex @@ -0,0 +1,262 @@ +\documentclass[a4paper,10pt]{article} +\usepackage{myXsim} + +% Title Page +\title{DS8 \hfill GERMAIN Anaïs} +\tribe{TST} +\date{\hfillÀ render pour le Mercredi 7 avril} + +\xsimsetup{ + solution/print = true +} + +\begin{document} +\maketitle + +\begin{exercise}[subtitle={Automatismes}] + \textit{Toutes les questions de cette exercice sont indépendantes et peuvent être répondus séparément} + \begin{enumerate} + \item De janvier à septembre, une quantité a augmenté de $27\,\%$. Faire un schéma pour représenter la situation puis calculer le taux d'évolution moyen mensuel. + \item Une quantité augmente de $27\,\%$ par ans. En 2020, elle est de 132\euro. Quelle était sa valeur en 2019? Faire un schéma pour représenter la situation. + \item Déterminer l'équation de la droite \\ + \begin{tikzpicture}[xscale=0.8, yscale=0.5] + \tkzInit[xmin=-5,xmax=5,xstep=1, + ymin=-5,ymax=5,ystep=1] + \tkzGrid + \tkzAxeXY + \tkzFct[domain=-5:5,color=red,very thick]% + {2.0*\x -4}; + \end{tikzpicture} + \item Résoudre l'équation $7 \times 0.05^x = 18$ + \end{enumerate} +\end{exercise} + +\begin{solution} + \begin{enumerate} + \item On veut partager cette évolution en 8 évolutions. + \[ + \left(1 + \frac{27}{100}\right)^{\frac{1}{8}} = 1.0303 + \] + Donc le taux d'évolution moyen est + \[ + t_m = 1.0303 - 1 = 0.030299999999999994 + \] + \item Coefficient multiplicateur pour revenir en arrière + \[ + CM = (1 + \frac{27}{100})^{-1} = 0.7874 + \] + On en déduit la quantité en 2019 + \[ + 132 * 0.7874 = 103.9368 + \] + \item L'équation de la droite est + \[ + y = 2.0 x -4 + \] + \item Il faut penser à faire la division à par $7$ avant d'utiliser le log car sinon, on ne peut pas utiliser la formule $\log(a^n) = n\times \log(a)$. + + \[x = \frac{\log(2.57)}{\log(0.05)}\] + \end{enumerate} +\end{solution} + +\begin{exercise}[subtitle={Restaurant}] + Un \emph{food truck}, ouvert le midi et le soir, propose deux types de formules : + + \setlength\parindent{10mm} + \begin{itemize} + \item la formule \emph{Burger} ; + \item la formule \emph{Wok}. + \end{itemize} + \setlength\parindent{0mm} + + \medskip + + Le gérant a remarqué que 38\,\% de ses ventes ont lieu le midi. Le quart des ventes du midi correspondent à la formule \emph{Burger}, alors que 63\,\% des ventes du soir correspondent à la formule \emph{Wok}. + + Le gérant se constitue un fichier en notant, pour chaque vente, la formule choisie et le moment de cette vente (midi ou soir). + + On prélève une fiche de façon équiprobable. On définit les quatre évènements suivants: + + \begin{enumerate} + \item $M$ : \og la fiche correspond à une vente du midi\fg{} ; + \item $S$ : \og la fiche correspond à une vente du soir\fg {}; + \item $W$ : \og la fiche correspond à une formule \emph{Wok} \fg{} ; + \item $B$ : \og la fiche correspond à une formule \emph{Burger} \fg. + \end{enumerate} + \setlength\parindent{0mm} + + \medskip + + \begin{enumerate} + \item Recopier puis compléter l'arbre pondéré + + \begin{center} + \begin{tikzpicture}[sloped] + \node {.} + child {node {$M$} + child {node {$W$} + edge from parent + node[above] {...} + } + child {node {$B$} + edge from parent + node[above] {...} + } + edge from parent + node[above] {...} + } + child[missing] {} + child { node {$S$} + child {node {$W$} + edge from parent + node[above] {...} + } + child {node {$B$} + edge from parent + node[above] {...} + } + edge from parent + node[above] {...} + } ; + \end{tikzpicture} + \end{center} + + \item Calculer la probabilité de l'évènement $M \cap W$. Interpréter ce résultat dans le contexte de l'exercice. + \item Montrer que la probabilité que la fiche choisie corresponde à une formule \emph{Burger} est égale à $0.3244$. + \item On a prélevé une fiche correspondant à la formule \emph{Burger}. Quelle est la probabilité, arrondie au millième, que la vente ait eu lieu le soir? + \end{enumerate} +\end{exercise} + +\begin{solution} + \begin{enumerate} + \item + \begin{center} + \begin{tikzpicture}[sloped] + \node {.} + child {node {$M$} + child {node {$W$} + edge from parent + node[above] {$0.75$} + } + child {node {$B$} + edge from parent + node[above] {$0.25$} + } + edge from parent + node[above] {$0.38$} + } + child[missing] {} + child { node {$S$} + child {node {$W$} + edge from parent + node[above] {$0.63$} + } + child {node {$B$} + edge from parent + node[above] {$0.37$} + } + edge from parent + node[above] {$0.62$} + } ; + \end{tikzpicture} + \end{center} + \item On calcule la probabilité que la vente soit un wok et ait eu lieu à midi + \[ P(M\cap W) = P(M) \times P_M(W) = 0.38 \times 0.75 = 0.285 \] + \item Probabilité que la vente soit un burger. + \[ + P(B) = P(M\cap B) + P(S\cap B) = 0.38 \times 0.75 + 0.62 \times 0.63 = 0.3244 + \] + \item On cherche à calculer la quantité $P_B(S)$. Pour cela on utilise la formule de Bayes + \[ + P_B(S) = \frac{P(B\cap S)}{P(B)} = \frac{P_S(B) \times P(S)}{P(B)} = \frac{0.37\times 0.62}{0.3244} = 0.7071516646115905 \approx 0.707 + \] + \end{enumerate} +\end{solution} + +\begin{exercise}[subtitle={Continent plastique}] + \textit{Les quantités évoqués dans cette exercice sont générés au hasard et sont donc complètement farfelus.} + \medskip + Le \og continent de plastique\fg{} est la plus grande des plaques de déchets plastiques évoluant sur les océans. Elle occupe actuellement dans l'océan Pacifique une surface dont l'aire est évaluée à plus de $1,6$ million de km$^2$, entre Hawaï et la Californie. + + En 2017, des scientifiques ont estimé qu'il y avait $18$ millions de tonnes de déchets plastiques qui était déversé chaque année dans les océans et que cette quantité augmentait de $28\n\%$ par chaque année. + + On modélise l'évolution de la masse de ces déchets plastiques déversée chaque année, si rien n'est fait pour la réduire, par une suite géométrique $\left(u_n\right)$. L'arrondi au centième du terme $u_n$ représente la masse de ces déchets déversée chaque année, exprimée en million de tonnes, pour l'année $(2017 + n)$. + + \medskip + + \begin{enumerate} + \item Expliquer pourquoi la suite $u_n$ est géométrique? + \item Calculer $u_1$ et $u_2$. + \item Exprimer $u_n$ en fonction de $n$. + \item Au début de l'année 2017, il y avait $300$ millions de tonnes de déchets plastique. Calculer la quantité totale de déchets plastiques en 2030. + \item On souhaite déterminer en quelle année la masse totale de ces déchets plastiques aura pour la première fois augmenté de $50$\,\% par rapport à sa valeur de 2017. + \begin{enumerate} + \item Recopier et compléter l'algorithme ci-dessous pour que la variable $N$ contienne la réponse au problème posé. + + \begin{center} + \begin{tabularx}{0.4\linewidth}{|X|}\hline + $N = 2017$\\ + $U = 18$ \\ + $S = 300 + U$ \\ + while $S < 450$: \\ + \hspace{1cm} $N = \ldots$\\ + \hspace{1cm} $U = \ldots$\\ + \hspace{1cm} $S = \ldots$\\ + \hline + \end{tabularx} + \end{center} + \item Que contiennent les variables $S$, $U$ et $N$ après exécution de cet algorithme ? + + Interpréter les résultats dans le contexte de l'exercice. + \end{enumerate} + \end{enumerate} +\end{exercise} + +\begin{solution} + \begin{enumerate} + \item Une augmentation de $28\,\%$ revient à multiplier la quantité par $1.28$. La suite est donc bien géométrique. Son premier terme est $u_0 = 18$ et sa raison est $q = 1.28$ + \item + \[ + u_1 = u_0 * 1.28 = 23.04 + \] + \[ + u_2 = u_0 * 1.28^2 = 29.4912 + \] + \item + \[ + u_n = u_0 \times q^n = 18 \times 1.28^n + \] + \item On calcule la quantité totale déversée entre 2017 et 2030. + \[ + \sum_{n = 0}^{13} u_n = u_0 \times \frac{1-q^{13}}{1-q} = 18 \times \frac{1 - 1.28^{13}}{1 - 1.28} = 1527.35 + \] + On en déduit la quantité totale de déchets en 2030 + \[ + 300 + 1527.35 = 1827.35 + \] + \item + \begin{enumerate} + \item ~ + \begin{center} + \begin{tabularx}{0.4\linewidth}{|X|}\hline + $N \gets 2017$\\ + $U \gets 18$ \\ + $S \gets 300 + U$ \\ + Tant que $S < 450$ \\ + \hspace{1cm} $N \gets N + 1$\\ + \hspace{1cm} $U \gets U * 1.28$\\ + \hspace{1cm} $S \gets S + u$\\ + Fin Tant que\\\hline + \end{tabularx} + \end{center} + \item \textit{Pas de correction automatisé} + \end{enumerate} + \end{enumerate} +\end{solution} + +\end{document} + +%%% Local Variables: +%%% mode: latex +%%% TeX-master: "master" +%%% End: diff --git a/TST/DS/DS_21_04_07/TST3/corr_09_210407_DS8.tex b/TST/DS/DS_21_04_07/TST3/corr_09_210407_DS8.tex new file mode 100644 index 0000000..114cd58 --- /dev/null +++ b/TST/DS/DS_21_04_07/TST3/corr_09_210407_DS8.tex @@ -0,0 +1,262 @@ +\documentclass[a4paper,10pt]{article} +\usepackage{myXsim} + +% Title Page +\title{DS8 \hfill HADJRAS Mohcine} +\tribe{TST} +\date{\hfillÀ render pour le Mercredi 7 avril} + +\xsimsetup{ + solution/print = true +} + +\begin{document} +\maketitle + +\begin{exercise}[subtitle={Automatismes}] + \textit{Toutes les questions de cette exercice sont indépendantes et peuvent être répondus séparément} + \begin{enumerate} + \item De janvier à septembre, une quantité a augmenté de $19\,\%$. Faire un schéma pour représenter la situation puis calculer le taux d'évolution moyen mensuel. + \item Une quantité augmente de $19\,\%$ par ans. En 2020, elle est de 113\euro. Quelle était sa valeur en 2019? Faire un schéma pour représenter la situation. + \item Déterminer l'équation de la droite \\ + \begin{tikzpicture}[xscale=0.8, yscale=0.5] + \tkzInit[xmin=-5,xmax=5,xstep=1, + ymin=-5,ymax=5,ystep=1] + \tkzGrid + \tkzAxeXY + \tkzFct[domain=-5:5,color=red,very thick]% + {0.5*\x -1}; + \end{tikzpicture} + \item Résoudre l'équation $4 \times 0.54^x = 6$ + \end{enumerate} +\end{exercise} + +\begin{solution} + \begin{enumerate} + \item On veut partager cette évolution en 8 évolutions. + \[ + \left(1 + \frac{19}{100}\right)^{\frac{1}{8}} = 1.022 + \] + Donc le taux d'évolution moyen est + \[ + t_m = 1.022 - 1 = 0.02200000000000002 + \] + \item Coefficient multiplicateur pour revenir en arrière + \[ + CM = (1 + \frac{19}{100})^{-1} = 0.8403 + \] + On en déduit la quantité en 2019 + \[ + 113 * 0.8403 = 94.9539 + \] + \item L'équation de la droite est + \[ + y = 0.5 x -1 + \] + \item Il faut penser à faire la division à par $4$ avant d'utiliser le log car sinon, on ne peut pas utiliser la formule $\log(a^n) = n\times \log(a)$. + + \[x = \frac{\log(1.5)}{\log(0.54)}\] + \end{enumerate} +\end{solution} + +\begin{exercise}[subtitle={Restaurant}] + Un \emph{food truck}, ouvert le midi et le soir, propose deux types de formules : + + \setlength\parindent{10mm} + \begin{itemize} + \item la formule \emph{Burger} ; + \item la formule \emph{Wok}. + \end{itemize} + \setlength\parindent{0mm} + + \medskip + + Le gérant a remarqué que 68\,\% de ses ventes ont lieu le midi. Le quart des ventes du midi correspondent à la formule \emph{Burger}, alors que 43\,\% des ventes du soir correspondent à la formule \emph{Wok}. + + Le gérant se constitue un fichier en notant, pour chaque vente, la formule choisie et le moment de cette vente (midi ou soir). + + On prélève une fiche de façon équiprobable. On définit les quatre évènements suivants: + + \begin{enumerate} + \item $M$ : \og la fiche correspond à une vente du midi\fg{} ; + \item $S$ : \og la fiche correspond à une vente du soir\fg {}; + \item $W$ : \og la fiche correspond à une formule \emph{Wok} \fg{} ; + \item $B$ : \og la fiche correspond à une formule \emph{Burger} \fg. + \end{enumerate} + \setlength\parindent{0mm} + + \medskip + + \begin{enumerate} + \item Recopier puis compléter l'arbre pondéré + + \begin{center} + \begin{tikzpicture}[sloped] + \node {.} + child {node {$M$} + child {node {$W$} + edge from parent + node[above] {...} + } + child {node {$B$} + edge from parent + node[above] {...} + } + edge from parent + node[above] {...} + } + child[missing] {} + child { node {$S$} + child {node {$W$} + edge from parent + node[above] {...} + } + child {node {$B$} + edge from parent + node[above] {...} + } + edge from parent + node[above] {...} + } ; + \end{tikzpicture} + \end{center} + + \item Calculer la probabilité de l'évènement $M \cap W$. Interpréter ce résultat dans le contexte de l'exercice. + \item Montrer que la probabilité que la fiche choisie corresponde à une formule \emph{Burger} est égale à $0.3524$. + \item On a prélevé une fiche correspondant à la formule \emph{Burger}. Quelle est la probabilité, arrondie au millième, que la vente ait eu lieu le soir? + \end{enumerate} +\end{exercise} + +\begin{solution} + \begin{enumerate} + \item + \begin{center} + \begin{tikzpicture}[sloped] + \node {.} + child {node {$M$} + child {node {$W$} + edge from parent + node[above] {$0.75$} + } + child {node {$B$} + edge from parent + node[above] {$0.25$} + } + edge from parent + node[above] {$0.68$} + } + child[missing] {} + child { node {$S$} + child {node {$W$} + edge from parent + node[above] {$0.43$} + } + child {node {$B$} + edge from parent + node[above] {$0.57$} + } + edge from parent + node[above] {$0.32$} + } ; + \end{tikzpicture} + \end{center} + \item On calcule la probabilité que la vente soit un wok et ait eu lieu à midi + \[ P(M\cap W) = P(M) \times P_M(W) = 0.68 \times 0.75 = 0.51 \] + \item Probabilité que la vente soit un burger. + \[ + P(B) = P(M\cap B) + P(S\cap B) = 0.68 \times 0.75 + 0.32 \times 0.43 = 0.3524 + \] + \item On cherche à calculer la quantité $P_B(S)$. Pour cela on utilise la formule de Bayes + \[ + P_B(S) = \frac{P(B\cap S)}{P(B)} = \frac{P_S(B) \times P(S)}{P(B)} = \frac{0.57\times 0.32}{0.3524} = 0.5175936435868331 \approx 0.518 + \] + \end{enumerate} +\end{solution} + +\begin{exercise}[subtitle={Continent plastique}] + \textit{Les quantités évoqués dans cette exercice sont générés au hasard et sont donc complètement farfelus.} + \medskip + Le \og continent de plastique\fg{} est la plus grande des plaques de déchets plastiques évoluant sur les océans. Elle occupe actuellement dans l'océan Pacifique une surface dont l'aire est évaluée à plus de $1,6$ million de km$^2$, entre Hawaï et la Californie. + + En 2017, des scientifiques ont estimé qu'il y avait $4$ millions de tonnes de déchets plastiques qui était déversé chaque année dans les océans et que cette quantité augmentait de $10\n\%$ par chaque année. + + On modélise l'évolution de la masse de ces déchets plastiques déversée chaque année, si rien n'est fait pour la réduire, par une suite géométrique $\left(u_n\right)$. L'arrondi au centième du terme $u_n$ représente la masse de ces déchets déversée chaque année, exprimée en million de tonnes, pour l'année $(2017 + n)$. + + \medskip + + \begin{enumerate} + \item Expliquer pourquoi la suite $u_n$ est géométrique? + \item Calculer $u_1$ et $u_2$. + \item Exprimer $u_n$ en fonction de $n$. + \item Au début de l'année 2017, il y avait $300$ millions de tonnes de déchets plastique. Calculer la quantité totale de déchets plastiques en 2030. + \item On souhaite déterminer en quelle année la masse totale de ces déchets plastiques aura pour la première fois augmenté de $50$\,\% par rapport à sa valeur de 2017. + \begin{enumerate} + \item Recopier et compléter l'algorithme ci-dessous pour que la variable $N$ contienne la réponse au problème posé. + + \begin{center} + \begin{tabularx}{0.4\linewidth}{|X|}\hline + $N = 2017$\\ + $U = 4$ \\ + $S = 300 + U$ \\ + while $S < 450$: \\ + \hspace{1cm} $N = \ldots$\\ + \hspace{1cm} $U = \ldots$\\ + \hspace{1cm} $S = \ldots$\\ + \hline + \end{tabularx} + \end{center} + \item Que contiennent les variables $S$, $U$ et $N$ après exécution de cet algorithme ? + + Interpréter les résultats dans le contexte de l'exercice. + \end{enumerate} + \end{enumerate} +\end{exercise} + +\begin{solution} + \begin{enumerate} + \item Une augmentation de $10\,\%$ revient à multiplier la quantité par $1.1$. La suite est donc bien géométrique. Son premier terme est $u_0 = 4$ et sa raison est $q = 1.1$ + \item + \[ + u_1 = u_0 * 1.1 = 4.4 + \] + \[ + u_2 = u_0 * 1.1^2 = 4.84 + \] + \item + \[ + u_n = u_0 \times q^n = 4 \times 1.1^n + \] + \item On calcule la quantité totale déversée entre 2017 et 2030. + \[ + \sum_{n = 0}^{13} u_n = u_0 \times \frac{1-q^{13}}{1-q} = 4 \times \frac{1 - 1.1^{13}}{1 - 1.1} = 98.09 + \] + On en déduit la quantité totale de déchets en 2030 + \[ + 300 + 98.09 = 398.09000000000003 + \] + \item + \begin{enumerate} + \item ~ + \begin{center} + \begin{tabularx}{0.4\linewidth}{|X|}\hline + $N \gets 2017$\\ + $U \gets 4$ \\ + $S \gets 300 + U$ \\ + Tant que $S < 450$ \\ + \hspace{1cm} $N \gets N + 1$\\ + \hspace{1cm} $U \gets U * 1.1$\\ + \hspace{1cm} $S \gets S + u$\\ + Fin Tant que\\\hline + \end{tabularx} + \end{center} + \item \textit{Pas de correction automatisé} + \end{enumerate} + \end{enumerate} +\end{solution} + +\end{document} + +%%% Local Variables: +%%% mode: latex +%%% TeX-master: "master" +%%% End: diff --git a/TST/DS/DS_21_04_07/TST3/corr_10_210407_DS8.tex b/TST/DS/DS_21_04_07/TST3/corr_10_210407_DS8.tex new file mode 100644 index 0000000..e1a86c2 --- /dev/null +++ b/TST/DS/DS_21_04_07/TST3/corr_10_210407_DS8.tex @@ -0,0 +1,262 @@ +\documentclass[a4paper,10pt]{article} +\usepackage{myXsim} + +% Title Page +\title{DS8 \hfill HENRIST Maxime} +\tribe{TST} +\date{\hfillÀ render pour le Mercredi 7 avril} + +\xsimsetup{ + solution/print = true +} + +\begin{document} +\maketitle + +\begin{exercise}[subtitle={Automatismes}] + \textit{Toutes les questions de cette exercice sont indépendantes et peuvent être répondus séparément} + \begin{enumerate} + \item De janvier à septembre, une quantité a augmenté de $19\,\%$. Faire un schéma pour représenter la situation puis calculer le taux d'évolution moyen mensuel. + \item Une quantité augmente de $19\,\%$ par ans. En 2020, elle est de 133\euro. Quelle était sa valeur en 2019? Faire un schéma pour représenter la situation. + \item Déterminer l'équation de la droite \\ + \begin{tikzpicture}[xscale=0.8, yscale=0.5] + \tkzInit[xmin=-5,xmax=5,xstep=1, + ymin=-5,ymax=5,ystep=1] + \tkzGrid + \tkzAxeXY + \tkzFct[domain=-5:5,color=red,very thick]% + {1.0*\x -2}; + \end{tikzpicture} + \item Résoudre l'équation $7 \times 0.82^x = 19$ + \end{enumerate} +\end{exercise} + +\begin{solution} + \begin{enumerate} + \item On veut partager cette évolution en 8 évolutions. + \[ + \left(1 + \frac{19}{100}\right)^{\frac{1}{8}} = 1.022 + \] + Donc le taux d'évolution moyen est + \[ + t_m = 1.022 - 1 = 0.02200000000000002 + \] + \item Coefficient multiplicateur pour revenir en arrière + \[ + CM = (1 + \frac{19}{100})^{-1} = 0.8403 + \] + On en déduit la quantité en 2019 + \[ + 133 * 0.8403 = 111.7599 + \] + \item L'équation de la droite est + \[ + y = 1.0 x -2 + \] + \item Il faut penser à faire la division à par $7$ avant d'utiliser le log car sinon, on ne peut pas utiliser la formule $\log(a^n) = n\times \log(a)$. + + \[x = \frac{\log(2.71)}{\log(0.82)}\] + \end{enumerate} +\end{solution} + +\begin{exercise}[subtitle={Restaurant}] + Un \emph{food truck}, ouvert le midi et le soir, propose deux types de formules : + + \setlength\parindent{10mm} + \begin{itemize} + \item la formule \emph{Burger} ; + \item la formule \emph{Wok}. + \end{itemize} + \setlength\parindent{0mm} + + \medskip + + Le gérant a remarqué que 20\,\% de ses ventes ont lieu le midi. Le quart des ventes du midi correspondent à la formule \emph{Burger}, alors que 56\,\% des ventes du soir correspondent à la formule \emph{Wok}. + + Le gérant se constitue un fichier en notant, pour chaque vente, la formule choisie et le moment de cette vente (midi ou soir). + + On prélève une fiche de façon équiprobable. On définit les quatre évènements suivants: + + \begin{enumerate} + \item $M$ : \og la fiche correspond à une vente du midi\fg{} ; + \item $S$ : \og la fiche correspond à une vente du soir\fg {}; + \item $W$ : \og la fiche correspond à une formule \emph{Wok} \fg{} ; + \item $B$ : \og la fiche correspond à une formule \emph{Burger} \fg. + \end{enumerate} + \setlength\parindent{0mm} + + \medskip + + \begin{enumerate} + \item Recopier puis compléter l'arbre pondéré + + \begin{center} + \begin{tikzpicture}[sloped] + \node {.} + child {node {$M$} + child {node {$W$} + edge from parent + node[above] {...} + } + child {node {$B$} + edge from parent + node[above] {...} + } + edge from parent + node[above] {...} + } + child[missing] {} + child { node {$S$} + child {node {$W$} + edge from parent + node[above] {...} + } + child {node {$B$} + edge from parent + node[above] {...} + } + edge from parent + node[above] {...} + } ; + \end{tikzpicture} + \end{center} + + \item Calculer la probabilité de l'évènement $M \cap W$. Interpréter ce résultat dans le contexte de l'exercice. + \item Montrer que la probabilité que la fiche choisie corresponde à une formule \emph{Burger} est égale à $0.394$. + \item On a prélevé une fiche correspondant à la formule \emph{Burger}. Quelle est la probabilité, arrondie au millième, que la vente ait eu lieu le soir? + \end{enumerate} +\end{exercise} + +\begin{solution} + \begin{enumerate} + \item + \begin{center} + \begin{tikzpicture}[sloped] + \node {.} + child {node {$M$} + child {node {$W$} + edge from parent + node[above] {$0.75$} + } + child {node {$B$} + edge from parent + node[above] {$0.25$} + } + edge from parent + node[above] {$0.2$} + } + child[missing] {} + child { node {$S$} + child {node {$W$} + edge from parent + node[above] {$0.57$} + } + child {node {$B$} + edge from parent + node[above] {$0.43$} + } + edge from parent + node[above] {$0.8$} + } ; + \end{tikzpicture} + \end{center} + \item On calcule la probabilité que la vente soit un wok et ait eu lieu à midi + \[ P(M\cap W) = P(M) \times P_M(W) = 0.2 \times 0.75 = 0.15 \] + \item Probabilité que la vente soit un burger. + \[ + P(B) = P(M\cap B) + P(S\cap B) = 0.2 \times 0.75 + 0.8 \times 0.57 = 0.394 + \] + \item On cherche à calculer la quantité $P_B(S)$. Pour cela on utilise la formule de Bayes + \[ + P_B(S) = \frac{P(B\cap S)}{P(B)} = \frac{P_S(B) \times P(S)}{P(B)} = \frac{0.43\times 0.8}{0.394} = 0.8730964467005077 \approx 0.873 + \] + \end{enumerate} +\end{solution} + +\begin{exercise}[subtitle={Continent plastique}] + \textit{Les quantités évoqués dans cette exercice sont générés au hasard et sont donc complètement farfelus.} + \medskip + Le \og continent de plastique\fg{} est la plus grande des plaques de déchets plastiques évoluant sur les océans. Elle occupe actuellement dans l'océan Pacifique une surface dont l'aire est évaluée à plus de $1,6$ million de km$^2$, entre Hawaï et la Californie. + + En 2017, des scientifiques ont estimé qu'il y avait $5$ millions de tonnes de déchets plastiques qui était déversé chaque année dans les océans et que cette quantité augmentait de $16\n\%$ par chaque année. + + On modélise l'évolution de la masse de ces déchets plastiques déversée chaque année, si rien n'est fait pour la réduire, par une suite géométrique $\left(u_n\right)$. L'arrondi au centième du terme $u_n$ représente la masse de ces déchets déversée chaque année, exprimée en million de tonnes, pour l'année $(2017 + n)$. + + \medskip + + \begin{enumerate} + \item Expliquer pourquoi la suite $u_n$ est géométrique? + \item Calculer $u_1$ et $u_2$. + \item Exprimer $u_n$ en fonction de $n$. + \item Au début de l'année 2017, il y avait $300$ millions de tonnes de déchets plastique. Calculer la quantité totale de déchets plastiques en 2030. + \item On souhaite déterminer en quelle année la masse totale de ces déchets plastiques aura pour la première fois augmenté de $50$\,\% par rapport à sa valeur de 2017. + \begin{enumerate} + \item Recopier et compléter l'algorithme ci-dessous pour que la variable $N$ contienne la réponse au problème posé. + + \begin{center} + \begin{tabularx}{0.4\linewidth}{|X|}\hline + $N = 2017$\\ + $U = 5$ \\ + $S = 300 + U$ \\ + while $S < 450$: \\ + \hspace{1cm} $N = \ldots$\\ + \hspace{1cm} $U = \ldots$\\ + \hspace{1cm} $S = \ldots$\\ + \hline + \end{tabularx} + \end{center} + \item Que contiennent les variables $S$, $U$ et $N$ après exécution de cet algorithme ? + + Interpréter les résultats dans le contexte de l'exercice. + \end{enumerate} + \end{enumerate} +\end{exercise} + +\begin{solution} + \begin{enumerate} + \item Une augmentation de $16\,\%$ revient à multiplier la quantité par $1.16$. La suite est donc bien géométrique. Son premier terme est $u_0 = 5$ et sa raison est $q = 1.16$ + \item + \[ + u_1 = u_0 * 1.16 = 5.8 + \] + \[ + u_2 = u_0 * 1.16^2 = 6.728 + \] + \item + \[ + u_n = u_0 \times q^n = 5 \times 1.16^n + \] + \item On calcule la quantité totale déversée entre 2017 et 2030. + \[ + \sum_{n = 0}^{13} u_n = u_0 \times \frac{1-q^{13}}{1-q} = 5 \times \frac{1 - 1.16^{13}}{1 - 1.16} = 183.93 + \] + On en déduit la quantité totale de déchets en 2030 + \[ + 300 + 183.93 = 483.93 + \] + \item + \begin{enumerate} + \item ~ + \begin{center} + \begin{tabularx}{0.4\linewidth}{|X|}\hline + $N \gets 2017$\\ + $U \gets 5$ \\ + $S \gets 300 + U$ \\ + Tant que $S < 450$ \\ + \hspace{1cm} $N \gets N + 1$\\ + \hspace{1cm} $U \gets U * 1.16$\\ + \hspace{1cm} $S \gets S + u$\\ + Fin Tant que\\\hline + \end{tabularx} + \end{center} + \item \textit{Pas de correction automatisé} + \end{enumerate} + \end{enumerate} +\end{solution} + +\end{document} + +%%% Local Variables: +%%% mode: latex +%%% TeX-master: "master" +%%% End: diff --git a/TST/DS/DS_21_04_07/TST3/corr_11_210407_DS8.tex b/TST/DS/DS_21_04_07/TST3/corr_11_210407_DS8.tex new file mode 100644 index 0000000..fe8e03d --- /dev/null +++ b/TST/DS/DS_21_04_07/TST3/corr_11_210407_DS8.tex @@ -0,0 +1,262 @@ +\documentclass[a4paper,10pt]{article} +\usepackage{myXsim} + +% Title Page +\title{DS8 \hfill INFANTES Antoine} +\tribe{TST} +\date{\hfillÀ render pour le Mercredi 7 avril} + +\xsimsetup{ + solution/print = true +} + +\begin{document} +\maketitle + +\begin{exercise}[subtitle={Automatismes}] + \textit{Toutes les questions de cette exercice sont indépendantes et peuvent être répondus séparément} + \begin{enumerate} + \item De janvier à septembre, une quantité a augmenté de $18\,\%$. Faire un schéma pour représenter la situation puis calculer le taux d'évolution moyen mensuel. + \item Une quantité augmente de $18\,\%$ par ans. En 2020, elle est de 130\euro. Quelle était sa valeur en 2019? Faire un schéma pour représenter la situation. + \item Déterminer l'équation de la droite \\ + \begin{tikzpicture}[xscale=0.8, yscale=0.5] + \tkzInit[xmin=-5,xmax=5,xstep=1, + ymin=-5,ymax=5,ystep=1] + \tkzGrid + \tkzAxeXY + \tkzFct[domain=-5:5,color=red,very thick]% + {2.6666666666666665*\x -4}; + \end{tikzpicture} + \item Résoudre l'équation $3 \times 0.31^x = 15$ + \end{enumerate} +\end{exercise} + +\begin{solution} + \begin{enumerate} + \item On veut partager cette évolution en 8 évolutions. + \[ + \left(1 + \frac{18}{100}\right)^{\frac{1}{8}} = 1.0209 + \] + Donc le taux d'évolution moyen est + \[ + t_m = 1.0209 - 1 = 0.02089999999999992 + \] + \item Coefficient multiplicateur pour revenir en arrière + \[ + CM = (1 + \frac{18}{100})^{-1} = 0.8475 + \] + On en déduit la quantité en 2019 + \[ + 130 * 0.8475 = 110.175 + \] + \item L'équation de la droite est + \[ + y = 2.6666666666666665 x -4 + \] + \item Il faut penser à faire la division à par $3$ avant d'utiliser le log car sinon, on ne peut pas utiliser la formule $\log(a^n) = n\times \log(a)$. + + \[x = \frac{\log(5.0)}{\log(0.31)}\] + \end{enumerate} +\end{solution} + +\begin{exercise}[subtitle={Restaurant}] + Un \emph{food truck}, ouvert le midi et le soir, propose deux types de formules : + + \setlength\parindent{10mm} + \begin{itemize} + \item la formule \emph{Burger} ; + \item la formule \emph{Wok}. + \end{itemize} + \setlength\parindent{0mm} + + \medskip + + Le gérant a remarqué que 33\,\% de ses ventes ont lieu le midi. Le quart des ventes du midi correspondent à la formule \emph{Burger}, alors que 13\,\% des ventes du soir correspondent à la formule \emph{Wok}. + + Le gérant se constitue un fichier en notant, pour chaque vente, la formule choisie et le moment de cette vente (midi ou soir). + + On prélève une fiche de façon équiprobable. On définit les quatre évènements suivants: + + \begin{enumerate} + \item $M$ : \og la fiche correspond à une vente du midi\fg{} ; + \item $S$ : \og la fiche correspond à une vente du soir\fg {}; + \item $W$ : \og la fiche correspond à une formule \emph{Wok} \fg{} ; + \item $B$ : \og la fiche correspond à une formule \emph{Burger} \fg. + \end{enumerate} + \setlength\parindent{0mm} + + \medskip + + \begin{enumerate} + \item Recopier puis compléter l'arbre pondéré + + \begin{center} + \begin{tikzpicture}[sloped] + \node {.} + child {node {$M$} + child {node {$W$} + edge from parent + node[above] {...} + } + child {node {$B$} + edge from parent + node[above] {...} + } + edge from parent + node[above] {...} + } + child[missing] {} + child { node {$S$} + child {node {$W$} + edge from parent + node[above] {...} + } + child {node {$B$} + edge from parent + node[above] {...} + } + edge from parent + node[above] {...} + } ; + \end{tikzpicture} + \end{center} + + \item Calculer la probabilité de l'évènement $M \cap W$. Interpréter ce résultat dans le contexte de l'exercice. + \item Montrer que la probabilité que la fiche choisie corresponde à une formule \emph{Burger} est égale à $0.6654$. + \item On a prélevé une fiche correspondant à la formule \emph{Burger}. Quelle est la probabilité, arrondie au millième, que la vente ait eu lieu le soir? + \end{enumerate} +\end{exercise} + +\begin{solution} + \begin{enumerate} + \item + \begin{center} + \begin{tikzpicture}[sloped] + \node {.} + child {node {$M$} + child {node {$W$} + edge from parent + node[above] {$0.75$} + } + child {node {$B$} + edge from parent + node[above] {$0.25$} + } + edge from parent + node[above] {$0.33$} + } + child[missing] {} + child { node {$S$} + child {node {$W$} + edge from parent + node[above] {$0.13$} + } + child {node {$B$} + edge from parent + node[above] {$0.87$} + } + edge from parent + node[above] {$0.67$} + } ; + \end{tikzpicture} + \end{center} + \item On calcule la probabilité que la vente soit un wok et ait eu lieu à midi + \[ P(M\cap W) = P(M) \times P_M(W) = 0.33 \times 0.75 = 0.2475 \] + \item Probabilité que la vente soit un burger. + \[ + P(B) = P(M\cap B) + P(S\cap B) = 0.33 \times 0.75 + 0.67 \times 0.13 = 0.6654 + \] + \item On cherche à calculer la quantité $P_B(S)$. Pour cela on utilise la formule de Bayes + \[ + P_B(S) = \frac{P(B\cap S)}{P(B)} = \frac{P_S(B) \times P(S)}{P(B)} = \frac{0.87\times 0.67}{0.6654} = 0.8760144274120831 \approx 0.876 + \] + \end{enumerate} +\end{solution} + +\begin{exercise}[subtitle={Continent plastique}] + \textit{Les quantités évoqués dans cette exercice sont générés au hasard et sont donc complètement farfelus.} + \medskip + Le \og continent de plastique\fg{} est la plus grande des plaques de déchets plastiques évoluant sur les océans. Elle occupe actuellement dans l'océan Pacifique une surface dont l'aire est évaluée à plus de $1,6$ million de km$^2$, entre Hawaï et la Californie. + + En 2017, des scientifiques ont estimé qu'il y avait $12$ millions de tonnes de déchets plastiques qui était déversé chaque année dans les océans et que cette quantité augmentait de $26\n\%$ par chaque année. + + On modélise l'évolution de la masse de ces déchets plastiques déversée chaque année, si rien n'est fait pour la réduire, par une suite géométrique $\left(u_n\right)$. L'arrondi au centième du terme $u_n$ représente la masse de ces déchets déversée chaque année, exprimée en million de tonnes, pour l'année $(2017 + n)$. + + \medskip + + \begin{enumerate} + \item Expliquer pourquoi la suite $u_n$ est géométrique? + \item Calculer $u_1$ et $u_2$. + \item Exprimer $u_n$ en fonction de $n$. + \item Au début de l'année 2017, il y avait $300$ millions de tonnes de déchets plastique. Calculer la quantité totale de déchets plastiques en 2030. + \item On souhaite déterminer en quelle année la masse totale de ces déchets plastiques aura pour la première fois augmenté de $50$\,\% par rapport à sa valeur de 2017. + \begin{enumerate} + \item Recopier et compléter l'algorithme ci-dessous pour que la variable $N$ contienne la réponse au problème posé. + + \begin{center} + \begin{tabularx}{0.4\linewidth}{|X|}\hline + $N = 2017$\\ + $U = 12$ \\ + $S = 300 + U$ \\ + while $S < 450$: \\ + \hspace{1cm} $N = \ldots$\\ + \hspace{1cm} $U = \ldots$\\ + \hspace{1cm} $S = \ldots$\\ + \hline + \end{tabularx} + \end{center} + \item Que contiennent les variables $S$, $U$ et $N$ après exécution de cet algorithme ? + + Interpréter les résultats dans le contexte de l'exercice. + \end{enumerate} + \end{enumerate} +\end{exercise} + +\begin{solution} + \begin{enumerate} + \item Une augmentation de $26\,\%$ revient à multiplier la quantité par $1.26$. La suite est donc bien géométrique. Son premier terme est $u_0 = 12$ et sa raison est $q = 1.26$ + \item + \[ + u_1 = u_0 * 1.26 = 15.120000000000001 + \] + \[ + u_2 = u_0 * 1.26^2 = 19.0512 + \] + \item + \[ + u_n = u_0 \times q^n = 12 \times 1.26^n + \] + \item On calcule la quantité totale déversée entre 2017 et 2030. + \[ + \sum_{n = 0}^{13} u_n = u_0 \times \frac{1-q^{13}}{1-q} = 12 \times \frac{1 - 1.26^{13}}{1 - 1.26} = 885.01 + \] + On en déduit la quantité totale de déchets en 2030 + \[ + 300 + 885.01 = 1185.01 + \] + \item + \begin{enumerate} + \item ~ + \begin{center} + \begin{tabularx}{0.4\linewidth}{|X|}\hline + $N \gets 2017$\\ + $U \gets 12$ \\ + $S \gets 300 + U$ \\ + Tant que $S < 450$ \\ + \hspace{1cm} $N \gets N + 1$\\ + \hspace{1cm} $U \gets U * 1.26$\\ + \hspace{1cm} $S \gets S + u$\\ + Fin Tant que\\\hline + \end{tabularx} + \end{center} + \item \textit{Pas de correction automatisé} + \end{enumerate} + \end{enumerate} +\end{solution} + +\end{document} + +%%% Local Variables: +%%% mode: latex +%%% TeX-master: "master" +%%% End: diff --git a/TST/DS/DS_21_04_07/TST3/corr_12_210407_DS8.tex b/TST/DS/DS_21_04_07/TST3/corr_12_210407_DS8.tex new file mode 100644 index 0000000..3990570 --- /dev/null +++ b/TST/DS/DS_21_04_07/TST3/corr_12_210407_DS8.tex @@ -0,0 +1,262 @@ +\documentclass[a4paper,10pt]{article} +\usepackage{myXsim} + +% Title Page +\title{DS8 \hfill MAGRO Robin} +\tribe{TST} +\date{\hfillÀ render pour le Mercredi 7 avril} + +\xsimsetup{ + solution/print = true +} + +\begin{document} +\maketitle + +\begin{exercise}[subtitle={Automatismes}] + \textit{Toutes les questions de cette exercice sont indépendantes et peuvent être répondus séparément} + \begin{enumerate} + \item De janvier à septembre, une quantité a augmenté de $11\,\%$. Faire un schéma pour représenter la situation puis calculer le taux d'évolution moyen mensuel. + \item Une quantité augmente de $11\,\%$ par ans. En 2020, elle est de 143\euro. Quelle était sa valeur en 2019? Faire un schéma pour représenter la situation. + \item Déterminer l'équation de la droite \\ + \begin{tikzpicture}[xscale=0.8, yscale=0.5] + \tkzInit[xmin=-5,xmax=5,xstep=1, + ymin=-5,ymax=5,ystep=1] + \tkzGrid + \tkzAxeXY + \tkzFct[domain=-5:5,color=red,very thick]% + {3.0*\x -3}; + \end{tikzpicture} + \item Résoudre l'équation $5 \times 0.12^x = 40$ + \end{enumerate} +\end{exercise} + +\begin{solution} + \begin{enumerate} + \item On veut partager cette évolution en 8 évolutions. + \[ + \left(1 + \frac{11}{100}\right)^{\frac{1}{8}} = 1.0131 + \] + Donc le taux d'évolution moyen est + \[ + t_m = 1.0131 - 1 = 0.01309999999999989 + \] + \item Coefficient multiplicateur pour revenir en arrière + \[ + CM = (1 + \frac{11}{100})^{-1} = 0.9009 + \] + On en déduit la quantité en 2019 + \[ + 143 * 0.9009 = 128.8287 + \] + \item L'équation de la droite est + \[ + y = 3.0 x -3 + \] + \item Il faut penser à faire la division à par $5$ avant d'utiliser le log car sinon, on ne peut pas utiliser la formule $\log(a^n) = n\times \log(a)$. + + \[x = \frac{\log(8.0)}{\log(0.12)}\] + \end{enumerate} +\end{solution} + +\begin{exercise}[subtitle={Restaurant}] + Un \emph{food truck}, ouvert le midi et le soir, propose deux types de formules : + + \setlength\parindent{10mm} + \begin{itemize} + \item la formule \emph{Burger} ; + \item la formule \emph{Wok}. + \end{itemize} + \setlength\parindent{0mm} + + \medskip + + Le gérant a remarqué que 37\,\% de ses ventes ont lieu le midi. Le quart des ventes du midi correspondent à la formule \emph{Burger}, alors que 32\,\% des ventes du soir correspondent à la formule \emph{Wok}. + + Le gérant se constitue un fichier en notant, pour chaque vente, la formule choisie et le moment de cette vente (midi ou soir). + + On prélève une fiche de façon équiprobable. On définit les quatre évènements suivants: + + \begin{enumerate} + \item $M$ : \og la fiche correspond à une vente du midi\fg{} ; + \item $S$ : \og la fiche correspond à une vente du soir\fg {}; + \item $W$ : \og la fiche correspond à une formule \emph{Wok} \fg{} ; + \item $B$ : \og la fiche correspond à une formule \emph{Burger} \fg. + \end{enumerate} + \setlength\parindent{0mm} + + \medskip + + \begin{enumerate} + \item Recopier puis compléter l'arbre pondéré + + \begin{center} + \begin{tikzpicture}[sloped] + \node {.} + child {node {$M$} + child {node {$W$} + edge from parent + node[above] {...} + } + child {node {$B$} + edge from parent + node[above] {...} + } + edge from parent + node[above] {...} + } + child[missing] {} + child { node {$S$} + child {node {$W$} + edge from parent + node[above] {...} + } + child {node {$B$} + edge from parent + node[above] {...} + } + edge from parent + node[above] {...} + } ; + \end{tikzpicture} + \end{center} + + \item Calculer la probabilité de l'évènement $M \cap W$. Interpréter ce résultat dans le contexte de l'exercice. + \item Montrer que la probabilité que la fiche choisie corresponde à une formule \emph{Burger} est égale à $0.5209$. + \item On a prélevé une fiche correspondant à la formule \emph{Burger}. Quelle est la probabilité, arrondie au millième, que la vente ait eu lieu le soir? + \end{enumerate} +\end{exercise} + +\begin{solution} + \begin{enumerate} + \item + \begin{center} + \begin{tikzpicture}[sloped] + \node {.} + child {node {$M$} + child {node {$W$} + edge from parent + node[above] {$0.75$} + } + child {node {$B$} + edge from parent + node[above] {$0.25$} + } + edge from parent + node[above] {$0.37$} + } + child[missing] {} + child { node {$S$} + child {node {$W$} + edge from parent + node[above] {$0.32$} + } + child {node {$B$} + edge from parent + node[above] {$0.68$} + } + edge from parent + node[above] {$0.63$} + } ; + \end{tikzpicture} + \end{center} + \item On calcule la probabilité que la vente soit un wok et ait eu lieu à midi + \[ P(M\cap W) = P(M) \times P_M(W) = 0.37 \times 0.75 = 0.2775 \] + \item Probabilité que la vente soit un burger. + \[ + P(B) = P(M\cap B) + P(S\cap B) = 0.37 \times 0.75 + 0.63 \times 0.32 = 0.5209 + \] + \item On cherche à calculer la quantité $P_B(S)$. Pour cela on utilise la formule de Bayes + \[ + P_B(S) = \frac{P(B\cap S)}{P(B)} = \frac{P_S(B) \times P(S)}{P(B)} = \frac{0.68\times 0.63}{0.5209} = 0.8224227298905741 \approx 0.822 + \] + \end{enumerate} +\end{solution} + +\begin{exercise}[subtitle={Continent plastique}] + \textit{Les quantités évoqués dans cette exercice sont générés au hasard et sont donc complètement farfelus.} + \medskip + Le \og continent de plastique\fg{} est la plus grande des plaques de déchets plastiques évoluant sur les océans. Elle occupe actuellement dans l'océan Pacifique une surface dont l'aire est évaluée à plus de $1,6$ million de km$^2$, entre Hawaï et la Californie. + + En 2017, des scientifiques ont estimé qu'il y avait $16$ millions de tonnes de déchets plastiques qui était déversé chaque année dans les océans et que cette quantité augmentait de $17\n\%$ par chaque année. + + On modélise l'évolution de la masse de ces déchets plastiques déversée chaque année, si rien n'est fait pour la réduire, par une suite géométrique $\left(u_n\right)$. L'arrondi au centième du terme $u_n$ représente la masse de ces déchets déversée chaque année, exprimée en million de tonnes, pour l'année $(2017 + n)$. + + \medskip + + \begin{enumerate} + \item Expliquer pourquoi la suite $u_n$ est géométrique? + \item Calculer $u_1$ et $u_2$. + \item Exprimer $u_n$ en fonction de $n$. + \item Au début de l'année 2017, il y avait $300$ millions de tonnes de déchets plastique. Calculer la quantité totale de déchets plastiques en 2030. + \item On souhaite déterminer en quelle année la masse totale de ces déchets plastiques aura pour la première fois augmenté de $50$\,\% par rapport à sa valeur de 2017. + \begin{enumerate} + \item Recopier et compléter l'algorithme ci-dessous pour que la variable $N$ contienne la réponse au problème posé. + + \begin{center} + \begin{tabularx}{0.4\linewidth}{|X|}\hline + $N = 2017$\\ + $U = 16$ \\ + $S = 300 + U$ \\ + while $S < 450$: \\ + \hspace{1cm} $N = \ldots$\\ + \hspace{1cm} $U = \ldots$\\ + \hspace{1cm} $S = \ldots$\\ + \hline + \end{tabularx} + \end{center} + \item Que contiennent les variables $S$, $U$ et $N$ après exécution de cet algorithme ? + + Interpréter les résultats dans le contexte de l'exercice. + \end{enumerate} + \end{enumerate} +\end{exercise} + +\begin{solution} + \begin{enumerate} + \item Une augmentation de $17\,\%$ revient à multiplier la quantité par $1.17$. La suite est donc bien géométrique. Son premier terme est $u_0 = 16$ et sa raison est $q = 1.17$ + \item + \[ + u_1 = u_0 * 1.17 = 18.72 + \] + \[ + u_2 = u_0 * 1.17^2 = 21.9024 + \] + \item + \[ + u_n = u_0 \times q^n = 16 \times 1.17^n + \] + \item On calcule la quantité totale déversée entre 2017 et 2030. + \[ + \sum_{n = 0}^{13} u_n = u_0 \times \frac{1-q^{13}}{1-q} = 16 \times \frac{1 - 1.17^{13}}{1 - 1.17} = 630.46 + \] + On en déduit la quantité totale de déchets en 2030 + \[ + 300 + 630.46 = 930.46 + \] + \item + \begin{enumerate} + \item ~ + \begin{center} + \begin{tabularx}{0.4\linewidth}{|X|}\hline + $N \gets 2017$\\ + $U \gets 16$ \\ + $S \gets 300 + U$ \\ + Tant que $S < 450$ \\ + \hspace{1cm} $N \gets N + 1$\\ + \hspace{1cm} $U \gets U * 1.17$\\ + \hspace{1cm} $S \gets S + u$\\ + Fin Tant que\\\hline + \end{tabularx} + \end{center} + \item \textit{Pas de correction automatisé} + \end{enumerate} + \end{enumerate} +\end{solution} + +\end{document} + +%%% Local Variables: +%%% mode: latex +%%% TeX-master: "master" +%%% End: diff --git a/TST/DS/DS_21_04_07/TST3/corr_13_210407_DS8.tex b/TST/DS/DS_21_04_07/TST3/corr_13_210407_DS8.tex new file mode 100644 index 0000000..fe4ffab --- /dev/null +++ b/TST/DS/DS_21_04_07/TST3/corr_13_210407_DS8.tex @@ -0,0 +1,262 @@ +\documentclass[a4paper,10pt]{article} +\usepackage{myXsim} + +% Title Page +\title{DS8 \hfill MORFIN Chloé} +\tribe{TST} +\date{\hfillÀ render pour le Mercredi 7 avril} + +\xsimsetup{ + solution/print = true +} + +\begin{document} +\maketitle + +\begin{exercise}[subtitle={Automatismes}] + \textit{Toutes les questions de cette exercice sont indépendantes et peuvent être répondus séparément} + \begin{enumerate} + \item De janvier à septembre, une quantité a augmenté de $18\,\%$. Faire un schéma pour représenter la situation puis calculer le taux d'évolution moyen mensuel. + \item Une quantité augmente de $18\,\%$ par ans. En 2020, elle est de 119\euro. Quelle était sa valeur en 2019? Faire un schéma pour représenter la situation. + \item Déterminer l'équation de la droite \\ + \begin{tikzpicture}[xscale=0.8, yscale=0.5] + \tkzInit[xmin=-5,xmax=5,xstep=1, + ymin=-5,ymax=5,ystep=1] + \tkzGrid + \tkzAxeXY + \tkzFct[domain=-5:5,color=red,very thick]% + {4.0*\x -4}; + \end{tikzpicture} + \item Résoudre l'équation $3 \times 1.0^x = 32$ + \end{enumerate} +\end{exercise} + +\begin{solution} + \begin{enumerate} + \item On veut partager cette évolution en 8 évolutions. + \[ + \left(1 + \frac{18}{100}\right)^{\frac{1}{8}} = 1.0209 + \] + Donc le taux d'évolution moyen est + \[ + t_m = 1.0209 - 1 = 0.02089999999999992 + \] + \item Coefficient multiplicateur pour revenir en arrière + \[ + CM = (1 + \frac{18}{100})^{-1} = 0.8475 + \] + On en déduit la quantité en 2019 + \[ + 119 * 0.8475 = 100.8525 + \] + \item L'équation de la droite est + \[ + y = 4.0 x -4 + \] + \item Il faut penser à faire la division à par $3$ avant d'utiliser le log car sinon, on ne peut pas utiliser la formule $\log(a^n) = n\times \log(a)$. + + \[x = \frac{\log(10.67)}{\log(1.0)}\] + \end{enumerate} +\end{solution} + +\begin{exercise}[subtitle={Restaurant}] + Un \emph{food truck}, ouvert le midi et le soir, propose deux types de formules : + + \setlength\parindent{10mm} + \begin{itemize} + \item la formule \emph{Burger} ; + \item la formule \emph{Wok}. + \end{itemize} + \setlength\parindent{0mm} + + \medskip + + Le gérant a remarqué que 24\,\% de ses ventes ont lieu le midi. Le quart des ventes du midi correspondent à la formule \emph{Burger}, alors que 57\,\% des ventes du soir correspondent à la formule \emph{Wok}. + + Le gérant se constitue un fichier en notant, pour chaque vente, la formule choisie et le moment de cette vente (midi ou soir). + + On prélève une fiche de façon équiprobable. On définit les quatre évènements suivants: + + \begin{enumerate} + \item $M$ : \og la fiche correspond à une vente du midi\fg{} ; + \item $S$ : \og la fiche correspond à une vente du soir\fg {}; + \item $W$ : \og la fiche correspond à une formule \emph{Wok} \fg{} ; + \item $B$ : \og la fiche correspond à une formule \emph{Burger} \fg. + \end{enumerate} + \setlength\parindent{0mm} + + \medskip + + \begin{enumerate} + \item Recopier puis compléter l'arbre pondéré + + \begin{center} + \begin{tikzpicture}[sloped] + \node {.} + child {node {$M$} + child {node {$W$} + edge from parent + node[above] {...} + } + child {node {$B$} + edge from parent + node[above] {...} + } + edge from parent + node[above] {...} + } + child[missing] {} + child { node {$S$} + child {node {$W$} + edge from parent + node[above] {...} + } + child {node {$B$} + edge from parent + node[above] {...} + } + edge from parent + node[above] {...} + } ; + \end{tikzpicture} + \end{center} + + \item Calculer la probabilité de l'évènement $M \cap W$. Interpréter ce résultat dans le contexte de l'exercice. + \item Montrer que la probabilité que la fiche choisie corresponde à une formule \emph{Burger} est égale à $0.3792$. + \item On a prélevé une fiche correspondant à la formule \emph{Burger}. Quelle est la probabilité, arrondie au millième, que la vente ait eu lieu le soir? + \end{enumerate} +\end{exercise} + +\begin{solution} + \begin{enumerate} + \item + \begin{center} + \begin{tikzpicture}[sloped] + \node {.} + child {node {$M$} + child {node {$W$} + edge from parent + node[above] {$0.75$} + } + child {node {$B$} + edge from parent + node[above] {$0.25$} + } + edge from parent + node[above] {$0.24$} + } + child[missing] {} + child { node {$S$} + child {node {$W$} + edge from parent + node[above] {$0.58$} + } + child {node {$B$} + edge from parent + node[above] {$0.42$} + } + edge from parent + node[above] {$0.76$} + } ; + \end{tikzpicture} + \end{center} + \item On calcule la probabilité que la vente soit un wok et ait eu lieu à midi + \[ P(M\cap W) = P(M) \times P_M(W) = 0.24 \times 0.75 = 0.18 \] + \item Probabilité que la vente soit un burger. + \[ + P(B) = P(M\cap B) + P(S\cap B) = 0.24 \times 0.75 + 0.76 \times 0.58 = 0.3792 + \] + \item On cherche à calculer la quantité $P_B(S)$. Pour cela on utilise la formule de Bayes + \[ + P_B(S) = \frac{P(B\cap S)}{P(B)} = \frac{P_S(B) \times P(S)}{P(B)} = \frac{0.42\times 0.76}{0.3792} = 0.8417721518987342 \approx 0.842 + \] + \end{enumerate} +\end{solution} + +\begin{exercise}[subtitle={Continent plastique}] + \textit{Les quantités évoqués dans cette exercice sont générés au hasard et sont donc complètement farfelus.} + \medskip + Le \og continent de plastique\fg{} est la plus grande des plaques de déchets plastiques évoluant sur les océans. Elle occupe actuellement dans l'océan Pacifique une surface dont l'aire est évaluée à plus de $1,6$ million de km$^2$, entre Hawaï et la Californie. + + En 2017, des scientifiques ont estimé qu'il y avait $17$ millions de tonnes de déchets plastiques qui était déversé chaque année dans les océans et que cette quantité augmentait de $16\n\%$ par chaque année. + + On modélise l'évolution de la masse de ces déchets plastiques déversée chaque année, si rien n'est fait pour la réduire, par une suite géométrique $\left(u_n\right)$. L'arrondi au centième du terme $u_n$ représente la masse de ces déchets déversée chaque année, exprimée en million de tonnes, pour l'année $(2017 + n)$. + + \medskip + + \begin{enumerate} + \item Expliquer pourquoi la suite $u_n$ est géométrique? + \item Calculer $u_1$ et $u_2$. + \item Exprimer $u_n$ en fonction de $n$. + \item Au début de l'année 2017, il y avait $300$ millions de tonnes de déchets plastique. Calculer la quantité totale de déchets plastiques en 2030. + \item On souhaite déterminer en quelle année la masse totale de ces déchets plastiques aura pour la première fois augmenté de $50$\,\% par rapport à sa valeur de 2017. + \begin{enumerate} + \item Recopier et compléter l'algorithme ci-dessous pour que la variable $N$ contienne la réponse au problème posé. + + \begin{center} + \begin{tabularx}{0.4\linewidth}{|X|}\hline + $N = 2017$\\ + $U = 17$ \\ + $S = 300 + U$ \\ + while $S < 450$: \\ + \hspace{1cm} $N = \ldots$\\ + \hspace{1cm} $U = \ldots$\\ + \hspace{1cm} $S = \ldots$\\ + \hline + \end{tabularx} + \end{center} + \item Que contiennent les variables $S$, $U$ et $N$ après exécution de cet algorithme ? + + Interpréter les résultats dans le contexte de l'exercice. + \end{enumerate} + \end{enumerate} +\end{exercise} + +\begin{solution} + \begin{enumerate} + \item Une augmentation de $16\,\%$ revient à multiplier la quantité par $1.16$. La suite est donc bien géométrique. Son premier terme est $u_0 = 17$ et sa raison est $q = 1.16$ + \item + \[ + u_1 = u_0 * 1.16 = 19.72 + \] + \[ + u_2 = u_0 * 1.16^2 = 22.8752 + \] + \item + \[ + u_n = u_0 \times q^n = 17 \times 1.16^n + \] + \item On calcule la quantité totale déversée entre 2017 et 2030. + \[ + \sum_{n = 0}^{13} u_n = u_0 \times \frac{1-q^{13}}{1-q} = 17 \times \frac{1 - 1.16^{13}}{1 - 1.16} = 625.37 + \] + On en déduit la quantité totale de déchets en 2030 + \[ + 300 + 625.37 = 925.37 + \] + \item + \begin{enumerate} + \item ~ + \begin{center} + \begin{tabularx}{0.4\linewidth}{|X|}\hline + $N \gets 2017$\\ + $U \gets 17$ \\ + $S \gets 300 + U$ \\ + Tant que $S < 450$ \\ + \hspace{1cm} $N \gets N + 1$\\ + \hspace{1cm} $U \gets U * 1.16$\\ + \hspace{1cm} $S \gets S + u$\\ + Fin Tant que\\\hline + \end{tabularx} + \end{center} + \item \textit{Pas de correction automatisé} + \end{enumerate} + \end{enumerate} +\end{solution} + +\end{document} + +%%% Local Variables: +%%% mode: latex +%%% TeX-master: "master" +%%% End: diff --git a/TST/DS/DS_21_04_07/TST3/corr_14_210407_DS8.tex b/TST/DS/DS_21_04_07/TST3/corr_14_210407_DS8.tex new file mode 100644 index 0000000..f5bdc1d --- /dev/null +++ b/TST/DS/DS_21_04_07/TST3/corr_14_210407_DS8.tex @@ -0,0 +1,262 @@ +\documentclass[a4paper,10pt]{article} +\usepackage{myXsim} + +% Title Page +\title{DS8 \hfill PERES RAMALHO Emeric} +\tribe{TST} +\date{\hfillÀ render pour le Mercredi 7 avril} + +\xsimsetup{ + solution/print = true +} + +\begin{document} +\maketitle + +\begin{exercise}[subtitle={Automatismes}] + \textit{Toutes les questions de cette exercice sont indépendantes et peuvent être répondus séparément} + \begin{enumerate} + \item De janvier à septembre, une quantité a augmenté de $23\,\%$. Faire un schéma pour représenter la situation puis calculer le taux d'évolution moyen mensuel. + \item Une quantité augmente de $23\,\%$ par ans. En 2020, elle est de 118\euro. Quelle était sa valeur en 2019? Faire un schéma pour représenter la situation. + \item Déterminer l'équation de la droite \\ + \begin{tikzpicture}[xscale=0.8, yscale=0.5] + \tkzInit[xmin=-5,xmax=5,xstep=1, + ymin=-5,ymax=5,ystep=1] + \tkzGrid + \tkzAxeXY + \tkzFct[domain=-5:5,color=red,very thick]% + {1.3333333333333333*\x -2}; + \end{tikzpicture} + \item Résoudre l'équation $5 \times 0.46^x = 2$ + \end{enumerate} +\end{exercise} + +\begin{solution} + \begin{enumerate} + \item On veut partager cette évolution en 8 évolutions. + \[ + \left(1 + \frac{23}{100}\right)^{\frac{1}{8}} = 1.0262 + \] + Donc le taux d'évolution moyen est + \[ + t_m = 1.0262 - 1 = 0.0262 + \] + \item Coefficient multiplicateur pour revenir en arrière + \[ + CM = (1 + \frac{23}{100})^{-1} = 0.813 + \] + On en déduit la quantité en 2019 + \[ + 118 * 0.813 = 95.934 + \] + \item L'équation de la droite est + \[ + y = 1.3333333333333333 x -2 + \] + \item Il faut penser à faire la division à par $5$ avant d'utiliser le log car sinon, on ne peut pas utiliser la formule $\log(a^n) = n\times \log(a)$. + + \[x = \frac{\log(0.4)}{\log(0.46)}\] + \end{enumerate} +\end{solution} + +\begin{exercise}[subtitle={Restaurant}] + Un \emph{food truck}, ouvert le midi et le soir, propose deux types de formules : + + \setlength\parindent{10mm} + \begin{itemize} + \item la formule \emph{Burger} ; + \item la formule \emph{Wok}. + \end{itemize} + \setlength\parindent{0mm} + + \medskip + + Le gérant a remarqué que 11\,\% de ses ventes ont lieu le midi. Le quart des ventes du midi correspondent à la formule \emph{Burger}, alors que 18\,\% des ventes du soir correspondent à la formule \emph{Wok}. + + Le gérant se constitue un fichier en notant, pour chaque vente, la formule choisie et le moment de cette vente (midi ou soir). + + On prélève une fiche de façon équiprobable. On définit les quatre évènements suivants: + + \begin{enumerate} + \item $M$ : \og la fiche correspond à une vente du midi\fg{} ; + \item $S$ : \og la fiche correspond à une vente du soir\fg {}; + \item $W$ : \og la fiche correspond à une formule \emph{Wok} \fg{} ; + \item $B$ : \og la fiche correspond à une formule \emph{Burger} \fg. + \end{enumerate} + \setlength\parindent{0mm} + + \medskip + + \begin{enumerate} + \item Recopier puis compléter l'arbre pondéré + + \begin{center} + \begin{tikzpicture}[sloped] + \node {.} + child {node {$M$} + child {node {$W$} + edge from parent + node[above] {...} + } + child {node {$B$} + edge from parent + node[above] {...} + } + edge from parent + node[above] {...} + } + child[missing] {} + child { node {$S$} + child {node {$W$} + edge from parent + node[above] {...} + } + child {node {$B$} + edge from parent + node[above] {...} + } + edge from parent + node[above] {...} + } ; + \end{tikzpicture} + \end{center} + + \item Calculer la probabilité de l'évènement $M \cap W$. Interpréter ce résultat dans le contexte de l'exercice. + \item Montrer que la probabilité que la fiche choisie corresponde à une formule \emph{Burger} est égale à $0.7573$. + \item On a prélevé une fiche correspondant à la formule \emph{Burger}. Quelle est la probabilité, arrondie au millième, que la vente ait eu lieu le soir? + \end{enumerate} +\end{exercise} + +\begin{solution} + \begin{enumerate} + \item + \begin{center} + \begin{tikzpicture}[sloped] + \node {.} + child {node {$M$} + child {node {$W$} + edge from parent + node[above] {$0.75$} + } + child {node {$B$} + edge from parent + node[above] {$0.25$} + } + edge from parent + node[above] {$0.11$} + } + child[missing] {} + child { node {$S$} + child {node {$W$} + edge from parent + node[above] {$0.18$} + } + child {node {$B$} + edge from parent + node[above] {$0.82$} + } + edge from parent + node[above] {$0.89$} + } ; + \end{tikzpicture} + \end{center} + \item On calcule la probabilité que la vente soit un wok et ait eu lieu à midi + \[ P(M\cap W) = P(M) \times P_M(W) = 0.11 \times 0.75 = 0.0825 \] + \item Probabilité que la vente soit un burger. + \[ + P(B) = P(M\cap B) + P(S\cap B) = 0.11 \times 0.75 + 0.89 \times 0.18 = 0.7573 + \] + \item On cherche à calculer la quantité $P_B(S)$. Pour cela on utilise la formule de Bayes + \[ + P_B(S) = \frac{P(B\cap S)}{P(B)} = \frac{P_S(B) \times P(S)}{P(B)} = \frac{0.82\times 0.89}{0.7573} = 0.9636867819886439 \approx 0.964 + \] + \end{enumerate} +\end{solution} + +\begin{exercise}[subtitle={Continent plastique}] + \textit{Les quantités évoqués dans cette exercice sont générés au hasard et sont donc complètement farfelus.} + \medskip + Le \og continent de plastique\fg{} est la plus grande des plaques de déchets plastiques évoluant sur les océans. Elle occupe actuellement dans l'océan Pacifique une surface dont l'aire est évaluée à plus de $1,6$ million de km$^2$, entre Hawaï et la Californie. + + En 2017, des scientifiques ont estimé qu'il y avait $19$ millions de tonnes de déchets plastiques qui était déversé chaque année dans les océans et que cette quantité augmentait de $22\n\%$ par chaque année. + + On modélise l'évolution de la masse de ces déchets plastiques déversée chaque année, si rien n'est fait pour la réduire, par une suite géométrique $\left(u_n\right)$. L'arrondi au centième du terme $u_n$ représente la masse de ces déchets déversée chaque année, exprimée en million de tonnes, pour l'année $(2017 + n)$. + + \medskip + + \begin{enumerate} + \item Expliquer pourquoi la suite $u_n$ est géométrique? + \item Calculer $u_1$ et $u_2$. + \item Exprimer $u_n$ en fonction de $n$. + \item Au début de l'année 2017, il y avait $300$ millions de tonnes de déchets plastique. Calculer la quantité totale de déchets plastiques en 2030. + \item On souhaite déterminer en quelle année la masse totale de ces déchets plastiques aura pour la première fois augmenté de $50$\,\% par rapport à sa valeur de 2017. + \begin{enumerate} + \item Recopier et compléter l'algorithme ci-dessous pour que la variable $N$ contienne la réponse au problème posé. + + \begin{center} + \begin{tabularx}{0.4\linewidth}{|X|}\hline + $N = 2017$\\ + $U = 19$ \\ + $S = 300 + U$ \\ + while $S < 450$: \\ + \hspace{1cm} $N = \ldots$\\ + \hspace{1cm} $U = \ldots$\\ + \hspace{1cm} $S = \ldots$\\ + \hline + \end{tabularx} + \end{center} + \item Que contiennent les variables $S$, $U$ et $N$ après exécution de cet algorithme ? + + Interpréter les résultats dans le contexte de l'exercice. + \end{enumerate} + \end{enumerate} +\end{exercise} + +\begin{solution} + \begin{enumerate} + \item Une augmentation de $22\,\%$ revient à multiplier la quantité par $1.22$. La suite est donc bien géométrique. Son premier terme est $u_0 = 19$ et sa raison est $q = 1.22$ + \item + \[ + u_1 = u_0 * 1.22 = 23.18 + \] + \[ + u_2 = u_0 * 1.22^2 = 28.2796 + \] + \item + \[ + u_n = u_0 \times q^n = 19 \times 1.22^n + \] + \item On calcule la quantité totale déversée entre 2017 et 2030. + \[ + \sum_{n = 0}^{13} u_n = u_0 \times \frac{1-q^{13}}{1-q} = 19 \times \frac{1 - 1.22^{13}}{1 - 1.22} = 1059.17 + \] + On en déduit la quantité totale de déchets en 2030 + \[ + 300 + 1059.17 = 1359.17 + \] + \item + \begin{enumerate} + \item ~ + \begin{center} + \begin{tabularx}{0.4\linewidth}{|X|}\hline + $N \gets 2017$\\ + $U \gets 19$ \\ + $S \gets 300 + U$ \\ + Tant que $S < 450$ \\ + \hspace{1cm} $N \gets N + 1$\\ + \hspace{1cm} $U \gets U * 1.22$\\ + \hspace{1cm} $S \gets S + u$\\ + Fin Tant que\\\hline + \end{tabularx} + \end{center} + \item \textit{Pas de correction automatisé} + \end{enumerate} + \end{enumerate} +\end{solution} + +\end{document} + +%%% Local Variables: +%%% mode: latex +%%% TeX-master: "master" +%%% End: diff --git a/TST/DS/DS_21_04_07/TST3/corr_15_210407_DS8.tex b/TST/DS/DS_21_04_07/TST3/corr_15_210407_DS8.tex new file mode 100644 index 0000000..5e9411c --- /dev/null +++ b/TST/DS/DS_21_04_07/TST3/corr_15_210407_DS8.tex @@ -0,0 +1,262 @@ +\documentclass[a4paper,10pt]{article} +\usepackage{myXsim} + +% Title Page +\title{DS8 \hfill RADOUAA Saleh} +\tribe{TST} +\date{\hfillÀ render pour le Mercredi 7 avril} + +\xsimsetup{ + solution/print = true +} + +\begin{document} +\maketitle + +\begin{exercise}[subtitle={Automatismes}] + \textit{Toutes les questions de cette exercice sont indépendantes et peuvent être répondus séparément} + \begin{enumerate} + \item De janvier à septembre, une quantité a augmenté de $27\,\%$. Faire un schéma pour représenter la situation puis calculer le taux d'évolution moyen mensuel. + \item Une quantité augmente de $27\,\%$ par ans. En 2020, elle est de 141\euro. Quelle était sa valeur en 2019? Faire un schéma pour représenter la situation. + \item Déterminer l'équation de la droite \\ + \begin{tikzpicture}[xscale=0.8, yscale=0.5] + \tkzInit[xmin=-5,xmax=5,xstep=1, + ymin=-5,ymax=5,ystep=1] + \tkzGrid + \tkzAxeXY + \tkzFct[domain=-5:5,color=red,very thick]% + {2.0*\x -4}; + \end{tikzpicture} + \item Résoudre l'équation $2 \times 0.88^x = 8$ + \end{enumerate} +\end{exercise} + +\begin{solution} + \begin{enumerate} + \item On veut partager cette évolution en 8 évolutions. + \[ + \left(1 + \frac{27}{100}\right)^{\frac{1}{8}} = 1.0303 + \] + Donc le taux d'évolution moyen est + \[ + t_m = 1.0303 - 1 = 0.030299999999999994 + \] + \item Coefficient multiplicateur pour revenir en arrière + \[ + CM = (1 + \frac{27}{100})^{-1} = 0.7874 + \] + On en déduit la quantité en 2019 + \[ + 141 * 0.7874 = 111.0234 + \] + \item L'équation de la droite est + \[ + y = 2.0 x -4 + \] + \item Il faut penser à faire la division à par $2$ avant d'utiliser le log car sinon, on ne peut pas utiliser la formule $\log(a^n) = n\times \log(a)$. + + \[x = \frac{\log(4.0)}{\log(0.88)}\] + \end{enumerate} +\end{solution} + +\begin{exercise}[subtitle={Restaurant}] + Un \emph{food truck}, ouvert le midi et le soir, propose deux types de formules : + + \setlength\parindent{10mm} + \begin{itemize} + \item la formule \emph{Burger} ; + \item la formule \emph{Wok}. + \end{itemize} + \setlength\parindent{0mm} + + \medskip + + Le gérant a remarqué que 1\,\% de ses ventes ont lieu le midi. Le quart des ventes du midi correspondent à la formule \emph{Burger}, alors que 99\,\% des ventes du soir correspondent à la formule \emph{Wok}. + + Le gérant se constitue un fichier en notant, pour chaque vente, la formule choisie et le moment de cette vente (midi ou soir). + + On prélève une fiche de façon équiprobable. On définit les quatre évènements suivants: + + \begin{enumerate} + \item $M$ : \og la fiche correspond à une vente du midi\fg{} ; + \item $S$ : \og la fiche correspond à une vente du soir\fg {}; + \item $W$ : \og la fiche correspond à une formule \emph{Wok} \fg{} ; + \item $B$ : \og la fiche correspond à une formule \emph{Burger} \fg. + \end{enumerate} + \setlength\parindent{0mm} + + \medskip + + \begin{enumerate} + \item Recopier puis compléter l'arbre pondéré + + \begin{center} + \begin{tikzpicture}[sloped] + \node {.} + child {node {$M$} + child {node {$W$} + edge from parent + node[above] {...} + } + child {node {$B$} + edge from parent + node[above] {...} + } + edge from parent + node[above] {...} + } + child[missing] {} + child { node {$S$} + child {node {$W$} + edge from parent + node[above] {...} + } + child {node {$B$} + edge from parent + node[above] {...} + } + edge from parent + node[above] {...} + } ; + \end{tikzpicture} + \end{center} + + \item Calculer la probabilité de l'évènement $M \cap W$. Interpréter ce résultat dans le contexte de l'exercice. + \item Montrer que la probabilité que la fiche choisie corresponde à une formule \emph{Burger} est égale à $0.0124$. + \item On a prélevé une fiche correspondant à la formule \emph{Burger}. Quelle est la probabilité, arrondie au millième, que la vente ait eu lieu le soir? + \end{enumerate} +\end{exercise} + +\begin{solution} + \begin{enumerate} + \item + \begin{center} + \begin{tikzpicture}[sloped] + \node {.} + child {node {$M$} + child {node {$W$} + edge from parent + node[above] {$0.75$} + } + child {node {$B$} + edge from parent + node[above] {$0.25$} + } + edge from parent + node[above] {$0.01$} + } + child[missing] {} + child { node {$S$} + child {node {$W$} + edge from parent + node[above] {$0.99$} + } + child {node {$B$} + edge from parent + node[above] {$0.01$} + } + edge from parent + node[above] {$0.99$} + } ; + \end{tikzpicture} + \end{center} + \item On calcule la probabilité que la vente soit un wok et ait eu lieu à midi + \[ P(M\cap W) = P(M) \times P_M(W) = 0.01 \times 0.75 = 0.0075 \] + \item Probabilité que la vente soit un burger. + \[ + P(B) = P(M\cap B) + P(S\cap B) = 0.01 \times 0.75 + 0.99 \times 0.99 = 0.0124 + \] + \item On cherche à calculer la quantité $P_B(S)$. Pour cela on utilise la formule de Bayes + \[ + P_B(S) = \frac{P(B\cap S)}{P(B)} = \frac{P_S(B) \times P(S)}{P(B)} = \frac{0.01\times 0.99}{0.0124} = 0.7983870967741936 \approx 0.798 + \] + \end{enumerate} +\end{solution} + +\begin{exercise}[subtitle={Continent plastique}] + \textit{Les quantités évoqués dans cette exercice sont générés au hasard et sont donc complètement farfelus.} + \medskip + Le \og continent de plastique\fg{} est la plus grande des plaques de déchets plastiques évoluant sur les océans. Elle occupe actuellement dans l'océan Pacifique une surface dont l'aire est évaluée à plus de $1,6$ million de km$^2$, entre Hawaï et la Californie. + + En 2017, des scientifiques ont estimé qu'il y avait $5$ millions de tonnes de déchets plastiques qui était déversé chaque année dans les océans et que cette quantité augmentait de $17\n\%$ par chaque année. + + On modélise l'évolution de la masse de ces déchets plastiques déversée chaque année, si rien n'est fait pour la réduire, par une suite géométrique $\left(u_n\right)$. L'arrondi au centième du terme $u_n$ représente la masse de ces déchets déversée chaque année, exprimée en million de tonnes, pour l'année $(2017 + n)$. + + \medskip + + \begin{enumerate} + \item Expliquer pourquoi la suite $u_n$ est géométrique? + \item Calculer $u_1$ et $u_2$. + \item Exprimer $u_n$ en fonction de $n$. + \item Au début de l'année 2017, il y avait $300$ millions de tonnes de déchets plastique. Calculer la quantité totale de déchets plastiques en 2030. + \item On souhaite déterminer en quelle année la masse totale de ces déchets plastiques aura pour la première fois augmenté de $50$\,\% par rapport à sa valeur de 2017. + \begin{enumerate} + \item Recopier et compléter l'algorithme ci-dessous pour que la variable $N$ contienne la réponse au problème posé. + + \begin{center} + \begin{tabularx}{0.4\linewidth}{|X|}\hline + $N = 2017$\\ + $U = 5$ \\ + $S = 300 + U$ \\ + while $S < 450$: \\ + \hspace{1cm} $N = \ldots$\\ + \hspace{1cm} $U = \ldots$\\ + \hspace{1cm} $S = \ldots$\\ + \hline + \end{tabularx} + \end{center} + \item Que contiennent les variables $S$, $U$ et $N$ après exécution de cet algorithme ? + + Interpréter les résultats dans le contexte de l'exercice. + \end{enumerate} + \end{enumerate} +\end{exercise} + +\begin{solution} + \begin{enumerate} + \item Une augmentation de $17\,\%$ revient à multiplier la quantité par $1.17$. La suite est donc bien géométrique. Son premier terme est $u_0 = 5$ et sa raison est $q = 1.17$ + \item + \[ + u_1 = u_0 * 1.17 = 5.85 + \] + \[ + u_2 = u_0 * 1.17^2 = 6.8445 + \] + \item + \[ + u_n = u_0 \times q^n = 5 \times 1.17^n + \] + \item On calcule la quantité totale déversée entre 2017 et 2030. + \[ + \sum_{n = 0}^{13} u_n = u_0 \times \frac{1-q^{13}}{1-q} = 5 \times \frac{1 - 1.17^{13}}{1 - 1.17} = 197.02 + \] + On en déduit la quantité totale de déchets en 2030 + \[ + 300 + 197.02 = 497.02 + \] + \item + \begin{enumerate} + \item ~ + \begin{center} + \begin{tabularx}{0.4\linewidth}{|X|}\hline + $N \gets 2017$\\ + $U \gets 5$ \\ + $S \gets 300 + U$ \\ + Tant que $S < 450$ \\ + \hspace{1cm} $N \gets N + 1$\\ + \hspace{1cm} $U \gets U * 1.17$\\ + \hspace{1cm} $S \gets S + u$\\ + Fin Tant que\\\hline + \end{tabularx} + \end{center} + \item \textit{Pas de correction automatisé} + \end{enumerate} + \end{enumerate} +\end{solution} + +\end{document} + +%%% Local Variables: +%%% mode: latex +%%% TeX-master: "master" +%%% End: diff --git a/TST/DS/DS_21_04_07/TST3/corr_16_210407_DS8.tex b/TST/DS/DS_21_04_07/TST3/corr_16_210407_DS8.tex new file mode 100644 index 0000000..dc9ab83 --- /dev/null +++ b/TST/DS/DS_21_04_07/TST3/corr_16_210407_DS8.tex @@ -0,0 +1,262 @@ +\documentclass[a4paper,10pt]{article} +\usepackage{myXsim} + +% Title Page +\title{DS8 \hfill TAY Ummuhan} +\tribe{TST} +\date{\hfillÀ render pour le Mercredi 7 avril} + +\xsimsetup{ + solution/print = true +} + +\begin{document} +\maketitle + +\begin{exercise}[subtitle={Automatismes}] + \textit{Toutes les questions de cette exercice sont indépendantes et peuvent être répondus séparément} + \begin{enumerate} + \item De janvier à septembre, une quantité a augmenté de $14\,\%$. Faire un schéma pour représenter la situation puis calculer le taux d'évolution moyen mensuel. + \item Une quantité augmente de $14\,\%$ par ans. En 2020, elle est de 116\euro. Quelle était sa valeur en 2019? Faire un schéma pour représenter la situation. + \item Déterminer l'équation de la droite \\ + \begin{tikzpicture}[xscale=0.8, yscale=0.5] + \tkzInit[xmin=-5,xmax=5,xstep=1, + ymin=-5,ymax=5,ystep=1] + \tkzGrid + \tkzAxeXY + \tkzFct[domain=-5:5,color=red,very thick]% + {1.3333333333333333*\x -2}; + \end{tikzpicture} + \item Résoudre l'équation $6 \times 0.14^x = 14$ + \end{enumerate} +\end{exercise} + +\begin{solution} + \begin{enumerate} + \item On veut partager cette évolution en 8 évolutions. + \[ + \left(1 + \frac{14}{100}\right)^{\frac{1}{8}} = 1.0165 + \] + Donc le taux d'évolution moyen est + \[ + t_m = 1.0165 - 1 = 0.01649999999999996 + \] + \item Coefficient multiplicateur pour revenir en arrière + \[ + CM = (1 + \frac{14}{100})^{-1} = 0.8772 + \] + On en déduit la quantité en 2019 + \[ + 116 * 0.8772 = 101.7552 + \] + \item L'équation de la droite est + \[ + y = 1.3333333333333333 x -2 + \] + \item Il faut penser à faire la division à par $6$ avant d'utiliser le log car sinon, on ne peut pas utiliser la formule $\log(a^n) = n\times \log(a)$. + + \[x = \frac{\log(2.33)}{\log(0.14)}\] + \end{enumerate} +\end{solution} + +\begin{exercise}[subtitle={Restaurant}] + Un \emph{food truck}, ouvert le midi et le soir, propose deux types de formules : + + \setlength\parindent{10mm} + \begin{itemize} + \item la formule \emph{Burger} ; + \item la formule \emph{Wok}. + \end{itemize} + \setlength\parindent{0mm} + + \medskip + + Le gérant a remarqué que 28\,\% de ses ventes ont lieu le midi. Le quart des ventes du midi correspondent à la formule \emph{Burger}, alors que 80\,\% des ventes du soir correspondent à la formule \emph{Wok}. + + Le gérant se constitue un fichier en notant, pour chaque vente, la formule choisie et le moment de cette vente (midi ou soir). + + On prélève une fiche de façon équiprobable. On définit les quatre évènements suivants: + + \begin{enumerate} + \item $M$ : \og la fiche correspond à une vente du midi\fg{} ; + \item $S$ : \og la fiche correspond à une vente du soir\fg {}; + \item $W$ : \og la fiche correspond à une formule \emph{Wok} \fg{} ; + \item $B$ : \og la fiche correspond à une formule \emph{Burger} \fg. + \end{enumerate} + \setlength\parindent{0mm} + + \medskip + + \begin{enumerate} + \item Recopier puis compléter l'arbre pondéré + + \begin{center} + \begin{tikzpicture}[sloped] + \node {.} + child {node {$M$} + child {node {$W$} + edge from parent + node[above] {...} + } + child {node {$B$} + edge from parent + node[above] {...} + } + edge from parent + node[above] {...} + } + child[missing] {} + child { node {$S$} + child {node {$W$} + edge from parent + node[above] {...} + } + child {node {$B$} + edge from parent + node[above] {...} + } + edge from parent + node[above] {...} + } ; + \end{tikzpicture} + \end{center} + + \item Calculer la probabilité de l'évènement $M \cap W$. Interpréter ce résultat dans le contexte de l'exercice. + \item Montrer que la probabilité que la fiche choisie corresponde à une formule \emph{Burger} est égale à $0.214$. + \item On a prélevé une fiche correspondant à la formule \emph{Burger}. Quelle est la probabilité, arrondie au millième, que la vente ait eu lieu le soir? + \end{enumerate} +\end{exercise} + +\begin{solution} + \begin{enumerate} + \item + \begin{center} + \begin{tikzpicture}[sloped] + \node {.} + child {node {$M$} + child {node {$W$} + edge from parent + node[above] {$0.75$} + } + child {node {$B$} + edge from parent + node[above] {$0.25$} + } + edge from parent + node[above] {$0.28$} + } + child[missing] {} + child { node {$S$} + child {node {$W$} + edge from parent + node[above] {$0.8$} + } + child {node {$B$} + edge from parent + node[above] {$0.2$} + } + edge from parent + node[above] {$0.72$} + } ; + \end{tikzpicture} + \end{center} + \item On calcule la probabilité que la vente soit un wok et ait eu lieu à midi + \[ P(M\cap W) = P(M) \times P_M(W) = 0.28 \times 0.75 = 0.21 \] + \item Probabilité que la vente soit un burger. + \[ + P(B) = P(M\cap B) + P(S\cap B) = 0.28 \times 0.75 + 0.72 \times 0.8 = 0.214 + \] + \item On cherche à calculer la quantité $P_B(S)$. Pour cela on utilise la formule de Bayes + \[ + P_B(S) = \frac{P(B\cap S)}{P(B)} = \frac{P_S(B) \times P(S)}{P(B)} = \frac{0.2\times 0.72}{0.214} = 0.6728971962616822 \approx 0.673 + \] + \end{enumerate} +\end{solution} + +\begin{exercise}[subtitle={Continent plastique}] + \textit{Les quantités évoqués dans cette exercice sont générés au hasard et sont donc complètement farfelus.} + \medskip + Le \og continent de plastique\fg{} est la plus grande des plaques de déchets plastiques évoluant sur les océans. Elle occupe actuellement dans l'océan Pacifique une surface dont l'aire est évaluée à plus de $1,6$ million de km$^2$, entre Hawaï et la Californie. + + En 2017, des scientifiques ont estimé qu'il y avait $7$ millions de tonnes de déchets plastiques qui était déversé chaque année dans les océans et que cette quantité augmentait de $26\n\%$ par chaque année. + + On modélise l'évolution de la masse de ces déchets plastiques déversée chaque année, si rien n'est fait pour la réduire, par une suite géométrique $\left(u_n\right)$. L'arrondi au centième du terme $u_n$ représente la masse de ces déchets déversée chaque année, exprimée en million de tonnes, pour l'année $(2017 + n)$. + + \medskip + + \begin{enumerate} + \item Expliquer pourquoi la suite $u_n$ est géométrique? + \item Calculer $u_1$ et $u_2$. + \item Exprimer $u_n$ en fonction de $n$. + \item Au début de l'année 2017, il y avait $300$ millions de tonnes de déchets plastique. Calculer la quantité totale de déchets plastiques en 2030. + \item On souhaite déterminer en quelle année la masse totale de ces déchets plastiques aura pour la première fois augmenté de $50$\,\% par rapport à sa valeur de 2017. + \begin{enumerate} + \item Recopier et compléter l'algorithme ci-dessous pour que la variable $N$ contienne la réponse au problème posé. + + \begin{center} + \begin{tabularx}{0.4\linewidth}{|X|}\hline + $N = 2017$\\ + $U = 7$ \\ + $S = 300 + U$ \\ + while $S < 450$: \\ + \hspace{1cm} $N = \ldots$\\ + \hspace{1cm} $U = \ldots$\\ + \hspace{1cm} $S = \ldots$\\ + \hline + \end{tabularx} + \end{center} + \item Que contiennent les variables $S$, $U$ et $N$ après exécution de cet algorithme ? + + Interpréter les résultats dans le contexte de l'exercice. + \end{enumerate} + \end{enumerate} +\end{exercise} + +\begin{solution} + \begin{enumerate} + \item Une augmentation de $26\,\%$ revient à multiplier la quantité par $1.26$. La suite est donc bien géométrique. Son premier terme est $u_0 = 7$ et sa raison est $q = 1.26$ + \item + \[ + u_1 = u_0 * 1.26 = 8.82 + \] + \[ + u_2 = u_0 * 1.26^2 = 11.1132 + \] + \item + \[ + u_n = u_0 \times q^n = 7 \times 1.26^n + \] + \item On calcule la quantité totale déversée entre 2017 et 2030. + \[ + \sum_{n = 0}^{13} u_n = u_0 \times \frac{1-q^{13}}{1-q} = 7 \times \frac{1 - 1.26^{13}}{1 - 1.26} = 516.25 + \] + On en déduit la quantité totale de déchets en 2030 + \[ + 300 + 516.25 = 816.25 + \] + \item + \begin{enumerate} + \item ~ + \begin{center} + \begin{tabularx}{0.4\linewidth}{|X|}\hline + $N \gets 2017$\\ + $U \gets 7$ \\ + $S \gets 300 + U$ \\ + Tant que $S < 450$ \\ + \hspace{1cm} $N \gets N + 1$\\ + \hspace{1cm} $U \gets U * 1.26$\\ + \hspace{1cm} $S \gets S + u$\\ + Fin Tant que\\\hline + \end{tabularx} + \end{center} + \item \textit{Pas de correction automatisé} + \end{enumerate} + \end{enumerate} +\end{solution} + +\end{document} + +%%% Local Variables: +%%% mode: latex +%%% TeX-master: "master" +%%% End: diff --git a/TST/DS/DS_21_04_07/TST3/corr_17_210407_DS8.tex b/TST/DS/DS_21_04_07/TST3/corr_17_210407_DS8.tex new file mode 100644 index 0000000..f9158a6 --- /dev/null +++ b/TST/DS/DS_21_04_07/TST3/corr_17_210407_DS8.tex @@ -0,0 +1,262 @@ +\documentclass[a4paper,10pt]{article} +\usepackage{myXsim} + +% Title Page +\title{DS8 \hfill VIALON-DUPERRON Victorien} +\tribe{TST} +\date{\hfillÀ render pour le Mercredi 7 avril} + +\xsimsetup{ + solution/print = true +} + +\begin{document} +\maketitle + +\begin{exercise}[subtitle={Automatismes}] + \textit{Toutes les questions de cette exercice sont indépendantes et peuvent être répondus séparément} + \begin{enumerate} + \item De janvier à septembre, une quantité a augmenté de $20\,\%$. Faire un schéma pour représenter la situation puis calculer le taux d'évolution moyen mensuel. + \item Une quantité augmente de $20\,\%$ par ans. En 2020, elle est de 137\euro. Quelle était sa valeur en 2019? Faire un schéma pour représenter la situation. + \item Déterminer l'équation de la droite \\ + \begin{tikzpicture}[xscale=0.8, yscale=0.5] + \tkzInit[xmin=-5,xmax=5,xstep=1, + ymin=-5,ymax=5,ystep=1] + \tkzGrid + \tkzAxeXY + \tkzFct[domain=-5:5,color=red,very thick]% + {2.6666666666666665*\x -4}; + \end{tikzpicture} + \item Résoudre l'équation $10 \times 0.26^x = 42$ + \end{enumerate} +\end{exercise} + +\begin{solution} + \begin{enumerate} + \item On veut partager cette évolution en 8 évolutions. + \[ + \left(1 + \frac{20}{100}\right)^{\frac{1}{8}} = 1.0231 + \] + Donc le taux d'évolution moyen est + \[ + t_m = 1.0231 - 1 = 0.0230999999999999 + \] + \item Coefficient multiplicateur pour revenir en arrière + \[ + CM = (1 + \frac{20}{100})^{-1} = 0.8333 + \] + On en déduit la quantité en 2019 + \[ + 137 * 0.8333 = 114.16210000000001 + \] + \item L'équation de la droite est + \[ + y = 2.6666666666666665 x -4 + \] + \item Il faut penser à faire la division à par $10$ avant d'utiliser le log car sinon, on ne peut pas utiliser la formule $\log(a^n) = n\times \log(a)$. + + \[x = \frac{\log(4.2)}{\log(0.26)}\] + \end{enumerate} +\end{solution} + +\begin{exercise}[subtitle={Restaurant}] + Un \emph{food truck}, ouvert le midi et le soir, propose deux types de formules : + + \setlength\parindent{10mm} + \begin{itemize} + \item la formule \emph{Burger} ; + \item la formule \emph{Wok}. + \end{itemize} + \setlength\parindent{0mm} + + \medskip + + Le gérant a remarqué que 87\,\% de ses ventes ont lieu le midi. Le quart des ventes du midi correspondent à la formule \emph{Burger}, alors que 47\,\% des ventes du soir correspondent à la formule \emph{Wok}. + + Le gérant se constitue un fichier en notant, pour chaque vente, la formule choisie et le moment de cette vente (midi ou soir). + + On prélève une fiche de façon équiprobable. On définit les quatre évènements suivants: + + \begin{enumerate} + \item $M$ : \og la fiche correspond à une vente du midi\fg{} ; + \item $S$ : \og la fiche correspond à une vente du soir\fg {}; + \item $W$ : \og la fiche correspond à une formule \emph{Wok} \fg{} ; + \item $B$ : \og la fiche correspond à une formule \emph{Burger} \fg. + \end{enumerate} + \setlength\parindent{0mm} + + \medskip + + \begin{enumerate} + \item Recopier puis compléter l'arbre pondéré + + \begin{center} + \begin{tikzpicture}[sloped] + \node {.} + child {node {$M$} + child {node {$W$} + edge from parent + node[above] {...} + } + child {node {$B$} + edge from parent + node[above] {...} + } + edge from parent + node[above] {...} + } + child[missing] {} + child { node {$S$} + child {node {$W$} + edge from parent + node[above] {...} + } + child {node {$B$} + edge from parent + node[above] {...} + } + edge from parent + node[above] {...} + } ; + \end{tikzpicture} + \end{center} + + \item Calculer la probabilité de l'évènement $M \cap W$. Interpréter ce résultat dans le contexte de l'exercice. + \item Montrer que la probabilité que la fiche choisie corresponde à une formule \emph{Burger} est égale à $0.2864$. + \item On a prélevé une fiche correspondant à la formule \emph{Burger}. Quelle est la probabilité, arrondie au millième, que la vente ait eu lieu le soir? + \end{enumerate} +\end{exercise} + +\begin{solution} + \begin{enumerate} + \item + \begin{center} + \begin{tikzpicture}[sloped] + \node {.} + child {node {$M$} + child {node {$W$} + edge from parent + node[above] {$0.75$} + } + child {node {$B$} + edge from parent + node[above] {$0.25$} + } + edge from parent + node[above] {$0.87$} + } + child[missing] {} + child { node {$S$} + child {node {$W$} + edge from parent + node[above] {$0.47$} + } + child {node {$B$} + edge from parent + node[above] {$0.53$} + } + edge from parent + node[above] {$0.13$} + } ; + \end{tikzpicture} + \end{center} + \item On calcule la probabilité que la vente soit un wok et ait eu lieu à midi + \[ P(M\cap W) = P(M) \times P_M(W) = 0.87 \times 0.75 = 0.6525 \] + \item Probabilité que la vente soit un burger. + \[ + P(B) = P(M\cap B) + P(S\cap B) = 0.87 \times 0.75 + 0.13 \times 0.47 = 0.2864 + \] + \item On cherche à calculer la quantité $P_B(S)$. Pour cela on utilise la formule de Bayes + \[ + P_B(S) = \frac{P(B\cap S)}{P(B)} = \frac{P_S(B) \times P(S)}{P(B)} = \frac{0.53\times 0.13}{0.2864} = 0.24057262569832405 \approx 0.241 + \] + \end{enumerate} +\end{solution} + +\begin{exercise}[subtitle={Continent plastique}] + \textit{Les quantités évoqués dans cette exercice sont générés au hasard et sont donc complètement farfelus.} + \medskip + Le \og continent de plastique\fg{} est la plus grande des plaques de déchets plastiques évoluant sur les océans. Elle occupe actuellement dans l'océan Pacifique une surface dont l'aire est évaluée à plus de $1,6$ million de km$^2$, entre Hawaï et la Californie. + + En 2017, des scientifiques ont estimé qu'il y avait $14$ millions de tonnes de déchets plastiques qui était déversé chaque année dans les océans et que cette quantité augmentait de $17\n\%$ par chaque année. + + On modélise l'évolution de la masse de ces déchets plastiques déversée chaque année, si rien n'est fait pour la réduire, par une suite géométrique $\left(u_n\right)$. L'arrondi au centième du terme $u_n$ représente la masse de ces déchets déversée chaque année, exprimée en million de tonnes, pour l'année $(2017 + n)$. + + \medskip + + \begin{enumerate} + \item Expliquer pourquoi la suite $u_n$ est géométrique? + \item Calculer $u_1$ et $u_2$. + \item Exprimer $u_n$ en fonction de $n$. + \item Au début de l'année 2017, il y avait $300$ millions de tonnes de déchets plastique. Calculer la quantité totale de déchets plastiques en 2030. + \item On souhaite déterminer en quelle année la masse totale de ces déchets plastiques aura pour la première fois augmenté de $50$\,\% par rapport à sa valeur de 2017. + \begin{enumerate} + \item Recopier et compléter l'algorithme ci-dessous pour que la variable $N$ contienne la réponse au problème posé. + + \begin{center} + \begin{tabularx}{0.4\linewidth}{|X|}\hline + $N = 2017$\\ + $U = 14$ \\ + $S = 300 + U$ \\ + while $S < 450$: \\ + \hspace{1cm} $N = \ldots$\\ + \hspace{1cm} $U = \ldots$\\ + \hspace{1cm} $S = \ldots$\\ + \hline + \end{tabularx} + \end{center} + \item Que contiennent les variables $S$, $U$ et $N$ après exécution de cet algorithme ? + + Interpréter les résultats dans le contexte de l'exercice. + \end{enumerate} + \end{enumerate} +\end{exercise} + +\begin{solution} + \begin{enumerate} + \item Une augmentation de $17\,\%$ revient à multiplier la quantité par $1.17$. La suite est donc bien géométrique. Son premier terme est $u_0 = 14$ et sa raison est $q = 1.17$ + \item + \[ + u_1 = u_0 * 1.17 = 16.38 + \] + \[ + u_2 = u_0 * 1.17^2 = 19.1646 + \] + \item + \[ + u_n = u_0 \times q^n = 14 \times 1.17^n + \] + \item On calcule la quantité totale déversée entre 2017 et 2030. + \[ + \sum_{n = 0}^{13} u_n = u_0 \times \frac{1-q^{13}}{1-q} = 14 \times \frac{1 - 1.17^{13}}{1 - 1.17} = 551.66 + \] + On en déduit la quantité totale de déchets en 2030 + \[ + 300 + 551.66 = 851.66 + \] + \item + \begin{enumerate} + \item ~ + \begin{center} + \begin{tabularx}{0.4\linewidth}{|X|}\hline + $N \gets 2017$\\ + $U \gets 14$ \\ + $S \gets 300 + U$ \\ + Tant que $S < 450$ \\ + \hspace{1cm} $N \gets N + 1$\\ + \hspace{1cm} $U \gets U * 1.17$\\ + \hspace{1cm} $S \gets S + u$\\ + Fin Tant que\\\hline + \end{tabularx} + \end{center} + \item \textit{Pas de correction automatisé} + \end{enumerate} + \end{enumerate} +\end{solution} + +\end{document} + +%%% Local Variables: +%%% mode: latex +%%% TeX-master: "master" +%%% End: diff --git a/TST/DS/DS_21_04_07/TST3/corr_18_210407_DS8.tex b/TST/DS/DS_21_04_07/TST3/corr_18_210407_DS8.tex new file mode 100644 index 0000000..8e911c1 --- /dev/null +++ b/TST/DS/DS_21_04_07/TST3/corr_18_210407_DS8.tex @@ -0,0 +1,262 @@ +\documentclass[a4paper,10pt]{article} +\usepackage{myXsim} + +% Title Page +\title{DS8 \hfill ZENAGUI Yanis} +\tribe{TST} +\date{\hfillÀ render pour le Mercredi 7 avril} + +\xsimsetup{ + solution/print = true +} + +\begin{document} +\maketitle + +\begin{exercise}[subtitle={Automatismes}] + \textit{Toutes les questions de cette exercice sont indépendantes et peuvent être répondus séparément} + \begin{enumerate} + \item De janvier à septembre, une quantité a augmenté de $29\,\%$. Faire un schéma pour représenter la situation puis calculer le taux d'évolution moyen mensuel. + \item Une quantité augmente de $29\,\%$ par ans. En 2020, elle est de 142\euro. Quelle était sa valeur en 2019? Faire un schéma pour représenter la situation. + \item Déterminer l'équation de la droite \\ + \begin{tikzpicture}[xscale=0.8, yscale=0.5] + \tkzInit[xmin=-5,xmax=5,xstep=1, + ymin=-5,ymax=5,ystep=1] + \tkzGrid + \tkzAxeXY + \tkzFct[domain=-5:5,color=red,very thick]% + {2.0*\x -4}; + \end{tikzpicture} + \item Résoudre l'équation $6 \times 0.41^x = 34$ + \end{enumerate} +\end{exercise} + +\begin{solution} + \begin{enumerate} + \item On veut partager cette évolution en 8 évolutions. + \[ + \left(1 + \frac{29}{100}\right)^{\frac{1}{8}} = 1.0323 + \] + Donc le taux d'évolution moyen est + \[ + t_m = 1.0323 - 1 = 0.032299999999999995 + \] + \item Coefficient multiplicateur pour revenir en arrière + \[ + CM = (1 + \frac{29}{100})^{-1} = 0.7752 + \] + On en déduit la quantité en 2019 + \[ + 142 * 0.7752 = 110.0784 + \] + \item L'équation de la droite est + \[ + y = 2.0 x -4 + \] + \item Il faut penser à faire la division à par $6$ avant d'utiliser le log car sinon, on ne peut pas utiliser la formule $\log(a^n) = n\times \log(a)$. + + \[x = \frac{\log(5.67)}{\log(0.41)}\] + \end{enumerate} +\end{solution} + +\begin{exercise}[subtitle={Restaurant}] + Un \emph{food truck}, ouvert le midi et le soir, propose deux types de formules : + + \setlength\parindent{10mm} + \begin{itemize} + \item la formule \emph{Burger} ; + \item la formule \emph{Wok}. + \end{itemize} + \setlength\parindent{0mm} + + \medskip + + Le gérant a remarqué que 30\,\% de ses ventes ont lieu le midi. Le quart des ventes du midi correspondent à la formule \emph{Burger}, alors que 33\,\% des ventes du soir correspondent à la formule \emph{Wok}. + + Le gérant se constitue un fichier en notant, pour chaque vente, la formule choisie et le moment de cette vente (midi ou soir). + + On prélève une fiche de façon équiprobable. On définit les quatre évènements suivants: + + \begin{enumerate} + \item $M$ : \og la fiche correspond à une vente du midi\fg{} ; + \item $S$ : \og la fiche correspond à une vente du soir\fg {}; + \item $W$ : \og la fiche correspond à une formule \emph{Wok} \fg{} ; + \item $B$ : \og la fiche correspond à une formule \emph{Burger} \fg. + \end{enumerate} + \setlength\parindent{0mm} + + \medskip + + \begin{enumerate} + \item Recopier puis compléter l'arbre pondéré + + \begin{center} + \begin{tikzpicture}[sloped] + \node {.} + child {node {$M$} + child {node {$W$} + edge from parent + node[above] {...} + } + child {node {$B$} + edge from parent + node[above] {...} + } + edge from parent + node[above] {...} + } + child[missing] {} + child { node {$S$} + child {node {$W$} + edge from parent + node[above] {...} + } + child {node {$B$} + edge from parent + node[above] {...} + } + edge from parent + node[above] {...} + } ; + \end{tikzpicture} + \end{center} + + \item Calculer la probabilité de l'évènement $M \cap W$. Interpréter ce résultat dans le contexte de l'exercice. + \item Montrer que la probabilité que la fiche choisie corresponde à une formule \emph{Burger} est égale à $0.544$. + \item On a prélevé une fiche correspondant à la formule \emph{Burger}. Quelle est la probabilité, arrondie au millième, que la vente ait eu lieu le soir? + \end{enumerate} +\end{exercise} + +\begin{solution} + \begin{enumerate} + \item + \begin{center} + \begin{tikzpicture}[sloped] + \node {.} + child {node {$M$} + child {node {$W$} + edge from parent + node[above] {$0.75$} + } + child {node {$B$} + edge from parent + node[above] {$0.25$} + } + edge from parent + node[above] {$0.3$} + } + child[missing] {} + child { node {$S$} + child {node {$W$} + edge from parent + node[above] {$0.33$} + } + child {node {$B$} + edge from parent + node[above] {$0.67$} + } + edge from parent + node[above] {$0.7$} + } ; + \end{tikzpicture} + \end{center} + \item On calcule la probabilité que la vente soit un wok et ait eu lieu à midi + \[ P(M\cap W) = P(M) \times P_M(W) = 0.3 \times 0.75 = 0.225 \] + \item Probabilité que la vente soit un burger. + \[ + P(B) = P(M\cap B) + P(S\cap B) = 0.3 \times 0.75 + 0.7 \times 0.33 = 0.544 + \] + \item On cherche à calculer la quantité $P_B(S)$. Pour cela on utilise la formule de Bayes + \[ + P_B(S) = \frac{P(B\cap S)}{P(B)} = \frac{P_S(B) \times P(S)}{P(B)} = \frac{0.67\times 0.7}{0.544} = 0.8621323529411764 \approx 0.862 + \] + \end{enumerate} +\end{solution} + +\begin{exercise}[subtitle={Continent plastique}] + \textit{Les quantités évoqués dans cette exercice sont générés au hasard et sont donc complètement farfelus.} + \medskip + Le \og continent de plastique\fg{} est la plus grande des plaques de déchets plastiques évoluant sur les océans. Elle occupe actuellement dans l'océan Pacifique une surface dont l'aire est évaluée à plus de $1,6$ million de km$^2$, entre Hawaï et la Californie. + + En 2017, des scientifiques ont estimé qu'il y avait $4$ millions de tonnes de déchets plastiques qui était déversé chaque année dans les océans et que cette quantité augmentait de $18\n\%$ par chaque année. + + On modélise l'évolution de la masse de ces déchets plastiques déversée chaque année, si rien n'est fait pour la réduire, par une suite géométrique $\left(u_n\right)$. L'arrondi au centième du terme $u_n$ représente la masse de ces déchets déversée chaque année, exprimée en million de tonnes, pour l'année $(2017 + n)$. + + \medskip + + \begin{enumerate} + \item Expliquer pourquoi la suite $u_n$ est géométrique? + \item Calculer $u_1$ et $u_2$. + \item Exprimer $u_n$ en fonction de $n$. + \item Au début de l'année 2017, il y avait $300$ millions de tonnes de déchets plastique. Calculer la quantité totale de déchets plastiques en 2030. + \item On souhaite déterminer en quelle année la masse totale de ces déchets plastiques aura pour la première fois augmenté de $50$\,\% par rapport à sa valeur de 2017. + \begin{enumerate} + \item Recopier et compléter l'algorithme ci-dessous pour que la variable $N$ contienne la réponse au problème posé. + + \begin{center} + \begin{tabularx}{0.4\linewidth}{|X|}\hline + $N = 2017$\\ + $U = 4$ \\ + $S = 300 + U$ \\ + while $S < 450$: \\ + \hspace{1cm} $N = \ldots$\\ + \hspace{1cm} $U = \ldots$\\ + \hspace{1cm} $S = \ldots$\\ + \hline + \end{tabularx} + \end{center} + \item Que contiennent les variables $S$, $U$ et $N$ après exécution de cet algorithme ? + + Interpréter les résultats dans le contexte de l'exercice. + \end{enumerate} + \end{enumerate} +\end{exercise} + +\begin{solution} + \begin{enumerate} + \item Une augmentation de $18\,\%$ revient à multiplier la quantité par $1.18$. La suite est donc bien géométrique. Son premier terme est $u_0 = 4$ et sa raison est $q = 1.18$ + \item + \[ + u_1 = u_0 * 1.18 = 4.72 + \] + \[ + u_2 = u_0 * 1.18^2 = 5.5696 + \] + \item + \[ + u_n = u_0 \times q^n = 4 \times 1.18^n + \] + \item On calcule la quantité totale déversée entre 2017 et 2030. + \[ + \sum_{n = 0}^{13} u_n = u_0 \times \frac{1-q^{13}}{1-q} = 4 \times \frac{1 - 1.18^{13}}{1 - 1.18} = 168.87 + \] + On en déduit la quantité totale de déchets en 2030 + \[ + 300 + 168.87 = 468.87 + \] + \item + \begin{enumerate} + \item ~ + \begin{center} + \begin{tabularx}{0.4\linewidth}{|X|}\hline + $N \gets 2017$\\ + $U \gets 4$ \\ + $S \gets 300 + U$ \\ + Tant que $S < 450$ \\ + \hspace{1cm} $N \gets N + 1$\\ + \hspace{1cm} $U \gets U * 1.18$\\ + \hspace{1cm} $S \gets S + u$\\ + Fin Tant que\\\hline + \end{tabularx} + \end{center} + \item \textit{Pas de correction automatisé} + \end{enumerate} + \end{enumerate} +\end{solution} + +\end{document} + +%%% Local Variables: +%%% mode: latex +%%% TeX-master: "master" +%%% End: diff --git a/TST/DS/DS_21_04_07/TST3/corr_19_210407_DS8.tex b/TST/DS/DS_21_04_07/TST3/corr_19_210407_DS8.tex new file mode 100644 index 0000000..122e844 --- /dev/null +++ b/TST/DS/DS_21_04_07/TST3/corr_19_210407_DS8.tex @@ -0,0 +1,262 @@ +\documentclass[a4paper,10pt]{article} +\usepackage{myXsim} + +% Title Page +\title{DS8 \hfill VIALON-DUPERRON Victorien} +\tribe{TST} +\date{\hfillÀ render pour le Mercredi 7 avril} + +\xsimsetup{ + solution/print = true +} + +\begin{document} +\maketitle + +\begin{exercise}[subtitle={Automatismes}] + \textit{Toutes les questions de cette exercice sont indépendantes et peuvent être répondus séparément} + \begin{enumerate} + \item De janvier à septembre, une quantité a augmenté de $10\,\%$. Faire un schéma pour représenter la situation puis calculer le taux d'évolution moyen mensuel. + \item Une quantité augmente de $10\,\%$ par ans. En 2020, elle est de 120\euro. Quelle était sa valeur en 2019? Faire un schéma pour représenter la situation. + \item Déterminer l'équation de la droite \\ + \begin{tikzpicture}[xscale=0.8, yscale=0.5] + \tkzInit[xmin=-5,xmax=5,xstep=1, + ymin=-5,ymax=5,ystep=1] + \tkzGrid + \tkzAxeXY + \tkzFct[domain=-5:5,color=red,very thick]% + {2.0*\x -3}; + \end{tikzpicture} + \item Résoudre l'équation $6 \times 0.69^x = 39$ + \end{enumerate} +\end{exercise} + +\begin{solution} + \begin{enumerate} + \item On veut partager cette évolution en 8 évolutions. + \[ + \left(1 + \frac{10}{100}\right)^{\frac{1}{8}} = 1.012 + \] + Donc le taux d'évolution moyen est + \[ + t_m = 1.012 - 1 = 0.01200000000000001 + \] + \item Coefficient multiplicateur pour revenir en arrière + \[ + CM = (1 + \frac{10}{100})^{-1} = 0.9091 + \] + On en déduit la quantité en 2019 + \[ + 120 * 0.9091 = 109.092 + \] + \item L'équation de la droite est + \[ + y = 2.0 x -3 + \] + \item Il faut penser à faire la division à par $6$ avant d'utiliser le log car sinon, on ne peut pas utiliser la formule $\log(a^n) = n\times \log(a)$. + + \[x = \frac{\log(6.5)}{\log(0.69)}\] + \end{enumerate} +\end{solution} + +\begin{exercise}[subtitle={Restaurant}] + Un \emph{food truck}, ouvert le midi et le soir, propose deux types de formules : + + \setlength\parindent{10mm} + \begin{itemize} + \item la formule \emph{Burger} ; + \item la formule \emph{Wok}. + \end{itemize} + \setlength\parindent{0mm} + + \medskip + + Le gérant a remarqué que 43\,\% de ses ventes ont lieu le midi. Le quart des ventes du midi correspondent à la formule \emph{Burger}, alors que 68\,\% des ventes du soir correspondent à la formule \emph{Wok}. + + Le gérant se constitue un fichier en notant, pour chaque vente, la formule choisie et le moment de cette vente (midi ou soir). + + On prélève une fiche de façon équiprobable. On définit les quatre évènements suivants: + + \begin{enumerate} + \item $M$ : \og la fiche correspond à une vente du midi\fg{} ; + \item $S$ : \og la fiche correspond à une vente du soir\fg {}; + \item $W$ : \og la fiche correspond à une formule \emph{Wok} \fg{} ; + \item $B$ : \og la fiche correspond à une formule \emph{Burger} \fg. + \end{enumerate} + \setlength\parindent{0mm} + + \medskip + + \begin{enumerate} + \item Recopier puis compléter l'arbre pondéré + + \begin{center} + \begin{tikzpicture}[sloped] + \node {.} + child {node {$M$} + child {node {$W$} + edge from parent + node[above] {...} + } + child {node {$B$} + edge from parent + node[above] {...} + } + edge from parent + node[above] {...} + } + child[missing] {} + child { node {$S$} + child {node {$W$} + edge from parent + node[above] {...} + } + child {node {$B$} + edge from parent + node[above] {...} + } + edge from parent + node[above] {...} + } ; + \end{tikzpicture} + \end{center} + + \item Calculer la probabilité de l'évènement $M \cap W$. Interpréter ce résultat dans le contexte de l'exercice. + \item Montrer que la probabilité que la fiche choisie corresponde à une formule \emph{Burger} est égale à $0.2899$. + \item On a prélevé une fiche correspondant à la formule \emph{Burger}. Quelle est la probabilité, arrondie au millième, que la vente ait eu lieu le soir? + \end{enumerate} +\end{exercise} + +\begin{solution} + \begin{enumerate} + \item + \begin{center} + \begin{tikzpicture}[sloped] + \node {.} + child {node {$M$} + child {node {$W$} + edge from parent + node[above] {$0.75$} + } + child {node {$B$} + edge from parent + node[above] {$0.25$} + } + edge from parent + node[above] {$0.43$} + } + child[missing] {} + child { node {$S$} + child {node {$W$} + edge from parent + node[above] {$0.68$} + } + child {node {$B$} + edge from parent + node[above] {$0.32$} + } + edge from parent + node[above] {$0.57$} + } ; + \end{tikzpicture} + \end{center} + \item On calcule la probabilité que la vente soit un wok et ait eu lieu à midi + \[ P(M\cap W) = P(M) \times P_M(W) = 0.43 \times 0.75 = 0.3225 \] + \item Probabilité que la vente soit un burger. + \[ + P(B) = P(M\cap B) + P(S\cap B) = 0.43 \times 0.75 + 0.57 \times 0.68 = 0.2899 + \] + \item On cherche à calculer la quantité $P_B(S)$. Pour cela on utilise la formule de Bayes + \[ + P_B(S) = \frac{P(B\cap S)}{P(B)} = \frac{P_S(B) \times P(S)}{P(B)} = \frac{0.32\times 0.57}{0.2899} = 0.6291824767161089 \approx 0.629 + \] + \end{enumerate} +\end{solution} + +\begin{exercise}[subtitle={Continent plastique}] + \textit{Les quantités évoqués dans cette exercice sont générés au hasard et sont donc complètement farfelus.} + \medskip + Le \og continent de plastique\fg{} est la plus grande des plaques de déchets plastiques évoluant sur les océans. Elle occupe actuellement dans l'océan Pacifique une surface dont l'aire est évaluée à plus de $1,6$ million de km$^2$, entre Hawaï et la Californie. + + En 2017, des scientifiques ont estimé qu'il y avait $7$ millions de tonnes de déchets plastiques qui était déversé chaque année dans les océans et que cette quantité augmentait de $11\n\%$ par chaque année. + + On modélise l'évolution de la masse de ces déchets plastiques déversée chaque année, si rien n'est fait pour la réduire, par une suite géométrique $\left(u_n\right)$. L'arrondi au centième du terme $u_n$ représente la masse de ces déchets déversée chaque année, exprimée en million de tonnes, pour l'année $(2017 + n)$. + + \medskip + + \begin{enumerate} + \item Expliquer pourquoi la suite $u_n$ est géométrique? + \item Calculer $u_1$ et $u_2$. + \item Exprimer $u_n$ en fonction de $n$. + \item Au début de l'année 2017, il y avait $300$ millions de tonnes de déchets plastique. Calculer la quantité totale de déchets plastiques en 2030. + \item On souhaite déterminer en quelle année la masse totale de ces déchets plastiques aura pour la première fois augmenté de $50$\,\% par rapport à sa valeur de 2017. + \begin{enumerate} + \item Recopier et compléter l'algorithme ci-dessous pour que la variable $N$ contienne la réponse au problème posé. + + \begin{center} + \begin{tabularx}{0.4\linewidth}{|X|}\hline + $N = 2017$\\ + $U = 7$ \\ + $S = 300 + U$ \\ + while $S < 450$: \\ + \hspace{1cm} $N = \ldots$\\ + \hspace{1cm} $U = \ldots$\\ + \hspace{1cm} $S = \ldots$\\ + \hline + \end{tabularx} + \end{center} + \item Que contiennent les variables $S$, $U$ et $N$ après exécution de cet algorithme ? + + Interpréter les résultats dans le contexte de l'exercice. + \end{enumerate} + \end{enumerate} +\end{exercise} + +\begin{solution} + \begin{enumerate} + \item Une augmentation de $11\,\%$ revient à multiplier la quantité par $1.11$. La suite est donc bien géométrique. Son premier terme est $u_0 = 7$ et sa raison est $q = 1.11$ + \item + \[ + u_1 = u_0 * 1.11 = 7.7700000000000005 + \] + \[ + u_2 = u_0 * 1.11^2 = 8.6247 + \] + \item + \[ + u_n = u_0 \times q^n = 7 \times 1.11^n + \] + \item On calcule la quantité totale déversée entre 2017 et 2030. + \[ + \sum_{n = 0}^{13} u_n = u_0 \times \frac{1-q^{13}}{1-q} = 7 \times \frac{1 - 1.11^{13}}{1 - 1.11} = 183.48 + \] + On en déduit la quantité totale de déchets en 2030 + \[ + 300 + 183.48 = 483.48 + \] + \item + \begin{enumerate} + \item ~ + \begin{center} + \begin{tabularx}{0.4\linewidth}{|X|}\hline + $N \gets 2017$\\ + $U \gets 7$ \\ + $S \gets 300 + U$ \\ + Tant que $S < 450$ \\ + \hspace{1cm} $N \gets N + 1$\\ + \hspace{1cm} $U \gets U * 1.11$\\ + \hspace{1cm} $S \gets S + u$\\ + Fin Tant que\\\hline + \end{tabularx} + \end{center} + \item \textit{Pas de correction automatisé} + \end{enumerate} + \end{enumerate} +\end{solution} + +\end{document} + +%%% Local Variables: +%%% mode: latex +%%% TeX-master: "master" +%%% End: diff --git a/TST/DS/DS_21_04_07/TST3/corr_20_210407_DS8.tex b/TST/DS/DS_21_04_07/TST3/corr_20_210407_DS8.tex new file mode 100644 index 0000000..bc5f35d --- /dev/null +++ b/TST/DS/DS_21_04_07/TST3/corr_20_210407_DS8.tex @@ -0,0 +1,262 @@ +\documentclass[a4paper,10pt]{article} +\usepackage{myXsim} + +% Title Page +\title{DS8 \hfill ZENAGUI Yanis} +\tribe{TST} +\date{\hfillÀ render pour le Mercredi 7 avril} + +\xsimsetup{ + solution/print = true +} + +\begin{document} +\maketitle + +\begin{exercise}[subtitle={Automatismes}] + \textit{Toutes les questions de cette exercice sont indépendantes et peuvent être répondus séparément} + \begin{enumerate} + \item De janvier à septembre, une quantité a augmenté de $23\,\%$. Faire un schéma pour représenter la situation puis calculer le taux d'évolution moyen mensuel. + \item Une quantité augmente de $23\,\%$ par ans. En 2020, elle est de 135\euro. Quelle était sa valeur en 2019? Faire un schéma pour représenter la situation. + \item Déterminer l'équation de la droite \\ + \begin{tikzpicture}[xscale=0.8, yscale=0.5] + \tkzInit[xmin=-5,xmax=5,xstep=1, + ymin=-5,ymax=5,ystep=1] + \tkzGrid + \tkzAxeXY + \tkzFct[domain=-5:5,color=red,very thick]% + {0.5*\x -1}; + \end{tikzpicture} + \item Résoudre l'équation $3 \times 0.55^x = 39$ + \end{enumerate} +\end{exercise} + +\begin{solution} + \begin{enumerate} + \item On veut partager cette évolution en 8 évolutions. + \[ + \left(1 + \frac{23}{100}\right)^{\frac{1}{8}} = 1.0262 + \] + Donc le taux d'évolution moyen est + \[ + t_m = 1.0262 - 1 = 0.0262 + \] + \item Coefficient multiplicateur pour revenir en arrière + \[ + CM = (1 + \frac{23}{100})^{-1} = 0.813 + \] + On en déduit la quantité en 2019 + \[ + 135 * 0.813 = 109.755 + \] + \item L'équation de la droite est + \[ + y = 0.5 x -1 + \] + \item Il faut penser à faire la division à par $3$ avant d'utiliser le log car sinon, on ne peut pas utiliser la formule $\log(a^n) = n\times \log(a)$. + + \[x = \frac{\log(13.0)}{\log(0.55)}\] + \end{enumerate} +\end{solution} + +\begin{exercise}[subtitle={Restaurant}] + Un \emph{food truck}, ouvert le midi et le soir, propose deux types de formules : + + \setlength\parindent{10mm} + \begin{itemize} + \item la formule \emph{Burger} ; + \item la formule \emph{Wok}. + \end{itemize} + \setlength\parindent{0mm} + + \medskip + + Le gérant a remarqué que 9\,\% de ses ventes ont lieu le midi. Le quart des ventes du midi correspondent à la formule \emph{Burger}, alors que 11\,\% des ventes du soir correspondent à la formule \emph{Wok}. + + Le gérant se constitue un fichier en notant, pour chaque vente, la formule choisie et le moment de cette vente (midi ou soir). + + On prélève une fiche de façon équiprobable. On définit les quatre évènements suivants: + + \begin{enumerate} + \item $M$ : \og la fiche correspond à une vente du midi\fg{} ; + \item $S$ : \og la fiche correspond à une vente du soir\fg {}; + \item $W$ : \og la fiche correspond à une formule \emph{Wok} \fg{} ; + \item $B$ : \og la fiche correspond à une formule \emph{Burger} \fg. + \end{enumerate} + \setlength\parindent{0mm} + + \medskip + + \begin{enumerate} + \item Recopier puis compléter l'arbre pondéré + + \begin{center} + \begin{tikzpicture}[sloped] + \node {.} + child {node {$M$} + child {node {$W$} + edge from parent + node[above] {...} + } + child {node {$B$} + edge from parent + node[above] {...} + } + edge from parent + node[above] {...} + } + child[missing] {} + child { node {$S$} + child {node {$W$} + edge from parent + node[above] {...} + } + child {node {$B$} + edge from parent + node[above] {...} + } + edge from parent + node[above] {...} + } ; + \end{tikzpicture} + \end{center} + + \item Calculer la probabilité de l'évènement $M \cap W$. Interpréter ce résultat dans le contexte de l'exercice. + \item Montrer que la probabilité que la fiche choisie corresponde à une formule \emph{Burger} est égale à $0.8324$. + \item On a prélevé une fiche correspondant à la formule \emph{Burger}. Quelle est la probabilité, arrondie au millième, que la vente ait eu lieu le soir? + \end{enumerate} +\end{exercise} + +\begin{solution} + \begin{enumerate} + \item + \begin{center} + \begin{tikzpicture}[sloped] + \node {.} + child {node {$M$} + child {node {$W$} + edge from parent + node[above] {$0.75$} + } + child {node {$B$} + edge from parent + node[above] {$0.25$} + } + edge from parent + node[above] {$0.09$} + } + child[missing] {} + child { node {$S$} + child {node {$W$} + edge from parent + node[above] {$0.11$} + } + child {node {$B$} + edge from parent + node[above] {$0.89$} + } + edge from parent + node[above] {$0.91$} + } ; + \end{tikzpicture} + \end{center} + \item On calcule la probabilité que la vente soit un wok et ait eu lieu à midi + \[ P(M\cap W) = P(M) \times P_M(W) = 0.09 \times 0.75 = 0.0675 \] + \item Probabilité que la vente soit un burger. + \[ + P(B) = P(M\cap B) + P(S\cap B) = 0.09 \times 0.75 + 0.91 \times 0.11 = 0.8324 + \] + \item On cherche à calculer la quantité $P_B(S)$. Pour cela on utilise la formule de Bayes + \[ + P_B(S) = \frac{P(B\cap S)}{P(B)} = \frac{P_S(B) \times P(S)}{P(B)} = \frac{0.89\times 0.91}{0.8324} = 0.9729697260932244 \approx 0.973 + \] + \end{enumerate} +\end{solution} + +\begin{exercise}[subtitle={Continent plastique}] + \textit{Les quantités évoqués dans cette exercice sont générés au hasard et sont donc complètement farfelus.} + \medskip + Le \og continent de plastique\fg{} est la plus grande des plaques de déchets plastiques évoluant sur les océans. Elle occupe actuellement dans l'océan Pacifique une surface dont l'aire est évaluée à plus de $1,6$ million de km$^2$, entre Hawaï et la Californie. + + En 2017, des scientifiques ont estimé qu'il y avait $4$ millions de tonnes de déchets plastiques qui était déversé chaque année dans les océans et que cette quantité augmentait de $19\n\%$ par chaque année. + + On modélise l'évolution de la masse de ces déchets plastiques déversée chaque année, si rien n'est fait pour la réduire, par une suite géométrique $\left(u_n\right)$. L'arrondi au centième du terme $u_n$ représente la masse de ces déchets déversée chaque année, exprimée en million de tonnes, pour l'année $(2017 + n)$. + + \medskip + + \begin{enumerate} + \item Expliquer pourquoi la suite $u_n$ est géométrique? + \item Calculer $u_1$ et $u_2$. + \item Exprimer $u_n$ en fonction de $n$. + \item Au début de l'année 2017, il y avait $300$ millions de tonnes de déchets plastique. Calculer la quantité totale de déchets plastiques en 2030. + \item On souhaite déterminer en quelle année la masse totale de ces déchets plastiques aura pour la première fois augmenté de $50$\,\% par rapport à sa valeur de 2017. + \begin{enumerate} + \item Recopier et compléter l'algorithme ci-dessous pour que la variable $N$ contienne la réponse au problème posé. + + \begin{center} + \begin{tabularx}{0.4\linewidth}{|X|}\hline + $N = 2017$\\ + $U = 4$ \\ + $S = 300 + U$ \\ + while $S < 450$: \\ + \hspace{1cm} $N = \ldots$\\ + \hspace{1cm} $U = \ldots$\\ + \hspace{1cm} $S = \ldots$\\ + \hline + \end{tabularx} + \end{center} + \item Que contiennent les variables $S$, $U$ et $N$ après exécution de cet algorithme ? + + Interpréter les résultats dans le contexte de l'exercice. + \end{enumerate} + \end{enumerate} +\end{exercise} + +\begin{solution} + \begin{enumerate} + \item Une augmentation de $19\,\%$ revient à multiplier la quantité par $1.19$. La suite est donc bien géométrique. Son premier terme est $u_0 = 4$ et sa raison est $q = 1.19$ + \item + \[ + u_1 = u_0 * 1.19 = 4.76 + \] + \[ + u_2 = u_0 * 1.19^2 = 5.6644 + \] + \item + \[ + u_n = u_0 \times q^n = 4 \times 1.19^n + \] + \item On calcule la quantité totale déversée entre 2017 et 2030. + \[ + \sum_{n = 0}^{13} u_n = u_0 \times \frac{1-q^{13}}{1-q} = 4 \times \frac{1 - 1.19^{13}}{1 - 1.19} = 180.98 + \] + On en déduit la quantité totale de déchets en 2030 + \[ + 300 + 180.98 = 480.98 + \] + \item + \begin{enumerate} + \item ~ + \begin{center} + \begin{tabularx}{0.4\linewidth}{|X|}\hline + $N \gets 2017$\\ + $U \gets 4$ \\ + $S \gets 300 + U$ \\ + Tant que $S < 450$ \\ + \hspace{1cm} $N \gets N + 1$\\ + \hspace{1cm} $U \gets U * 1.19$\\ + \hspace{1cm} $S \gets S + u$\\ + Fin Tant que\\\hline + \end{tabularx} + \end{center} + \item \textit{Pas de correction automatisé} + \end{enumerate} + \end{enumerate} +\end{solution} + +\end{document} + +%%% Local Variables: +%%% mode: latex +%%% TeX-master: "master" +%%% End: diff --git a/TST/DS/DS_21_04_07/TST3/corr_all_210407_DS8.pdf b/TST/DS/DS_21_04_07/TST3/corr_all_210407_DS8.pdf new file mode 100644 index 0000000..05caa04 Binary files /dev/null and b/TST/DS/DS_21_04_07/TST3/corr_all_210407_DS8.pdf differ diff --git a/TST/DS/DS_21_04_07/tpl_210407_DS8.tex b/TST/DS/DS_21_04_07/tpl_210407_DS8.tex index 0087f72..f8661a0 100644 --- a/TST/DS/DS_21_04_07/tpl_210407_DS8.tex +++ b/TST/DS/DS_21_04_07/tpl_210407_DS8.tex @@ -20,7 +20,7 @@ \item De janvier à septembre, une quantité a augmenté de $\Var{t}\,\%$. Faire un schéma pour représenter la situation puis calculer le taux d'évolution moyen mensuel. %- set valeur = randint(110, 150) \item Une quantité augmente de $\Var{t}\,\%$ par ans. En 2020, elle est de \Var{valeur}\euro. Quelle était sa valeur en 2019? Faire un schéma pour représenter la situation. - \item Déterminer l'équation de la droite + \item Déterminer l'équation de la droite \\ %- set b = randint(-4, -1) %- set denom = randint(2, 4) %- set a = -2*b/denom @@ -192,7 +192,7 @@ \end{solution} \begin{exercise}[subtitle={Continent plastique}] - \textit{Les quantités évoqués dans cette exercices sont générés au hasard et sont donc complètement farfelus.} + \textit{Les quantités évoqués dans cette exercice sont générés au hasard et sont donc complètement farfelus.} \medskip %- set u0 = randint(2, 20) %- set t = round(0.1 + 0.2*random(), 2) @@ -216,14 +216,14 @@ \begin{center} \begin{tabularx}{0.4\linewidth}{|X|}\hline - $N \gets 2017$\\ - $U \gets \Var{u0}$ \\ - $S \gets 300 + U$ \\ - Tant que $S < 450$ \\ - \hspace{1cm} $N \gets \ldots$\\ - \hspace{1cm} $U \gets \ldots$\\ - \hspace{1cm} $S \gets \ldots$\\ - Fin Tant que\\\hline + $N = 2017$\\ + $U = \Var{u0}$ \\ + $S = 300 + U$ \\ + while $S < 450$: \\ + \hspace{1cm} $N = \ldots$\\ + \hspace{1cm} $U = \ldots$\\ + \hspace{1cm} $S = \ldots$\\ + \hline \end{tabularx} \end{center} \item Que contiennent les variables $S$, $U$ et $N$ après exécution de cet algorithme ?